You are on page 1of 274

ĐỀ THI DUYÊN HẢI BẮC BỘ

MÔN TIẾNG ANH

vectorstock.com/31086119

Ths Nguyễn Thanh Tú


eBook Collection

TỔNG HỢP ĐỀ ĐỀ XUẤT KÌ THI HSG KHU


VỰC DUYÊN HẢI VÀ ĐỒNG BẰNG BẮC BỘ
MÔN TIẾNG ANH KHỐI 10, 11 NĂM 2023
(CÓ ĐÁP ÁN VÀ PHẦN NGHE)
WORD VERSION | 2023 EDITION
ORDER NOW / CHUYỂN GIAO QUA EMAIL
[email protected]

Tài liệu chuẩn tham khảo


Phát triển kênh bởi
Ths Nguyễn Thanh Tú
Đơn vị tài trợ / phát hành / chia sẻ học thuật : 
Nguyen Thanh Tu Group

Hỗ trợ trực tuyến


Fb www.facebook.com/DayKemQuyNhon
Mobi/Zalo 0905779594
HỘI CÁC TRƯỜNG THPT CHUYÊN ĐỀ THI MÔN ANH KHỐI 10
VÙNG DUYÊN HẢI VÀ ĐỒNG BẰNG BẮC BỘ NĂM 2023
TRƯỜNG THPT CHUYÊN BIÊN HÒA Thời gian làm bài 180 phút

L
TỈNH HÀ NAM Đề thi gồm …..trang

A
ĐỀ THI ĐỀ XUẤT

CI
I. LISTENING (50 points)
Part 1. You will hear part of an interview with a music journalist called Pip Rogers and

FI
a musician called Heath Francis about the renewed popularity of vinyl records. For
questions 1- 6, choose the best answer (A, B, C or D). (20 points)
1. What surprised Pip about the comeback of vinyl records?

OF
A It has not created as much income as expected.
B It is happening only now rather than in the 1990s.
C. Some of the old record factories haven’t closed down.
D Vinyl manufacturers are struggling to meet demand.

ƠN
2. In Heath's opinion, what is the main attraction of records?
A. They are an object you can possess.
B. They can become a serious hobby.
C. They are often cheaper than CDs.
NH
D. They are durable and long lasting.
3. What does Pip say about young people and records?
A. They prefer listening to current music on records.
B. Vinyl records offer them a novel experience.
Y

C. They are frustrated by vinyl because of its limitations.


D They have alternative ways of expressing their musical tastes.
QU

4. When asked about the future of records, Pip reveals she is


A. confident that people will want to invest in the industry
B. unsure whether the requirements for growth will be met.
C. concerned people will lose interest in vinyl eventually.
M

D. surprised that vinyl is perceived as a passing trend.


5. What both Pip and Health appreciate about digital music is

A. not having to pay for every song you hear.


B. being able to listen to music anywhere.
C. having access to a wide variety of music.
D. being able to share music instantaneously.
Y

Your answers:
1. 2. 3. 4. 5.
DẠ

1
Part 2. Listen to a podcast on climate change and decide whether the following statements
are True (T), False (F), or Not Given (NG) according to what you hear. (10 points)
1._________ A 1.5 degree rise in temperature makes virtually no difference to the world we are

L
living in.

A
2._________ There is little likelihood of the aviation undergoing carbon neutral due to some
governments’ objections.

CI
3._________ Every year, the number of people suffering from diseases related to exceptional
heatwaves increases by 65 million.

FI
4._________ Notwithstanding the melted ice rivers, the number of people lacking water for daily
lives will increase twofold.
5._________ Continually rising carbon emission is synonymous with a world ridden with

OF
disasters.
Your answers:
1. 2. 3. 4. 5.

ƠN
Part 3. Listen to a talk about the unique characteristics of glass and fill each blanks with
NO MORE THAN FOUR WORDS. (20 points)
NH
Glass is technically known as any (1)_________________________, whose structures of atoms
and molecules are in disorder.
In ice, the water molecules (2)____________________________ each other and lock themselves
into a repeating crystal pattern.
As glass cools, its molecules (3)_________________________ until they stop moving
Y

altogether.
QU

One study from 2017 estimated that if a cathedral were to stand at room temperature for a billion
years, it’s glass would flow (4)____________________________________.
Another research team from Spain examined samples of 110 million-year-old amber, a naturally
occurring variety of glass (5)_______________________________, and found that over its long
existence it had become about (6)__________________________________.
M

Glass, if cooled slowly and infinitely until it hardens, would have


(7)_____________________________ like a crystal’s.

Non-ideal glass is thought to be riddled with two-level systems, which, when going near absolute
zero, can (8)________________________________, absorbing heat in the process.
In a very different glassmaking technique that makes use of (9)_________________________,
glass is built one molecule at a time.
Y

Old cathedral glass is believed to be thicker at the bottom because it’s


(10)___________________________. However, that’s just due to the technique used to make
DẠ

the glass.

2
II. LEXICO-GRAMMAR (40 points)
Part 1. Choose the correct answer (A, B, C or D) to complete the sentences. (20 points)

L
1. Walking down the street, I noticed that_________ had a beautiful garden.
A. entire houses B. either of the houses

A
C. each and every house D. enough houses

CI
2. The fire spread quickly, leaving devastation in its _________.
A. zone B. blaze C. wake D. line
3. The dog’s owner has no idea what may have _________ the attack.

FI
A. spelled B. provoked C. embodied D. heralded
4. I’m not sure I believe Jason’s story- it is a bit_________.
A. plausible B. inhospitable C. far-fetched

OF
D. endless
5. His health was gradually ________ by drink and drugs.
A. ravaged B. injured C. destroyed D. rectified
6. ________, I had to buy a new one.
A. My dictionary losing B. My dictionary had been lost

ƠN
C. My dictionary having been lost D. Because my dictionary lost
7. The sheep were huddled into a ____________ to protect them from overnight frosts.
A. cage B. kennel C. pen D. hutch
NH
8. We stand on the ____________ of a new era in space exploration.
A. threshold B. basis C. brink D. surface
9. Three lives were lost in an accident at the____________ of a driver who had been drinking.
A. mercy B. expense C. high D. hands
10. The _______ of chickens in cages is cruel, so many farmers allow them to wander freely.
Y

A. constraint B. confinement C. distress D. slaughter


QU

11. We can’t always rely on ________ on time.


A. the buses that arriving B. the arriving of buses
C. the buses to arrive D. the buses’ arriving
12. The ________ of marriage in Viet Nam remains popular in spite of the high divorce rate recently.
A. state B. habit C. practice D. institution
M

13. At the ________ of organizing committee, personnel must wear their identity badges at all time.
A. request B. assistance C. demands D. interests

14. IU is known for being a ________ girl who can take the roles of a singer, a song writer and an
actress as well.
A. versatile B. changeable C. variable D. diverse
15. This isn’t a family any longer, no one seems to care about solving ________ problem.
Y

A. anyone’s else B. anyone else’s C. any other D. any other’s


DẠ

16. Since the torture scandal several months ago, there have been ill ________ between area
residents and police.
A. emotions B. feelings C. affections D. sensation

3
17. The new development project of Phu Ly city has begun to ________ on the surrounding
green belt.
A. enter B. intrude C. encroach D. reach

L
18. Ha Nam has been attracting foreign investment due to the present favorable political ________ of

A
the province.
A. temperature B. climate C. state D. weather

CI
19. Don’t worry, Will’s been going through a rebellious phase as any other teenager. He’ll gradually
________ it.

FI
A. grow into B. get through to
C. grow out of D. get on with it
20. I’ve ________ as inconsiderate a person as Chander.

OF
A. ever to have met B. been met
C. yet to meet D. never been meeting
Your answers:

ƠN
1. 2. 3. 4. 5. 6. 7. 8. 9. 10.

11. 12. 13. 14. 15. 16. 17. 18. 19. 20.

Part 2. Give the correct form of each word in the blanket to complete the following sentences.
NH
(10 points)
1. How we ourselves were treated by our parents in our youth can have an ______________
effect on who we become as parents. APPRECIATE
2. Employees who have been nursing a ______________ against their manager are encouraged
to discuss it with the board. GRIEF
Y

3. Although air travel is admittedly fast, passengers are still subject to the unexpected
QU

______________ which seem to be inherent in any form of travel. HOLD


4. He became an ______________ by defying the law. LAW
5. That old concrete block of flats is a real ______________ in such a modern city. EYE
6. Passing the final exam with flying colour, I was overcome by relief and ______________ joy.
ADULT
M

7. The _____________ of the movement of Mars by Kepler led to the formation of his three
laws of planetary motion. DOCUMENT

8. I think this printer has _____________ its usefulness and should be sold. LIVE
9. It would be erroneous to believe that they are born weaker, more sensitive or with a greater
natural _____________ to cry. Boys have feelings too. INCLINE
10. The headmaster, Bowyer, was a severe ______________ but respected by his pupils.
Y

DISCIPLINE
DẠ

Your answers:
1. 2. 3. 4. 5.
6. 7. 8. 9. 10.

4
Part 3. Fill in each blank with one or two prepositions/particles. (10 points)
1. This evening Manchester United takes ________ Barcelona.
2. When John and Frank first met, they hit it ________ immediately. They have been good friends

L
ever since.

A
3. You might have gotten a higher grade if you didn't cram ________ that exam at the last minute.
4. Another way of saying 'old-fashioned' is '________ the times'.

CI
5. Growing environmental fears have made climate research all the more important,
and Europe is forging ________ in this field.

FI
6. The thing I don't like about my present job is that we often have to work up to 10 hours
_______ a stretch.
7. He manipulates people and tries to bend them ________ his will.

OF
8. The authorities plan to bring _________ new regulations on the application of pesticides.
9. If I could do it ______ again, I’d do it differently.
10. I wish you wouldn’t fly_______ me like that every time I make a mistake.
Your answers:

ƠN
1. 2. 3. 4. 5.
6. 7. 8. 9. 10.

SECTION C: READING (60 points)


NH
Part 1. Read the following article and decide which answer (A, B, C or D) best fits each gap.
(15 points)
WE REALLY CAN TELL IF WE ARE BEING WATCHED
Stories about how people somehow know when they are being watched have been going
Y

around for years. However, few (0) ____________ have been made to investigate the phenomenon
scientifically. Now, with the completion of the largest ever study of the so-called staring effect,
QU

there is impressive evidence that this is a recognizable and (1) __________ sixth sense. The study
(2) ___________hundreds of children. For the experiments, they sat with their eyes (3)
____________ so they could not see, and with their backs to other children, who were told to
either stare at them or look away. Time and time again the results showed that the children who
M

could not see were able to (4) __________ when they were being stared at. In a (5) ____________
of more than 18,000 trials (6) ____________ worldwide, the children (7) ____________ sensed

when they were being watched almost 70% of the time. The experiment was repeated with the (8)
__________precaution of putting the children who were being watched outside the room, (9)
____________ from the starers by the windows. This was done just in case there was some (10)
_________ going on with the children telling each other whether they were looking or not.
Y

0. A. tries B. tests C. attempts D. aims


1. A. genuine B. accepted C. received D. sure
DẠ

2. A. involved B. contained C. comprised D. enclosed


3. A. shaded B. wrapped C. masked D. covered
4. A. find B. notice C. tell D. reveal
5
5. A. sum B. collection C. mass D. total
6. A. worked over B. worked through C. carried on D. carried out
7. A. correctly B. exactly C. thoroughly D. perfectly

L
8. A. attached B. added C. connected D. increased

A
9. A. separated B. parted C. split D. divided
10. A. pretending B. lying C. cheating D. deceiving

CI
Your answers:
1. 2. 3. 4. 5.

FI
6. 7. 8. 9. 10.

OF
Part 2. Read the following text and fill in one word which best fits each gap. Use only ONE
word in each gap. (15 points)
One of the hazards that electronic media like the television, radio or computers (1) _______
these days is the decline in book reading.

ƠN
The concern applies mainly (2) _______ the younger generations who are strongly tempted
by the glamour of the silver screen and, (3) _______, don’t recognize the importance of acquiring
first-hand information from books.
To encourage reading for pleasure and to propagate a wide array of publications (4)
NH
_______ encyclopedias, reference books, manuals or fiction, radical solutions should be applied.
Firstly, more (5) _____ ought to be put on the educational factor. Youngsters should be made to
feel comfortable while reading either for information or self-satisfaction in (6) _____ places like
airports, buses or on the beach. Secondly, libraries must be subsidized (7) _____ accurately in order
Y

to provide the potential readers with ample choice of publications and to become more publically
active so as to put books at people’s (8) _____ rather than keep them under lock and key. Fund
QU

collecting actions organized by libraries might also raise the public awareness of the advantages of
becoming engrossed in a good book. (9) _____, the mass media themselves might contribute
substantially by recommending the purchase of valuable best-sellers (10) _____ inspiring their
viewers to enrich knowledge and erudition, and thus helping them to develop the habit of
M

spontaneous every reading.


Your answers:
1. 2. 3. 4. 5.

6. 7. 8. 9. 10.
Y
DẠ

6
Part 3. Read the passage and choose the best option (A, B, C, or D) to answer the questions.
Write your answers in the corresponding numbered boxes. (15 points)
HISTORY OF THE CHICKENPOX VACCINE

L
Chickenpox is a highly contagious infectious disease caused by the Varicella zoster virus; sufferers

A
develop a fleeting itchy rash that can spread throughout the body. The disease can last for up to 14
days and can occur in both children and adults, though the young are particularly vulnerable.

CI
Individuals infected with chickenpox can expect to experience a high but tolerable level of
discomfort and a fever as the disease works its way through the system. The ailment was once

FI
considered to be a “rite of passage” by parents in the U.S. and thought to provide children
with greater and improved immunity to other forms of sickness later in life. This view,
however, was altered after additional research by scientists demonstrated unexpected dangers

OF
associated with the virus. Over time, the fruits of this research have transformed attitudes toward
the disease and the utility of seeking preemptive measures against it.
A vaccine against chickenpox was originally invented by Michiaki Takahashi, a Japanese doctor
and research scientist, in the mid-1960s. Dr. Takahashi began his work to isolate and grow the

ƠN
virus in 1965 and in 1972 began clinical trials with a live but weakened form of the virus that
caused the human body to create antibodies. Japan and several other countries began widespread
chickenpox vaccination programs in 1974. However, it took over 20 years for the chickenpox
vaccine to be approved by the U.S. Food & Drug Administration (FDA), finally earning the U.S.
NH
government’s seal of approval for widespread use in 1995. Yet even though the chickenpox
vaccine was available and recommended by the FDA, parents did not immediately choose to
vaccinate their children against this disease. Mothers and fathers typically cited the notion that
chickenpox did not constitute a serious enough disease against which a person needed to be
Y

vaccinated.
Strong belief in that view eroded when scientists discovered the link between Varicella zoster, the
QU

virus that causes chickenpox, and shingles, a far more serious, harmful, and longer-lasting disease
in older adults that impacts the nervous system. They reached the conclusion that Varicella zoster
remains dormant inside the body, making it significantly more likely for someone to develop
shingles. As a result, the medical community in the U.S. encouraged the development, adoption,
M

and use of a vaccine against chickenpox to the public. Although the appearance of chickenpox and
shingles within one person can be many years apart - generally many decades - the increased risk

in developing shingles as a younger adult (30-40 years old rather than 60-70 years old) proved to
be enough to convince the medical community that immunization should be preferred to the
traditional alternative.
Another reason that the chickenpox vaccine was not immediately accepted and used by parents in
Y

the U.S. centered on observations made by scientists that the vaccine simply did not last long
enough and did not confer a lifetime of immunity. In other words, scientists considered the benefits
DẠ

of the vaccine to be temporary when given to young children. They also feared that it increased
the odds that a person could become infected with chickenpox later as a young adult, when the
rash is more painful and prevalent and can last up to three or four weeks. Hence, allowing young
7
children to develop chickenpox rather than take a vaccine against it was believed to be the “lesser
of two evils.” This idea changed over time as booster shots of the vaccine elongated immunity
and countered the perceived limits on the strength of the vaccine itself.

L
Today, use of the chickenpox vaccine is common throughout the world. Pediatricians suggest an

A
initial vaccination shot after a child turns one year old, with booster shots recommended after the
child turns eight. The vaccine is estimated to be up to 90% effective and has reduced worldwide

CI
cases of chickenpox infection to 400,000 cases per year from over 4,000,000 cases before
vaccination became widespread. ■ (A) In light of such statistics, most doctors insist that the

FI
potential risks of developing shingles outweigh the benefits of avoiding rare complications
associated with inoculations. ■ (B) Of course, many parents continue to think of the disease as an
innocuous ailment, refusing to take preemptive steps against it. ■ (C) As increasing numbers of

OF
students are vaccinated and the virus becomes increasingly rarer, however, even this trend among
parents has failed to halt the decline of chickenpox among the most vulnerable populations. ■ (D)
1. The word “tolerable” in the 1st passage is closest in meaning to
(A) sudden. (B) bearable. (C) infrequent. (D) unexpected.

ƠN
2. According to paragraph 1, which of the following is true of the chickenpox virus?
(A) It leads to a potentially deadly disease in adults.
(B) It is associated with a possibly permanent rash.
(C) It is easily transmittable by an infected individual.
NH
(D) It has been virtually eradicated in the modern world.
3. Which of the following best expresses the essential information in the highlighted sentence?
(A) U.S. parents believed that having chickenpox benefited their children.
(B) U.S. parents believed that chickenpox led to immunity against most sickness.
Y

(C) U.S. parents wanted to make sure that their children developed chickenpox.
(D) U.S. parents did not think that other vaccinations were needed after chickenpox.
QU

4. what can be inferred from paragraph 2 about the clinical trials for the chickenpox vaccine?
(A) They took longer than expected. (B) They cost a lot of money to complete.
(C) They took a long time to finish. (D) They were ultimately successful.
nd
5. The word “notion” in the 2 passage is closest in meaning to
M

(A) history. (B) findings. (C) fact. (D) belief.


6. According to paragraph 3, which of the following is true of Varicella Zoster?

(A) It typically attacks adults who are over 60 years old.


(B) It is linked to a serious disease that occurs more commonly in adults.
(C) It likely is not a serious enough threat to human health to require a vaccine.
(D) It is completely eradicated from the body after chickenpox occurs.
Y

7. According to paragraph 3, all of the following is true about the chickenpox virus EXCEPT:
(A) It causes two distinct yet related ailments.
DẠ

(B) People did not view it as a serious public health threat.


(C) It tended to quickly become dormant and remain inoperative over time.
(D) Vaccination against it would help prevent the onset of shingles.
8
8. The author uses “booster shots” as an example of
(A) a way to increase the effectiveness of the chickenpox vaccine.
(B) a preferred method of chickenpox rash and fever treatment.

L
(C) a scientifically approved medicine to eliminate chickenpox.

A
(D) a strategy for parents to avoid vaccinating their child altogether.
9. According to paragraph 4, many parents did not choose the chickenpox vaccine because

CI
(A) they believed that the virus was weak and not especially harmful.
(B) they thought that scientists did not have enough data to reach a conclusion.

FI
(C) they were unsure about the utility of the vaccine given its expected duration.
(D) they were convinced it was potentially very toxic, particularly for older children.
10. Look at the four squares [■] that indicate where the following sentence could be added to the

OF
passage.
Meanwhile, some continue to remain unconvinced, citing a supposed potential of the
vaccine to do harm.
Where would the sentence fit best?

ƠN
(A) A (B) B (C) C (D) D
Your answers
1. 2. 3. 4. 5.
NH
6. 7. 8. 9. 10.

Part 4. Read the text and do the following tasks. (15 points)
Stadiums: past, present and future
A Stadiums are among the oldest forms of urban architecture: vast stadiums where the public
Y

could watch sporting events were at the centre of western city life as far back as the ancient
QU

Greek and Roman Empires, well before the construction of the great medieval cathedrals
and the grand 19th- and 20th-century railway stations which dominated urban skylines in
later eras.
Today, however, stadiums are regarded with growing scepticism. Construction costs can
soar above £1 billion, and stadiums finished for major events such as the Olympic Games or
M

the FIFA World Cup have notably fallen into disuse and disrepair.
But this need not be the case. History shows that stadiums can drive urban development and

adapt to the culture of every age. Even today, architects and planners are finding new ways
to adapt the mono-functional sports arenas which became emblematic of modernization
during the 20th century.
B The amphitheatre1 of Arles in southwest France, with a capacity of 25,000 spectators, is
Y

perhaps the best example of just how versatile stadiums can be. Built by the Romans in 90
DẠ

1amphitheatre: (especially in Greek and Roman architecture) an open circular or oval building with a
central space surrounded by tiers of seats for spectators, for the presentation of dramatic or sporting
events
9
AD, it became a fortress with four towers after the fifth century, and was then transformed
into a village containing more than 200 houses. With the growing interest in conservation
during the 19th century, it was converted back into an arena for the staging of bullfights,

L
thereby returning the structure to its original use as a venue for public spectacles.

A
Another example is the imposing arena of Verona in northern Italy, with space for 30,000
spectators, which was built 60 years before the Arles amphitheatre and 40 years before

CI
Rome’s famous Colosseum. It has endured the centuries and is currently considered one of
the world’s prime sites for opera, thanks to its outstanding acoustics.

FI
C The area in the centre of the Italian town of Lucca, known as the Piazza dell’Anfiteatro, is
yet another impressive example of an amphitheatre becoming absorbed into the fabric of the
city. The site evolved in a similar way to Arles and was progressively filled with buildings

OF
from the Middle Ages until the 19th century, variously used as houses, a salt depot and a
prison. But rather than reverting to an arena, it became a market square, designed by
Romanticist architect Lorenzo Nottolini. Today, the ruins of the amphitheatre remain
embedded in the various shops and residences surrounding the public square.

ƠN
D There are many similarities between modern stadiums and the ancient amphitheatres
intended for games. But some of the flexibility was lost at the beginning of the 20th century,
as stadiums were developed using new products such as steel and reinforced concrete, and
made use of bright lights for night-time matches.
NH
Many such stadiums are situated in suburban areas, designed for sporting use only and
surrounded by parking lots. These factors mean that they may not be as accessible to the
general public, require more energy to run and contribute to urban heat.
E But many of today’s most innovative architects see scope for the stadium to help improve
Y

the city. Among the current strategies, two seem to be having particular success: the stadium
as an urban hub, and as a power plant.
QU

There’s a growing trend for stadiums to be equipped with public spaces and services that
serve a function beyond sport, such as hotels, retail outlets, conference centres, restaurants
and bars, children’s playgrounds and green space. Creating mixed-use developments such
as this reinforces compactness and multi-functionality, making more efficient use of land
M

and helping to regenerate urban spaces.


This opens the space up to families and a wider cross-section of society, instead of catering

only to sportspeople and supporters. There have been many examples of this in the UK: the
mixed-use facilities at Wembley and Old Trafford have become a blueprint for many other
stadiums in the world.
F The phenomenon of stadiums as power stations has arisen from the idea that energy
Y

problems can be overcome by integrating interconnected buildings by means of a smart grid,


which is an electricity supply network that uses digital communications technology to detect
DẠ

and react to local changes in usage, without significant energy losses. Stadiums are ideal for
these purposes, because their canopies have a large surface area for fitting photovoltaic
panels and rise high enough (more than 40 metres) to make use of micro wind turbines.
10
Freiburg Mage Solar Stadium in Germany is the first of a new wave of stadiums as power
plants, which also includes the Amsterdam Arena and the Kaohsiung Stadium. The latter,
inaugurated in 2009, has 8,844 photovoltaic panels producing up to 1.14 GWh of electricity

L
annually. This reduces the annual output of carbon dioxide by 660 tons and supplies up to

A
80 percent of the surrounding area when the stadium is not in use. This is proof that a stadium
can serve its city, and have a decidedly positive impact in terms of reduction of CO2

CI
emissions.
G Sporting arenas have always been central to the life and culture of cities. In every era, the

FI
stadium has acquired new value and uses: from military fortress to residential village, public
space to theatre and most recently a field for experimentation in advanced engineering. The
stadium of today now brings together multiple functions, thus helping cities to create a

OF
sustainable future.
Questions 1–4: Reading Passage 2 has seven sections, A–G. Which section contains the
following information?
NB You may use any letter more than once.

ƠN
1. a mention of negative attitudes towards stadium building projects.
2. figures demonstrating the environmental benefits of a certain stadium.
3. examples of the wide range of facilities available at some new stadiums.
4. reference to the disadvantages of the stadiums built during a certain era.
NH
Questions 5-8: Complete the summary below. Choose ONE WORD ONLY from the passage for
each answer.
Roman amphitheatres
The Roman stadiums of Europe have proved very versatile. The amphitheatre of Arles, for
Y

example, was converted first into a 5. __________, then into a residential area and finally into an
arena where spectators could watch 6. __________. Meanwhile, the arena in Verona, one of the
QU

oldest Roman amphitheatres, is famous today as a venue where opera is performed. The site of
Lucca’s amphitheatre has also been used for many purposes over the centuries, including the
storage of 7. __________. It is now a market square with 8. __________ and homes incorporated
into the remains of the Roman amphitheatre.
M

Questions 9-10: Choose TWO letters, A–E. Which TWO advantages of modern stadium
design does the writer mention?

A. offering improved amenities for the enjoyment of sports events.


B. bringing community life back into the city environment.
C. facilitating research into solar and wind energy solutions.
D. enabling local residents to reduce their consumption of electricity.
Y

E. providing a suitable site for the installation of renewable power generators.


Your answers
DẠ

1. 2. 3. 4. 5.

11
6. 7. 8. 9. 10.

L
II. WRITING (50 points)

A
Part 1. 20 points
The table below shows the percentage of the population by age groups in one town who rode

CI
bicycles in 2011.
Summarise the information by selecting and reporting the main features and make comparisons
where relevant.

FI
OF
ƠN
NH
Part 2. 30 points
Write an essay of about 250 words on the following topic:
Some people feel that we should not keep animals in zoos because animals suffer in captivity.
Y

Others feel zoos play an important educational role in teaching us about wildlife. Do you believe
zoos still have a place in the modern world? Explain your opinion and give specific details and
QU

reasons to support your viewpoint.

---------- THE END ----------


M

Y
DẠ

12
HỘI CÁC TRƯỜNG THPT CHUYÊN ĐỀ THI MÔN ANH KHỐI 10
VÙNG DUYÊN HẢI VÀ ĐỒNG BẰNG BẮC BỘ NĂM 2023

L
TRƯỜNG THPT CHUYÊN BIÊN HÒA HƯỚNG DẪN CHẤM

A
TỈNH HÀ NAM
ĐỀ THI ĐỀ XUẤT

CI
I. LISTENING (50 points)

FI
Part 1. You will hear part of an interview with a music journalist called Pip Rogers
and a musician called Heath Francis about the renewed popularity of vinyl records.

OF
For questions 1- 6, choose the best answer (A, B, C or D). (20 points)
1. C 2. A 3. B 4. B 5. D

Part 2. Listen to a podcast on climate change and decide whether the following
statements are True (T), False (F), or Not Given (NG) according to what you hear. (10

ƠN
points)
https://www.youtube.com/watch?v=oqMTm5S8FxY#:~:text=every%20year%20every
%20choice%20matters,that%20we%20can%20barely%20even.
NH
1. F 2. T 3. NG 4. T 5. T

Part 3. Listen to a talk about the unique characteristics of glass and fill each blanks with
NO MORE THAN FOUR WORDS. (20 points)
https://www.youtube.com/watch?v=8Jsbyx3nMfU
Y

1. Rigid amorphous solid


QU

2. Tug on
3. Contract
4. A single nanometer
5. Derived from tree sap
M

6. 2% denser
7. ( Low )entropy

8. Quantum tunnel between configurations


9. Vapor disposition
10. Sagged over time
II. LEXICO-GRAMMAR (40 points)
Y

Part 1. Choose the correct answer (A, B, C or D) to complete the sentences. (20 points)
DẠ

1. C 2. C 3. B 4. C 5. A 6. C 7. C 8. A 9. D 10. B
11. D 12. D 13. A 14. A 15. B 16. B 17. C 18. B 19. C 20. C

1
Part 2. Give the correct form of each word in the blanket to complete the following
sentences. (10 points)

L
1. Appreciable

A
2. Grievance
3. Hold- ups

CI
4. Outlaw
5. Eyesore
6. Unadulterated

FI
7. Documentation
8. Outlived

OF
9. Inclination
10. Disciplinarian
Part 3. Fill in each blank with one or two prepositions/particles. (10 points)
1. on 2. off 3. for 4. behind 5. ahead

ƠN
6. at 7. to 8. in 9. over 10. at
III. READING (60 points)
Part 1. Read the following article and decide which answer (A, B, C or D) best fits each
gap. (15 points)
NH
1. A 2. A 3. D 4. C 5. D

6. D 7. A 8. B 9. A 10. C

Part 2. Read the following text and fill in one word which best fits each gap. Use only
Y

ONE word in each gap. (15 points)


1. Pose/ cause/ 2. To 3. Consequently/ 4. Like 5. Emphasis/
QU

create hence/ therefore focus


6. Public 7. More 8. Disposal 9. Finally 10. And

Part 3. Read the passage and choose the best option (A, B, C, or D) to answer the
M

questions. Write your answers in the corresponding numbered boxes. (15 points)

1. B 2. C 3. A 4. D 5. D
6. B 7. C 8. A 9. C 10. B

Part 4. Read the text and do the following tasks. (15 points)
Y

1. A 2. F 3. E 4. D 5. 6. 7. 8. 9-10. B, E
DẠ

FORTRESS BULLFIGHTS SALT SHOPS

2
IV. WRITING (50 points)
Part 1. 20 points

L
The table below shows the percentage of the population by age groups in one town who rode

A
bicycles in 2011. Summarise the information by selecting and reporting the main features and
make comparisons where relevant.

CI
*Striking features:
- This means of transport was most popular among the youngest age group(0-9)

FI
- A higher percentage of females cycled every day in comparison with males.
1. Completion (02 points): The report covers the table and has a clear structure of three parts:
Introduction, Overview and Body.

OF
2. Content (10 points)
- The report MUST cover the following points:
• Introduce the table (01 point) and state the general and striking features (05 points).
• Describe main features with relevant data from both charts and make relevant comparisons

ƠN
(05 points).
- The report MUST NOT contain personal opinions.
(A penalty of 01 point to 02 points will be given to personal opinions found in the answer.)
3. Organisation (02 points)
NH
- Ideas are well organised.
- The report is sensibly divided into paragraphs.
4. Punctuation and spelling (01 point)
The report should demonstrate the correct use of spelling and punctuations.
Y

5. Language use (05 points)


The report should:
QU

- Demonstrate a wide variety of lexical and grammatical structures.


- Make correct use of words (verb tenses, word forms, voice, etc.).
Part 2. 30 points
Write an essay of about 250 words on the following topic:
M

Some people feel that we should not keep animals in zoos because animals suffer in captivity.
Others feel zoos play an important educational role in teaching us about wildlife. Do you

believe zoos still have a place in the modern world? Explain your opinion and give specific
details and reasons to support your viewpoint.
The mark given to part 3 is based on the following scheme:
1. Content (10 points)
Y

- ALL requirements of the task are sufficiently addressed.


- Ideas are adequately supported and elaborated with relevant and reliable explanations,
DẠ

examples, evidence, personal experience, etc.


2. Organisation and Presentation (08 points)
- Ideas are well organised and presented with coherence, cohesion, and clarity.
3
- The essay is well-structured:
• The introduction is presented with a clear thesis statement.

L
• The body paragraphs are written with unity, coherence and cohesion. Each body

A
paragraph must have a topic sentence and supporting details and examples where necessary.
• The conclusion summarises the main points and offers personal opinions on the issue.

CI
3. Language (09 points)
- Demonstration of a variety of vocabulary and structures appropriate to the level of English
language gifted upper-secondary school students.

FI
- Good use and control of grammatical structures.
- Use of vocabulary must not create confusion for readers.

OF
4. Handwriting, punctuation, and spelling (03 points)
- Good punctuation and no spelling mistakes.
- Legible handwriting.

ƠN
---------- THE END ----------
NH
Y
QU
M

Y
DẠ

4
HỘI CÁC TRƯỜNG THPT CHUYÊN ĐỀ THI CHỌN HỌC SINH GIỎI LẦN THỨ XIII
KHU VỰC DUYÊN HẢI, ĐỒNG BẰNG BẮC BỘ MÔN THI: TIẾNG ANH – KHỐI 11
TRƯỜNG THPT CHUYÊN BIÊN HOÀ, T. HÀ NAM Ngày thi 15-16/07/2023

L
___________________ Thời gian làm bài 180 phút
ĐỀ THI ĐỀ XUẤT (Đề thi gồm … trang)

A
CI
SECTION A. LISTENING (50 points)
Part 1. You hear two people speaking about their friendship. Each given question has four
options A, B, C and D. Choose the best option for each question.

FI
1. One speaker felt her watershed moment was when_____
A. she watched a film under a blanket.
B. she turned fifty.

OF
C. she had an argument with a good friend.
D. she got married.
2. One speaker made a tongue-in-cheek comment about
A. The Tate.
B. the food they both like.
C. taking up knitting.

ƠN
D. singing in a choir.
3. One speaker expressed the view that a quilt of fear
A. was ideal for daydreaming.
B. was great for keeping you warm in front of the television.
C. protected you from imaginary concerns.
NH
D. made your wishes come true.
4. The speakers failed to agree about
A. the number of times they fell out.
B. where the quilt should be displayed.
C. the number of pieces the quilt should have.
Y

D. the best age to form lasting friendships.


5. What conclusion did they put forward?
QU

A. Friendships made in your teenage years reflect your aspirations.


B. Good friendships are hard to find.
C. The longer the friendship, the better it becomes.
D. It's best not to mix friends.
Your answers:
1. 2. 3. 4. 5.
M

Part 2. Listen to the recording and decide which of the following statements are True (F) and
which ones are False (F)?
1. The queen's personal income is primarily derived from her lucrative investment portfolio.
2. The Sovereign Grant is provided by the government to cover official duties and expenditures of
the royal family.
Y

3. The Crown Estate and the Royal Collection Trust are both separate entities that hold valuable
assets associated with the royal family.
DẠ

4. The monarchy's brand contributes over 1 billion to the national economy annually.
5. The Duchy of Cornwall serves as the primary source of income for the Duke of Lancaster.
Your answers:
1. 2. 3. 4. 5.

1 of 14
Part 3. Listen to the recording and fill in the gaps with NO MORE THAN THREE WORDS
and/or A NUMBER
The Shinkansen Bullet Train in Japan faced a noise problem when it exited tunnels due to the (1)

L
_______________ it created. To address this issue, an engineering team used (2)
___________________ to design a quieter and faster train.

A
The redesigned train took inspiration from birds such as owls, whose feathers influenced the design
of the (3) ________________.

CI
The pantograph's supporting shaft was redesigned based on the smooth body of the Adelie penguin
to reduce wind resistance and achieve a (4) _____________________.
The Kingfisher's beak served as a model for the train's nose design, which helped minimize noise by
reducing pressure waves and splashes, achieving the (5) ________________.

FI
The redesigned train, with its biomimetic components, achieved remarkable results, including being
10% faster and using (6) ___________________.
However, people who shape the world often lack experience in biology. Consequently, they are (7)

OF
___________________ when it comes to understanding the workings of the world.
Biomimicry has influenced various fields, including healthcare, where researchers have looked at
shark skin to develop bacteria-resistant surfaces for hospitals, mimicking the shark's (8)
________________________.
Mimicking natural processes, such as how ants communicate to efficiently find resources, has been

ƠN
applied in the development of software, including the movement of autonomous cars in a (9)
_____________________.
The idea of the (10) _______________________ promotes using materials in a way that eliminates
waste by continuously upcycling them, similar to how materials are reused and transformed in natural
ecosystems.
NH
Your answers:
1. 2. 3. 4. 5.

6. 7. 8. 9. 10.
Y

Part 4. Listen to a recording about Frederick, the duke of York and answer the following
QU

questions. Write NO MORE THAN THREE WORDS and/or A NUMBER for each answer.
1. In what type of composition was Frederick, the duke, made famous?
_______________________________________________
2. Who did his battles end in total disaster against?
_______________________________________________
3. What is the derogatory term employed to depict a collective of individuals perceived as
M

contemptible and devoid of value?


_______________________________________________
4. When did the Duke of York find himself subjected to ridicule through satirical songs?

_______________________________________________
5. During his involvement in a scandalous affair, what specific position did the Duke hold?
_______________________________________________

SECTION B. GRAMMAR & VOCABULARY (30 points)


Y

Part 1: Choose the answer A, B, C, or D that best completes each of the following
sentences. Write your answers in the corresponding numbered boxes. (20 points)
DẠ

1. They regularly hold elections without a _____ of corruption or violence.


A. scent B. breath C. sniff D. whiff
2. The lyrics came to him in _____ during this and other catnaps.
A. few and far B. length and breadth C. bits and pieces D. leaps and bounds

2 of 14
3. Should the longevity link also apply to human beings, it could well lead to the development
of drugs that mimic the effects of calorie _____ while allowing people to maintain their
normal diet.

L
A. constraint B. restriction C. prevention D. restraint
4. Right after I got married, I got a big promotion at work, so I'm really _____ at the moment!

A
A. on the fence B. on the crest of the wave
C. on cloud nine D. B&C are correct

CI
5. Joe's been walking with a _____ ever since he found out he was getting a promotion.
A. feather in his cap B. spring in his step C. nail on his head D. chip on his shoulder
6. They would much sooner ____ than _____ by car.
A. walk/going B. walking/going C. walking/go D. walk/go

FI
7. Both reporters cut their journalistic _____ on the same provincial newspaper.
A. tooth B. teeth C. foot D. feet
8. (NP) It’s _____ as bad as he said

OF
A. more like B. much C. nothing like D. a great deal
9. The old bridge fell into _____ several years ago.
A. disuse B. misuse C. unuse D. abuse
10. (NP) Lest anyone _____ my story, I have brought documents to attest to its truth.
A. must doubt B. should doubt C. doubts D. doubted

ƠN
11. Let me make a _____ concession at the start.
A. square B. triangle C. round D. cross
12. (NP) There has been _____ increasing number of cases of _____ disease.
A. an/the B. an/Ø (zero article) C. the/a D. the/the
13. He announced he could see the _____ of recovery in the job market.
NH
A. green shoots B. blue chip C. white goods D. golden hello
14. Colin's girlfriend dumped him weeks ago, but the poor guy's heart is still in his _____ .
A. shoes B. heels C. boots D. hoods
15. I chose to pursue a career in medical research so that I might someday _____ in the world with
a groundbreaking discovery.
A. make my grade B. make a move C. make my day D. make my mark
Y

16. There is concern that overfishing could snuff _____ some species.
A. up B. out C. in D. about
QU

17. This is a good exercise for toning ____ the thighs.


A. up B. out C. in D. at
18. (NP) __________ that it now carries nearly two million passenger each day.
A. So popular the system has become B. So popular has the system become
C. Such popular was the system D. Such was popular the system
19. (NP) I’ll see you on Saturday. What ____________ in the afternoon?
M

A. will you do B. will you be doing C. do you do D. are you doing


20. The company has changed __________ several times but is still on the verge of bankruptcy.

A. places B. tune C. subject D. hands


Your answers
1. 2. 3. 4. 5. 6. 7. 8. 9.

11. 12. 13. 14. 15. 16. 17. 18. 19.


Y

Part 2: Give the correct form of each bracketed words. Write your answers in
DẠ

the corresponding numbered boxes. (10 points)


1. He had a(n) _____ smile on his face when he saw me. (ENIGMA)
2. In Greek myth, love is _____ by the goddess Aphrodite. (PERSON)
3. I have only a _____ knowledge of Spanish history. (PATCH)

3 of 14
4. After several setbacks it's hard not to grow _____. (HEART)
5. The book is an _____ summary of issues in pensions, relying on quotes from and references
to others. (ORIGIN)

L
6. Pinga is _____ and begins to cry, to the extent of remaining against a wall. (CONSOLE)
7. The report contained conflicting evidence and plenty of _____. (CONSISTENT)

A
8. _____ by her own fears, she never left the house. (PRISON)
9. The inclusion of _____ explanations and justifications was by no means standard

CI
practice. (SOLICIT)
10. Given the observed difference between men and women as regards to psychological distress, all
_____ were conducted separately for men and women. (ANALYSE)
Your answers

FI
1. 6.

2. 7.

OF
3. 8.

4. 9.

ƠN
5. 10.

SECTION C. READING (60 points)


Part 1: Read the text below and think of the word which best fits each space. Use only
NH
ONE word in each space. Write your answers in the corresponding numbered boxes. (15
points) “Extreme sports” is a broad term to describe any action or adventure sports that include
a high dosage of risk, height, speed, natural challenges, and physical struggle. Most extreme sports
are considered as an alternative (1) _____ mainstream sports with their adrenaline-rushing
thrills. Extreme sports are popularized in (2) _____ 1990s with lots of TV coverage and they keep
on increasing their popularity every year. Bungee Jumping is a simple yet thrilling
Y

recreational activity that involves head-first jumping from a tall structure (3) _____ an elastic cord.
QU

Bungee jump as we (4) _____ it was first practiced in 1979 but its different forms used to be a
tribal coming-of-age ritual in many cultures. Now, bungee jumping is one of the most popular
and available extreme sports in the world. Paragliding is an aerial extreme sport involving
gliding through the air with a parachute that is attached into your body. Participants descend from
(5) _____ altitudes such as a mountain, cliff or an aircraft. Abseiling is a recreational activity
that involves sliding down through a rope in controlled conditions from a mountain, cliff or a man
M

made structure. In spring the weather is more welcoming and warmer for abseiling and since the (6)
_____ is mostly taking place in nature; spring is the most convenient season for it. Kitesurfing is
riding and gliding across the water (7) _____ holding onto a large hand-controlled kite that

is powered by the wind. Mountain biking is an off-road bicycle racing sport that is set on
rough terrain like a mountain, desert, or rocks with specially (8) _____ mountain bikes. Most
mountain bikers like to ride on a dry terrain (9) _____ snow or rain. Skateboarding is an action-
filled recreational activity and a professional sport that involves performing tricks on a
skateboard. Skateboarding has also created its (10) _____ subculture from its slang to music.
Y

(Adapted from https://www.flypgs.com/en/extreme-sports )


Your answers
DẠ

1. 2. 3. 4. 5.

6. 7. 8. 9. 10.

4 of 14
Part 2. Read the following passage and choose the answer (A, B, C or D) which fits
best according to the text. Write your answers in the corresponding numbered boxes.

L
(10 points)
MARCO POLO

A
“Here begins the introduction of this book, which is called “The Description of the World”.
Lords, Emperors, and Kings, Dukes, and Marquesses, Counts, Knights, and Burgessess, and all

CI
people who wish to know the different generations of men and the diversities of the different regions
of the world, then take this book and have it read and here you will fine all the greatest marvels
and the great diversities…”
So begins Marco Polo’s book, “The Description of the World,” as presented in Arthur

FI
Christopher Moule’s masterful English translation of a version of Marco Polo’s book known to
scholars and the “F” text. The storied Venetian trader escaped bandits, pirates, rampaging rivers and
sandstorms on his epic eastbound journey. Sailing the treacherous coasts of Southeast Asia and India,

OF
Marco Polo returned to Venicce in 1295, after 24 years, rich in gems, and wild tales of unimagined
lands. Shortly after his return to Venice, Marco Polo was captured at sea, possibly by pirates.
One tradition suggests he was imprisoned in Genoa’s Palazzo and that he devoted his prison time
to composing his book. On his deathbed in 1324, the legendary adventurer reflected that he had
many more stories to tell.

ƠN
“The Description of the World,” the original product of Marco Polo’s collaboration with a
romance writer named Rustichello has been lost, and so scholars are left to sift through the some
150 versions known to exist, no two exactly alike. Scholars divide the 150 versions into two
groups, labeled “A”, and “B”. The “F” text, which falls into the “A” group, is housed in the
Bibliotheque Nationale in Paris. Considered one of the best and very close to the original, it is written
NH
in a Franco-Italian language described by one scholars as “uncouth French much mingled with
Italian.” Some of these “A” texts are notorious for variations that show the biases, mistakes and
editorial judgments of their copiers. For example, when some translators were presented with the
news that three Magi were buried at Saveh in Persia rather than in Cologne, they inserted that the
people of Saveh tell many lies. As these books were translated from language to language, the
opportunities for error multiplied; one text from the early 16th century is a Tuscan translation of a
Y

Latin translation of an earlier Tuscan translation of the original Franco-Italian language. Although
we have no confirmation of the Marco-Rustichello collaboration other than the book itself,
QU

Marco seems to have approved of at least some of its versions, for in 1307 he presented a
French translation of it to an envoy of Charles of Valois.
The second group of manuscripts, known as the “B” group, provides some provocative material not
found in the “A” texts. From this “B” group, for example, we learn that the people around Yarkand
in western China suffer from goiter – a problem for them even today. Until the 1930s the only
examples of “B” texts were a few odd bits of manuscript and a printed text by Giambattista Ramusio
M

that appeared in 1559, two years after his death. Ramusio tells his readers that his Italian version was
produced with the help of different copies.” [A] The foundation of his work appears to be a Latin

text dating from before 1320, with influences from other identifiable versions. [B] What is distinctive
about Ramusio’s work is that about twenty percent of it was, until 1932, considered unique. [C] That
twenty percent is thought to have destroyed in a 1558 fire. [D]
A second version containing much of Ramusio’s original material surfaced in Toledo, Spain in 1932.
Most of this Latin manuscript agrees with the “F” manuscript, but it also contains some 200 passages
not found in “F”. About 120 of those, however, are found in Ramusio’s book. Because the remaining
Y

80 offer valuable historical and geographical material and even help to clarify some obscure passages
of “F”, this manuscript is thought to be a copy of something that was very close to an original.
DẠ

In sorting this out, scholars have come to conclude that Marco Polo probably wrote two versions of
his book. The second version, presented by the “B” texts, may have been a revision and expansion
done for a select group of readers who had already made their way through the first book. It is

5 of 14
unlikely that we will ever know exactly what form the first book took, but the versions we have still
make for a very good read.
(Adapted from CPE express book 2)

L
1. According to the introduction to the book, readers can expect to
A. learn about Marco Polo’s life. B. learn about differences among generations.

A
C. travel to far regions of the world. D. read descriptions of places.
2. According to the second paragraphs, stories about Marco Polo’s life

CI
A. are well-supported B. are all imaginary
C. take place at sea D. are sometimes unreliable
3. What is the “F” text?
A. The authentic text written by Marco Polo and Rustichello.

FI
B. The script with the greatest affinity to the original source.
C. Not one of the 150 versions of Marco’s original book.
D. A good version of the “B” texts written in Franco-Italian.

OF
4. What is one of the main problems with the “A” texts?
A. All translators manipulated the truth.
B. Editing is now difficult and unreliable.
C. The early versions were remote from the original text.
D. Later translations distorted the original.

ƠN
5. The “B” group of manuscripts
A. contained previously undocumented information.
B. were compilations of manuscripts printed by Ramusio.
C. dealt with health and culture in China.
D. were published two years after Ramusio’s death.
NH
6. What was found in Spain in 1932?
A. A Latin version containing valuable information about Ramusio
B. A text which was very close to the “F” manuscript.
C. A manuscript of 200 passages that do not appear in the “F” text.
D. The original book written by Marco Polo.
7. In relation to the book, “The Description of the World”, the author suggests that
Y

A. despite its uncertain origins, it is a fascinating piece of literature.


B. scholars should discover who the true author was.
QU

C. Marco Polo wrote many versions of the same book.


D. Marco Polo intended his original book for an elite readership.
8. Which of the following statements is TRUE
A. Marco Polo was captured at sea by pirates for two years.
B. There are about 150 versions of the book “The Description of the World”.
C. “B” texts have more biases, mistakes and editorial judgments than “B” texts.
M

D. Texts by Ramusio were destroyed in a fire.


9. The word “envoy” in paragraph 4 refers to

A. a person B. a dynasty C. a department D. a country


10. Which of the following square brackets [A], [B], [C], or [D] best indicates where in
the paragraph the following sentence can be inserted?
In any event, the source has never been found.
A. [A] B. [B] C. [C] D. [D]
Your answers
Y

1. 2. 3. 4. 5.
DẠ

6. 7. 8. 9. 10.

6 of 14
Part 3. Read the passage and do the tasks that follow. Write your answers in
the corresponding numbered boxes. (13points)
List of headings

L
i. How the study of body language has changed
ii. A possible business application of body language

A
iii. Using body language as a tool to deceive others
iv. Communicating a wide range of messages

CI
v. A branch of an older academic field
vi. The need for skill when interpreting body language
vii. Recognising a positive attitude without realising it
viii. How power is linked with certain family roles

FI
ix. A form of body language that can be misinterpreted
x. Imitating the chief person in a group
xi. Ignoring signals from other people

OF
1. Paragraph A: _____
2. Paragraph B: _____
3. Paragraph C: _____
4. Paragraph D: _____

ƠN
5. Paragraph E: _____
6. Paragraph F: _____
7. Paragraph G: _____
8. Paragraph H: _____
NH
KINESICS
A. Psychology is a well-established subject, but one area of it, “kinesics”- the study of body language
– was not identified until 1952, and research only began in earnest in the 1960s. Kinesics is based on
the behavioral patterns of non-verbal communication. Clinical studies have revealed the extent to
which body language can actually contradict verbal communications. A classic example is the young
Y

woman who told her psychiatrist that she loved her boyfriend very much while shaking her head
from side to side in subconscious denial.
QU

B. Body language also sheds light on the dynamics of interfamily relationships. A family
sitting together can give a revealing picture of itself simply by the way its members move their arms
and legs. If the mother, for example, crosses her legs first and the rest of the family then follows suit,
she has set the lead for the family action, though she, as well as the rest of the family may not be aware
she is doing it. In fact, her words may deny her leadership as she asks her husband or children for
advice. But the unspoken, follow-the-leader clue in the actions of the family members gives the
M

family set-up away to someone knowledgeable in kinesics.


C. Another kinesic signal is the unconscious widening of a person’s pupils when their eyes

see something pleasant. Experiments have shown that we become aware of how that person
feels, although we are conscious neither of seeing the signal nor of giving it meaning.
D. This kinesic principle has been used on a commercial level to detect the effect of a
television advertisement. While the ad is being shown to a selected audience, their eyes are
photographed. Later, the film is carefully studied to detect just when there is any widening of the
eye; in other words, when there is any unconscious, positive response to the advertisement.
Y

E. Body language can include any volutionary or involutionary movement of a part or all the
body, used by a person to communicate an emotional message to the outside world. To understand
DẠ

this unspoken body language, kinesics experts often have to take into consideration cultural
and environmental differences. The average person, unschooled in cultural nuances of body
language, is often mistaken when decoding what he or she sees.

7 of 14
F. In addition to sending and receiving messages, body language can also serve to break
through defences, if it is used skillfully. Often the swiftest and most obvious type of body language
is touch. The touch of a hand, or an arm around someone’s shoulder, can spell a more vivid and

L
direct message of friendliness than dozens of words. But such a touch must come at the right
moment and in the right context, or the other person may take it as an intrusion into his or her

A
personal space. For every situation there must be two elements to body language: the delivery of
the message and the reception of the message.

CI
G. However, some people are “touchers”, compulsive touchers, who seem completely impervious to
all messages they may get from friends and acquaintances. They are people who will touch others
even though they are bombarded with body-language requests not to. There are also people who
avoid touching others altogether.

FI
H. We act out our state of being with non-verbal body language. We lift one eyebrow for
disbelief, shrug our shoulders for indifference, tap our fingers for impatience. The gestures are
numerous, and while some are deliberate and others are almost deliberate, there are some, such as

OF
rubbing under our noses for puzzlement or clasping our arms to protect ourselves, that are
mostly unconscious. Kinesics is a study of the mixture of all body movements, from the very
deliberate to the completely unconscious, from those that apply only in one culture to those that cut
across all cultural barriers.

ƠN
Questions 9-13. Do the following statements reflect the claims of the writer in the
reading passage? Write
YES if the statement reflects the claims of the writer
NO if the statement contradicts the claims of the writer
NOT GIVEN if it is impossible to say what the writer thinks about this
NH
9. Little work was done in the field of kinesics for the first few years after the subject had
been identified.
10. Family leaders consistently show their dominance through speech and body language.
11. The use of kinesics in connection with television advertising has increased sales of products.
12. Touching may be regarded as an unwelcome gesture.
Y

Question 13. Choose the correct letter A, B, C or D


QU

The writer’s intention is


A. to present recent findings in kinesics to specialists.
B. to introduce kinesics to general readers.
C. to examine weaknesses in kinesics.
D. to identify the scientific basis of kinesics.
(Adapted from Cambridge Objective IELTS advanced)
M

Your answers:
1. 2. 3. 4. 5.

6. 7. 8. 9. 10.

11. 12. 13.


Y

Part 4. You are going to read an extract from an article. Seven paragraphs have
DẠ

been removed from the extract. Choose from the paragraphs A - H the one which fits each
gap (1- 7). There is one extra paragraph which you do not need to use. Write your answers in
the corresponding numbered boxes. (7 points)

8 of 14
OpenAI last week opened up access to ChatGPT, an AI-powered chatbot that interacts with users in
an eerily convincing and conversational way. Its ability to provide lengthy, thoughtful and thorough
responses to questions and prompts – even if inaccurate – has stunned users, including academics

L
and some in the tech industry.
1.

A
CI
“There’s a certain feeling that happens when a new technology adjusts your thinking
about computing. Google did it. Firefox did it. AWS did it. iPhone did it. OpenAI is doing it
with ChatGPT,” Levie said on Twitter. But as with other AI-powered tools, it also poses

FI
possible concerns, including for how it could disrupt creative industries, perpetuate biases and
spread misinformation.
2.

OF
After signing up for ChatGPT, users can ask the AI system to field a range of questions, such as “Who
was the president of the United States in 1955,” or summarize difficult concepts into something a

ƠN
second grader could understand. It’ll even tackle open-ended questions, such as “What’s the meaning
of life?” or “What should I wear if it’s 40 degrees out today?”
3.
NH
But some users are getting very creative. One person asked the chatbot to rewrite the 90s hit
song, “Baby Got Back,” in the Style of “The Canterbury Tales;” another wrote a letter to remove a
bad account from a credit report (rather than using a credit repair lawyer). Other colorful
examples including asking for fairy-tale inspired home décor tips and giving it an AP English exam
question (it responded with a 5 paragraph essay about Wuthering Heights.)
Y

4.
QU

While ChatGPT successfully fielded a variety of questions submitted by CNN, some responses were
noticeably off. In fact, Stack Overflow – a Q&A platform for coders and programmers – temporarily
banned users from sharing information from ChatGPT, noting that it’s “substantially harmful to the
site and to users who are asking or looking for correct answers.”
M

5.

“While we’ve made efforts to make the model refuse inappropriate requests, it will
sometimes respond to harmful instructions or exhibit biased behavior,” Open AI said on its website.
“We’re using the Moderation API to warn or block certain types of unsafe content, but we expect it
to have some false negatives and positives for now. We’re eager to collect user feedback to aid
Y

our ongoing work to improve this system.”


6.
DẠ

9 of 14
“It is very easy for the model to give plausible-sounding but incorrect or nonsensical answers,”
he said. “It guessed when it was supposed to clarify and sometimes responded to harmful
instructions or exhibited biased behavior. It also lacks regional and country-specific understanding.”

L
7.

A
CI
While the DALL-E tool is free, it does put a limit on the number of prompts a user can do
before having to pay. When Elon Musk, a co-founder of OpenAI, recently asked Altman on Twitter
about the average cost per ChatGPT chat, Altman said: “We will have to monetize it somehow at
some point; the compute costs are eye-watering.”

FI
(Adapted from https://edition.cnn.com/2022/12/05/tech/chatgpt-trnd/index.html)
A. “It depends on what activities you plan to do. If you plan to be outside, you should wear a
light jacket or sweater, long pants, and closed-toe shoes,” ChatGPT responded. “If you plan to be

OF
inside, you can wear a t-shirt and jeans or other comfortable clothing.”
B. The tool quickly went viral. On Monday, Open AI’s co-founder Sam Altman, a prominent Silicon
Valley investor, said on Twitter that ChatGPT crossed one million users. It also captured the attention
of some prominent tech leaders, such as Box CEO Aaron Levie.
C. Still, Lian Jye Su, a research director at market research firm ABI Research, warns the chatbot is

ƠN
operating “without a contextual understanding of the language.”
D. In a blog post last week, OpenAI said the “format makes it possible for the tool to answer follow-
up questions, admit its mistakes, challenge incorrect premises, and reject inappropriate requests.”
As of Monday morning, the page to try ChatGPT was down, citing “exceptionally high demand.”
“Please hang tight as we work on scaling our systems,” the message said. (It now appears to be
NH
back online).
E. At the same time, however, it does provide a glimpse into how companies may be able to capitalize
on developing more robust virtual assistance, as well as patient and customer care solutions.
F. Beyond the issue of spreading incorrect information, the tool could also threaten some
written professions, be used to explain problematic concepts, and as with all AI tools, perpetuate
biases based on the pool of data on which it’s trained. Typing a prompt involving a CEO, for
Y

example, could prompt a response assuming that the individual is white and male, for example.
G. Like ChatGPT, the new Google Search and Bard are built on a large language model. They
QU

are trained on vast troves of data online in order to generate compelling responses to user
prompts, but these tools are also known to get responses wrong or “hallucinate” answers.
H. ChatGPT is a large language model trained on a massive trove of information online to
create its responses. It comes from the same company behind DALL-E, which generates a
seemingly limitless range of images in response to prompts from users. It’s also the next iteration
of text generator GPT-3.
M

Your answers
1. 2. 3. 4. 5. 6. 7.

Part 5. For questions 1-10, select the travel writer (A-E) using the separate answer
sheet. Each travel writer may be selected more than once. (15 points)
GENETIC ENGINEERING- THE WAY OF THE FUTURE?
Y

To examine the issue, we’ve asked the opinions of six experts


A. Dr Robert Rodriguez – bioethics lecturer
DẠ

Is it so surprising that there is widespread public suspicion and mistrust? Incidentally, the
public’s negative view of GM cannot be attributed to ignorance because mistrust tends to increase
with education on the topic. This is despite an ever-growing body of research that can find no
evidence of harm. Of course, this doesn’t mean there isn’t any, even if we assume the best intentions

10 of 14
of the people involved; it’s a subject that is not completely understood. Anyway why should the
public assume that best intentions are behind the research? Look at the past. Look what happened
with BSE, better known as mad cow disease. Agricultural practices did not protect the public,

L
it endangered them. Look back further to the pesticide DDT. We do not have a track record
that encourages public confidence.

A
B. Dr Lisa Khan – geographer
There is no doubt that people are starving today in many parts of the world. And with

CI
global population growth projections – we seem set to add a billion people every twelve to fifteen
years – there is absolutely no debate that we will be unable to feed the population in the future
unless things change fundamentally. Genetically modifying crops is certainly one way to achieve
this change. But it is not the only way, and it may not be the most effective. Take, for comparison,

FI
the “green revolution” of the 1950s, which greatly increased productivity by using new strains
of crops, new mechanical tools and petrochemical pesticides and fertilizers; it has created its own
set of problems. People may starve because of lack of food, but the food is there; other things –

OF
social or economic issues – stand in the way of it getting where it needs to be.
C. Dr Sylvia Johnson – doctor
I would like to point out that GM organisms have made very important contributions to medicine. I’m
not talking about GM foods; that is a separate issue. But if we consider insulin being produced in
tobacco plants, there is a tremendous benefit. A difficult to obtain substance is made available safely

ƠN
at a lower cost. The plants are cultivated in a greenhouse, under controlled circumstances, the product
they synthesise is purified in a stringent process, and the modified plants are destroyed; there is very
little risk involved. The potential for creating a wide range of difficult- to- produce and life-saving
proteins and pharmaceuticals, even vaccines, at costs low enough for third world utilization should
not be ignored.
NH
D. Dr Gary Wilson – population geneticist
These days there is little question about gene flow. If you plant GM crops, the genes will end up in
non-GM crops, in wild weed populations, in soil bacteria; they cannot be contained. They will move;
they have moved. There are studies that prove it. In the early research, they were citing probabilities
of pollination events or gene transfer events that were miniscule; these studies were used to support
the cause of GM. But if you have enough chances, if you cultivate enough acres, the improbable will
Y

happen. And it has. And it will continue to. For a population geneticist there is no surprise here. At
the risk of being incendiary, I will say the conclusions drawn from this early research could be
QU

considered an example of willful misinterpretation, of statistical probability. Well, it has become a


profitable industry.
E. Dr. Daphne Alexander – ecologist
I would like to mention the monarch butterfly. This insect carries out a unique migration from
the Northern US and Canada all the way to Mexico, and is reliant on a range of environmental
and temporal patterns throughout this whole geographic area. It came out a while ago that one
M

strain of GM corn that was engineered to contain a toxin, originally from bacteria, to kill any insect
that attempted to eat its leaves, also expressed this toxin in its pollen. It was not supposed to do

this, other strains did not. And this pollen was falling on the leaves of the milkweed plants, on
which the monarch larvae are dependent for a food source, killing or stunting the larvae. This is
an example of the kind of unintended consequence that is impossible to foresee.
F. Dr. Andrew Wright – lawyer
In the EU, by law, food products that contain GM ingredients must be labeled. But it is not quite as
simple as that. In fact, it is not simple at all; it is unbelievably convoluted. Tomato sauce made from
Y

GM tomatoes is simple; but it must be labeled. But what about meat, milk, cheese, or eggs produced
from animals fed GM corn or soya in their feed? The feed must be labeled, but not the final product.
DẠ

And what about enzymes, like those used to make cheese for example, that have been produced by
GM microorganisms? Furthermore, there can legally be up to 9 percent contamination with GM
products, with no labeling required, as long as the producer can prove it was accidental and

11 of 14
unavoidable! I would urge everyone to read widely; there is no other way to keep informed and a lot
has already transpired, while most of us were unaware.
Which person gives each of the following opinions about genetic engineering?

L
1. _________ GM genes are already present in wild populations.
2. _________ Some people may have misled others when they did not interpret data correctly.

A
3. _________ Current research might not be correct.
4. _________ People haven’t been paying attention to developments.

CI
5. _________ Even unlikely events happen.
6. _________ It is impossible to account for all the possible effects in advance.
7. _________ People have no reason to believe what the authorities say.
8. _________ Unintentional presence of GM items in food is not regulated.

FI
9. _________ There is more than one way to solve a problem.
10. _________ In some cases the benefits clearly outweigh the risks.

OF
Your answers
1. 2. 3. 4. 5. 6. 7. 8. 9.

ƠN
SECTION D. WRITING (60 points)
Part 1. Read the following text and use your own words to summarize it. Your
summary should be about 100-120 words long. You MUST NOT copy the original. (15 points)
Man is forever changing the face of nature. He has been doing so since he first appeared on the
NH
earth. Yet, all that man has done is not always to the ultimate advantage of the earth or
himself. Man has, in fact, destroyed more than necessary.
In his struggle to live and extract the most out of life, man has destroyed many species of wildlife;
directly by sheer physical destruction, and indirectly by the destruction or alteration of habitats. Some
species may be able to withstand disruptions to their habitat while others may not be able to cope.
Take the simple act of farming. When a farmer tills a rough ground, he makes it unsuitable for the
Y

survival of certain species. Every change in land use brings about a change in the types of plant and
animals found on that land.When man builds a new town, this means the total destruction of vast
QU

areas of farmland or woodland. Here, you have the complete destruction of entire habitats and it is
inevitable.
It follows therefore, that every form of human activity unavoidably upsets or changes the wildlife
complex of the area. Man has destroyed many forms of wildlife for no reasonable purpose. They
have also made many great blunders in land use, habitat destruction and the extermination of many
forms of wildlife
M

Man's attitude towards animals depends on the degree to which his own survival is affected. He sets
aside protection for animals that he hunts for sport and wages a war on any other creature that may
pose a danger or inconvenience to him. This creates many problems and man has made irreversible,

serious errors in his destruction of predators. He has destroyed animals and birds which are useful to
farmers as pest controllers. The tragedy that emerges is that all the killing of predators did not in any
way increase the number of game birds.
……………………………………………………………………………………………………
……………………………………………………………………………………………………
Y

……………………………………………………………………………………………………
……………………………………………………………………………………………………
DẠ

……………………………………………………………………………………………………
……………………………………………………………………………………………………
……………………………………………………………………………………………………
……………………………………………………………………………………………………

12 of 14
……………………………………………………………………………………………………
……………………………………………………………………………………………………
……………………………………………………………………………………………………

L
……………………………………………………………………………………………………
………………………………………………………………

A
……………………………………………………………………………………………………
……………………………………………………………………………………………………

CI
……………………………………………………………………………………………………
……………………………………………………………………………………………………
…………………………………………………………………………………………………………
…………………………

FI
Part 2. Report writing (15points)
The charts below show the numbers of people in different age groups who go sailing or play
basketball in Australia.

OF
Summarise the information by selecting and reporting the main features, and
make comparisons where relevant.
Write at least 150 words.
13

ƠN
NH
Y
QU
M

……………………………………………………………………………………………………
……………………………………………………………………………………………………
Y

……………………………………………………………………………………………………
……………………………………………………………………………………………………
DẠ

…………………………………………………………………………………………………………
……………………………………………………………………………………………………
……………………………………………………………………………………………………
……………………………………………………………………………………………………

13 of 14
……………………………………………………………………………………………………
……………………………………………………………………………………………………
……………………………………… ……………………………………… ……………………….

L
……………………………………………………………………………………………………
……………………………………………………………………………………………………

A
……………………………………………………………………………………………………
……………………………………………………………………………………………………

CI
……………………………………………………………………………………………………
……………………………………… ………………………………………………………………
Part 3. Write an essay (30points)
It is said that the characteristics we are born with have much more influence on our personality

FI
and development than any experiences we may have in our life. To what extent do you agree or
disagree with that idea?
Present your argumentation to highlight your opinion on this matter. Give reasons and

OF
specific examples to support your opinion(s).
Write an essay (about 350 words) to express your opinion.
……………………………………………………………………………………………………
……………………………………………………………………………………………………
……………………………………………………………………………………………………

ƠN
……………………………………………………………………………………………………
……………………………………………………………………………………………………
……………………………………………………………………………………………………
……………………………………………………………………………………………………
……………………………………………………………………………………………………
NH
……………………………………………………………………………………………………
……………………………………………………………………………………………………
……………………………………………………………………………………………………
……………………………………………………………………………………………………
……………………………………………………………………………………………………
……………………………………………………………………………………………………
Y

……………………………………………………………………………………………………
……………………………………………………………………………………………………
QU

……………………………………………………………………………………………………
……………………………………………………………………………………………………
……………………………………………………………………………………………………
……………………………………………………………………………………………………
……………………………………………………………………………………………………
……………………………………………………………………………………………………
M

……………………………………………………………………………………………………
……………………………………………………………………………………………………

……………………………………………………………………………………………………
……………………………………………………………………………………………………
……………………………………………………………………………………………………
……………………………………………………………………………………………………
……………………………………………………………………………………………………
……………………………………………………………………………………………………
Y

……………………………………………………………………………………………………
THE END
DẠ

Người ra đề:
Lại Thanh Tình (Lexico-grammar, Reading, Writing)
Đỗ Hồng Ngọc Diệp (Listening)

14 of 14
HỘI CÁC TRƯỜNG THPT CHUYÊN ĐỀ THI CHỌN HỌC SINH GIỎI LẦN THỨ
KHU VỰC DUYÊN HẢI, ĐỒNG BẰNG BẮC BỘ XIII
TRƯỜNG THPT CHUYÊN BIÊN HOÀ, T. HÀ NAM MÔN THI: TIẾNG ANH – KHỐI 11

L
Ngày thi 15-16/07/2023
Thời gian làm bài 180 phút

A
(Đề thi gồm … trang)

CI
FI
OF
SECTION A. LISTENING (50 points)
Part 1: CPE
Your answers: https://www.englishaula.com/en/cambridge-english-test-exam-preparation/cambridge-
english-proficiency-c2-proficiency/listening/exam-parts/exercise-practice-test/0-3-5-
5317875497697280/

ƠN
1. B 2. A 3. C 4. A 5. A

Part 2: authentic
Your answers: https://www.youtube.com/watch?v=pI19xAquCdM
NH
1. F 2. T 3. T 4. T 5. F

Part 3: authentic
Your answers: https://www.youtube.com/watch?v=iMtXqTmfta0&t=5s
1. sonic boom 2. biomimicry 3. pantograph 4. drag-reducing 5. lotus effect
Y

pattern
QU

6. 15% less 7. Novices 8. same 9. prairie dog 10. circular


electricity serrations burrows economy
M

Part 4: IELTS short answer (Simulation Test 8, part 4)


1. (In) Rhyme / song
3. French Revolutionary Army

3. Scum of Earth
4. After military fiasco
5. Commander in Chief
SECTION B. GRAMMAR & VOCABULARY (30 points)
Y

Part 1: (5 câu ngữ pháp, 15 câu từ vựng nguồn CPE)


DẠ

1. D 2. C 3. B 4. D 5. B 6. D 7. B 8. C 9. A 10. B

11. A 12. A 13. A 14. C 15. D 16. B 17. A 18. B 19. B 20. D
Part 2: 10 câu rời, nguồn CPE

L
1. enigmatic 6. inconsolable

A
2. personified 7. inconsistencies

CI
3. patchy 8. Imprisoned

4. disheartened 9. unsolicited

FI
5. unoriginal 10. analyses

OF
SECTION C. READING (60 points)
Part 1: (https://www.flypgs.com/en/extreme-sports)
1. to 2. the 3. with 4. know 5. high

ƠN
6. sport 7. while/when 8. designed 9. without 10. own

Part 2: (CPE Express book 2, test 2)


NH
1. D 2. D 3. B 4. D 5. A 6. B 7. A 8. B 9. A 10. D

Part 3: (Cambridge Objective IELTS advanced)


1. v 2.x 3.vii 4.ii 5.vi
Y

6.ix 7.xi 8.iv 9.YES 10.NO


QU

11.NG 12.YES 13.B

Part 4: (https://edition.cnn.com/2022/12/05/tech/chatgpt-trnd/index.html)
1. B 2. H 3. A 4. D 5. F 6. C 7. E
M

Part 5: (CPE express Book 3)


1. D 2. D 3. A 4. F 5. D 6. E 7. A 8. F 9. B 10. C
Y

SECTION D. WRITING (60 points)


DẠ

Part 1:
Key points:
- Man has done more harm than good to the Earth.
- Man has destroyed some wild species both directly and indirectly
- Some species survive, but some have gone extinct due to human activities
- The only protection of animals done by man is only useful to man’s survival, man kills animals that
cause harm and inconvenience to him. -> creating the imbalance in the ecology.
Part 2:
Striking features/trends:

L
- sailing attracts more people from older age groups than younger ones
- basketball attracts more pepple from younger age groups than older ones

A
- more men take part in these two sports than women.
- the gap between the number of men and women taking part in basketball is negligible in the age

CI
groups of 25-34 and 35-44

Sample:

FI
OF
ƠN
NH
Y
QU

\
M

Y
DẠ
ĐỀ THI ĐỀ XUẤT TRẠI HÈ DUYÊN HẢI – ĐBBB NĂM 2023
TRƯỜNG THPT CHUYÊN SƠN LA – TỈNH SƠN LA
MÔN: TIẾNG ANH 10
(Đề thi có 14 trang)

A L
SECTION I: LISTENING (50pts)
Part 1: Questions 1-5. Complete the notes below. Write ONE WORD AND/ OR A NUMBER for each

CI
answer. (10pts)
Customer Satisfaction Survey

FI
Customer details
Name: Sophie Bird

OF
Occupation: 1………………………………..
Journey information
Name of station returning to: Staunfirth
Type of ticket purchased: standard 2…………………………….ticket

ƠN
Where ticket was bought: online
Satisfaction with the journey
Most satisfied with: the Wi-Fi
NH
Least satisfied with: the 3……………………………….this morning
Satisfaction with station facilities
Most satisfied with: how much 4…………………………………….was provided
Least satisfied with: lack of seats, particularly on the platforms
Y

Neither satisfied nor dissatisfied with: the 5……………………………..available


QU

(Cambridge IELTS ACADEMIC 15)


Your Answers:
1. 2. 3. 4. 5.
M

Part 2. For questions 6-10, listen to a talk about fear and decide whether these statements are True (T)
or False (F). Write your answers in the corresponding numbered boxes provided. (10 points)

6. Fear and worry can be easily overcome once you understand awareness in the mind.
7. Worry and fear exist only in the future, not in the past.
8. The person in the story missed their favorite radio show because they got stuck in the snow.
Y

9. Developing concentration and willpower can help in keeping awareness on what you're doing and
prevent it from going into the future in an uncontrolled way.
DẠ

10. It is beneficial to let awareness go into the future to think about things in a negative way.
https://www.youtube.com/watch?v=9WicYGH5X4M

Page 1 of 14
Your answers:
6. 7. 8. 9. 10.

L
Part 3: You will hear part of an interview in which a career coach called Charles Norris is discussing
job-hunting. For questions 11-15, choose the answer (A, B, or C) which fits best according to what you

A
hear. (10pts)

CI
11. What is NOT included in the role of a career coach?
A. Recognizing a client’s job-hunting weaknesses.

FI
B. Advising clients how to update their CVs
C. Locating and securing employment for clients

OF
D. Assisting clients in improving their interview techniques
12. According to Charles, the covering letter is important because______.
A. It is an integral part of the CV.
B. It contains all the details of the applicant.

ƠN
C. It is the same for every position
D. It is the only part of the CV employers read.
13. When discussing the internet, Charles suggests that job seekers should______.
NH
A. socialize on the internet to meet employers.
B. Clean up data that exists about them on the internet.
C. Search for other job-seekers online.
D. Apply to employers that use networking sites.
Y

14. Charles believes one of his clients succeeded in his job search because he______.
QU

A. uploaded his CV to his profile. B. had a detailed profile.


C. included keywords in his profile. D. Had a catchy headline on his profile.
15. What does Charles say about asking questions at an interview?
A. That you should only do if you have done your research.
M

B. That you should never ask about a company’s competitor.


C. That you should have a list prepared before you go.

D. That you should search for questions online.


(On Screen workbook and Grammar book C1)
Your answers:
11. 12. 13. 14. 15.
Y
DẠ

Part 4. For questions 16-25, listen to Listen to a recording and complete the sentences with NO MORE
THAN THREE WORDS AND/OR A NUMBER taken from the recording for each answer in the space
provided.
- Scientists are searching the hidden corners of the world for viruses that are poised to become

Page 2 of 14
(16) __________________________.
- In many cases, they’re searching for Disease. That’s not the name of an actual illness. The next big worldwide
disease could be caused by what we call (17) ________________________________.

L
- (18) ___________________ is the main tool they’re using to do that. This is actually something I’m using
in my own research on bacterial communities, so let’s break it down together!

A
- In (19) ______________________ we take a sample—that can be soil, ocean water, a bodily fluid—and we

CI
purify it down to the genetic material of just the stuff we’re looking for. In this case, let’s say it’s viruses. So
now we have the genomes of all the viruses in our sample.

FI
- All of these genomes have to be read by our (20) ______________________________. And, if you can
picture, it’s much faster for lots of people to each read a sentence. And they can each read their sentence at

OF
the same time a sentence than it is for one person to read a whole page, one at a time.
- A large number of (21) _______________________ are quickly identified by metagenomic sequencing, an
incredibly useful tool.
- Scientists have been surprised at how many they’re finding, everywhere, as they have turned that powerful

ƠN
lens on our world to (22) _______________________________.
- In the ocean, in our wastewater, inside the spiders in our gardens and of course, inside us. They’re preying
on other marine microorganisms, releasing nutrients. This is probably the (23)
NH
______________________________ of the ocean’s food web.
- It’s likely that the balance of other organisms would rapidly get out of whack if viruses suddenly disappeared
from the planet.
- Maybe, a piece of (24) _______________________________ that jumped over to its mammalian host about
Y

130 million years ago enabled modern humans to give birth. That gave us our ability to grow a placenta. So,
QU

yep. We got hacked, and it’s probably the reason we don’t lay eggs.
- Scientists don’t know how many more viruses are out there, waiting to be discovered. Sequencing at least
some of those nonillion viruses out there is the first step of finding how many of them may be harmful to us.
Identifying what they’re living inside...and how exactly these tiny, invisible puppeteers are (25)
M

____________________________ of our natural world is the next step.


https://youtu.be/uRABqa8wPQw

Your answers:
16. 17.
18. 19.
20. 21.
Y

22. 23.
DẠ

24. 25.

Page 3 of 14
SECTION II: LEXICO-GRAMMAR (40pts)
Part 1: Choose one word or phrase which best completes each sentence. Circle its corresponding letter
A, B, C, or D to indicate your answer. (20pts)

L
1. The teachers are not very qualified, but in all ______ to Principal Montara, the school hasn't received
funding to hire better teachers.

A
A. fairness B. tenderness C. fondness D. hardiness

CI
2. Words with strongly positive ______often persuade readers to see your ideas in a favorable way.
A. calculations B. connotations C. considerations D. compilations

FI
3. She was kept in ______ of his true identity.
A. acceptance B. intolerance C. annoyance D. ignorance

OF
4. My brother says that snails have teeth but I don't know if he's ______ or if he's serious.
A. giving me the cold shoulder B. twisting my arm
C. pulling my leg D. raising his eyebrows
5. It was a very ______ article that truly made me stop and think.

ƠN
A. long-lasting B. absent-minded C. thought-provoking D. strong-willed
6. He was greatly ______ by the tragic news of the assassination of the Archduke and his wife.
A. baffled B. charmed C. distressed D. intrigued
NH
7. Be ______ enough to learn from your mistakes.
A. spurious B. wary C. humble D. eloquent
8. She gave him a look that made words ______.
A. tedious B. self-effacting C. eclectic D. superfluous
Y

9. Moving in together was a mistake, and things rapidly went ______.


QU

A. down bad B. off C. a long way D. downhill


10. "______" is an American English metaphorical idiom for an obvious problem or risk that no one wants
to discuss.
A. Lone wolf B. Crocodile tears
M

C. Elephant in the room D. Butterflies in the stomach


11. I don’t suppose you are telling us the true version of the story, ______?

A. do I B. are you C. aren’t you D. will you


12. Not having written about the required topic, ________ a low mark.
A. my presentation was given B. the teacher gives me
C. the teacher gave me D. I was given
Y

13. Thomas received a warning for speeding. He _________ so fast.


DẠ

A. shouldn’t have driven C. must have driven


B. would have driven D. might have driven
14.________, they resumed their journey.
A. Rising the sun B. The sun having risen

Page 4 of 14
C. The sun being risen D. Having risen the sun
15. The doctor insisted that his patient ________.
A. did not work too hard for three months B. take it easy for three months

L
C. take it easy inside of three months D. could take a vacation for three months
16. She walked indolently along, with a mind at rest, its peace ______ in her innocent face.

A
A. reflected B. reflecting C. being reflected D. having reflected

CI
17. ________ technically proficient; it also explores psychological questions.
A. Not only is Barbara Astman's artwork B. Not only Barbara Astman's artwork

FI
C. Barbara Astman's artwork not only D. Barbara Astman’s artwork not only
18_________ talking of running for election again, after such a crushing defeat, is surely proof of his

OF
resilience.
A. Should he be B. That he is C. Had he been D. That he were
19. We took the shortcut round the market ______ late for class this morning.
A. so that we will not be B. lest we be not

ƠN
C. for fear that we should be D. in order that we not be
20. “The President says the company is making more cars this year than last year.” “_______attribute the
increase in productivity?”
NH
A. Does he B. What to C. To what does he D. What does he
Your answers:
1. 2. 3. 4. 5. 6. 7. 8. 9. 10.
11. 12. 13. 14. 15. 16. 17. 18. 19. 20.
Y
QU

Part 2: Use the word given in capitals at the end of each line to form a word that fits in the gap in the
same line.
1. Though her novels do not feature the themes of Romanticism, Jane Austen’s work
was ______ with that of Wordsworth and Byron. TEMPORAL
M

2. A______angel is believed to be a good spirit that protects each of us. GUARD


3. She spoke with______ of his impudent behaviour. APPROVE

4. The Princess looked ______ beautiful at the wedding. STUN


5. They tested the______ of water in the village wells. TRANSPARENT
6. I counted on your ______ and you let me down. DISCREET
7. The principal took______ measures against the culprits. DISCIPLINE
Y

8. During the war, a lot of______ fled to America. REFUGE


DẠ

9. After the death of his uncle, he became the______ owner of the castle. LAW
10. She has had several______ of her illness lately. RECUR

Page 5 of 14
Your answers:
1. 6.
2. 7.

L
3 8.

A
4. 9.
5. 10.

CI
Part 3. Complete each of the following sentences with suitable preposition(s). Write your answer in the

FI
boxes provided on the answer sheet. (10 points)
1. He left the country …………………………threat of arrest if he returned.

OF
2. Come sit ____ the fire and I’ll get you a hot cup of coffee.
3. Many of the passengers looked shaken ____ when the pilot announced that the plane was having
mechanical problems.
4. Tommy was bummed ____ about his favorite TV show getting cancelled.

ƠN
5. I was ____ the zone at my basketball game last night. I couldn’t miss.
6. Some of the neighbors’ kids threw eggs ____ our house on Mischief Night.
7. Before I learned how to do my job, I felt like I was just getting ____ the way of my coworkers.
NH
8. Can I substitute something else ____ the eggs in this recipe? I’m a vegan.
9. Before beginning her research career, my aunt went to college and graduated ____ a degree in
biology.
10. Are organic fruits and vegetables really superior ____ conventional produce?
Y

Your answers:
QU

1. 2. 3. 4. 5.
6. 7. 8. 9. 10.

SECTION C: READING (60 points)


M

Part 1. Read the following passage and decide which option (A, B, C, or D) best fits each gap. Write your
answers in corresponding numbered boxes on the answer sheet. (15 points)

The concept of artificial intelligence (AI) has (1) ________ the boundaries of what was once thought possible.
With its ability to process vast amounts of data and perform complex tasks, AI has (2) ________ potential in
various fields.
Y

One area where AI has made significant strides is in the field of healthcare. It has the capacity to (3) ________
patterns and anomalies in medical images, aiding in the early detection and diagnosis of diseases. Moreover,
DẠ

AI-powered virtual assistants can (4) ________ patient data, enabling healthcare professionals to make more
informed decisions. However, there are ethical concerns regarding the privacy and security of sensitive
medical information.

Page 6 of 14
In the realm of transportation, AI is shaping the future of autonomous vehicles. Self-driving cars (5) ________
advanced algorithms and sensors to navigate roads, potentially reducing accidents and improving traffic flow.
Nevertheless, questions about legal liability and the ethics of decision-making in life-or-death situations (6)

L
________ unresolved.
AI has also revolutionized the way we interact with technology through natural language processing. Voice

A
assistants like Siri and Alexa can (7) ________ user commands and respond with relevant information.

CI
However, concerns about data privacy and the potential for misuse has been raised.
Moreover, AI's impact extends to the job market. Automation powered by AI has led to the (8) ________ of

FI
certain jobs, while simultaneously creating new roles that require expertise in AI and data analysis. The (10)
________ between humans and AI in the workforce is a topic of ongoing discussion and debate.

OF
In conclusion, the advancement of AI presents immense possibilities and challenges across various domains.
As we harness the power of AI, it is crucial to address ethical, legal, and societal implications to (10)______a
balanced and responsible integration.
Questions:

ƠN
1. A expanded B. extended C. stretched D. pushed
2. A. unlimited B. boundless C. immense D. vast
3. A identify B. recognize C. distinguish D. detect
NH
4. A. analyze B. assess C. evaluate D. examine
5. A. employ B. utilize C. harness D. exploit
6. A remain B. persist C. endure D. linger
7. A. comprehend B. decode C. interpret D. understand
Y

8. A elimination B. eradication C. depletion D. obsolescence


QU

9. A. interaction B. collaboration C. integration D. cooperation


10. A. assure B. ensure C. secure D. confirm
Your answers:

1. 2. 3. 4. 5.
M

6. 7. 8. 9. 10.

Part 2. Fill each of the following numbered blanks with ONE suitable word and write your answers in the
corresponding numbered boxes on the answer sheet. (15 points)
Organic nutrition or organic food is a phrase that is mentioned a lot nowadays. Organic food is the most
Y

(1)______food from housewives because of the health benefits it brings. But (2)______people understand
what organic food is, what nutritional benefits this food brings, as well as how to choose these foods.
DẠ

The term “organic food” refers to foods that are grown, handled and prepared in a way that is safe for the
environment. For agricultural products, it is a product that is grown (3)______the use of most conventional

Page 7 of 14
pesticides, using artificial fertilizers, sewage sludge, radiation and genetically modified organisms. For animal
meat, antibiotics or hormones are not injected.

Organic food is usually good for the environment. (4)______, it is also relatively more expensive: the USDA

L
(US Department of Agriculture) reports that the cost of organic fruits and vegetables is often 20% more than

A
conventional products. Sometimes the difference can be much higher, (5)______for certain items like organic
milk and eggs.

CI
Advocates say organic food is safer, possibly more nutritious, and often tastier than non-organic food. They
also say that organic production is (6)______for the environment and is also safer for animals.

FI
And more and more consumers seem to be convinced by that. Although organic foods often cost more — or
a lot more — sales in the industry are steadily increasing.

OF
Food scientist Alyson E. Mitchell, and his colleagues at the University of California, Davis have studied
compounds (7)______flavonoids. Recent evidence suggests that these micronutrients play an important role
in preventing cancer and heart disease.

ƠN
Flavonoids are also compounds with plant protection effects. They protect against UV (8)______. They help
fight fungi and bacteria. In addition, flavonoids do not have a favorable taste for pests. Normally, if a plant is
(9)______by bugs, it starts to produce more flavonoids. Mitchell thinks plants sprayed with insecticides and
NH
fungicides won't produce as many flavonoids as plants grown organically.

So her research team compared the flavonoid levels in fruits and vegetables grown in the same place but with
different methods. The results showed that Flavonoids were found more in (10)______grown vegetables.
Y

https://www.vinmec.com/vi/news/health-news/nutrition/what-is-organic-food/
QU

Your answers:

1. 2. 3. 4. 5.
6. 7. 8. 9. 10.
M

Part 3. Read the following passage and circle the best answer to each of the following questions. Write your

answers in corresponding numbered boxes provided on the answer sheet. (15 points)
1. There has, in recent years, been an outpouring of information about the impact of buildings on the natural
environment; Information which explains and promotes green and sustainable construction design, strives to
convince others of its efficacy and warns of the dangers of ignoring the issue. Seldom do these documents
Y

offer any advice to practitioners, such as those designing mechanical and electrical systems for a building, on
DẠ

how to utilise this knowledge on a practical level.

2. While the terms green and sustainable are often considered synonymous, in that they both symbolise nature,
green does not encompass all that is meant by sustainability, which can be defined as minimizing the negative
impacts of human activities on the natural environment, in particular those which have long-term and

Page 8 of 14
irreversible effects. Some elements of green design may be sustainable too, for example those which reduce
energy usage and pollution, while others, such as ensuring internal air quality, may be considered green despite
having no influence on the ecological balance.

L
3. Although there are a good many advocates of ‘green’ construction in the architectural industry, able to cite

A
ample reasons why buildings should be designed in a sustainable way, not to mention a plethora of
architectural firms with experience in green design, this is not enough to make green construction come into

CI
being. The driving force behind whether a building is constructed with minimal environmental impact lies
with the owner of the building; that is, the person financing the project. If the owner considers green design

FI
unimportant, or of secondary importance, then more than likely, it will not be factored into the design.

4. The commissioning process plays a key role in ensuring the owner gets the building he wants, in terms of

OF
design, costs and risk. At the predesign stage, the owner’s objectives, criteria and the type of design envisaged
are discussed and documented. This gives a design team a solid foundation on which they can build their ideas,
and also provides a specific benchmark against which individual elements, such as costs, design and

ƠN
environmental impact can be judged.

5. Owners who skip the commissioning process, or fail to take ‘green’ issues into account when doing so,
often come a cropper once their building is up and running. Materials and equipment are installed as planned,
NH
and, at first glance, appear to fulfil their purpose adequately. However, in time, the owner realises that
operational and maintenance costs are higher than necessary, and that the occupants are dissatisfied with the
results. These factors in turn lead to higher ownership costs as well as increased environmental impact.

6. In some cases, an owner may be aware of the latest trends in sustainable building design. He may have done
Y

research into it himself, or he may have been informed of the merits of green design through early discussion
QU

with professionals. However, firms should not take it as read that someone commissioning a building already
has a preconceived idea of how green he intends the structure to be. Indeed, this initial interaction between
owner and firm is the ideal time for a designer to outline and promote the ways that green design can meet the
client’s objectives, thus turning a project originally not destined for green design into a potential candidate.
M

7. Typically, when considering whether or not to adopt a green approach, an owner will ask about additional

costs, return for investment and to what extent green design should be the limiting factor governing decisions
in the design process. (1) Many of these costs are incurred by the increased cooperation between the various
stakeholders, such as the owner, the design professionals, contractors and end-users. (2) However, in green
design, they must be involved from the outset, since green design demands interaction between these
Y

disciplines. (3) This increased coordination clearly requires additional expenditure. (4) A client may
DẠ

initially balk at these added fees, and may require further convincing of the benefits if he is to proceed. It is
up to the project team to gauge the extent to which a client wants to get involved in a green design project and
provide a commensurate service.

Page 9 of 14
8. Of course, there may be financial advantage for the client in choosing a greener design. Case studies cite
examples of green / sustainable designs which have demonstrated lower costs for long-term operation,
ownership and even construction. Tax credits and rebates are usually available on a regional basis for projects

L
with sustainable design or low emissions, among others.

A
1. The writer’s main purpose is to______.

CI
A. explain to professionals how they can influence clients to choose greener designs
B. explain the importance of green building design in reducing long-term damage to the environment

FI
C. explain to owners commissioning a building why ignoring green issues is costly and dangerous
D. explain to professionals why it is important to follow the correct procedures when a building is

OF
commissioned
2 The examples of green and sustainable designs given in paragraph 2 show that______.
A. designs must be sustainable in order for them to be described as green
B. for the purposes of this paper, the terms green and sustainable have the same meaning

ƠN
C. some sustainable designs are green, while others are not
D. some designs are termed green, even though they are not sustainable
3 According to paragraph 3, the reason for the lack of green buildings being designed is that______.
NH
A. few firms have any experience in design and constructing buildings to a green design
B. construction companies are unaware of the benefits of green and sustainable designs
C. firms do not get to decide whether a building is to be constructed sustainably
D. firms tend to convince clients that other factors are more important than sustainability
Y

4. In paragraph 4, what does the phrase ‘envisaged’ mean?


QU

A. envisioned B. decided C. fixed D. enclosed


5 Which of the following is NOT true about the commissioning process?
A. It is conducted before the building is designed.
B. It is a stage that all clients go through when constructing a building.
M

C. It is a step in the design procedure in which the client’s goals are identified.
D. It provides the firm with a measure of how well they did their job.

6 In paragraph 5, what does the phrase ‘come a cropper’ mean?


A. experience misfortune B. change one’s mind
C. notice the benefits D. make a selection
7 In paragraph 6, the writer implies that______.
Y

A. most clients enter the commissioning process with a clear idea of whether or not they want a green
DẠ

building
B. designers are usually less concerned about green design than the clients are
C. the commissioning process offers a perfect opportunity to bring up the subject of green design
D. firms should avoid working with clients who reject green designs in their buildings

Page 10 of 14
8. Where in paragraph 7 does this sentence belong?
In a typical project, landscape architects and mechanical, electrical and plumbing engineers do not become
involved until a much later stage.

L
A. 1 B. 2 C.3 D.4
9 In paragraph 7, what does ‘balk at’ mean?

A
A. display shock towards B. agree to pay

CI
C. question the reason for D. understand the need for
10. Green buildings are most likely to incur more expense than conventional buildings due to______.

FI
A. higher taxes incurred on sustainable buildings B. higher long-term operational costs
C. the higher cost of green construction materials D. increased coordination between construction teams

OF
Your answers:

1. 2. 3. 4. 5.
6. 7. 8. 9. 10.

ƠN
Part 4. Read the passage and do the following tasks. Write your answers in the corresponding numbered
boxes on the answer sheet. (15 points)
The Maori are the indigenous people of the islands of New Zealand. Their traditional medicine, which is
NH
believed to date back as far as the 13th century, was a system of healing that was passed down through the
generations orally. It comprised diverse practices and placed an emphasis on the spiritual dimension of health.
Its practice included remedies made from herbs, and physical therapies such as massage to relieve discomfort
in the muscles and bones.
Y

Maori systems for treating illness were well developed before European arrived in New Zealand in the late
QU

1700s: they had quite detailed knowledge of anatomy and recognition of the healing properties of various
plants. When Europeans first visited New Zealand, the average age of death for Maori adults was around 30.
However, apart from this, the people were fit and healthy, and troubled by few diseases.
Illness was often seen as spiritually based. Maori saw themselves as guardians of the earth, and the focus of
M

their existence was to remain at one with the natural and supernatural world. Rather than a medical problem,
sickness was often viewed as a symptom of disharmony with natures.

In Maori culture, illnesses were divided into diseases of the gods (mate atua) and physical diseases (mate
tangata). Diseases sent by the gods were often attributed to attacks by evil spirits, because the person had
broken a religious rule. For instance, for Maori, Places where people had died, or places where their ancestors
Y

were buried were sacred, so if someone took food from a river where someone had died, or took a stick form
DẠ

a tree that had held their ancestor's bones and placed it on a cooking fire, it was believed that the gods could
punish them for their disrespectful acts by making them SICK.
More than 200 plants were used medicinally by Maori. The leaves of the flax plant were used to treat skin
infections and food poisoning, and the hard part of the leaf was also used as a splint or brace for broken bones

Page 11 of 14
and injured backs. Flax fibers were used along with a sharpened stick to sew up bad cuts. The bark and leaves
of the pepper tree were used to heal cuts, wounds and stomach pain. People who had toothache were instructed
to chew the leaves of this same tree, and this was found to be of considerable benefit. The pepper tree was

L
also used in vapor baths to treat people with painful joints.
Colonization by European in the 1800s had a significant effect on traditional Maori healing. Europeans

A
brought many new diseases with them which Maori healers had limited ability to combat. Though Western

CI
medicine was also relatively ineffectual at the time, this failure still strongly affected Maori confidence in
their healers. Some western missionaries attributed the spread of disease to the fact the Maori did not believe

FI
in Christianity, and as Maori healers appeared powerless, many Maori accepted this explanation and turned to
Christianity. Over time the schools of higher learning which ahd trained healers started to close and the

OF
tradition of the Maori healer declined.
From the late 20th century, there was renewed Maori interest in their traditional medicine. This was due to
several factors. There was a resurgence of all aspects of Maori culture in New Zealand. Furthermore, people
started to be less trusting of Western medicine-statistics from the 1970s came out revealing that Maori health

ƠN
continued to be poorer than that of other New Zealanders. There were also problems with access to health care
for Maori. Additionally, there was and still a today a perceived lack of a spiritual dimension in Western health
services.
NH
Although Maori today largely accepted Western concepts of health and illness, and use the mainstream health
system, there is significant demand for traditional medicine. This is true for unusual illnesses, or those that
fail to respond to standard medical treatment, but also for common ailments such as the cold and influenza.
Today's healers differ significantly from those of old times. Training is highly variable, usually informal, and
Y

often less tribally bound than the rigorous education of the traditional houses of higher learning. Many modern
QU

healers work in urban clinics, some alongside mainstream health professionals. They experiment,
incorporating knowledge from Western and other medical systems. As a result, their modern day work has no
standard system of diagnosis or widespread agreement about treatments. Despite this, many healers are
recognized as having knowledge and ability that has been passed down from their ancestors. The Maori
M

language is also seen as important by many of those receiving treatment.


Question 1-5

In boxes 1-5 on your answer sheet, write


TRUE if the statement agrees with the information
FALSE if the statement contradicts the information
NOT GIVEN if there is no information on this
Y

1. Early Maori healers learned their skills through studying written texts.
DẠ

2. Diseases of the gods were believed to be more serious than physical diseases.
3. The leaves of the pepper tree were used to treat toothache.
4. Western religion was one reason why traditional Maori medicine became less popular.
5. Modern day Maori healers often reach the same conclusion about the type of treatment which is best.

Page 12 of 14
Your answers:
1. 2. 3. 4. 5.

L
Questions 6-10
Complete the notes below.

A
Choose ONE WORD ONLY from the passage for each answer.

CI
Write your answers in boxes 6-10 on your answer sheet.
A short history of Maori healing

FI
Pre-European arrival
• Maori were using plant based remedies, as well as treatment including massage

OF
• Diseases sent from the gods were thought to be caused by disobeying a spiritual 6.______________
• Sickness could be attributed to eating food from a sacred 7. _____________or burning sacred wood
After European arrival
1800s

ƠN
• The inability of Maori healers to cure new diseases meant the Maori people lost 8 __________in
them.
• Eventually the schools for Maori healing began shutting down
NH
1970s
• Published 9. _______________showed that Maori were not as healthy as Europeans
2000s
• Maori healers can be seen working with Western doctors in 10_________________ in cities
Y

• Many patients appreciate the fact that the Maoris language in used by healers
QU

6. 7. 8. 9. 10.

D. WRITING (50 points)


Part 1. (20 points)
The table below shows the number of hours per week spent by students of different nationalities
M

studying different subjects at a university in the UK.


Summarize the information by selecting and reporting the main features, and make comparisons where

relevant.

Nationality English Maths History Science Art


UK 4 3 2 4 3
Y

China 6 5 2 4 1
DẠ

India 3 2 4 3 1
USA 4 2 2 4 2
Japan 3 3 3 3 3

Page 13 of 14
Part 2. Essay writing (30 points)
Some people believe that children should spend more time on individual studies at home, while others argue
that they should participate in group activities and sports.

L
Discuss both views and give your own opinion.

A
Write at least 250 words.

CI
FI
OF
ƠN
NH
Y
QU
M

Y
DẠ

Page 14 of 14
HƯỚNG DẪN CHẤM
ĐỀ THI ĐỀ XUẤT TRẠI HÈ DUYÊN HẢI – ĐBBB NĂM 2023
TRƯỜNG THPT CHUYÊN SƠN LA – TỈNH SƠN LA

L
MÔN: TIẾNG ANH 10

A
(HDC gồm có: 08 trang)
SECTION I: LISTENING (50pts)
Part 1: Questions 1-5. Complete the notes below. Write ONE WORD AND/ OR A NUMBER for each

CI
answer. (10pts)
1. journalist 2. return 3. delay 4. information 5. parking

FI
Part 2. For questions 6-10, listen to a talk about fear and decide whether these statements are True (T)
or False (F). Write your answers in the corresponding numbered boxes provided. (10 points)

OF
6. TRUE 7. TRUE 8. FALSE 9. TRUE 10. FALSE

Part 3: You will hear part of an interview in which a career coach called Charles Norris is discussing
job-hunting. For questions 11-15, choose the answer (A, B, or C) which fits best according to what you

ƠN
hear. (10pts)
11. D 12. A 13. A 14. B 15. D

Part 4. For questions 16-25, listen to Listen to a recording and complete the sentences with NO MORE
THAN THREE WORDS AND/OR A NUMBER taken from the recording for each answer in the space
NH
provided.

16. human pathogens 21. microbes


17. hypothetical infection 22. catalogue viruses
18. Metagenomics 23. bottommost foundation
Y

19. metagenomic sequencing 24. viral genetic code


20. sequencing machine 25. pulling the strings
QU

SECTION II: LEXICO-GRAMMAR (40pts)


Part 1: Choose one word or phrase which best completes each sentence. Circle its corresponding letter
A, B, C, or D to indicate your answer. (20pts)

1. A 2. B 3. D 4. C 5. C 6. C 7. C 8. D 9. D 10. C
M

11. B 12. D 13. A 14. B 15. B 16. A 17. A 18. B 19. C 20. C

Part 2: Use the word given in capitals at the end of each line to form a word that fits in the gap in the
same line.
1. contemporaneous 6. discretion
2. guardian 7. disciplinary
3 disapproval 8. refugees
Y

4. . stunningly 9. lawful
5. transparency 10. recover
DẠ

Part 3. Complete each of the following sentences with suitable preposition(s). Write your answer in the
boxes provided on the answer sheet. (10 points)
1. under 2. by 3. up 4. out 5. in

Page 1|8
6. at 7. in 8. for 9. with 10. to

SECTION C: READING (60 points)

L
Part 1. Read the following passage and decide which option (A, B, C, or D) best fits each gap. Write your
answers in corresponding numbered boxes on the answer sheet. (15 points)

A
1. A 2. C 3. D 4. A 5. B

CI
6. A 7. C 8. D 9. A 10. B

Part 2. Fill each of the following numbered blanks with ONE suitable word and write your answers in the

FI
corresponding numbered boxes on the answer sheet. (15 points)
1. chosen 2. few 3. without 4. However 5. especially

OF
6. sustainable 7. called 8. radiation 9. attacked 10. organically

Part 3. Read the following passage and circle the best answer to each of the following questions. Write your
answers in corresponding numbered boxes provided on the answer sheet. (15 points)

ƠN
1. A 2. D 3. C 4. A 5. B
6. A 7. C 8. B 9. A 10. D
NH
Part 4. Read the passage and do the following tasks. Write your answers in the corresponding numbered
boxes on the answer sheet. (15 points)

1. FALSE 2. NOT GIVEN 3. TRUE 4. TRUE 5. FALSE


6. rule 7. river 8. confidence 9. statistics 10. clinics
Y
QU

D. WRITING (60 points)


Part 1: 20 points
Contents (10 points)
The report MUST cover the following points:
- Introduce the charts (2 points) and state the overall trends and striking features (2 points)
M

- Describe main features with relevant data from the charts and make relevant comparisons (6 points)

Language use (10 points)


The report:
- should demonstrate a wide variety of lexical and grammatical structures
- should have correct use of words (verb tenses, word forms, voice...) and mechanics (spelling,
Y

punctuations,)
DẠ

Part 2: (30 points)


The mark given to part 3 is based on the following criteria:
1. Organization (5 points)
a. Ideas are well organized and presented with coherence, cohesion and unity.
Page 2|8
b. The essay is well-structured:
* Introduction is presented with clear thesis statement.
* Body paragraph are written with unity, coherence and cohesion.
Each body paragraph must have a topic sentence and supporting details and examples when

L
necessary.

A
* Conclusion summarizes the main points and offers personal opinions (prediction,

CI
recommendation, consideration ...) on the issue.
2. Content (15 points)
a. All requirements of the task are sufficiently addressed.

FI
b. Ideas are adequately supported and elaborated with relevant and reliable explanations,
examples, evidence....

OF
3. Language use (10 points)
a. Demonstration of a variety of topic-related vocabulary.
b. Excellent use and control of grammatical structures (verb tenses, word forms, voice...) and
mechanics (spelling, punctuations...)

PART 1. IELTS 15. TEST 4. PART 1


TRANSCRIPTS
ƠN
NH
Y
QU
M

Y
DẠ

Page 3|8
A L
CI
FI
OF
ƠN
NH
Y
QU
M

Y
DẠ

PART 2. https://www.youtube.com/watch?v=9WicYGH5X4M
So how do you overcome fear and worry? Right this is something we all experience in different
degrees from time to time. This is very easy to overcome once you understand awareness in
the mind. Let's look at a simple view of time. Right you have the past, you have the present
which is now, and the future. Right this is a very simplified way of looking at time.
Page 4|8
Worry and fear only exists in the future. You cannot worry anything about anything in the
past. It's already happened, but you can be concerned or fearful of what the consequences that
may take place in the future. Fear and worry is future based. Fear and worry is awareness
leaving the present moment going into the future, creating a situation that has not happened

L
coming back to the present, and worrying about it. I'll tell you a story that my guru shared with
me when he was seven years old. This was in 1934 up in Lake Tahoe. His family was coming

A
back in the car and it was snowing really heavily, and he was really worried that he would
miss his favorite radio program called captain midnight. And back then if you miss your

CI
favorite radio show there's no internet that you can go and like listen to it again. Right, there's
somebody on the phone going captain midnight. Right, and that's all you get to listen you get
one shot. So sitting in the car, it was snowing really heavily, the car was driving and he was

FI
thinking “my god i'm going to miss my show. We're going to get stuck in the snow.
I'm not going to get home in time, not going to be able to watch my rad, listen to my radio

OF
show.” And then he saw what was happening in his mind. He saw his awareness going into
the future, creating a situation in his mind where they were getting stuck in the snow and then
he was coming back his awareness, came back to the present and started worrying about this
thing that had not happened yet. Then he asked himself “Are we stuck in the snow?” and his
answer was “no”. “Are we still moving” “Yes am I still on schedule to get home to watch my

ƠN
show”. “Yes, so what am I worried about?” right and he said to himself I am all right right
now. And that's a beautiful affirmation to say every time you're fearful or worry say to yourself.
I am all right right now but to understand and overcome fear and worry you have to understand
awareness. And the mind fear and worry is based on the fact that awareness is leaving the
NH
present moment going into the future in your head, creating a situation that has not happened.
coming back to the present and worrying about that or fearing that. So every time awareness
goes into the future in an uncontrolled way and creates a situation in your mind. Right that is
not wholesome bring it back to the present and say I'm not going to let it do it and that's why
you want to develop concentration, and willpower right. So you can keep awareness on what
Y

you're doing. It's okay to let awareness go into the future as long as it goes into the future to
think out things in a very positive way.
QU

PART 3. ON SCREEN C1. MODULE 3- 3d. listening


M

Y
DẠ

Page 5|8
DẠ
Y

M
QU
Y
NH
ƠN
OF
FI
CI
A

Page 6|8
L
A L
CI
FI
OF
PART 4. https://youtu.be/uRABqa8wPQw

ƠN
The current pandemic has highlighted how important it is for us to know what viruses may be
lurking out there. But while we keep one wary eye out for those potentially harmful ones,
we’re also uncovering just how much viruses have actually shaped humans—and the rest of
NH
life on Earth—in some really surprising ways. ‘Virus hunters’, as they’re known, are scientists
who search the hidden corners of the world for viruses that are poised to become human
pathogens. In many cases, they’re searching for Disease. That’s not the name of an actual
illness—it's what we call the hypothetical infection that could cause the next big global
disease. The idea is to find it before it finds us. But other researchers are delving into what else
Y

viruses are doing on our planet besides making people sick. It’s estimated that there are about
this many individual viruses on our planet. And we’ve only made the most miniscule dent in
QU

that number: we’ve found and classified about 9,000 of the viruses on Earth. And of those,
only about 200 are known to cause disease in humans. But over the past decade, scientists have
been identifying new viruses by the thousands. And the main tool they’re using to do that...is
metagenomics. This is actually something I’m using in my own research on bacterial
communities, so let’s break it down together.! In metagenomic sequencing, we take a
M

sample—that can be soil, ocean water, a bodily fluid—and we purify it down to the genetic
material of just the stuff we’re looking for. In this case, let’s say it’s viruses. So now we have

the genomes of all the viruses in our sample. Lets picture each virus's genome as a piece of
paper with sentences on it. Because the next step—and this sounds nuts— is that we rip each
piece of paper up into little strips with just sentences on them. We do this because our
sequencing machine has to ‘read’ all of these genomes. And, if you can picture, it’s much
faster for lots of people to each read a sentence. And they can each read their sentence at the
Y

same time a sentence than it is for one person to read a whole page, one at a time. That’s what
we’re doing when we divide our genome up into little chunks. Now that we’ve read the
DẠ

sentences in a super efficient way, we can put the sentences back together into the full page,
or the full genome. Then we compare that to a library full of known genomes. So we can say,
‘this unknown page is the same as this known page that we have on file’. It’s a match! We’ve
ID’ed the virus in our sample. And we do this for every virus in our sample. Metagenomic

Page 7|8
sequencing is an incredibly useful tool that lets us identify huge numbers of microbes really
fast. And as scientists have turned that powerful lens on our world to catalogue
viruses...they’ve been astonished at just how many they’re finding, everywhere. In the ocean,
in our wastewater, inside the spiders in our gardens and of course, inside us. Most of them are

L
totally harmless to humans, and many of them are totally new to science! The viruses in the
ocean? They’re preying on other marine microorganisms, releasing nutrients. This may be

A
the bottommost foundation of the ocean’s food web. The viruses in every ecosystem? They
help maintain our planet’s biodiversity by keeping other species in check. If viruses suddenly

CI
vanished from the planet, it’s likely that the balance of other organisms would rapidly get out
of whack. And one other major result of all this sequencing is a new understanding of just
how thoroughly viruses have shaped life on Earth as we know it. Because they invade living

FI
cells to replicate, they’re equipped to take over a host’s genetic replication machinery—
sometimes, viral genetic material gets incorporated into the host’s. It’s estimated that around

OF
8% of human DNA actually came from a virus, including some of our most important skills.
Like, the ability of modern humans to give birth probably came from a piece of viral genetic
code that jumped over to its mammalian host about 130 million years ago. That gave us our
ability to grow a placenta. So, yep. We got hacked, and it’s probably the reason we don’t lay
eggs. Scientists don’t know how many more viruses are out there, waiting to be discovered.

ƠN
And of those, we have no idea how many of them may be harmful to us. But sequencing at
least some of those nonillion viruses out there is the first step. The next step is to identify what
they’re living inside...and how exactly these tiny, invisible puppeteers are pulling the strings
of our natural world. If you want more on just how many viruses are in our oceans and what
NH
they’re doing there, you can check out this video, and leave us a comment down below if
there’s another viral topic you’d like to see us cover.
Y
QU
M

Y
DẠ

Page 8|8
TRƯỜNG THPT CHUYÊN SƠN LA – TỈNH SƠN LA
ĐỀ ĐỀ XUẤT KỲ THI CHỌN HSG KHU VỰC DH VÀ ĐBBB

L
NĂM HỌC 2022-2023

A
MÔN: TIẾNG ANH – LỚP 11
(Đề thi có 20 trang)

CI
I. LISTENING (50 POINTS)

FI
Part 1: You will hear part of a discussion in which two business owners, Anya Stern
and Vincent Chambers, are talking about their experiences of launching their own
businesses. For questions 1-5, decide whether the following statements are TRUE (T) or

OF
FALSE (F). (10 points)
1. Anya's parents were successful in adapting to e-commerce.
2. Vincent worked for a large tech giant before starting his own business.
3. Anya wanted to bring Brazilian cuisine to the mass market.

ƠN
4. Anyone can have a unique business proposition.
5. You should always abandon your plans if you encounter adversity.
Your answers:
NH
1. 2. 3. 4. 5.
Part 2: Listen to a talk about a new approach in the field of genomics and answer the
questions. For questions 6-10, write NO MORE THAN THREE WORDS taken from the
recording for each answer in the corresponding numbered spaces provided. (10 points)
Y

6. In what aspect of biology does the scientific community has a limited understanding until
now?
QU

7. How is the current process of interpreting scientific outcomes described?


8. What element of the new approach enables the examination of biological systems in great
detail?
9. Along with a chemical process, what is utilized to conduct single-cell sequencing?
M

10. Apart from promoting research within biology, in what way can this advance in
genomics contribute scientifically?

Your answers:
6. 9.
7. 10.
8.
Y
DẠ

Part 3: You will hear a discussion in which academics Gordon Mackie and Sophie
Blackmore talk about how communication has changed in society. For questions 11–15,
select the best answer A, B, C or D. (10 points)
11. In his latest book, Gordon aims to challenge the notion that…
Page 1 of 20
A. technology has changed the purpose of communication.
B. linguistic standards in society are falling.
C. the media influences people’s use of language.

L
D. the evolution of language is inevitable.

A
12. Gordon says his main duty as an academic is to…
A. ensure language traditions survive in modern communication.

CI
B. record examples of linguistic patterns and trends.
C. explain the fundamental principles of correct language use.
D. evaluate different theories about language change.

FI
13. Sophie says the thing she values most in written communication is:
A style. B accuracy. C clarity. D tone.

OF
14.Why does Sophie use social media posts in her classes?
A. To raise awareness of what catches readers’ attention
B. To show how people alter their communication styles
C. To highlight the features of informal communication

ƠN
D. To prove that context changes the meaning of a message
15. What aspect of ‘text language’ do Sophie and Gordon disagree about?
A. Whether it will ever be accepted in education
NH
B. Why it may be favored by young people
C. How well it crosses cultural boundaries
D. Whether it influences spoken communication
Your answers:
Y

11. 12. 13. 14. 15.


Part 4: For questions 16-25, listen to a presenter talking about a phenomenon in the nature
QU

and complete the summary by writing NO MORE THAN THREE words and/or a number
in each gap. (20 points)
- Nutrients are the building blocks of plant life.
- Plants naturally absorb nutrients from the soil through their root system. However, farmers
M

turn to fertilizer when facing poor soil or massive 16. __________.


- Farmers tend to overuse fertilizer as a precautionary measure because it is difficult to 17.

_____________the amount of fertilizer needed.


- Excess fertilizer can runoff into bodies of water causing 18. ______________,
characterized by the rapid accumulation in the population of algae in marine water systems.
- Dense layers of algae form an 19. _________ on the surface of the water, blocking
Y

other plants in the water from getting the sunlight they need to survive.
DẠ

- Because water cannot support aquatic life, plants die off and sink to the bottom of the water
body, where decomposers 20. the dead bodies. This decomposition process consumes
plenty of dissolved oxygen and animals that rely on oxygen to breathe can, in fact, 21.

Page 2 of 20
________. A vicious cycle of degradation in aquatic life can be observed and a 22. _______is
created.
- If this process occurs in lakes, native species can give way to 23. _________. If this

L
happens in the ocean, the incidence of coral bleaching is inevitable.

A
- Nutrient-rich runoff, also known as 24. ___________, can also be the result of 25.
___________and farming activities.

CI
Your answers:
16. 21.

FI
17. 22.
18. 23.
19. 24.

OF
20. 25.

II. LEXICO AND GRAMMAR (30 POINTS)


Part 1: Choose the answer A, B, C, or D that best completes each of the following

ƠN
sentences. Write your answers in the corresponding numbered boxes. (20 points)
Grammar
26. The problem of pollution is a _____ topic.
NH
A. hotly debating B. debating hotly C. debated hotly D. hotly debated
27. I never bought a telephone answering machine. They’re just not ___________ useful.
A. that B. too C. that far D. that much
28. The laws of this country apply to everyone, ___________ you are rich or poor.
Y

A. either B. whether C. both D. including


29. Using English regularly is often the key ___________ fluent in it.
QU

A. in becoming B. to become C. to becoming D. becoming


30. This project is ___________ for the level of this class.
A. too much really hard B. much too really hard
C. really much too hard D. really hard too much
M

Vocabulary
31. When my sister has an exam to take, she either studies day and night or does nothing at

all. Shecan never find the____________.


A. a double bind B. a leading light C. the common touch D. the happy medium
32. Their business ____________better than expected and profits were slightly up.
A. came about B. brought up C. set off D. made out
Y

33. We can buy some more things if you want – that won’t break____________.
DẠ

A. the mold B. the ice C. the bank D. the budget


34. Tom wanted to be a professional footballer, but he didn't make the________ .
A. grade B. point C. mark D. best

Page 3 of 20
35. This software is called a “firewall” because it is an __________obstacle when someone
wants to access the system's data.
A. unyielding B. insurmountable C. infrangible D. invulnerable

L
36. North Korea is a small country but _________ a great punch on the world stage as it

A
possesses nuclear weapons.
A. throws B. blows C. packs D. lands

CI
37. I don’t want to make a decision ___________, so I need a couple of minutes to think about
it.
A. in one go B. there and then C. at a stroke D. on and off

FI
38. It’s their __________ of stupidity to go swimming in the beach in this stormy weather.
A. height B. depth C. source D. matter

OF
39. A little ___________ told me that you were accepted to Harvard. Is that correct?
A. dog’s B. birdie C. fox D. cat’s
40. The number of foreign tourists coming to Vietnam has been growing __________.
A. by leaps and bounds B. from time to time

ƠN
C. slow but sure D. by hook and crook
41. People turned out in _________ into My Dinh Stadium to watch the friendly match between
the Vietnamese team and the Japanese team
NH
A. volume B. mass C. force D. bulk
42. Not being able to find my email address is a pretty _________excuse for not
contacting me.
A. fragile B. frail C. faint D. feeble
Y

43. The pollution problems on the beaches have been ___________ by mass tourism in the
summer months.
QU

A. exacerbated B. developed C. augmented D. contributed


44. You’ll have to _________ yourself to the fact that you can’t always have what
you want.
A. acknowledge B. reconcile C. concede D. allow
M

45. It was a fantastic movie. I enjoyed it _________.


A. endlessly B. unendingly C. without end D. no end

Your answers:
26. 27. 28. 29. 30. 31. 32. 33. 34. 35.
36. 37. 38. 39. 40. 41. 42. 43. 44. 45.
Y

Part 2: Give the correct form of each given word in CAPITAL to complete the following
DẠ

sentences. Write your answers in the numbered boxes. (10 points)


46. Chemicals are easily ________if they are laid open in the atmosphere for a long time.
(ACTION)
47. His speech is __________ dull!
Page 4 of 20
(SPEAK)
48. The new students will be ________________ classified into different classes.
(QUALIFY)

L
49. A person with an___________ complex is generally quite shy.

A
(INFERIOR)
50. The New Year 2023’s _________ show involves almost every famous face of the showbiz.

CI
(STAR)
51. Lot of young people go abroad in ___________ of their study.
(PERSUE)

FI
52. Jason was ____________ from all the major social media sites for spreading a fake new.
(PLATFORM)

OF
53. The conflict had a seriously ___________ effect on the region.
(STABLE)
54. ______________ children often have poor concentration and require very little sleep.
(ACTIVE)

ƠN
55. High __________ among suppliers, distributors and customers via Internet is promoting
development of e-commerce in Vietnam.
(OPERATE)
NH
Your answers:
46. 51.
47. 52.
Y

48. 53.
49. 54.
QU

50. 55.

III. READING (60 POINTS)


Part 1: Read the text below and think of the word which best fits each space. Use only
M

ONE word in each space. Write your answers in the space provided. (15 points)
People are often (56) _________ off meditation by what they see as its many mystical

associations. Yet meditation is a straightforward technique which merely involves sitting and
resting the (57) _________ In addition to its simplicity, meditation offers powerful help in the
battle (58) _________stress. Hundreds of studies have shown that meditation, when (59)
_________in a principled way, can reduce hypertension which is related to stress in the (60)
Y

_________ Research has proved that certain types of meditation can substantially decrease key
DẠ

stress symptoms such as anxiety and irritability. In fact, those who practise meditation with
any (61) _________ see their doctors less and spend, on average, seventy per cent (62)
_________ days in hospital. They are said to have more stamina, a happier disposition and
even enjoy better relationships.
Page 5 of 20
When you learn to meditate, your teacher will give you a personal 'mantra' or word
which you use every time you practise the technique and which is supposedly chosen according
to your needs. Initial classes are taught individually but (63) _________ classes usually consist

L
of a group of students and take place over a period of about four days. The aim is to learn how

A
to slip into a deeper (64) _________ of consciousness for twenty minutes a day. The rewards
(65) _________ for themselves.

CI
Your answers:
56. 57. 58. 59. 60.

FI
61. 62. 63. 64. 65.

Part 2: Read the article below and choose the answer (A, B, C or D) which you think fits

OF
best according to the text. (10 points)
OIL AGAINST CARIBOU…
They hiked over mountains and canoed along crashing Arctic rivers. They were guided
by a sun that set for only an hour a night and braced by freezing winds. [A] When they finally

ƠN
made it to the log cabins of Arctic Village on Friday, their cries of joy and their dancing and
singing for a moment drowned out the deep fear that drove them on their odyssey. [B] The
100,000-strong Gwich’in tribe’s ‘millennium trek’ was a desperate plea for survival. One of
NH
the last tribes of native Americans to live by subsistence, they fear that they – and the caribou
on which they depend – are about to lose a 25-year battle that could end in one last Klondike-
like scramble for oil. [C] It is a battle that has pitched Alaska against the rest of America and
the Indians and environmentalists against Britain’s biggest oil company. [D]
Y

The age-old conflict between development and environment has nowhere been so stark
as in this fragile corner of the world. Nowhere are man’s thirst for oil – and the effects of global
QU

warming – having such an impact. The remote Gwich’in community of Arctic Village, 100
miles north of the Arctic Circle and with no access by road, nestles against the southern edge
of the vast Arctic National Wildlife Refuge. The refuge is the last true wilderness in North
America, so abundant in wildlife it has been dubbed America’s Serengeti. Here are 20 million
M

acres of soaring snow-capped mountains, marshy coastal plains, Arctic tundra and forests of
stunted trees, with no signs of man: no roads nor car parks, no visitor centres nor hiking trails.

In the short summers the tundra explodes into vibrant reds and yellows. In winter it is
smothered in snow. Visitors talk about a primal landscape that takes them back to a time before
man, but to the environmentalists and the Gwich’in, it is the wildlife that makes it unique. It
has the only population of Alaskan polar bears that live on land; it has the near-extinct shaggy
Y

musk ox, the regal moose, grizzly bears, wolverine and lynx. The refuge is the breeding ground
DẠ

for 150 species of birds that migrate to the US, Asia and South America for the bitter winters,
when for three months the sun does not rise and temperatures drop below minus 50 centigrade.
Above all, it is the 130,000 caribou which dominate the landscape and on which the
Gwich’in depend for much of their food. Each year the caribou go on one of the last great
Page 6 of 20
mammal migrations on the planet, trekking from 1,000 miles away in Canada, through the
9,000ft Brooks range and swimming precariously across sweeping rivers, to the narrow coastal
strip of the refuge. Here, in the calving grounds sacred to the Gwich’in, the caribou fatten up

L
with cotton grass to get them through the Arctic winter and, protected from predators, the cows

A
give birth. But amid the majestic scenery are a few oil seeps, a tell-tale sign of what is
underneath: below the feet of the grazing caribou lies enough crude oil to fill up to 16 billion

CI
barrels – worth many tens of billions of pounds – and the oil companies are pushing hard to
start drilling.
Oil is already produced along much of the rest of the Alaskan coast but is starting to dry

FI
up. At the Prudhoe Bay field, just along from the refuge, production has fallen from two million
barrels a day a decade ago to a million now. And the momentum to raid the refuge for oil,

OF
which will further promote global warming, is building. Although not all shareholders are
enthusiastic, preferring money to be spent on renewable energy sources, the oil industry can
count on the support of the majority Alaskans. Oil has made this once poor and punishing state
rich and far more comfortable, and now accounts for 85 per cent of its economy. Anchorage,

ƠN
not so long ago a small, insignificant town, now boasts soaring, mirrored office blocks, and
every village now has a million-dollar school. Oil has also brought many thousands of well-
paid jobs, and opening up the refuge will create up to 700,000 more, and with current oil fields
NH
drying up, pressure for new jobs is intense.
Prudhoe Bay is one of the largest industrial developments in the world, with hundreds
of miles of pipelines snacking across the damp tundra, disappearing off to the horizon in every
direction. There are hundreds of well heads and processing facilities, flares, landing strips,
Y

roads, sea water processing plants and accommodation blocks. In between, the land is indeed
clean and major oil spills have been avoided. However, the company’s record here is far from
QU

unblemished: earlier this year it had to pay a $15m fine for toxic waste dumping.
For the Gwich’in, there is a far more fundamental issue. Faith Gemmill, their spokeswoman,
showed me around her ancestors’ cemetery, on a bend in a river overlooked by the mountains.
She then declared: “This is how we have lived for thousands of years. No one has a right to
M

take this away from us for money or greed. This is what they did to the Sioux when they killed
the buffalo. When is this country going to learn they can't do that to a people?

Questions:
66. Which of the square brackets [A], [B], [C], or [D] best indicates where in the first paragraph
the sentence “For 800 miles across the wilderness they trekked, from the Yukon in Canada
to remotest Alaska, far above the Arctic Circle.” can be inserted?
Y

A. [A] B. [B] C. [C] D. [D]


DẠ

67. The Gwich’in tribe seem to have…


A. travelled to gain publicity. B. taken part in a traditional dance ritual.
C. recently become aware of a threat. D. been at war with neighboring tribes.
68. What is meant by the phrase “live by subsistence” in the first paragraph?

Page 7 of 20
A. People depend only on the natural resources available to them in their immediate
environment for their survival.
B. People deal with a clash between industrial and environmental interest.

L
C. People live in an area where the caribou breed – perhaps one which is being threatened by

A
oil pollution.
D. People suffer one of the various forms of anti-environmental activity which threaten the

CI
caribou’s habitat.
69.Their destination, Arctic Village…
A. falls within a wildlife reserve. B. is home to many wild animals.

FI
C. is part of an area under threat. D. is completely inaccessible.
70. According to the article, locals and tourists…

OF
A. are struck by the diverse natural surroundings.
B. are mainly interested in birds.
C. are awed by the area's long history.
D. have different perceptions of the area.

ƠN
71. The caribou are unusual in that…
A. they survive in an oil field. B. they make seasonal journeys.
C. they are considered sacred. D. they can only eat certain grasses.
NH
72. Greatest support for the drilling to begin comes from…
A. oil companies. B. the Alaskan locals.
C. the unemployed. D. the Anchorage authorities
73. The writer implies that the installation at Prudhoe Bay…
Y

A. is a taste of things to come. B. is larger than it should be.


C. is without environmental impact. D. is aesthetically pleasing.
QU

74. What point is Faith Gemmill making when she mentions the Sioux and the buffalo in the
last paragraph?
A. Just as the traditional life of the Sioux depended on the survival of the buffalo, the traditional
life of the Gwich’in will be destroyed if the oil company destroys the habitat of the caribou.
M

B. The local population will support the oil industry because drilling has brought wealth and
prosperity to other Alaskan communities.

C. The Sioux are a tribe of North American Indians famed for their struggle to protect their
territories and buffalo from the white settlers in the mid-nineteenth century.
D. The benefits of drilling for oil in the area where caribou graze is: the huge quantity of crude
oil which lies beneath the caribou grazing ground; and the wealth which would greatly improve
Y

the quality of life for thousands of people living and working in the area.
DẠ

75. Regarding the clash between the environmentalists and the oil companies, the writer’s
attitude can best be described as…
A. cynical B. balanced C. balanced D. aloof
Your answers

Page 8 of 20
66. 67. 68. 69. 70. 71. 72. 73. 74. 75.

L
Part 3: Read the passage and do the tasks that follow. (13 points)

A
Adults and children are frequently confronted with statements about the alarming rate
of loss of tropical rainforests. For example, one graphic illustration to which children might

CI
readily relate is the estimate that rainforests are being destroyed at a rate equivalent to one
thousand football fields every forty minutes – about the duration of a normal classroom period.

FI
In the face of the frequent and often vivid media coverage, it is likely that children will have
formed ideas about rainforests – what and where they are, why they are important, what
endangers them – independent of any formal tuition. It is also possible that some of these ideas

OF
will be mistaken.
Many studies have shown that children harbour misconceptions about ‘pure’,
curriculum science. These misconceptions do not remain isolated but become incorporated into
a multifaceted, but organized, conceptual framework, making it and the component ideas, some

ƠN
of which are erroneous, more robust but also accessible to modification. These ideas may be
developed by children absorbing ideas through the popular media. Sometimes this information
may be erroneous. It seems schools may not be providing an opportunity for children to re-
NH
express their ideas and so have them tested and refined by teachers and their peers.
Despite the extensive coverage in the popular media of the destruction of rainforests,
little formal information is available about children’s ideas in this area. The aim of the present
study is to start to provide such information, to help teachers design their educational strategies
Y

to build upon correct ideas and to displace misconceptions and to plan programmes in
environmental studies in their schools.
QU

The study surveys children’s scientific knowledge and attitudes to rainforests.


Secondary school children were asked to complete a questionnaire containing five open-form
questions. The most frequent responses to the first question were descriptions which are self-
evident from the term ‘rainforest’. Some children described them as damp, wet or hot. The
M

second question concerned the geographical location of rainforests. The commonest responses
were continents or countries: Africa (given by 43% of children), South America (30%), Brazil

(25%). Some children also gave more general locations, such as being near the Equator.
Responses to question three concerned the importance of rainforests. The dominant
idea, raised by 64% of the pupils, was that rainforests provide animals with habitats. Fewer
students responded that rainforests provide plant habitats, and even fewer mentioned the
Y

indigenous populations of rainforests. More girls (70%) than boys (60%) raised the idea of
DẠ

rainforest as animal habitats.


Similarly, but at a lower level, more girls (13%) than boys (5%) said that rainforests
provided human habitats. These observations are generally consistent with our previous studies
of pupils’ views about the use and conservation of rainforests, in which girls were shown to be
Page 9 of 20
more sympathetic to animals and expressed views which seem to place an intrinsic value on
non-human animal life.
The fourth question concerned the causes of the destruction of rainforests. Perhaps

L
encouragingly, more than half of the pupils (59%) identified that it is human activities which

A
are destroying rainforests, some personalizing the responsibility by the use of terms such as
‘we are’. About 18% of the pupils referred specifically to logging activity.

CI
One misconception, expressed by some 10% of the pupils, was that acid rain is
responsible for rainforest destruction; a similar proportion said that pollution is destroying
rainforests. Here, children are confusing rainforest destruction with damage to the forests of

FI
Western Europe by these factors.
While two fifths of the students provided the information that the rainforests provide

OF
oxygen, in some cases this response also embraced the misconception that rainforest
destruction would reduce atmospheric oxygen, making the atmosphere incompatible with
human life on Earth.
In answer to the final question about the importance of rainforest conservation, the

ƠN
majority of children simply said that we need rainforests to survive. Only a few of the pupils
(6%) mentioned that rainforest destruction may contribute to global warming. This is
surprising considering the high level of media coverage on this issue. Some children expressed
NH
the idea that the conservation of rainforests is not important.
The results of this study suggest that certain ideas predominate in the thinking of
children about rainforests. Pupils’ responses indicate some misconceptions in basic scientific
knowledge of rainforests’ ecosystems such as their ideas about rainforests as habitats for
Y

animals, plants and humans and the relationship between climatic change and destruction of
rainforests.
QU

Pupils did not volunteer ideas that suggested that they appreciated the complexity of
causes of rainforest destruction. In other words, they gave no indication of an appreciation of
either the range of ways in which rainforests are important or the complex social, economic
and political factors which drive the activities which are destroying the rainforests. One
M

encouragement is that the results of similar studies about other environmental issues suggest
that older children seem to acquire the ability to appreciate, value and evaluate conflicting

views. Environmental education offers an arena in which these skills can be developed, which
is essential for these children as future decision–makers.
Questions 76-83
Do the following statements agree with the information given in Reading Passage 1? In
Y

boxes 76-83, write:


DẠ

TRUE if the statement agrees with the information


FALSE if the statement contradicts the information
NOT GIVEN if there is no information on this
76. The plight of the rainforests has largely been ignored by the media.

Page 10 of 20
77. Children only accept opinions on rainforests that they encounter in their classrooms.
78. It has been suggested that children hold mistaken views about the ‘pure’ science that they
study at school.

L
79. The fact that children’s ideas about science form part of a larger framework of ideas means

A
that it is easier to change them.
80. The study involved asking children a number of yes/no questions such as ‘Are there any

CI
rainforests in Africa?’
81. Girls are more likely than boys to hold mistaken views about the rain forests’ destruction.
82. The study reported here follows on from a series of studies that have looked at children’s

FI
understanding of rainforests.
83. A second study has been planned to investigate primary school children’s ideas about

OF
rainforests.
Questions 84-88
The box below gives a list of responses A–P to the questionnaire discussed in Reading
Passage 1. Answer the following questions by choosing the correct responses A–P. Write

ƠN
your answers in boxes 84–88.
84. What was the children’s most frequent response when asked where the rainforests were?
85. What was the most common response to the question about the importance of the
NH
rainforests?
86. What did most children give as the reason for the loss of the rainforests?
87. Why did most children think it important for the rainforests to be protected?
88. Which of the responses is cited as unexpectedly uncommon, given the amount of time spent
Y

on the issue by the newspapers and television?


QU

A. There is a complicated combination of reasons for the loss of the rainforests.


B. The rainforests are being destroyed by the same things that are destroying the forests
of Western Europe.
C. Rainforests are located near the Equator.
M

D. Brazil is home to the rainforests.


E. Without rainforests some animals would have nowhere to live.

F. Rainforests are important habitats for a lot of plants.


G. People are responsible for the loss of the rainforests.
H. The rainforests are a source of oxygen.
I. Rainforests are of consequence for a number of different reasons.
Y

J. As the rainforests are destroyed, the world gets warmer.


DẠ

K. Without rainforests there would not be enough oxygen in the air.


L. There are people for whom the rainforests are home.
M. Rainforests are found in Africa.
N. Rainforests are not really important to human life.

Page 11 of 20
O. The destruction of the rainforests is the direct result of logging activity.
P. Humans depend on the rainforests for their continuing existence.
Your answers:

L
76. 77. 78. 79. 80.

A
81. 82. 83. 84. 85.
86. 87. 88.

CI
Part 4: Read the text. Seven paragraphs have been removed. Choose from the paragraphs

FI
A- H the one which fits each gap (89-95). There is ONE extra paragraph which you do
not need to use. (0.7 pt)
Would I lie to you?

OF
Blatant dishonesty has invaded our culture. Sue Jackson explains how to spot a liar.
Who hasn’t told a lie? Even the most upstanding individual probably utters one occasionally
to help the day to run more smoothly. But, according to the experts, the extent to which people
regularly tell serious untruths has exploded. Lying has pervaded every aspect of our lives.

ƠN
89
Research in California reveals that people lie up to 20 times a day, while in a poll last
year, a quarter or respondents admitted being untruthful on a daily basis. Only 8 per cent
NH
claimed they had never lied – although there is always the chance, that even then they weren’t
being honest. Many of these will be sweet little lies, the type psychologists refer to as ‘false
positives’ and the sort we are all guilty of committing when we want to appear more
enthusiastic about something than we really are.
Y

90
Until recently it was thought that only manipulative and Machiavellian characters were
QU

prone to excessive fabrication of this sort, but research has proved otherwise. According to
experts, anyone under pressure or with the being enough incentive is prepared to say something
that isn’t true.
91
M

That figure rose to one in three among people with university qualifications. Apparently,
this sort of background gives people the vocabulary and the confidence to deceive. They are

more sophisticated and plausible than you might elsewhere in society.


92
The proliferation of lying in corporate culture means that there are huge profits to be
Y

gained by companies who can weed out fraudsters before employing them. Numerous studies
have been conducted, including using video cameras, to analyze people who lie. There are two
DẠ

main methods of ousting liars, although one, the mechanical lie detector or polygraph, requires
subjects to be trussed up in electrodes, so it hardly lends itself to interviews. That leaves body
language and psychological testing.
93
Page 12 of 20
However, sometimes the subconscious takes over. Liars often start blinking fast, a
visual sight that the brain is concentrating hard on the task in hand, and are likely to frequently
touch their body and face with their hands. Liars are also more likely to tap or swing a foot as

L
they speak.

A
94
Everyone seems to agree that good liars don’t show non-verbal signals, so you need to

CI
know what to look for. Lying takes a lot of effort, so often they will rely on past experience to
see them through and reduce a cognitive load.

FI
95
Experts, however, agree that the one person you shouldn’t deceive is yourself – and that,
once you begin to do so, it is a sure sign that your untruthfulness is getting out of hand.

OF
Paragraphs to choose:
A. This makes detecting the charlatan who fibs his way through a CV very difficult. In the
film Liar, Liar the comedian Jim Carrey played a smooth-talking lawyer and consummate liar

ƠN
who specializes in dealing with untrustworthy clients whom no one else will take on. Only
when his young son made a wish to see his father get through an entire day without lying was
Carrey’s character forced to tell the truth. Mayhem ensued.
NH
B. Things like ‘That was delicious, thank you’, ’You look great in that dress’ and ‘Of
course I want to see you’. They are mostly considered harmless social fabricants. But at the
other end of the spectrum are compulsive liars who are effortlessly dishonest.
C. Visual clues are not wholly reliable, as experienced deceivers are aware of the
Y

common give-away signs and take calculated measures to avoid them. Shifty eyes, for
instance, are traditionally thought to be a sure way to tell whether someone is being dishonest,
QU

but experienced fabricators will capitalize on this myth.


D. So, for instance, people who are lying about where they have been may declare they
were at the cinema or the gym so that their untruths doesn’t take too much mental planning. It
is easier to make up story about something they know well and have done many times.
M

E. ‘We are experiencing an epidemic of lying’ says Professor Leo Damak, an expert in lie
detection at a leading university. ‘It has always been around, but we are much more aware of

it now.’ In one study of college students, 85 per cent of couples reported that one or both of
them had lied about past relationships or recent events. In another, it was found that dating
partners lie to each other in about a third of their conversations.
F. A recent study found that pathological liars are just as likely to be self-confident,
Y

attractive and popular as they are introverted and withdrawn. It also seems that the better
DẠ

educated a person is, the higher level of deceit. It was found that falsehoods typically occurred
in one fifth of all ten-minute conversations they have.

Page 13 of 20
G. Obviously, many won’t stand for ambiguity any more. By being more aware of how
and why someone will tell a lie, they have more chance of catching him before he tells another
and causes real harm.

L
H. However, vocabulary and sounds are generally considered more reliable indicators than

A
body movements. Liars tend to use fewer words, take longer to start answering a question and
a pause a lot as if to mentally rehearse what they are about to say. Their voices may adopt a

CI
high pitch and they are prone to repetition.
Your answers:
89. 90. 91. 92. 93. 94. 95.

FI
Part 5: You are going to read a set of science book reviews. For questions 96–105, choose

OF
from the reviews (A-D). The reviews may be chosen more than once. Write your answer
in the numbered boxes. (15 points)
Prairie Fever

ƠN
A new book chronicles the efforts of 19th century British aristocrats to create a corner of
England in the American west.
A. How the British aristocracy was drawn to the frontier lands of 19th-century America is
perhaps the most bizarre episode in the country's epic immigration story, and is revealed in a
NH
remarkable new book, Prairie Fever, by veteran BBC documentary maker Peter Pagnamenta.
Lured by romantic tales of the American outdoors by writers such as James Fennimore Cooper,
and the real-life gun-slinging escapades of Wild Bill Hickock, these eccentric newcomers
wanted the U.S. on their own terms. In settlements with reassuringly British names, such as
Y

Runnymede and Victoria, the British aristocracy set about ensuring that there was one corner
of America that was forever England.
QU

B. The pioneers started arriving in the 1830s. Some were sportsmen drawn by the promise
of unlimited buffalo to hunt, others true adventurers. They were led by Scotsman Sir William
Stewart, a Waterloo veteran who spent seven years trekking through the Rockies, rubbing
shoulders with mountain men, and fending off marauding bears and Indians. His companion,
M

Charles Murray, son of the Earl of Dunmore, lived for a spell with the Pawnee Indians. The
Old Etonian had to swallow his pride when his hosts ate his dog, but he impressed with rock-

throwing contests in which he used skills honed in the Highland Games. Sadly, few of
the lords that followed were nearly so adaptable. They often treated the locals and their customs
with utter contempt. Sir George Gore — a classic example of the breed -- went on a $100,000,
three-year hunting expedition beginning in 1854 in Missouri. American officials later accused
Y

him of slaughtering 6,000 buffalo, single-handedly endangering the Plains Indians Food
DẠ

supply. Later, the English settlers wound up the Americans even more because of their air of
superiority.
C. By the 1870s, however, their American hosts had more to complain about than
aristocratic rudeness – the British wanted to settle permanently. The British ruling classes had
Page 14 of 20
realised that the American West wasn't just a good place to hunt and carouse, but also the
perfect dumping ground for younger sons with few prospects at home, America, desperate for
new settlers to farm prairie states like Kansas and Iowa, welcomed them with open arms. Back

L
in Britain, the Press followed the settlers closely. ‘It was hot but everyone looked happy ... how

A
much more sensible and useful lives they live there than they would live here at home!’ the
Times reported. Yet more astute observers noted that the British settlers never grasped the

CI
American work ethic. For them, running their farms came a poor second to hunting and
enjoying themselves.
D. The prairie states were already dotted with ‘colonies’, each made up exclusively of

FI
workers from one part of America or one group of immigrants such as Danes or Russians. In
1873, an enterprising Scottish gentleman farmer named George Grant had a brainwave - a

OF
colony in western Kansas populated entirely by the British upper classes, by stipulating that
they had to have at least £2,000 in funds and would each get no less than a square mile of land,
he kept out the rabble. Victoria, as Grant patriotically called his settlement, was talked back
home as a ‘Second Eden’, but the new arrivals - many of whom had never farmed in their lives

ƠN
- soon discovered it was a hard place to play the country gentleman. No rain would fall for
months and the temperature could soar to 1050F in the shade. Worst of all, nobody had
mentioned the dense clouds of grasshoppers that would suddenly arrive and eat everything.
NH
Despite their neighbors’ derision at these remittance men’ (so named because they relied on
allowances from their parents), the two hundred or so colonists gamely battled on.
E. In general, the colonists’ dreams came to nothing, and many headed home. But there
was one event that definitively ended the British aristocracy’s love affair with the West.
Y

Encouraged by the vast sums to be made from cattle ranching, some wealthy British investors
bought huge tracts of land. One investor alone amassed 1.75 million acres and 100,000 cattle.
QU

Enough was enough. Tolerant when the British were buffoonish adventurers, Americans felt
threatened once they became too rich. U.S. politicians stoked anti-foreigner resentment, aided
by widows out of their homes and rumours that some were so snobby they referred to their
cowboys as ‘cow-servants’. Congress passed the Alien Land Act limiting foreign companies
M

to buying no more than 5,000 acres in future.


In which paragraph is each of the followings mentioned? Your answers


86. the opinion that the settlers never got their priorities right 96. ________
87. the fact the settlers wanted nothing less than a home away from home 97. ________
88. the wish to maintain exclusivity in the British colonies 98. ________
Y

89. the inability of the settlers to become truly independent of Britain 99. ________
DẠ

90. the view that the English were naive in their expectations of the USA 100. _______
91. a difference in the locals’ and settlers’ cultural taboos 101. _______
92. the view that the British settlers were victims of their own success 102. _______
93. the fact that America offered a solution to a problem 103. _______

Page 15 of 20
94. a newspaper showing lack of perception 104. _______
95. the disregard of the settlers for the locals’ way of life 105. _______
Your answers:

L
96. 97. 98. 99. 100. 101. 102. 103. 104. 105.

A
IV. WRITING (60 POINTS)

CI
Part 1: Read the following extract and use your own words to summarize it. Your
summary should be about 100-120 words long. (15 pts)

FI
The sea turtles, especially the leatherbacks, are undoubtedly one of nature amazing
creatures. They roam the warm seas of the world throughout their come ashore only to lay
eggs. Malaysia with its long sandy shorelines has destined as one of their breeding grounds.

OF
Unfortunately, the number of leatherback turtles landing on the beaches has been declining
over the years. According WWF estimates, about 2000 leatherbacks arrived on Malaysian
beaches in 19 1989, only between 30 and 60 leatherbacks were found. The drastic drop within
a short span is certainly a great cause for concern. Where have they gone to? Are avoiding the

ƠN
beaches of Malaysia? Whatever it is, the message is disturbingly cl they may become an extinct
species soon, that is, if no concerted effort is taken to check the decline now.
The belief that the declining landings of turtles in Malaysia is the result of increasing
NH
landings in others parts of the world is a fallacy. This is because scenario is the same in other
countries known to have been visited by the turtles. What has caused the species to dwindle at
such a rapid rate? There are many reasons but an obvious one is none other than man's greed.
As we know, turtles are killed for their meat. In the days before refrigeration, turtles had been
Y

a source of fresh food for sailing ships. Today, turtle soup is a favorite dish among the Asians.
Their shells have become coveted items for decorations and jewelry. Their eggs which are
QU

meant to hatched into young turtles, are instead harvested and eaten. When deep sea fishing
nets inadvertently trap the turtles, fisherman often kill the turtles instead of cutting their nets
to release them. Pollution of the sea has also reduced the number of turtles. Many are choked
to death by the plastic bags that they mistake for jelly fish. It appears that the turtles are no
M

longer safe in the sea where they spend most of the lives.
Neither are they safe when the females come ashore to lay eggs. In fact, this is the time

when they are particularly vulnerable as their movements are slow on land. The nesting places
for these turtles have also been greatly reduced. As more and more beaches are taken over for
tourism with the construction of hotels, chalets and condominiums, the breeding grounds are
reduced in the process. The intrusion of tourists into these places make it difficult for the turtles
Y

to lay their eggs. They have to look elsewhere to places that are still quiet and undisturbed to
DẠ

lay their eggs. Unfortunately, these ideal places are few to come by now. Too much
development has taken place even along the coastline in most countries.
Perhaps all is not lost yet. Sincere efforts are being taken to check the decline. In
Malaysia, it is heartening to note that concrete steps have been taken to protect the turtles that
Page 16 of 20
come ashore to lay eggs. The indiscriminate collection of turtles’ eggs on the beaches is no
more allowed. Turtle sanctuaries have been set up in Rantau Abang in Terengganu. The eggs
collected by designated officials are sent to hatcheries in the sanctuaries. In this way, the loss

L
of eggs and the rate of mortality among the baby turtles are reduced. In other words, more baby

A
turtles are now able to return to sea and grow into adulthood.
In an effort to discourage the public from eating turtle eggs a Turtle Enactment Act has

CI
been introduced to prohibit the sale of leatherback eggs. The WWF has also launched the ‘Save
the Turtle Campaign’ to create an awareness among the public to help save the endangered
species. In this way, the consumption of turtle eggs and perhaps turtle meat will be discouraged,

FI
thus putting a stop to the illegal sale of eggs and trapping of turtles. Let us hope that it is not
too late to save these fascinating creatures from becoming extinct.

OF
Your answer:

ƠN
NH
Y
QU
M

Y
DẠ

Page 17 of 20
Part 2: Graph description (15 points)
The charts below give information about the way in which water was used in different
countries in 2022. Summarize the information by selecting and reporting the main features,

L
and make comparisons where relevant.

A
You should write at least 150 words.

CI
FI
OF
Your answer:
ƠN
NH
Y
QU
M

Y
DẠ

Page 18 of 20
Part 3: Essay writing (30 points)
It is evident that a significant proportion of current high school students engage in
cheating during tests and exams, or consider cheating to be acceptable. Some people blame

L
this trend on the intense academic pressure and excessive workload placed upon students.

A
Others think that it is no more than the problem of students’ dishonesty and insufficient self-
respect.

CI
Discuss both views and state your opinion.
Use specific reasons and examples to support your answer.

FI
You should write at least 350 words.
Your answer:

OF
ƠN
NH
Y
QU
M

Y
DẠ

Page 19 of 20
AL
CI
FI
OF
ƠN
NH
Y
QU
M

- THE END -

Y

GV ra đề: Hà Huy Khánh – Trường THPT Chuyên Sơn La


Số ĐT:
DẠ

Email:

Page 20 of 20
TRƯỜNG THPT CHUYÊN SƠN LA – TỈNH SƠN LA
HƯỚNG DẪN CHẤM
ĐỀ ĐỀ XUẤT KỲ THI CHỌN HSG KHU VỰC DH VÀ ĐBBB

L
NĂM HỌC 2022-2023

A
MÔN: TIẾNG ANH – LỚP 11

CI
(Hướng dẫn chấm có 06 trang)

I. LISTENING (50 POINTS)


Part 1: You will hear part of a discussion in which two business owners, Anya Stern

FI
and Vincent Chambers, are talking about their experiences of launching their own
businesses. For questions 1-5, decide whether the following statements are TRUE (T) or

OF
FALSE (F). (10 points)
1. False (They struggled to adapt and their clothes shop never recovered.)
2. True
3. True
4. True

ƠN
5. False (You should not automatically abandon your plans if you encounter adversity.)

Part 2: Listen to a talk about a new approach in the field of genomics and answer the
NH
questions. For questions 6-10, write NO MORE THAN THREE WORDS taken from the
recording for each answer in the corresponding numbered spaces provided. (10 points)
1. inner workings 2. incremental and slow 3. technology suite
4. proprietary tagging method 5. fuel medical breakthroughs
Y

Part 3: You will hear a discussion in which academics Gordon Mackie and Sophie
QU

Blackmore talk about how communication has changed in society. For questions 11–15,
select the best answer A, B, C or D. (10 points)
11. B 12. B 13. C 14. D 15. A
M

Part 4: For questions 16-25, listen to a presenter talking about a phenomenon in the nature
and complete the summary by writing NO MORE THAN THREE words and/or a number

in each gap. (20 points)


(https://www.youtube.com/watch?v=mLbDbmmV6Qc&t=26s)
16. erosion or leaching 17. gauge 18. algal bloom
19. impenetrable roof 20. feast on 21. suffocate
Y

22. positive feedback loop 23. invasive 24. eutrophication


25. clear-cutting
DẠ

Page 1/10
II. LEXICO AND GRAMMAR (30 POINTS)
Part 1: Choose the answer A, B, C, or D that best completes each of the following
sentences. Write your answers in the corresponding numbered boxes. (20 points)

L
26. A 27. A 28. B 29. C 30. C 31. D 32. D 33. C 34. A 35. B
36. C 37. B 38. A 39. B 40. A 41. C 42. D 43. A 44. B 45. D

A
CI
Part 2: Give the correct form of each given word in CAPITAL to complete the following
sentences. Write your answers in the numbered boxes. (10 points)
46. deactivated 51. pursuit

FI
57. unspeakably 52. deplatformed
48. qualifiedly 53. destabilizing
49. inferiority 54. Hyperactive

OF
50. all – star 55. interoperability

III. READING (60 POINTS)


Part 1: Read the text below and think of the word which best fits each space. Use only
ONE word in each space. Write your answers in the space provided. (15 points)

ƠN
56. put 57. mind 58. against 59. undertaken 60. body
61. regularity 62. fewer 63. subsequent 64. state 65. speak

Part 2: Read the article below and choose the answer (A, B, C or D) which you think fits
NH
best according to the text. (10 points)
OIL AGAINST CARIBOU…
66. A 67. A 68. A 69. C 70. D 71. B 72. B 73. A 74. A 75. C

Part 3: Read the passage and do the tasks that follow. (13 points)
Y

76. False. In the third sentence of Paragraph One the opposite is said – the media gives
QU

‘frequent and often vivid’ coverage of deforestation.


77. False. Third sentence of Paragraph one mentions children probably having formed their
own opinion on the problem of deforestation ‘independent of any formal tuition’, i. e. the
information that influenced their opinions didn’t come from their classrooms, so the opposite
is true.
78. True. First sentence of the second paragraph uses a rather difficult phrasing that contains
M

the answer: children ‘harbour misconceptions’ about the pure science taught at school. To
harbour misconceptions about something is to have wrong ideas about it.

79. True. Sentence two of Paragraph Two phrases the idea in a complicated way. The ideas
about ‘pure’ science are not isolated, but are a part of some bigger system of knowledge. This
fact makes it easier to change these ideas, they are ‘accessible to modification’.
80. False. Paragraph Four, second sentence mentions ‘open-form questions’, which is the
Y

direct opposite of yes/no questions. Therefore, False should be used for the answer. Remember
that when the opposite statement is true, then the answer should be ‘False’.
DẠ

81. Not given. Paragraph Six focuses on the differences in responses between boys and girls.
However, it does not make a comparison of how likely boys or girls are to have mistaken
views.

Page 2/10
82. True. Paragraph Six, second sentence confirms that this study follows the series of studies
focused on the importance of rainforests and how school pupils understand it.
83. Not given. No such or similar information can be found in the text.

L
84. M. Last sentence of Paragraph Four mentions Africa as the most prevalent answer.
85. E. Paragraph Four, the second sentence points out the popular opinion of children about

A
rainforests providing habitats for animals.
86. G. Paragraph Seven, sentence two says that number one reason of rainforests destruction

CI
according to pupils is human activity.
87. P. Paragraph Nine focuses on the popular misconception about potential volume of oxygen
which wouldn’t be produced were the rainforests completely destroyed. Hence the idea of why

FI
it is so important to protect the rainforests. First sentence of Paragraph Ten can also be used to
answer the question.

OF
88. J. Second sentence of Paragraph Ten has the author of the text surprised by how few of the
pupils are aware of the possible threat of global warming associated with deforestation.

Part 4: Read the text. Seven paragraphs have been removed. Choose from the paragraphs
A- H the one which fits each gap (89-95). There is ONE extra paragraph which you do

ƠN
not need to use. (0.7 pt)
Would I lie to you?

89. E 90. B 91. F 92. A 93. C 94. H 95. D


NH
Part 5: You are going to read a set of science book reviews. For questions 96–105, choose
from the reviews (A-D). The reviews may be chosen more than once. Write your answer
in the numbered boxes. (15 points)
Prairie Fever
Y

A new book chronicles the efforts of 19th century British aristocrats to create a corner of
England in the American west.
QU

96. C 97. A 98. D 99. D 100. D


101. B 102. E 103. C 104. C 105. B

IV. WRITING (60 POINTS)


M

Part 1: Read the following extract and use your own words to summarize it. Your
summary should be about 100-120 words long. (15 pts)

Students at least mention to two ideas:


1. How and why the number of leather back turtles has declined
2. What steps have been taken to prevent the decline
Y

Suggested Summary:
DẠ

The declining landings of turtles in Malaysia are due to uncontrolled catching for their meat,
shells and eggs, many are inadvertently caught in deep sea fishing nets. Pollution of the sea
has also reduced the number of turtles. Many die of suffocation by discarded plastic bags.

Page 3/10
Besides, turtles are most vulnerable when they come ashore to lay eggs. Their nesting places
have also been reduced due to development for tourism along the coasts. Nevertheless,
concrete steps have been taken to arrest the decline. Indiscriminate collection and sale of turtle

L
eggs are banned. Turtle sanctuaries have been set up, thereby reducing the rate of mortality
among the baby turtles. Lastly, an awareness campaign has been launched by WWF to

A
discourage the public from eating turtle eggs. (125 words)

CI
Part 2: Graph description (15 points)
The charts below give information about the way in which water was used in different
countries in 2022. Summarize the information by selecting and reporting the main features,

FI
and make comparisons where relevant.
You should write at least 150 words.

OF
ƠN
NH
Y

.
Suggested Description:
QU

The two charts give figures for water use in different parts of the world in 2000. The
first indicates that almost three-quarters of world consumption (70%) was for agriculture,
while 22% was used for industry and a mere 8% for domestic purposes. This pattern is almost
identical to that for China in 2022, whereas South Africa used even more water (92%) for
M

agriculture and only 8% for industrial and domestic sectors. In contrast, New Zealand used
almost equal proportions for agriculture and household use, 44% and 46% respectively, and

a slightly higher 10% was consumed by industry. The pattern in the USA is almost the reverse
of the world average, with a mere 8% of water consumed by agriculture and a massive 80%
by industry Only 12% was used by the domestic sector, which was almost a quarter of the New
Zealand industrial consumption. Overall, the data show that water use in the two developing
Y

countries is closer to the world patterns of consumption. (161 words)


DẠ

Page 4/10
Part 3: Essay writing (30 points)
It is evident that a significant proportion of current high school students engage in
cheating during tests and exams, or consider cheating to be acceptable. Some people blame

L
this trend on the intense academic pressure and excessive workload placed upon students.
Others think that it is no more than the problem of students’ dishonesty and insufficient self-

A
respect.

CI
Discuss both views and state your opinion.
Use specific reasons and examples to support your answer.
You should write at least 350 words.

FI
MARKING SCALES:

OF
The mark given to Part 3 is based on the following criteria:
1. Organization (5 pt)
a. Ideas are well organized and presented with coherence, cohesion and unity.
b. The essay is well-structured:

ƠN
* Introduction is presented with clear thesis statement.
* Body paragraphs are written with unity, coherence and cohesion. Each body
paragraph must have a topic sentence and supporting details and examples where
NH
necessary.
* Conclusion summarizes the main points and offers personal opinions on the issue.
2. Content (15 pt)
a. All requirements of the task are sufficiently addressed.
Y

b. Ideas are adequately supported and elaborated with relevant and reliable
QU

explanations, examples, evidence....


3. Language use (10 pt)
a. Demonstration of a variety of topic-related vocabulary.
b. Excellent use and control of grammatical structures (verb tenses, word forms,
M

voice...) and mechanics (spelling, punctuations...).


Suggested Essay

Cheating among high school students has become a noticeable and concerning trend in recent
years. A considerable number of students are either actively participating in cheating during
tests and exams or perceive it as a normal practice. This issue has sparked a debate among
experts and observers, with divergent opinions emerging regarding its underlying causes and
Y

implications.
DẠ

One school of thought attributes the prevalence of cheating to the intense academic
pressure and overwhelming workload experienced by students. In today's competitive
educational environment, students face immense pressure to excel academically and secure

Page 5/10
their future. This pressure often leads to a mindset where the end justifies the means, making
cheating appear as a viable option to achieve desired outcomes. Moreover, the heavy burden
of assignments, projects, and exams leaves students feeling overwhelmed and drained, pushing

L
some towards cheating as a means of coping with the workload.

A
On the other hand, some argue that cheating among students is primarily a reflection

CI
of their dishonesty and lack of self-respect. According to this viewpoint, the problem lies within
the moral values and ethics instilled in students. They believe that a strong emphasis on
character development and integrity should be in place to deter cheating behavior. Supporters

FI
of this perspective argue that students who possess a strong sense of self-respect and moral
principles are less likely to resort to cheating, even under high-pressure circumstances.

OF
In my opinion, both factors contribute to the prevalence of cheating among high school
students. The intense academic pressure and heavy workload undoubtedly create an
environment where cheating seems like an attractive option. However, the issue of dishonesty
and insufficient self-respect cannot be ignored either. It is crucial to address both aspects to

ƠN
effectively tackle this problem.
Educational institutions should adopt measures to alleviate academic pressure by
promoting a balanced approach to learning. This can be achieved by implementing well-
designed curriculum structures that prioritize understanding and critical thinking over rote
NH
memorization. Additionally, teachers and educators must emphasize the importance of ethical
conduct and foster a culture of integrity within the school community. Encouraging open
discussions about the consequences of cheating and providing guidance on time management
and study skills can also play a significant role in reducing cheating incidents.
Y

In conclusion, the prevalence of cheating among high school students is a complex issue
QU

with multiple contributing factors. While academic pressure and heavy workloads certainly
play a role, addressing the problem of dishonesty and promoting self-respect is equally
important. By adopting a comprehensive approach that combines academic reform and
character development, we can create an environment where cheating is discouraged, and
students are encouraged to pursue success with integrity.
M

-THE END-

GV ra đề: Hà Huy Khánh – Trường THPT Chuyên Sơn La


Số ĐT: 0912.453.903
Email: [email protected]
Y
DẠ

Page 6/10
LISTENING TRANSCRIPTS
(Transcripts có 4 trang)
Part 1:

L
Interviewer Today we’re talking about business start-ups. I’m joined by Anya Stern, who set up
the Pepper chain of restaurants, and Vincent Chambers, an IT entrepreneur. Anya, where does

A
your entrepreneurial drive come from? Was that encouraged at an early age?

CI
Speaker 1 Well, ‘influenced’ might be a better word. I passively absorbed useful business
insights from my parents. Of course, it was mainly things I’d only appreciate much later. Like
the importance of doing things your own way rather than worrying about what other people are

FI
doing. But one thing has stuck with me since my childhood. I saw my parents struggling to adapt
when e-commerce came in. They put in incredibly long hours to salvage their clothes shop but
sadly, it never recovered. It showed me that effort alone isn’t enough in business. That

OF
sparked my curiosity about why some businesses work and others fail.
Speaker 2 Unlike Anya, I didn’t have any business influences in my formative years.
Entrepreneurs were just the people I’d seen on TV making million-dollar deals buying and
selling things! It didn’t dawn on me that there could be IT entrepreneurs, or that I didn’t have to

ƠN
fit a specific mould to start my own business. That’s why I went down the standard route of
working for a large tech giant. Admittedly, it ensured my financial security. And of course, I
amassed a lot of invaluable business skills too. But the hardest obstacle was still convincing
myself that people like me could run their own business.
NH
Speaker 1 Vincent’s point about TV is interesting. There used to be limited opportunities to see
business portrayed onscreen. Even now, many programmes purport to be about business, but
they’re not really. I’m talking about reality competitions where contestants are trying to win
investment. I can see how young people might tune in, especially if there are contestants, they
personally relate to. But the contestants do things they’d never get away with in the real world.
Y

Presumably, the content is manipulated by the producers for the sake of entertainment. But how
QU

many people with great business ideas are discouraged because of the behaviour they see
on these shows? I’d prefer broadcasters to focus on more inspirational business programmes.
Interviewer But what is it that inspires people to want to get into business in the first place?
Speaker 1 One key factor’s probably wanting to be your own boss. We all want a say in how
things are managed. But it depends on the individual. I’ve even met people who launched
M

businesses as a hobby after they’d retired and their children had left home. For me, I was
desperate to bring Brazilian cuisine to the mass market because I felt the restaurant sector

was ready for something new.


Speaker 2 Yes, many entrepreneurs are spurred on by spotting an opportunity. For instance, I
wanted to devote my time to virtual reality, which was really compatible with my corporate role.
That was my personal tipping point, and I could see potential to transform IT. I hadn’t
Y

formulated any specific plan at that point, but it felt like it was too important an opportunity to
pass up.
DẠ

Page 7/10
Speaker 1 That’s exactly the kind of thinking that makes an entrepreneur. Anyone can have a
unique business proposition, but not everyone’s brave enough to take the plunge, or deal with the
negative feedback.

L
Speaker 2 Yes, because even if your business model isn’t particularly innovative, there’ll be

A
times when you encounter adversity. So above all, you must be ready to respond to issues as
they arise. I’m not saying you should automatically abandon your plans, but be prepared

CI
to analyze things logically and without emotion. Then you can deal with whatever comes
your way.
Interviewer And I suppose…

FI
Part 2:
Scientists have made incredible advances in recent years. Yet there's still much they don't understand

OF
about the inner workings of biology. Even after mapping the human genome, for example an
international effort that took more than a decade to complete scientists still don't know why some
people with a particular variation in their DNA develop cancer and others with the same variation
don't. One reason for the limited understanding is that the tools scientists have been using to study

ƠN
biological systems like cells or tissues are themselves limited. Tools that offer the ability to closely
examine biological samples at high resolution don't allow scientists to see the bigger picture. While
tools that show the bigger picture lack the resolution necessary to reveal the true biology of a sample.
Neither approach is ideal, and the work of piecing together the results is incremental and slow. What
a scientist really needs is a clear, full view of a biological system from the start. And that's precisely
NH
what Ten x genomics offers new perspectives on the inner workings of biology that can quickly lead
to new insights. Our technology suite enables scientists to study biological systems at an unprecedented
level of resolution. With our instrument and kits, a scientist can separate and encapsulate tens of
thousands of cells from a biological sample into individual droplets. Next, using a unique chemical
process and proprietary tagging method It, the scientists can capture DNA, RNA, protein and
Y

immunological information from individual cells. A sequencer then decodes the information. It.
QU

Finally, our software uses this decoded information to build an interactive reconstruction of the
biological sample for interpretation. These combined innovations represent a dramatic shift in the scale
and resolution of information that can be derived from the sequencing process, and they advance our
mission to accelerate research in critical areas of biology and fuel medical breakthroughs. A in fact,
Ten X Genomics has already helped scientists uncover important biological processes and identify
previously unrecognized cell types. These discoveries, and others sure to follow will lead to better
M

technologies, better treatments, and a better future.


Part 3:
Interviewer Today we’re discussing language, and the role it plays in society. I’m joined by
Professor Gordon Mackie and Dr Sophie Blackmore, from Belmont College. Gordon,
communication is at the heart of your latest book, isn’t it?
Speaker 1 Exactly so. You only need to watch news reports or read newspaper articles from just
Y

a few years ago to see that communication has changed dramatically in the last few decades. And
I’ve obviously devoted a lot of the book to digital communication. We simply cannot ignore how
DẠ

it’s transformed the way we express ourselves, and even the slang we use. But the point isn’t
that these changes are having an adverse impact on language. Not at all. My mission is to
show how that view’s a myth because, if anything, our language has never been richer.

Page 8/10
Speaker 2 It sounds fascinating! So often in our academic work we deal with the obscure and
highly theoretical aspects of linguistics rather than highlight the beauty of everyday
communication.

L
Speaker 1 Thanks, Sophie. Yes, and what you say about the nature of academic research is
particularly true in my field of expertise, historical linguistics. I spend my time exploring how

A
the so-called rules of grammar and spelling have evolved, and the origins of certain taboo
expressions. But it’s important that I never lose sight of my primary task, which is to gather

CI
evidence of the type of language being used at particular times. My role isn’t to suggest how
language should be, it’s to describe how it is, or in my case, how it once was. I preserve
documents, not language usage!

FI
Speaker 2 My role is somewhat different. I do discuss the features that make up effective
communication because I teach essay-writing classes. Many university students have a tendency
to confuse adopting an appropriate tone with introducing an unnecessarily complex style. Now

OF
clearly, university essays should fulfil the conventions of academic writing, and be error-free.
However, those things are important only insofar as they help you achieve the key goal,
which is to be understood. You may have persuasive arguments, clear evidence, maybe even
ground-breaking theories, and that’s great. But unless they’re expressed in way that is
accessible to the reader, your writing cannot be deemed a success.

ƠN
Speaker 1 And you also research pragmatics, don’t you? You’ve published a lot on the nature of
social interaction.
Speaker 2 That’s right. Writers choose which information to present, and whether to convey it in
an emotional or neutral way. They decide how best to engage the reader. Then, the reader
analyses that information and compares it with their existing knowledge. I highlight that point in
NH
my classes using the example of social media. When we post something to our network, our
followers are already aware of the background. Our intended audience will immediately
see the point we’re making without us joining all the dots for them. But a stranger might
see the same post and interpret it in a completely different way.
Speaker 1 Then there’s text language, which has its own particular style and structure. I know
it’s frowned upon by schools currently, but it’ll eventually be studied in the same way that pupils
Y

study any other form of writing. After all, it’s very inventive. And it’s remarkable how universal
QU

standards are being adopted within text language. For instance, young people from different
cultures can use instant messaging and text language to understand one another without language
barriers.
Speaker 2 Yes, it’s evolved very quickly. But do you really envisage it appearing on a school
syllabus? I can’t see any prospect of that. And that’s a shame because of course, text language
is increasingly accepted by more of the population.
M

Interviewer And do you…


Part 4:
Plants require several things to grow. Most of us know that they need things like water, sunlight, and
CO2, but plants also need a variety of nutrients as well, things like nitrogen and phosphorus. Usually,
plants will get all their required nutrients from the soil through their roots, but when the soil is bad or
Y

there's been a lot of erosion or leaching, farmers, or even just people trying to make their lawns green,
will put down something called fertilizer. What makes fertilizer so fertile is that it's been enriched with
DẠ

these nutrients that the plants need, again mostly fixed nitrogen and phosphorus. But plants aren't the
best at soaking up every last nutrient in the soil, and it's also hard to gauge just how much fertilizer a
field needs as soil quality can vary drastically over short distances. To be safe, farmers will usually

Page 9/10
apply excess fertilizer to a given plot of land, better safe than sorry, you know? But instead of staying
in the soil for years, most of the excess nutrients will be carried away by the rain or other forms of
irrigation. These nutrients mix with the water and find their way into bodies of water, like ponds, lakes,

L
reservoirs, and even the ocean sometimes. With all these nutrients added, the algae, phytoplankton,
and even plants in the water do the same thing the crops in the field do. They grow. Well actually, they

A
explode in numbers. This is called an algal bloom. And entire lakes can become covered in layers of
plant growth like this. To sum, this might seem like a good thing. Plants are good for the environment,

CI
right? Well, not always. First off, this floating layer of algae forms basically an impenetrable roof on
the water, not allowing sunlight through to the bottom of the lake. Without the presence of sunlight,
all plants below the surface cannot partake in photosynthesis. You know, metabolize, make glucose,

FI
live, that sort of stuff? But not even this is the bad part of it all. Many plants can store enough energy
in their bodies to weight out these conditions. The real problem comes when all the nutrients are used
up and the water can no longer support so much life. When this happens, the excess algae,

OF
phytoplankton, and plants die off and sink to the bottom of the body of water. Here, bacteria and other
decomposers feast on the dead bodies in a chemical process of decay which consumes oxygen. Now,
in a usual ecosystem, the amount of dead matter is relatively constant, so oxygen levels stay relatively
constant as well. But when a bloom occurs, far more organic matter is ready to decompose, and so
nearly all the oxygen in the water is used in the process of decomposition, and none is left for the

ƠN
animals living in the water. Without this, animals that use the dissolved oxygen to breathe, so things
like fish, can actually suffocate. This causes even more death, leading to more decomposition and more
oxygen usage. Basically, at this point, a positive feedback loop has been created. It can take a body of
water a very long time to recover, though each one is different, and recovery depends on a lot of things,
like how many nutrients leaked into the water, how big the body of water is, what organisms are present
NH
there, and so on. When this happens in lakes, native species can be suppressed and allow invasives to
come in while the environment is still disturbed. If this happens in the ocean, the lack of oxygen can
cause corals to bleach and possibly even die. All around, this can greatly damage many ecosystems
and leads to a decrease in biodiversity globally. Nutrient -⁠rich runoff can also be caused by things like
clear cutting, which releases the nutrients which were kept in the soil by the plants, or also by things
like animal farms where nutrient -⁠rich waste materials can leak into local bodies of water. And that's
Y

eutrophication, simply put. I hope you enjoyed the video and maybe think twice before using fertilizer
QU

or clearing a forest. If you want more people to see this video, give it a like, and if you want to see
more.

THE END
M

Y
DẠ

Page 10/10
SỞ GD & ĐT CAO BẰNG ĐỀ XUẤT ĐỀ THI HỌC SINH GIỎI

L
TRƯỜNG THPT CHUYÊN KHU VỰC DUYÊN HẢI – ĐBBB NĂM 2023

A
------------------------- Môn: Tiếng Anh – Lớp 10
A. LISTENING (50 points)

CI
Part 1: Complete the table below. Write NO MORE THAN THREE WORDS for each
answer. Write your answers in the corresponding numbered boxes. (10 points)
Date Event Importance for art

FI
3000 BC rice farmers from 1. ________ built temples with wood and stone
carvings settled in Bali

OF
14th century introduction of Hinduism artists employed by 2. ________ and
focused on epic narratives
1906 Dutch East Indies Company art became expression of opposition to
established 3. ________
1920s beginning of 4. ________ encouraged use of new materials,

ƠN
techniques and subjects
1945 independence new art with scene of 5. _________ (e.g.
harvests) reflecting national identity
Your answers:
NH
1. 2. 3. 4. 5.

Part 2. You will hear part of a discussion between Velm and Andrews, a lawyer, and
Sergeant William Bailey, a police officer. For questions 1-5, choose the answer (A, B,
Y

C, or D) which fits best according to what you hear. (10 points)


1. How did William feel the first time he gave evidence in court?
QU

A. humiliated B. nervous C. furious D. indifferent


2. Velm suggests that police officers giving evidence should ……….
A. study the evidence more carefully.
B. ignore the lawyer for the defense.
C. not take comments personally.
M

D. demonstrate that they are honest and reliable.


3. Velma compares a police officer's evidence to a piece in a jigsaw puzzle because ……….

A. it is unimportant unless it is part of a bigger picture.


B. it may not fit in with the rest of the evidence.
C. the defense lawyer will try to destroy it.
D the police officer should only talk about his or her evidence.
4. William suggests that lawyers ………….
Y

A adopt a special manner in the courtroom.


DẠ

B. can be detached from a case.


C. might actually be close friends.
D. do not take their work seriously.

1
5. William's main concern is that
A. a criminal could get away with his or her crime.

L
B. a court case could be confusing,

A
C. young police officers find courts terrifying.
D. police officers might argue with the lawyer.

CI
Your answers:
1. 2. 3. 4. 5.

FI
Part 3: You will hear two students, Bella and Tom, discussing an article they have
read about a woman astronaut. For questions 1-5, decide whether the following
statements are True (T) or False (F). (10 points)

OF
1. The speakers agree that being an astronaut is an unexpected job for women.
2. Bella particularly admires the astronaut Ellen Ochoa because of her determination to
fulfill a childhood dream.
3. Tom was surprised to learn that people who want to become astronauts should have

ƠN
experience as airplane pilots.
4. Bella thinks the most interesting part of Ellen’s life is coping with unexpected problems.
5. Tom and Bella both now decide to go to some talks on space travel in films and literature.
Your answers:
1. 2. 3. 4. 5.
NH
Part 4: For questions 1-10, listen to a piece of news from BBC about “What to wear?”
and supply the blanks with the missing information. Write NO MORE THAN
THREE WORDS and/or A NUMBER taken from the recording for each answer in
the spaces provided. (20 points)
Y

One of the biggest shocks, when you arrive in a new country, can be the clothes people are
QU

wearing. You may look fashionable at home, but you suddenly find you are
1____________________or simply someone to laugh at when you arrive abroad. With this
in mind, let's take a look at teenage fashion in the UK for girls.
One of the things that may shock an outsider most is piercings. These days it is not enough
to simply wear rings in your ears. You will see many teenagers with rings on their navel,
M

belly button, nose, lip, or even their eyebrows.


Some girls go for a 'glam' look. They wear T-shirts; trousers are usually preferred, blue or
black, and the look is 2_____________________ with metallic bags and shoes and arms

full of bracelets. Another alternative is the 3________________________ You start with


a T-shirt of your favorite band and tight jeans or a long skirt. On top of this, you can wear
a denim jacket. Jewellery tends to be large and metallic and to add color, wear a scarf.
If neither of these is for you, why not 4_________________________? T-shirts are
Y

usually tie-dyed in hot colors. Wear long shorts, short jeans or a denim skirt. And on your
feet? 5___________________________, of course! If you prefer something more
DẠ

feminine, there's the 6___________________________ Skirts are long, to the floor. Wear
a top with butterflies or flowers printed on it!

2
Finally, how about the 'Tom Boy' look? Wear 7________________________ and a T-shirt
with a logo. Don't forget your 8_______________________ of course!

L
Follow the fashion tips above, and you shouldn't 9______________________However, it's

A
important to remember to wear clothes and choose a look that you feel comfortable with.
Don't just be 10_______________- be yourself!

CI
Your answers:
1. 2. 3. 4. 5.

FI
6. 7. 8. 9. 10.

OF
B. LEXICO-GRAMMAR (40 points)
Part 1: For questions 1 - 20, choose the correct answer (A, B, C, or D) to each of the

ƠN
following questions and write your answers in the corresponding numbered boxes. (20
points)

1. In ____________, with the benefits of hindsight, it is clear that this was a bad decision.
NH
A. reflection B. retrospect C. status quo D. a second thought
2. Little did I imagine The Amazing Race would entail a long-winded journey and ups and
downs ____.
A. aplenty B. inexhaustibly
C. profusely D. superabundant
Y

3. None of his relatives really understood the full ____________ of his difficulty or his
desperation.
QU

A. degree B. dimension C. extent D. width


4. It’s quite unreasonable to ask me to organize the conference at such short ____________.
A. warning B. notice C. time D. advance
5. I slept badly last night and am feeling particularly ____________ this morning.
A. slow-witted B. far-reaching C. off-hand D. top-heavy
M

6. A few tears were _____________ by both parents when Maria finally left home.
A. flowed B. poured C. shed D. streamed

7. John did nine hours’ ____ studying a day for his exam.
A. strong B. solid C. heavy D. big
8. I suppose I____ lent the book to Simon, but I’m almost sure I didn’t.
A. might B. could C. might have D. must have
9. People were injured by ____ stones the size of tennis balls.
Y

A. sleet B. gale C. hail D. snow


10. The police ____ off the street when the bomb had gone off.
DẠ

A. cordoned B. battened C. fastened D. shuttered

3
11. Roger Williams was a clergyman, ______ the colony of Rhode Island, and an outspoken
advocate of religious and political freedom.

L
A. founded B. the founder of C. was the founder of D. the founded

A
12. ______ at home requires only three types of chemicals, several pieces of simple
equipment, and running water.

CI
A. For the development of film B. To develop film
C. When film is developed D. In developing film
13. The bark of a tree thickens ______.

FI
A. with age B. it gets older C. as older D. by age
14. Widely reproduced in magazines and books, ______.
A. Ansel Adams depicted the Western wilderness in his photographs.

OF
B. the Western wilderness was depicted in the photographs of Ansel Adams.
C. Ansel Adams’ photographs depicted the Western wilderness.
D. it was through his photographs that Ansel Adams depicted the Western wilderness.
15. This new glue is very useful for small repairs as it ______ very rapidly.
A. thickens B. stiffens C. sets D. fixes

ƠN
16. Advertisers often aim their campaigns at young people as they have considerable
spending ______.
A. power B. force C. energy D. ability
17. We’ve bought some ______ chairs for the garden so that they are easy to store away.
NH
A. adapting B. adjusting C. bending D. folding
18. Alan’s photo was slightly too large for the frame so he decided to ______ it.
A. hack B. chop C. slice D. trim
19. Although the patient received intensive treatment, there was no ______ improvement
in her condition.
Y

A. decipherable B. legible C. discernible D. intelligible


20. The painting was a valuable family possession, which had been ___________ from
QU

generation to generation.
A. handed over B. handed down C. handed across D. handed out
M

Your answers:
1. 2. 3. 4. 5.

6. 7. 8. 9. 10.
11. 12. 13. 14. 15.
16. 17. 18. 19. 20.
Y
DẠ

Part 2: Write the correct form of each bracketed word in corresponding numbered
boxes. (10 points)

4
1. A UN force has been sent in to try and ___________ the area worst affected by the civil
war. (PEACE)

L
2. During puberty, my children showed increasing signs of mental ________, which raised

A
lots of concern for their parents. (STABLE)
3. In fact, those who practice meditation with any________ see their doctors less and

CI
spend, on average, seventy percent fewer days in the hospital. (REGULAR)
4. There’s been yet another____________ of cholera in Delhi. (BREAK)

FI
5. Sarah stood there completely ________, so I had no idea at all what she was thinking.
(EXPRESS)

OF
6. The upper layers of Earth’s oceans have cooled ________ over the past two years, even
though the planet as a whole is warming up. (SIGNIFY)
7. This soft drink has a very fruity flavor to it and a_____after-taste. I really like it.

ƠN
(PLEASE)
8. ________ as it may seem, mammoths were alive only five thousand years ago.
(CREDIT)
9. The judge was forced to acquit, saying the evidence was wholly_________.
NH
(CONCLUDE)
10. Can you think of one______ reason why I should give you your job back?
(SOLITUDE)
Your answers:
Y

1. 2. 3. 4. 5.
QU

6. 7. 8. 9. 10.
Part 3: Fill in the blank with a suitable preposition or a particle to complete the following
sentences. (10 points)
1. Text messaging caught_____ because kids wanted to pass notes to each other during
M

class.
2. It is true that she plans to give _____ her prize money to charity.

3. If you act _____impulse, you risk making mistakes.


4. Don’t let one spoilt meal put you _____ cooking forever!
5. I got the job _______ the strength of your recommendation.
6. You must be weak _______ the head if you believe that.
Y

7. Prospects of success in the talks were put _______ zero.


8. His appearance was the subject _______ some critical comments.
DẠ

9. She went _______ the roof when I told her I’d crashed her car.
10. The government’s decision is a real kick _______ the teeth for the unions.
Your answers:

5
1. 2. 3. 4. 5.
6. 7. 8. 9. 10.

L
C. READING (60 points)

A
Part 1: Read the following passage and mark the letter A, B, C or D on your answer

CI
sheet to indicate the best answer to each of the following questions. (15 points)
In the past, not a very long time ago, most people traveled on foot, by train, or on

FI
horseback. (1) _____ had made it possible to travel rapidly over long distances. Bicycles
were also becoming (2) _____, after the invention of the air- filled (3) _____, which made

OF
cycling a lot more comfortable. Buses, trams, and underground railways had already been
invented, and cities all over the world already had traffic (4) _____. There were very few
private cars, and city streets were still full of horses. What a difference a hundred years

ƠN
have (5) _____! Nowadays we have got (6) _____to the problem of private cars, and some
cities are so noisy and (7) _____ that in many places (8) _____ have been banned from the
city centre. How will we be travelling in a hundred year’s time? Perhaps by then there will
NH
be only personal helicopters. There may be no need to (9) _____ to work or school in the
future, (10) _____everyone will have a computer at home. There might even be more
people walking and horse – riding, for pleasure and exercise.
Y

Question 1. A. Tracks B. Railways C. Ways D. Lines


QU

Question 2. A. then B. invented C. popular D. handlebars


Question 3. A. boot B. brake C. engine D. tyre
Question 4. A. blocks B. sticks C. knots D. jams
Question 5. A. taken B. done C. made D. got
M

Question 6. A. more B. them C. motorists D. used


Question 7. A. even B. so C. polluted D. poisoned


Question 8. A. traffic B. vehicles C. transport D. trips
Question 9. A. have B. transport C. decide D. commute
Y

Question 10. A. if B. since C. when D. unless


Your answers:
DẠ

1. 2. 3. 4. 5.
6. 7. 8. 9. 10.

6
Part 2: Fill in the blank with one suitable word. (15 points)

L
Of course, we all think our children are the best and brightest in the class, but have you
ever seriously thought that your child might be highly intelligent? Does he or she (1)

A
________ out in class as being far ahead of his or her classmates? How can we be sure?

CI
Child psychiatrists and educationists have compiled a profile of (2) ________ gifted
children so that parents can have the opportunity to see for themselves which characteristics
are true for their own offspring and then take the appropriate (3) ________. Have a look at
the profile below (4) ________ we hope will help you to form a truer opinion of your own

FI
child's intelligence.
Firstly, these children have an unusually wide vocabulary, enjoy reading and show (5)

OF
________ about the world and what makes things work. Secondly, they are perfectionists,
have the ability to work (6) _________, and are interested in adult topics such as religion,
war, politics, and so on.
Furthermore, highly gifted children are sensitive, creative, and adventurous. They have a
good sense of humor and usually have the ability to look (7) ________ the bright side of a

ƠN
difficult situation. They often show a desire to improve themselves and they dislike
conforming just for the (8) ________ of it. Finally, they are self-confident - especially in
the company of adults - adaptable and responsible.
All of the above characteristics (9) ________ to making up what is seen as a highly gifted
NH
child. If you feel that your child shows this particular combination, then it is advisable to
get in (10) __________ with one of the organizations that exist to give guidance to parents
where they will no longer feel "different " but, possibly for the first time, on equal footing
with their peers.
Your answers:
Y

1. 2. 3. 4. 5.
QU

6. 7. 8. 9. 10.
Part 3: Read the text and choose the answer (A, B, C or D) which you think fits best
according to the text. (15 points)
M

The attraction of valuable objects from ships sinking in the oceans is always great.
Until recently, hunting for treasure from shipwrecks was mostly fantasy; with recent

technological advances, however, the search for sunken treasure has become more popular
as a legitimate endeavor. This has caused a debate between those wanting to salvage the
wrecks and those wanting to preserve them.
Y

Treasure hunters are spurred on by the thought of finding caches of gold coins or
DẠ

other valuable objects on a sunken ship. One team of salvagers, for instance, searched the

7
wreck of the RMS Republic, which sank outside the Boston harbor in 1900. The search

L
party, using side-scan sonar, a device that projects sound waves across the ocean bottom

A
and produces a profile of the sea floor, located the wreck in just two and a half days. Before
the use of this new technology, such searches could take months or years. The team of 45

CI
divers searched the wreck for two months, finding silver tea services, crystal dinnerware,
and thousands of bottles of wine, but they did not find the five and a half tons of American

FI
Gold Eagle coins they were searching for.

OF
Preservationists focus on the historic value of a ship. They say that even if a
shipwreck’s treasure does not have a high monetary value, it can be an invaluable source
of historic artifacts that are preserved in nearly mint condition. But once a salvage team
has scoured a site, much of the archaeological value is lost. Maritime archaeologists who

ƠN
are preservationists worry that the success of salvagers will attract more treasure-hunting
expeditions and thus threaten remaining undiscovered wrecks. Preservationists are
NH
lobbying their state lawmakers to legally restrict underwater searches and unregulated
salvages. To counter their efforts, treasure hunters argue that without the lure of gold and
million-dollar treasures, the wrecks and their historical artifacts would never be recovered
at all.
Y

Question 1: What is the main idea of this passage?


QU

A. The popularity of treasure-seeking has spurred a debate between preservationists and


salvagers.
B. Maritime archaeologists are concerned about the unregulated searching of wrecks.
M

C. The search of the RMS Republic failed to produce the hoped-for coins.
D. Searching for wrecks is much easier with new technologies like side-scan sonar.

Question 2: The word “sunken” is closest in meaning to which of the following words?
A. underwater B. broken C. ancient D. hollow
Question 3: Which of the following could best replace the phrase “a profile” in the
Y

passage?
DẠ

A. a projection B. an execution C. an outline D. a highlight


Question 4: Which of the following statements is best supported by the author?

8
A. The value of a shipwreck depends on the quantity of its artifacts.

L
B. Preservationists are fighting the use of technological advances such as side-scan sonar.

A
C. Side-scan sonar has helped to legitimize salvaging.
D. The use of sound waves is crucial to locating shipwrecks.

CI
Question 5: The author uses the phrase “mint condition” to describe _____ .
A. something perfect B. something significant

FI
C. something tolerant D. something magical

OF
Question 6: All of the following were found on the RMS Republic EXCEPT _____ .
A. wine bottles B. silver tea services
C. American Gold Eagle coins D. crystal dinnerware
Question 7: From the passage, you can infer that a preservationist would be most likely to

ƠN
______ .
A. shun treasure-seeking salvagers B. be a diver
NH
C. put treasures in a museum D. do archaeological research
Question 8: The word “scoured” is most similar to which of the following?
A. scraped away B. scratched over C. scrambled around D. searched around
Question 9: What is the closest meaning to the word “lure” in the passage?
Y

A. knowledge B. attraction C. luxury D. glare


QU

Question 10: The second and third paragraphs are an example of _____ .
A. chronological order B. explanation
C. specific to general D. definition
M

Your answers:
1. 2. 3. 4. 5.

6. 7. 8. 9. 10.

Part 4: Read the passage and do the tasks that follow. (15 points)
Y
DẠ

The evolutional mystery: Crocodile survives


A

9
Crocodiles have been around for 200 million years, but they’re certainly not primitive. The
early forms of crocodiles are known as Crocodilian. Since they spent most of their life

L
beneath the water, accordingly their body adapted to the aquatic lifestyle. Due to the

A
changes formed within their body shape and tendency to adapt according to the climate,
they were able to survive when most of the reptiles of their period are just a part of history.

CI
In their tenure on Earth, they’ve endured the impacts of meteors, planetary refrigeration,
extreme upheavals of the Earth’s tectonic surface, and profound climate change. They were
around for the rise and fall of the dinosaurs, and even 65 million years of supposed

FI
mammalian dominance has failed to loosen their grip on the environments they inhabit.
Today’s crocodiles and alligators are little changed from their prehistoric ancestors, a
telling clue that these reptiles were (and remain) extremely well adapted to their

OF
environment.
B
The first crocodile-like ancestors appeared about 230 million years ago, with many of the
features that make crocs such successful stealth hunters already in place: streamlined
bodies, long tails, protective armor, and long jaws. They have a long head and a long tail

ƠN
that helps them to change their direction in the water while moving. They have four legs
which are short and are webbed. Never underestimate their ability to move on the ground.
When they move they can move at such a speed that won’t give you a second chance to
make a mistake by going close to them especially when hungry. They can lift their whole
NH
body within seconds from ground. The fastest way by which most species can move is a
sort of “belly run”, where the body moves like a snake, members huddled to the side
paddling away frenetically while the tail whips back and forth. When “belly running”
Crocodiles can reach speeds up to 10 or 11 km/h (about 7mph), and often faster if they are
sliding down muddy banks. Other form of movement is their “high walk”, where the body
Y

is elevated above the ground.


C
QU

Crocodilians have no lips. When submerged in their classic ‘sit and wait’ position, their
mouths filled with water. The nostrils on the tip of the elongated snout lead into canals that
run through bone to open behind the valve – allowing the crocodilian to breathe through
its nostrils even though its mouth is underwater. When the animal is totally submerged,
another valve seals the nostrils, so the crocodilian can open its mouth to catch prey with no
M

fear of drowning. The thin skin on the crocodilian head and face is covered with tiny,
pigmented domes, forming a network of neural pressure receptors that can detect barely

perceptible vibrations in the water. This enables a crocodile lying in silent darkness to
suddenly throw its head sideways and grasp with deadly accuracy small prey moving close
by.
D
Y

Like other reptiles, crocodiles are endothermic animals (cold-blooded, or whose body
temperature varies with the temperature of the surrounding environment) and, therefore,
DẠ

need to sunbathe, to raise the temperature of the body. On the contrary, if it is too hot, they
prefer being in water or in the shade. Being a cold-blooded species, the crocodilian heart
is unique in having an actively controlled valve that can redirect, at will, blood flow away

10
from the lungs and recirculate it around the body, taking oxygen to where it’s needed most.
In addition, their metabolism is a very slow one, so, they can survive for long periods

L
without feeding. Crocodiles are capable of slowing their metabolism even further allowing

A
them to survive for a full year without feeding. Compared to mammals and birds,
crocodilians have slow metabolisms that burn much less fuel, and are ideally suited to

CI
relatively unstable environments that would defeat mammals with their high food demands.
E
Crocodiles use a very effective technique to catch the prey. The prey remains almost

FI
unaware of the fact that there can be any crocodile beneath water. It is due to the fact that
when the crocodile sees its prey it moves under water without making any noise and
significant movement. It keeps only its eyes above water surface. When it feels it has

OF
reached sufficiently close to the target it whistles out of water with wide open jaws. 80
percent of their attempts are successful. They have very powerful jaws. Once the prey
trapped in its jaws they swallow it. Their power can be judged from the fact they can kill
the wild zebras which come to watery areas in search of water. They do not chew their
food. They normally feed on small animals, big fish, birds and even human flesh. As like

ƠN
some water creatures that interact by making sounds crocodiles also use many sounds to
communicate with other crocodiles. They exist where conditions have remained the same
and they are free of human interference. The crocodile is successful because it switches its
feeding methods. It hunts fish, grabs birds at the surface, hides among the water edge
NH
vegetation to wait for a gazelle to come by, and when there is a chance for an ambush, the
crocodile lunges forward, knocks the animal with its powerful tail and then drags it to water
where it quickly drowns. Another way is to wait motionless for an animal to come to the
water’s edge and grabs it by its nose where it is held to drown.
F
Y

In many places inhabited by crocodilians, the hot season brings drought that dries up their
hunting grounds and takes away the means to regulate their body temperature. They
QU

allowed reptiles to dominate the terrestrial environment. Furthermore, many crocs protect
themselves from this by digging burrows and entombing themselves in mud, waiting for
months without access to food or water, until the rains arrive. To do this, they sink into a
quiescent state called aestivation.
G
M

Most of (At least nine species of) crocodilians are thought to aestivate during dry periods.
Kennett and Christian’s six-year study of Australian freshwater crocodiles – Crocodylus

Johnstoni (the King Crocodiles). The crocodiles spent almost four months a year
underground without access to water. Doubly labeled water was used to measure field
metabolic rates and water flux, and plasma (and cloacal fluid samples were taken at
approximately monthly intervals during some years to monitor the effects of aestivation
Y

with respect to the accumulation of nitrogenous wastes and electrolyte concentrations.


Double found that the crocodiles’ metabolic engines tick over, producing waste and using
DẠ

up water and fat reserves. Waste products are stored in the urine, which gets increasingly
concentrated as the months pass. However, the concentration of waste products in the blood
changes very little, allowing the crocodiles to function normally. Furthermore, though the

11
animals lost water and body mass (just over one-tenth of their initial mass) while
underground, the losses were proportional: on emergence, the aestivating crocodiles were

L
not dehydrated and exhibited no other detrimental effects such as a decreased growth rate.

A
Kennett and Christian believe this ability of individuals to sit out the bad times and endure
long periods of enforced starvation must surely be key to the survival of the crocodilian

CI
line through time.

FI
Questions 1-5
The reading passage has seven paragraphs, A-G.

OF
Choose the correct heading for paragraph A-G from the list below.
Write the correct number, i-x, in boxes 1 – 5 on your answer sheet.
List of Headings
i The competitors with the dinosaur
ii A historical event for the Supreme survivors

ƠN
iii What makes the crocodile the fastest-running animal on land
iv Regulated body temperature by the surrounding environment
v Underwater aid in body structure offered to a successful predator
vi The perfectly designed body for a great land roamer
NH
vii Slow metabolisms which make crocodiles unique reptiles
viii The favorable features of the impact of a drought
xi Shifting Eating habits and food intake
x A unique finding has been achieved recently
Example: Paragraph A – ii
Y

1. Paragraph B _____
2. Paragraph C _____
QU

3. Paragraph D _____
4. Paragraph E _____
5. Paragraph F _____
Questions 6 - 10
M

Complete the summary and write the correct answer (NO MORE THAN TWO WORDS
OR A NUMBER) in boxes 6 – 10 on your answer sheet.
In many places inhabited by crocodilians, most types of crocodile have evolved a

successful scheme to survive in the drought brought by a (6)………………………


According to Kennett and Christian’s six-year study of Australian freshwater crocodiles’
aestivation, they found Aestivating crocodiles spent around (7)……………………… a
year without access to (8) ……………………… The absolute size of body water pools
Y

declined proportionately with (9) ………………………; thus there is no sign of (10)


……………………… and other health-damaging impacts in the crocodiles even after an
DẠ

aestivation period. This super capacity helps crocodiles endure the tough drought without
slowing their speed of growth significantly.
Your answers:

12
1. 2. 3. 4. 5.
6. 7. 8. 9. 10.

A L
D. WRITING (50 points)

CI
Part 1: The table below shows the proportion of different categories of families
living in poverty in Australia in 1999. (20 points)

FI
OF
ƠN
NH

………………………………………………………………………………………….…
Y

……………………………………………………………………………………………
QU

…….………………………………………………………………………………………
…………….………………………………………………………………………………
…………………….………………………………………………………………………
…………………………….………………………………………………………………
M

…………………………………….………………………………………………………

…………………………………………….………………………………………………
…………………………………………………….………………………………………
…………………………………………………………….………………………………
Y

…………………………………………………………………….………………………
DẠ

…………………………………………………………………………….………………
…………………………………………………………………………………….………

13
…………………………………………………………………………………………….

L
……………………………………………………………………………………………

A
……….……………………………………………………………………………………
……………….……………………………………………………………………………

CI
……………………….……………………………………………………………………
……………………………….……………………………………………………………

FI
……………………………………….……………………………………………………

OF
……………………………………………….……………………………………………
……………………………………………………….……………………………………
……………………………………………………………….……………………………
Part 2. Essay writing (30 points)

ƠN
Chat GPT is an advanced natural language processing tool developed by OpenAI. It has
gained a lot of popularity since its release in November 2022. Advantageous as it may be,
this tool poses lots of threats to our lives in many aspects.
NH
Write a 250-word essay to discuss both pros and cons of Chat GPT.

………………….…………………………………………………………………………
………………………….…………………………………………………………………
………………………………….…………………………………………………………
Y

………………………………………….…………………………………………………
………………………………………………….…………………………………………
QU

………………………………………………………….…………………………………
………………………………………………………………….…………………………
………………………………………………………………………….…………………
………………………………………………………………………………….…………
………………………………………………………………………………………….…
M

……………………………………………………………………….……………………
……………………………………………………………………………….……………

……………………………………………………………………………………….……
………………………………………………………………………………………………
.……………………………………………………………………………………………
……….……………………………………………………………………………………
……………….……………………………………………………………………………
Y

……………………….……………………………………………………………………
……………………………….……………………………………………………………
DẠ

……………………………………….……………………………………………………
……………………………………………….……………………………………………

14
…………………………….………………………………………………………………
…………………………………….………………………………………………………

L
…………………………………………….………………………………………………

A
…………………………………………………….………………………………………
…………………………………………………………….………………………………

CI
…………………………………………………………………….………………………
…………………………………………………………………………….………………
…………………………………………………………………………………….………

FI
…………………………………………………………………………………………….
………………………………………………………………………………………………
…….………………………………………………………………………….……………

OF
……………………………………………………………………………………….……
………………………………………………………………………………………………
.……………………………………………………………………………………………
……….……………………………………………………………………………………
……………….……………………………………………………………………………

ƠN
……………………….……………………………………………………………………
……………………………….……………………………………………………………
……………………………………….……………………………………………………
……………………………………………….……………………………………………
NH
……………………………………………………….……………………………………
…………………………………….………………………………………………………
…………………………………………….………………………………………………
…………………………………………………….………………………………………
…………………………………………………………….………………………………
Y

…………………………………………………………………….………………………
…………………………………………………………………………….………………
QU

…………………………………………………………………………………….………
…………………………………………………………………………………………….
………………………………………………………………………………………………
…….………………………………………………………………………………………
…………….………………………………………………………………………….……
M

………………………………………………………………………………………………
.……………………………………………………………………………………………

……….……………………………………………………………………………………
……………….……………………………………………………………………………
……………………….……………………………………………………………………
……………………………….……………………………………………………………
Y

……………………………………….……………………………………………………
……………………………………………….……………………………………………
DẠ

……………………………………………………….……………………………………
……………………………………………………………….……………
The end

15
DẠ
Y

M
QU
Y
NH
ƠN
OF
FI
CI
A

16
L
SỞ GD & ĐT CAO BẰNG HƯỚNG DẤN CHẤM

L
TRƯỜNG THPT CHUYÊN ĐỀ XUẤT ĐỀ THI HỌC SINH GIỎI

A
------------------------- KHU VỰC DUYÊN HẢI – ĐBBB NĂM 2023
Môn: Tiếng Anh – Lớp 10

CI
A. LISTENING (50 points)

FI
Part 1. Complete the table below. Write NO MORE THAN THREE WORDS for each answer.
Write your answers in the corresponding numbered boxes. (10 points)
1. China 2. (the) ruling families 3. colonisation 4. tourism 5. everyday life

OF
TAPESCRIPTS
Part 1:
Last week we looked at the traditional art of Japan. In this week’s lecture, we’re going to move

ƠN
south and look at the very special way in which art has developed in the beautiful island of Bali,
which is now part of Indonesia. I’ll begin by giving you a brief historical overview.
It’s thought that the first inhabitants of Bali were farmers who arrived around 3000 BC … at the
beginning of the Iron Age. They probably originally came from China, and in Bali they cultivated
rice and built temples ornamented with wood and stone carvings and statues. The Hindu religion
NH
was introduced in the 14th century AD, and this has remained the main religion on the island. This
was an important period in the artistic development of the island, when sculptors, poets, priests and
painters worked together in the service of the ruling families. Rather than painting everyday scenes,
artists concentrated on narrative paintings illustrating the epic stories of Hinduism. Bali’s rich
natural resources have always made it an alluring goal for merchants, and from the 17th century
Y

onwards, Dutch ships visited the island to trade in spices and luxury goods. Gradually the old royal
QU

families lost their power, and eventually in 1906 the Dutch East Indies Company was founded and
the island became a colony. In the 20th century, art then took on a very different role: as a tool
accessible to everyone in the fight of the Balinese people against colonisation rather than as the
property of a minority. Shortly after this, in the 1920s, stories of the beauty of the island of Bali
began to spread around the world, and Balinese art underwent another vast transformation with
advent of tourism to the island. At first, this was only on a small scale, but it had important effects.
M

Expatriate artists from Holland and Germany settled on the island bringing paper, Chinese ink and
other new materials with them. They worked with local artists, encouraging them to experiment

with concepts like naturalism, expressionism, light and perspective, as well as to move away from
the tradition focus on narrative painting towards something closer to their own experience. When
independence came in 1945, this desire for an art to match a new national identity became stronger
and the traditional narrative paintings started to give way to scenes showing the everyday life of the
Y

Balinese people –harvests, market scenes and daily tasks – as well as the myths and legends of their
history.
DẠ

Part 2. You will hear part of a discussion between Velm and Andrews, a lawyer, and Sergeant
William Bailey, a police officer. For questions 1-5, choose the answer (A, B, C or D) which fits
best according to what you hear. ( 10 points)

1
1. A 2. C 3. D 4. B 5. A

Part 2 (Exam essentials Practice Tests - CAE)

L
You will hear part of a discussion between Velma Andrews, a lawyer, and Sergeant William
Bailey, a police officer. For questions 1-5, choose the answer, A, B, C or D which fits best

A
according to what you hear.
Interviewer: Today on Legal Issues we have Velma Andrews, a lawyer, and Sergeant William

CI
Bailey, a police officer who helps to run a scheme that trains police officers In the art of giving
evidence In court. William, perhaps I can start by asking you why this training scheme is necessary.
William: Well, you must remember that in a criminal case, the police have gathered evidence to

FI
show that someone -the defendant - is guilty of a crime. And the defendant's lawyer is trying to
show that this evidence Is wrong or unreliable. Now, the way the defense lawyer goes about doing
this can be very tricky. For instance, the first time I gave evidence in court 25 years ago, the lawyer

OF
for the defense made me look like a right fool. He annoyed me by interrupting me all the time, and
when I tried to argue with him I got confused, and the people in court laughed at me. That made my
evidence look bad. I simply had no idea what I was up against.
Interviewer: Velma, you are a defense lawyer; do you agree with William?

ƠN
Velma: Absolutely. A police officer has to learn how the system works. You must get used to the
idea that the lawyers are just doing a job, and even if It seem s they are attacking your honesty in a
rude or brutal manner, they have nothing against you as an individual. Interviewer: It must be hard
to think like that when you're giving evidence and some lawyer is trying to trip you up.
Velma: It is, but a police officer has to develop the right attitude. You need to think of your evidence
NH
as one piece in a jigsaw puzzle, the picture being the whole case against the defendant. If you start
giving opinions about other pieces, other parts of the case that aren't your responsibility, it weakens
the case as a whole. Your piece of the puzzle is the only thing you should think about!
Interviewer: Do you find Velma's advice helpful for police officers on your training scheme,
William?
Y

William: Definitely. For a young officer, appearing in court is an intimidating experience. It's hard
QU

to get used to the system. I mean, there are two lawyers, one acting for the defendant and one for
the crown, and in the courtroom they are adversaries but they probably know each other
professionally. They may even go off together after the trial and have dinner. As if it were all a
game!
Interviewer: Would you advise William's trainees to treat a court case as a game, Velma?
M

Velma: I would tell them to remember that the defence lawyer is trying to discredit them and their
evidence. One tip to help you develop the right attitude so you don't get drawn into an argument
with the lawyer is to stand so you're facing the judge, and direct all your answers to the bench. That

should make it easier to avoid any sort of personal exchange with the lawyer.
Interviewer: William , is your training scheme having results?
William: Yes. I think police officers are more confident in court. And this is not just about making
people less nervous!
Y

I've seen some pretty terrible things happen in court. You get an inexperienced officer who starts
arguing with the lawyer and ends up making the judge and jury think there's something wrong with
DẠ

the police case ” there's a risk that dangerous criminals might be found not guilty and set free.
That's the main reason why officers need this training.
Interviewer: Velma Andrew s and William Bailey, thank you.

2
Part 3: You will hear two students, Bella and Tom, discussing an article they have read about a
woman astronaut. For questions 1-5, decide whether the following statements are True (T) or
False (F). (10 points)

L
1. T 2. F 3. F 4. F 5. T

A
Tapescript part 3

CI
Tom: That was a really interesting article the tutor recommended about Ellen Ochoa, wasn't it,
Bella?
Bella: It certainly was, Tom. I used to dream of going into space when I was a kid. Though I know

FI
now I wouldn't be the right kind of person for the job.
Tom: But surely they need all sorts of different types of people?
Bella: Mm. I'm not sure about that I’d get too panicky if there were problems I think. (1) Anyway,I

OF
found if particularly interesting to read about a spacewoman. There aren't too many of those around
Tom: You're right there!
Bella: Anyway. I'm sure they'd be just as able as men to cope with all the challenges of the job. The
article certainly suggests that Ellen was something special, the way she defied all the odds in her

ƠN
determination to do what she’d wanted to do ever since she was small. (2) What I liked best were
the sections that quoted her – I thought she was amazingly good at conveying what it was like to go
into space.
Tom: True. And she's also obviously a very talented scientist
Bella: Yes, she did amazing work and I think she sounds as if she must have been a wonderful
NH
colleague for the others in her team.
Tom: Absolutely, Bella. I also found the article interesting in what it said about the requirements
for becoming an astronaut. I knew you'd have to have done loads of flight hours as an ordinary
aircraft pilot of course. And l suppose it's not that surprising they want people who are good at sport
and who aren't either too small or too tall.
Y

Bella: Mm. well, I was surprised, Tom that being shorter than the average was not acceptable I
QU

wouldn't have expected that to be an issue. (3) But I never imagined that you needed to have a post-
graduate degree I don't think l even realised you had to be a graduate.
Tom: Me neither. I never imagined that. Anyway, she does have an interesting life, doesn't she?
Bella: Yes, Tom I know. Imagine walking in space and having to work inside and outside the capsule
when you're weightless.
Tom: Yes, that must be extraordinary. For me I think the most interesting bit would be having to
M

deal with all the little unexpected problems that arise, having to sort things out within your little
team. That must be amazing.

Bella: Yes, it must. (4) But I was particularly intrigued by her account of the role she sometimes
has as one of the people on earth who are in control of the mission. You know when she helps them
communicate with other astronauts in space. I thought that sounded really fascinating. Being the
one person present on the ground who really understands what life is like for the space crew. She
Y

must be able to make things go much more smoothly Anyway, all in all I thought it was a brilliant
article.
DẠ

Tom: Yes. it was. Though I thought it was a pity it didn’t tell us as much as it might have done about
the less pleasant sides of being in space. I can't believe that it's always straightforward, that all they
do is admire the views and carry out lots of ground-breaking scientific research There must be some
low points - even if it’s only being irritated by some annoying habit of a fellow crew member or
3
getting fed up with the same old food.
Bella: Well, we could always try to do a bit more research into that sort of area, if you liked. Tom.
I wondered even If Ellen Ochoa’s experiences might make a topic for that science coursework we’ve
got to do next term. Or, you know, we could see what we could learn about everyone who’s gone

L
into space from Yuri Gagarin to the present day.

A
Tom: Well, I’m not sure about that. I was quite keen to do something on the funding of space
research. Anyway, I suppose we could bear it in mind. (5) But what I do fancy doing is going to a

CI
series of lectures I've seen advertised on astronauts and how they’re portrayed in the cinema and
in books.
Bella: Wow! That sounds brilliant! Do you mind it I come along too?

FI
Tom: Of course not, Bella. It'd be good to do it together.

Part 4. For questions 1-10, listen to a piece of news from BBC about “What to wear?” and

OF
supply the blanks with the missing information. Write NO MORE THAN THREE WORDS
and/or A NUMBER taken from the recording for each answer in the spaces provided.
20 points
1. Behind the time
2. Finished off

ƠN
3. Rocker look
4. Go sporty
5. Beach sandals
6. Girly look
NH
7. flared jeans
8. waistcoat
9. feel out of place
10. one of the crowd
Y

B. LEXICO-GRAMMAR (40 points)


Part 1: For questions 1 - 20, choose the correct answer (A, B, C, or D) to each of the following
QU

questions and write your answers in the corresponding numbered boxes. (20 points)
1. B 2. A 3. C 4. B 5. A
6. C 7. B 8. C 9. C 10. A
M

11. B 12. B 13. A 14. C 15. C


16. A 17. D 18. D 19. C 20. B

Part 2: Write the correct form of each bracketed word in corresponding numbered boxes. (10
points)
1. pacify
Y

2. instability
3. regularity
DẠ

4.outbreak
5. expressionless
6. significantly
7. pleasant
4
8. incredible
9. inconclusive
10. solitary
Part 3: Fill in the blank with a suitable preposition or a particle to complete the following sentences.

L
(10 points)

A
1.on 6. in

CI
2.away 7. at
3. on 8. for

FI
4. off
9. through
5. on
10. in

OF
C. READING (60 points)
Part 1: (15 points)

ƠN
1. B 2. C 3. D 4. D 5. C 6. D 7. C 8. B 9. D 10. B
Part 2: Fill in the blank with one suitable word. (15 points)
1. stand. 2. highly 3. steps 4. which/that 5. curiosity
NH
6. Independently 7. on 8. sake 9. contribute 10. touch

Part 3: Read the passage below and choose the best answer to each question.
(15 points)
1. A 2. C 3. B 4. D 5. A 6. D 7. C 8. B 9. A 10. A
Y

Part 4: Read the passage and do the tasks that follow. (15 points)
QU

1. vi
2. v
3. iv
4. ix
5. viii
M

6. dry season/hot season/ dry period


7. four months

8. water
9. body mass
10. dehydration

D. WRITING
Y

Part 1: Chart description. (20 points)


DẠ

Contents (12 points):


- The report MUST have at least 2 paragraphs covering the following points:
• Introduce the charts (2 points) and state the overall trends & striking features (2 points)

5
• Describe the main features with relevant data from the charts and make relevant
comparisons
(6 points)
- The report MUST NOT contain personal opinions. (A penalty of 1 point to 2 points will be

L
given to personal opinions found in the answer.)

A
Language use (8 points)
The report:

CI
- should demonstrate a wide variety of lexical and grammatical structures,
- should have correct use of words (verb tenses, word forms, voice,…); and mechanics
(spelling, punctuations,....).

FI
Sample:
The table gives information about poverty rates among six types of households in Australia in the

OF
year 1999.

It is noticeable that levels of poverty were higher for single people than for couples, and people
with children were more likely to be poor than those without. Poverty rates were considerably

ƠN
lower among elderly people.

Overall, 11% of Australians, or 1,837,000 people, were living in poverty in 1999. Aged people
were the least likely to be poor, with poverty levels of 6% and 4% for single-aged people and
NH
aged couples respectively.

Just over one-fifth of single parents were living in poverty, whereas only 12% of parents living
with a partner were classed as poor. The same pattern can be seen for people with no children:
while 19% of single people in this group were living below the poverty line, the figure for couples
Y

was much lower, at only 7%.


QU

Part 2: Write an essay of about 250 words to express your opinion on the following topic.
(30 points)
1. Task achievement: (10 points)
a. ALL requirements of the task are sufficiently addressed.
M

b. Ideas are adequately supported and elaborated with relevant and reliable explanations,

examples, evidence, personal experience, etc.


2. Organization: (10 points)
a. Ideas are well organized and presented with coherence, cohesion, and unity.
Y

b. The essay is well-structured:


DẠ

• Introduction is presented with clear thesis statement.

6
• Body paragraphs are written with unity, coherence, and cohesion. Each body
paragraph must have a topic sentence and supporting details and examples when
necessary.

L
• Conclusion summarises the main points and offers personal opinions (prediction,

A
recommendation, consideration,…) on the issue.

CI
3. Language use: (5 points)
a. Demonstration of a variety of topic-related vocabulary

FI
b. Excellent use and control of grammatical structures
4. Punctuation, spelling, and handwriting (5 points)

OF
a. Correct punctuation and no spelling mistakes
b. Legible handwriting
-The end-

ƠN
NH
Y
QU
M

Y
DẠ

7
HỘI CÁC TRƯỜNG CHUYÊN VÙNG ĐỀ THI HSG KHU VỰC DH & ĐBBB
DUYÊN HẢI VÀ ĐỒNG BẰNG BẮC BỘ LẦN THỨ XV – NĂM 2023

L
MÔN: TIẾNG ANH - KHỐI 11
TRƯỜNG THPT CAO BẰNG
Thời gian: 180 phút

A
ĐỀ THI ĐỀ XUẤT Đề thi gồm: 20 trang

CI
PART A. LISTENING (50 pts)

FI
HƯỚNG DẪN PHẦN THI NGHE HIỂU
• Bài nghe gồm 4 phần, mỗi phần được nghe 2 lần, mỗi lần cách nhau 15 giây, mở

OF
đầu và kết thúc mỗi phần nghe có tín hiệu.
• Mở đầu và kết thúc bài nghe có tín hiệu nhạc. Thí sinh có 3 phút để hoàn chỉnh
bài trước tín hiệu nhạc kết thúc bài nghe.

ƠN
• Mọi hướng dẫn cho thí sinh (bằng tiếng Anh) đã có trong bài nghe.
Section 1. You will hear part of a radio interview in which the comedian and writer Jane
Clarkson is talking about her. For questions 1-5, choose the answer (A, B, C, or D) which fits
best according to what you hear. (10 pts)
NH
1. What did Jane find difficult about writing a book ?
A She couldn’t travel around the country.
B She didn’t get any instant reaction to her work.
C She had to spend time looking after her daughter.
D She found the process itself very challenging.
Y

2. According to Jane, why did some critics dislike her novel ?


QU

A They didn’t think her book was funny.


B They were dismissive of her initial success.
C They though her male colleagues were better writers.
D They thought she should stick to being a comedian
3. Which aspect of Jane’s work as a comedian helped her to write ?
M

A her patience B her ability to learn


C her habit of watching people D her rational way of thinking

4. According to Jane, how do many people react to female comedians ?


A They’re convinced women can’t tell jokes.
B They’re afraid the women will break down.
C They find women’s humour too intense.
Y

D They find women’s jokes embarrassing.


DẠ

5. What was the disadvantage of the stage image which Jane developed?
A It frightened the audience.
B It made the audience angry.

1
C People thought it reflected her real personality.
D People did not take her seriously anymore.

L
Your answers:

A
1. 2. 3. 4. 5.

CI
Section 2. Listen to the recording and decide whether the following statements are true (T)
or false (F). (10 pts)
6. The speaker has come from the Theosophical Society.

FI
7. One of the main points of the talk is to save money.
8. She thinks students should do more housework.

OF
9. She argues that plastic containers won’t biodegrade quickly.
10. She warns that asthma sufferers should be careful with her recipes.

Your answers:

ƠN
6. 7. 8. 9. 10.

Section 3. Listen to the talk about women in the workplace and answer the following
questions. (10 pts)
NH
11. How is the situation for women in the workplace changing?

…..............................................................................................................................

12. In which management role that women constitute 17% of the staff?
Y

…..............................................................................................................................
QU

13. How many percent of employees think that gender equality is a priority?

…..............................................................................................................................

14. What is the action companies should take to understand the problem of gender equality?
M

…..............................................................................................................................

15. What should companies do to make sure opportunities and advancement are equitable?

…..............................................................................................................................

Section 4: Listen and fill in the blanks with the missing information Write NO MORE
THAN THREE WORDS for each answer. (20 pts)
Y

Over the past few years as first lady, I have had the (16)______________ of traveling all
DẠ

across this country and everywhere I’ve gone and the people I've met and the stories I’ve heard, I
have seen the very best of the (17)______________. See, our life before moving to Washington
was, was filled with simple joys. Saturdays at soccer games, Sundays at grandma’s house, and a

2
date night for Barack and me was either dinner or movie because as an exhausted mum I couldn’t
stay awake for both. Even back then when Barack was a senator and (18)______________ to me

L
he was still the guy who picked me up for our dates in a car that was so (19)______________ that
I could actually see the pavement going by in a hole in the passenger side door. He was the guy

A
whose (20)______________ was a coffee table he'd found in a dumpster. Well today, after so

CI
many (21)______________ and moments that’ve tested my husband in ways I never could have
imagined, I have seen first-hand that being president doesn’t change who you are. No it
(22)______________who you are. When it comes to the health of our families, Barack refused to

FI
listen to all those folks to told him to leave (23)______________ for another day, another
president. He didn’t care whether it was the easy thing to do politically, no that's not how he was

OF
raised. He cared that it was the right thing to do. When we were first married our combined
monthly student loan bill was actually higher than our (24)______________. Yeah!! We were so
young, so in love, and so in debt. If we wanna give all of our children a foundation for their
dreams and opportunities worthy of their promise. If we wanna give them that sense of
(25)______________, that belief that here in America there was always something better out there

ƠN
if you're willing to work for it. Then we must work like never before, and we must once again
come together and stand together for the man we can trust to keep moving this great country
forward. My husband, our president, Barack Obama. Thank you, God bless you, God bless
NH
America.

Your answers:

16. 21.
17. 22.
Y

18. 23.
19. 24.
QU

20. 25.

PART B. LEXICO – GRAMMAR (30pts)


Section 1. Choose the word or phrase that best fits the gap in each sentence (20 pts)
1. I was promised a good job from January this year, but it’s April now and I’m afraid that they
M

are just ___________ me along


A. cheating B. swindling C. stringing D. bringing

2. Although we now believe this to be impossible, early scientists tried to produce __________
motion machines, that is, machines which would never stop.
A. perpetual B. everlasting C. undying D. forever
3. The company was so successful during the 1980s that it _________ 500 new employees in a
Y

period of six months


DẠ

A. put on B. took on C. caught on D. laid on


4. Mr. Discontent wanted to complain to the waiter but was afraid of making a(n)_______.
A. act B. drama C. play D. scene
5. I could see the tip of his cigarette _________ in the darkness.
3
A. glowing B. sparkling C. gleaming D. glinting
6. Is an inexperienced civil servant __________ to the task of running the company ?

L
A. capable B. skilled C. eligible D. suited
7. After the concert, everyone had to _________ home through the snow.

A
A. trace B. trend C. trudge D. trickle

CI
8. The students paid _________ attention to the distinguished professor.
A. respectable B. respected C. respectful D. respective
9. ______________, the people who came to this club are in their twenties and thirties.

FI
A. By and large B. Although C. To a degree D. Virtually
10. He is late for every conference without __________.

OF
A. suspicion B. fail C. doubt D. delay
11. Revenge at last! I always said I would __________ my own back on Steve, and now I have.
A. turn B. get C. do D. make
12. Look, will you stop _________ in and let me finish my sentence!
A. moving B. pushing C. butting D. plugging

ƠN
13. It’s the __________ of stupidity to go walking in the mountains in this weather.
A. height B. depth C. source D. matter
14. The school was closed for a month because of a serious ________ of fever.
NH
A. outcome B. outburst C. outset D. outbreak
15. Social work suits her ______ to the ground.
A. down B. for C. out D. round
16. The President accused his critics of being oversensitive and of __________.
A. playing it by ear B. cutting to the chase
Y

C. making a mountain out of a molehill D. splitting hairs


QU

17. My secretary left me in the ________ last month and I haven’t found a replacement yet.
A. rock B. lurch C. face D. fire
18. With patience and diplomacy, she eventually _________ the son of the billion-dollar company
into marrying her.
A. deluded B. inveigled C. cruised D. swindled
M

19. After our three-hour hike, I had a sandwich and it tasted like the best ________.
A. chef B. sauce C. seasoning D. flavor

20. You are being unnecessarily ________ by spending too much time on the details of your CV.
A. recalcitrant B. mendacious C. dogmatic D. pedantic
Your answers:
Y

1. 2. 3. 4. 5. 6. 7. 8. 9. 10.
11. 12. 13. 14. 15. 16. 17. 18. 19. 20.
DẠ

Section 2. Write the correct form of each bracketed word in each sentence. (10 pts)
1. His childhood spent in the harsh conditions of the civil war has completely ______ (SENSE)

4
the boy to the sight of blood and cruelty.
2. “What do you expect from your future wife, Mr. Robson?” - “I’d like one that is faithful and

L
______ (SUBMIT) so that she can fulfil all my requests.”
3. Our local newspaper is often full of stories that are hardly ______ (NEWS), but they need to

A
fill the pages somehow.

CI
4. Both of the fighting sides have agreed on a temporary ______ (ARM) just to let the civilians
safely leave the place.
5. The botanist studied the _____ (FLORAE) of the tropical rainforest, documenting numerous

FI
plant species.
6. The orator delivered a _____ (PERSUADE) speech that swayed the opinions of the entire

OF
audience.
7. The artist's paintings were filled with ______ (INTRICATE) details, requiring careful
observation to appreciate fully.
8. The philosopher's ideas were often ______ (ESOTERIC), understood only by a select few.
9. The archaeologist meticulously _______ (EXCAVATE) the ancient ruins, unearthing valuable

ƠN
artifacts.
10. The professor _______ (PROFICIENT) explained complex mathematical concepts with
remarkable clarity.
NH
Your answers:

1. 6.
2. 7.
3. 8.
Y

4. 9.
5. 10.
QU

PART C. READING (60 pts)

Section 1. Choose the word that best fits each of the blanks in the following passage (10 pts)

FOOTBALL AS AN ART FORM


M

When filmmakers Douglas Gordon and Phillipe Parreno set out to make an art house movie about
the legendary French footballer Zinedine Zidane, they chose to film just one match between Real

Madrid, the club for which he was playing at the (1) …..............., and their great rivals Villareal.
But instead of following the progress of the match, the ninety-minute film would show something
that had not been seen before; the (2) …............... detailed movements of one man during an entire
top-level football match. They hoped that the audience would disengage from the match itself, and
Y

focus on this portrait of greatness. Every (3) …............... gesture would be captured and they
DẠ

would see all of the player’s grace, athleticism and competitiveness in great detail.

The film (4) …............... is a fascinating work. Those who are not regular watchers of football
will be astonished at how (5) …............... Zidane becomes actively involved in the game. For
5
much of the ninety minutes he moves around the field relatively slowly; saying nothing,
expressing even less, and only occasionally (6) …............... into a lethargic jog. And then the ball

L
arrives at his feet, and there is a flurry of bewildering activity. The cameras (and there are
seventeen of them (7) …............... on him) struggle to keep up. The defenders don’t (8)

A
…............... a chance. In a few touches, a couple of checks and feints, Zidane has (9) …...............

CI
them all behind. He crosses from the tightest of (10) …............... and his teammate is left with
simplest of headers to score a goal.

FI
(Adapted from CAE Practice Tests Plus/88)

1. A. point B. moment C. time D. occasion

OF
2. A. clear B. steady C. precise D. slow

3. A. one B. single C. lone D. sole

4. A. following B. resulting C. concluding D. arising

ƠN
5. A. partly B. scarcely C. rarely D. hardly

6. A. breaking B. changing C. opening D. starting

7. A. trained B. looking C. pointed D. staring


NH
8. A. gain B. hold C. stand D. earn

9. A. missed B. left C. lost D. dropped

10. A. places B. positions C. areas D. angles


Y

Your answers:
QU

1. 2. 3. 4. 5.
6. 7. 8. 9. 10.

Section 2. Read the text below and think of one word which best fits each space. Use only
ONE WORD for each space. Write your answer in the space provided (15 pts)
M

Moths count!

Renowned conservationist Sir David Attenborough is launching a campaign today called


‘Moths Count’, to halt the drastically declining number of Britain’s native moths and improve
their poor image. A report (1) …............... ‘The State of Britain’s Larger Moths’ revealed last year
that in some areas, the moth population has almost (2) …............... since 1968. This has led the
Y

charity, ‘Butterfly Conservation’, of which Sir David is president, to develop a new strategy
DẠ

which will provide opportunities for real (3) …............... to broaden their (4) …............... and also
generate appreciation among the wider public. Moths, he insists, play an essential role in the
environment. Their loss (5) …............... the species of birds, bats and small mammals that (6)

6
…............... on them, and the plants they (7) …............... ‘Moths Count’ campaigner Richard Fox
says ‘Currently there’s an image problem, partly because there’s a (8) …............... that moths are

L
night creatures, although many are day-flying and only about half a dozen of Britain’s 2500
species damage clothes.’ Reasons for their decline include climate change and the loss of habitat.

A
Although the (9) …............... of moths has increased with the establishment of new species in

CI
Britain, overall their numbers have dropped, and for some, extinction now seems sadly (10)
…...............

FI
Your answers:

1. 2. 3. 4. 5.
6. 7. 8. 9. 10.

OF
Section 3. Read the following passage and answer the questions. (15 pts)
The history of aspirin is a product of a rollercoaster ride through time, of accidental
discoveries, intuitive reasoning and intense corporate rivalry.

ƠN
A.
In the opening pages of Aspirin: The Remarkable Story of a Wonder
Drug, Diarmuid Jeffreys describes this little white pill as ‘one of the most amazing
NH
creations in medical history, a drug so astonishingly versatile that it can relieve headache,
ease your aching limbs, lower your temperature and treat some of the deadliest human
diseases’.
B.
Its properties have been known for thousands of years. Ancient Egyptian physicians
Y

used extracts from the willow tree as an analgesic, or pain killer. Centuries later the Greek
QU

physician Hippocrates recommended the bark of the willow tree as a remedy for the pains
of childbirth and as a fever reducer. But it wasn't until the eighteenth and nineteenth
centuries that salicylates the chemical found in the willow tree became the subject of
serious scientific investigation. The race was on to identify the active ingredient and to
M

replicate it synthetically. At the end of the nineteenth century a German company,


Friedrich Bayer & Co. succeeded in creating a relatively safe and very effective chemical

compound, acetylsalicylic acid, which was renamed aspirin.


C.
The late nineteenth century was a fertile period for experimentation, partly because
of the hunger among scientists to answer some of the great scientific questions, but also
Y

because those questions were within their means to answer. One scientist in a laboratory
DẠ

with some chemicals and a test tube could make significant breakthroughs whereas today,
in order to map the human genome for instance, one needs ‘an army of researchers, a bank
of computers and millions and millions of dollars’.

7
D.
But an understanding of the nature of science and scientific inquiry is not enough

L
on its own to explain how society innovates. In the nineteenth century, scientific advance

A
was closely linked to the industrial revolution. This was a period when people frequently
had the means, motive and determination to take an idea and turn it into reality. In the case

CI
of aspirin that happened piecemeal - a series of minor, often unrelated advances, fertilised
by the century’s broader economic, medical and scientific developments, that led to one
big final breakthrough.

FI
E.
The link between big money and pharmaceutical innovation is also a significant

OF
one. Aspirin’s continued shelf life was ensured because for the first 70 years of its life,
huge amounts of money were put into promoting it as an ordinary everyday analgesic. In
the 1970s other analgesics, such as ibuprofen and paracetamol, were entering the market,
and the pharmaceutical companies then focused on publicising these new drugs. But just at

ƠN
the same time, discoveries were made regarding the beneficial role of aspirin in preventing
heart attacks, strokes and other afflictions. Had it not been for these findings, this
pharmaceutical marvel may well have disappeared.
NH
F.
So the relationship between big money and drugs is an odd one. Commercial
markets are necessary for developing new products and ensuring that they remain around
long enough for scientists to carry out research on them. But the commercial markets are
Y

just as likely to kill off' certain products when something more attractive comes along. In
the case of aspirin, a potential ‘wonder drug’ was around for over 70 years without
QU

anybody investigating the way in which it achieved its effects, because they were making
more than enough money out of it as it was. If ibuprofen or paracetamol had entered the
market just a decade earlier, aspirin might then not be here today. It would be just another
forgotten drug that people hadn't bothered to explore.
M

G.
None of the recent discoveries of aspirin's benefits were made by the big

pharmaceutical companies; they were made by scientists working in the public sector. 'The
reason for that is very simple and straightforward,' Jeffreys says in his book. 'Drug
companies will only pursue research that is going to deliver financial benefits. There's no
profit in aspirin any more. It is incredibly inexpensive with tiny profit margins and it has
Y

no patent any more, so anyone can produce it.' In fact, there's almost a disincentive for
DẠ

drug companies to further boost the drug, he argues, as it could possibly put them out of
business by stopping them from selling their more expensive brands.
H.
8
So what is the solution to a lack of commercial interest in further exploring the
therapeutic benefits of aspirin? More public money going into clinical trials, says Jeffreys.

L
‘If I were the Department of Health. I would say “this is a very inexpensive drug. There

A
may be a lot of other things we could do with it." We should put a lot more money into
trying to find out.'

CI
I.
Jeffreys' book which not only tells the tale of a 'wonder drug' but also explores the
nature of innovation and the role of big business, public money and regulation reminds us

FI
why such research is so important.
Questions 1-5. The Reading Passage has 9 paragraphs A-I.

OF
Choose the correct heading for each paragraph (A-E) from the list of headings below.
Write the correct number (i-x) in boxes 1-5.
List of headings:
i. The most powerful analgesic 1. Paragraph A

ƠN
ii. The profit potential of aspirin 2. Paragraph B
iii. Saved from oblivion by drug companies 3. Paragraph C
iv. Recognition of an important medicinal property 4. Paragraph D
NH
v. A double-edged sword 5. Paragraph E
vi. An unstructured pattern of development
vii. Major pharmaceutical companies
viii. A wonder drug
Y

ix. Roots of the scientific advancements in the 19th


century
QU

x. The discovery of new medical applications

Your answers
1. 2. 3. 4. 5.
M

Questions 6-10

Do the following statements agree with the views of the writer in Reading Passage?
In boxes 6-10 write:
YES if the statement agrees with the views of the writer
NO if the statement contradicts the views of the writer
Y

NOT GIVEN if it is impossible to say what the writer thinks about this
DẠ

6. The 19th century saw significant changes in the way in which scientific experiments were
carried out.

9
7. For nineteenth-century scientists, small-scale research was far from enough to make important
discoveries.

L
8. The creation of a market for aspirin as a painkiller was achieved through commercial
advertising campaigns.

A
9. In the 1970s sales of new analgesic drugs overtook sales of aspirin.

CI
10. Jeffrey suggests that there should be state support for further research into the possible
applications of aspirin.
Your answers

FI
6. 7. 8. 9. 10.

Section 4. Read the text and do the task as follow (13 pts)

OF
The Farmers! Parade of history

A. History of Fanner trading company: In 1909 Robert Laidlaw establishes mail-order company
Laidlaw Leeds in Fort Street, Auckland. Then, Branch expansion: purchase of Green and

ƠN
Colebrook chain store; further provincial stores in Auckland and Waikato to follow. Opening of
first furniture and boot factory. In 1920, Company now has 29 branches; Whangarei store
purchased. Doors open at Hobson Street for direct selling to public. The firm establishes London
and New York buying offices. With permission from the Harbour Board, the Large FARMERS
NH
electric sign on the Wyndham Street frontage is erected.

B. In 1935, if the merchandise has changed, the language of the catalogues hasn't Robert Laidlaw,
the Scottish immigrant who established die century-old business, might have been scripting a
modern-day television commercial when he told his earliest customers: Satisfaction, or your
Y

money back. "It was the first money back guarantee ever offered in New Zealand any firm," says
QU

Ian Hunter, business historian. "And his mission statement was, potentially, only the second one
ever found in the world." Laidlaw's stated aims were simple to build the greatest business in New
Zealand, to simplify every transaction, to eliminate all delays, to only sell goods it would pay the
customer to buy.
M

C. This year, the company that began as a mail-order business and employs 3500 staff across 58
stores turns 100. Its centenary will be celebrated withe release of a book and major community

fundraising project, to be announced next week. Hunter, who is writing the centenary history, says
in every 10 people had an account with die company. It was the place where teenage girl shopped
for their first bra, where newlyweds purchased their first dinner sets, where first pay cheques were
used to pay off hire purchase furniture, where Santa paraded every Christmas.
Y

D. Gary Blumenthal's mother shopped there, and so does he. The fondest memory for the Rotorua
resident? "We were on holiday in Auckland... I decided that upon the lookout tower on top of the
DẠ

Farmers building would be a unique place to fit the ring on my new fiancee’s finger." The
lovebirds, who had to wait for "an annoying youth" to leave the tower before they could enjoy
their engagement kiss, celebrate their 50th wedding anniversary in June.
10
E. Farmers, say Hunter, has always had a heart. This, from a 1993 North & South interview with a
former board chairman, Rawdon Busfield: " One day I was in the Hobson Street shop, and I saw a

L
woman with two small children. They were clean and tidily dressed, but poor, you could tell. That
week we had a special on a big bar of chocolate for one shilling. I heard the woman say to her

A
boy, 'no, your penny won't buy that'. He wasn't wearing shoes. So I went up to the body said,' Son,

CI
have you got your penny?' He handed it to me. It was hot he'd had it in his hand for hours. I took
the penny and gave him the chocolate.

F. Farmers was once the home of genteel tearooms, children's playground and an annual sale of

FI
celebration for birthday of Hector the Parrot (the store mascot died, aged 131, in the 1970s his
stuffed remains still occupy pride of place at the company's head office). You could buy houses

OF
from Farmers. Its saddle factory supplied the armed forces, and its upright grand overstrung
pianos offered "the acme of value" according to those early catalogues hand-drawn by Robert
Laidlaw himself. Walk through a Farmers store today and get hit by bright lights and big brands.
Its Albany branch houses 16 international cosmetics companies. It buys from approximately 500

ƠN
suppliers, and about 30% of those are locally owned.

G. "Eight, 10 years ago," says current chief executive Rod McDermott, "lots of brands wouldn't
partner with us. The stores were quite distressed. We were first price point focused, we weren't
NH
fashion focused. "Removed the rose-tinted nostalgia, and Farmers is, quite simply, a business,
doing business in hard times. Dancing with the Stars presenter Candy Lane launches a clothing
line? "We put a trial on, and we thought it was really lovely, but the uptake wasn't what we
thought it would be. It's got to be what the customer wants" says McDermott.
Y

H. He acknowledges retailers suffer in a recession: "We're celebrating 100 years because we can
and because we should" Farmers almost didn't pull though one economic crisis. By the mid 1980s,
QU

it had stores across the country. It had acquired the South Island's Calder Mackay chain of stores
and bought out Haywrights. Then, with sale topping $375 million, it was taken over by Chase
Corporation. Lincoln Laidlaw, now aged 88, and the son of the company's founder, remembers the
dark days following the stock market crash and the collapse of Chase. "I think, once, Farmers was
M

like a big family and all of the people who worked for it felt they were building something which
would ultimately be to their benefit and to the benefit of New Zealand... then the business was

being divided up and so that kind of family situation was dispelled and it hasn't been recovered."
For a turbulent few years, the stores were controlled, first by a consortium of Australian banks and
later Deka, the Maori Development Corporation and Foodland Associated Ltd. In 2003, it went
back to "family" ownership. with the purchase by the James Pascoe Group, owned by David and
Anne Norman the latter being the great-granddaughter of James Pascoe, whose first business
Y

interest was jewellery.


DẠ

I. "Sheer power of the brand", says McDermott," pulled Farmers through and now we're becoming
the brand it used to be again." Farmers was the company that, during World War n, topped up the
wages of any staff member disadvantaged by overseas service. Robert Laidlaw a committed
11
Christian who came to his faith at a 1902 evangelistic service in Dunedin concluded his original
mission statement with the words, "all at it, always it, win success". Next week, 58 Farmers stores

L
across the country will announce the local charities they will raise funds for in their centenary
celebration everything from guide dog services to hospices to volunteer fire brigades will benefit

A
Every dollar raised by the community will be matched by the company. "It's like a rebirth of an

CI
icon," says McDermott.

Question 1-5
The reading Passage has seven paragraphs A-I

FI
Which paragraph contains the following information?
Write the correct A-I, in boxes provided.

OF
1. Generosity offered in an occasion.

2. Innovation of offer made by the head of company.

3. Fashion was not its strong point.

ƠN
4. A romantic event on the roof of farmers.

5. Farmers were sold to a private owned company.


NH
Your answers:

1. 2. 3. 4. 5.

Question 6-10
Y

Complete the following summary of the paragraphs of Reading Passage


Using NO MORE THAN TWO WORDS from the Reading Passage for each answer.
QU

6. Farmers was first founded as a 36 ______ in Auckland by Mr Laidlaw.

7. Farmers developed fast and bought one 37______ then.

8. During oversea expansion, Farmers set up 38______ in cities such as London.


M

9. Farmers held a 39 ______once a year for the well-known parrot.


10. In the opinion of Lincoln Laidlaw, Farmers is like a 40______ for employees, not just for
themselves but for the whole country.

Your answers:
Y

6. 7. 8. 9. 10.
Question 11-13
DẠ

Use the information in the passage to match the people (listed A-C) with opinions or deeds
below.

12
Write the appropriate letters A-C in boxes provided.
NB You may use any letter more than once.

L
A Lincoln Laidlaw

A
B Rod McDermott

CI
C Ian Hunter

11. Product became worse as wrong aspect focused.

FI
12. An unprecedented statement made by Farmers in New Zealand.

13. Character of the company was changed.

OF
Your answer:

11. 12. 13.

ƠN
Section 5. You are going to read an article. Seven paragraphs have been removed. Choose
from the paragraphs A-H the one that fits each gap (1-7). There is one extra paragraph
which you do not need to use. (7.0 pts)
NH
THE IMPORTANCE OF SLEEP

A recent study claims to have found a new explanation for the puzzle that is one of our most
essential biological functions: sleep. Apparently, while we have our forty winks, a 'waste
management' system pumps cerebral fluid around the brain and flushes out molecular detritus, a
Y

natural by-product of neural activity that collects during waking hours. If allowed to pile up, these
waste toxins are responsible for a number of neurological disorders, including Alzheimer's
QU

1.

As a biological state, sleep is not restricted to humans. It is essential for every living creature and
tailored to suit their individual lifestyles as a species - from up to 20 hours a day to brown bats
hanging upside down to a 30-minute daily shut-eye for giraffes standing up. Sleep does not,
M

however, appear to make sense from a survival point of view, as, unable to operate our guards and
with significantly reduced response to external stimuli, we are vulnerable to predators. Yet, it is

hardly a choice for any of us.

2.

This is a pattern that nature doesn't let us break. Empirically, we have learnt more about the
Y

effects that are triggered by lack of sleep than the benefits we derive from it. Understand it or not,
DẠ

sleep is so vitally important that any deviation from what is within the perceived norms can impair
both our mental and physical health.

13
3.

Experiments on rats have shown that total sleep deprivation for about two weeks or more

L
inevitably led to death. In known cases where humans were prevented from sleeping, either due to

A
the existence of the syndrome or other imposed reasons, the body gradually degenerated to the
point of multiple organ failure.

CI
4.

The energy conservation theory: Sleep lowers our energy needs by as much as 10 per cent as

FI
compared to when we are restful and awake because our body reduced both its temperature and its
caloric consumption. This might have been crucial for the survival of our ancestors when food

OF
was not easily available as it gave them the advantage of being more efficient hunters.

5.

REM (rapid eye movement), for example, is a deep state of sleep which involves intense brain

ƠN
activity. It allows us to dream and it accounts for about one fifth of our total sleep time. REM
sleep plays an important role in restoring mental functions NREM (non-rapid eye movement)
sleep account for the remaining four fifths and is conducive to restoring physiological functions.

6.
NH
Mental activity during sleep has also been linked to brain plasticity, which is defined as the brain's
ability to modify its own structure and function to reflect changes in our body or the external
environment. There is evidence that sleep plays a critical role in brain development in infants and
young children.
Y

7.
QU

Although we have been unable to provide concrete proof for the above theories, it is essential that
we recognise the advances that have been made towards comprehending why we sleep and the
way in which research has helped us further our understanding of the mechanics of it. The bottom
line is that it might be too simplistic to assume there is a single answer to such a complex process
M

A. The study is just another attempt to gain some insight into what makes us humans tick.

Although we have a relatively satisfactory understanding of other life-sustaining activities, such as


eating (to fuel our body with essential nutrients so thit can grow and repair itself) and drinking
water (to keep our 70% topped up), we are still searching for a definitive answer to why we need
to sleep
Y

B. Up to now many theories have been put forward, including this recent one, but science has yet
DẠ

to come up with the definitive explanation of why we sleep. Though research and experimentation
we have gained valuable insight into the mechanisms that go to work in our body while we sleep.
We now know that sleep is related to our psychology, physiology, metabolism and digestion. Let's
have a look at a few of these theories
14
C. Finally, there's the theory that maintains that dreams, which only occur when we sleep, are out
way of coping with our emotions. Although dream research offers many theories of why we

L
dream, we know that dreams help us to do some problem solving, and deal with traumatic
experiences.

A
D. Sleep is controlled and dictated by our internal clock, which, in turn, is regulated by the

CI
process called Circadian rhythm, inherent in all living beings. It's a daily cycle of biological
activity which tells us when it is time to eat, rest or be active. It is set to respond to external
stimuli such as the presence of light, which wakes us up in the morning of the absence of it, which

FI
prompts us to go to bed at night

E. The restorative theory: Sleep give us the opportunity to do some essential repair work and

OF
'housekeeping'. It is the time when many of the major restorative function like tissue repair,
muscle growth, protein synthesis and the release of growth hormones take place in our body. The
same goes for our brain. It is important to note that there are two different states sleep during
which we switch from physical to mental maintenance.

ƠN
F. The scientist cautions that the simple brain of a mouse and the more complex brain of the
human are two different things, and that what is true for one is not necessarily true for the other. It
is important at this stage not to be either too optimistic or too dismissive of new indications but to
NH
continue research until we have enough evidence to from and assumption about its relevance to
the human brain.

G. The information consolidation theory: During sleep, and particularly during the first hours
when deep sleep occurs, our brain has the chance to process and consolidate the information it
Y

received the previous day, as well as prepare for the next day. Recently acquired information is
QU

either discarded as useless or committed to our long-term memory. The theory is supported by
research that indicates that sleep deprivation affects our ability to recall and retain information.

H. Chronic sleep deprivation, or, in other words, not enough sleep, which is the most common
complaint, has an adverse effect on our cognitive process, compromise our memory and can even
make us depressed, it increases the risk of serious illnesses like cardiovascular disease, diabetes
M

and some forms of cancer or even disease-related mortality.


PART D. WRITING (60pts)

Section 1. Read the following passage and use your own words to summarize it. Your
summary should be between 100 and 120 words. (15 pts)
Y

Buying things today is so simple. Just enter a shop, say a book store, choose the desired
book and pay for it. Long ago, before the invention of money, how did people trade? The most
DẠ

primitive way of exchange should be the barter trade. In this form of transaction, people used
goods to exchange for the things that they had in mind. For instance, if person A wanted a book
and he had a spare goat, he must look for someone who had the exact opposite, that is, that
15
someone, say person B, must have a spare book of person A's choice and is also in need of a goat.
Having found such a person, the problem does not end here. A big goat may worth not only one

L
book, hence person B may have to offer person A something else, say five chickens.

A
However, he runs the risk of person A rejecting the offer as he may not need the chickens.
The above example clearly illustrates the inefficiency of barter trading. Many years later, the

CI
cumbersome barter trade finally gave way to the monetary form of exchange when the idea of
money was invented. In the early days, almost anything could qualify as money: beads, shells and
even fishing hooks. Then in a region near Turkey, gold coins were used as money. In the

FI
beginning, each coin had a different denomination. It was only later, in about 700 BC, that Gyges,
the king of Lydia, standardized the value of each coin and even printed his name on the coins.

OF
Monetary means of transaction at first beat the traditional barter trade. However, as time
went by, the thought of carrying a ponderous pouch of coins for shopping appeared not only
troublesome but thieves attracting. Hence, the Greek and Roman traders who bought goods from
people faraway cities, invented checks to solve the problem. Not only are paper checks easy to

ƠN
carry around, they discouraged robbery as these checks can only be used by the person whose
name is printed on the notes. Following this idea, banks later issued notes in exchange for gold
deposited with them. These bank notes can then be used as cash. Finally, governments of today
adopted the idea and began to print paper money, backed by gold for the country's use.
NH
Today, besides enjoying the convenience of using paper notes as the mode of exchange,
technology has led man to invent other means of transaction too like the credit and cash cards.

………………………………………………………………………………………………………
………………………………………………………………………………………………………
Y

………………………………………………………………………………………………………
QU

………………………………………………………………………………………………………
………………………………………………………………………………………………………
………………………………………………………………………………………………………
………………………………………………………………………………………………………
………………………………………………………………………………………………………
M

………………………………………………………………………………………………………
………………………………………………………………………………………………………

………………………………………………………………………………………………………
………………………………………………………………………………………………………
………………………………………………………………………………………………………
………………………………………………………………………………………………………
Y

………………………………………………………………………………………………………
DẠ

………………………………………………………………………………………………………
Section 2. The chart shows the proportion of renewable energy in total energy supply in 4
countries from 1997 to 2010. Summarize the information and Write at least 150 words.
(15pts)
16
AL
CI
FI
OF
………………………………………………………………………………………………………

ƠN
………………………………………………………………………………………………………
………………………………………………………………………………………………………
………………………………………………………………………………………………………
………………………………………………………………………………………………………
NH
………………………………………………………………………………………………………
………………………………………………………………………………………………………
………………………………………………………………………………………………………
………………………………………………………………………………………………………
Y

………………………………………………………………………………………………………
………………………………………………………………………………………………………
QU

………………………………………………………………………………………………………
………………………………………………………………………………………………………
………………………………………………………………………………………………………
………………………………………………………………………………………………………
M

………………………………………………………………………………………………………
………………………………………………………………………………………………………

………………………………………………………………………………………………………
………………………………………………………………………………………………………
………………………………………………………………………………………………………
………………………………………………………………………………………………………
Y

………………………………………………………………………………………………………
………………………………………………………………………………………………………
DẠ

………………………………………………………………………………………………………
………………………………………………………………………………………………………
………………………………………………………………………………………………………
17
………………………………………………………………………………………………………
………………………………………………………………………………………………………

L
………………………………………………………………………………………………………
………………………………………………………………………………………………………

A
………………………………………………………………………………………………………

CI
………………………………………………………………………………………………………
Section 3. Write an essay of 350 words on the following topic. (30 pts)
Some people think that the best way to become successful in life is to get a university

FI
education, while others disagree and say this is no longer true nowadays.
Discuss these both views and give your own opinion.

OF
………………………………………………………………………………………………………
………………………………………………………………………………………………………
………………………………………………………………………………………………………
………………………………………………………………………………………………………

ƠN
………………………………………………………………………………………………………
………………………………………………………………………………………………………
………………………………………………………………………………………………………
NH
………………………………………………………………………………………………………
………………………………………………………………………………………………………
………………………………………………………………………………………………………
………………………………………………………………………………………………………
………………………………………………………………………………………………………
Y

………………………………………………………………………………………………………
QU

………………………………………………………………………………………………………
………………………………………………………………………………………………………
………………………………………………………………………………………………………
………………………………………………………………………………………………………
………………………………………………………………………………………………………
M

………………………………………………………………………………………………………
………………………………………………………………………………………………………

………………………………………………………………………………………………………
………………………………………………………………………………………………………
………………………………………………………………………………………………………
………………………………………………………………………………………………………
Y

………………………………………………………………………………………………………
DẠ

………………………………………………………………………………………………………
………………………………………………………………………………………………………
………………………………………………………………………………………………………
………………………………………………………………………………………………………
18
………………………………………………………………………………………………………
………………………………………………………………………………………………………

L
………………………………………………………………………………………………………
………………………………………………………………………………………………………

A
………………………………………………………………………………………………………

CI
………………………………………………………………………………………………………
………………………………………………………………………………………………………
………………………………………………………………………………………………………

FI
………………………………………………………………………………………………………
………………………………………………………………………………………………………

OF
………………………………………………………………………………………………………
………………………………………………………………………………………………………
………………………………………………………………………………………………………
………………………………………………………………………………………………………
………………………………………………………………………………………………………

ƠN
………………………………………………………………………………………………………
………………………………………………………………………………………………………
………………………………………………………………………………………………………
NH
………………………………………………………………………………………………………
………………………………………………………………………………………………………
………………………………………………………………………………………………………
………………………………………………………………………………………………………
………………………………………………………………………………………………………
Y

………………………………………………………………………………………………………
QU

………………………………………………………………………………………………………
………………………………………………………………………………………………………
………………………………………………………………………………………………………
Giáo viên ra đề
M

Lưu Thị Thúy Ngà

Số điện thoại:

Y
DẠ

19
DẠ
Y

M
QU
Y

20
NH
ƠN
OF
FI
CI
A L
HỘI CÁC TRƯỜNG CHUYÊN VÙNG ĐỀ THI HSG KHU VỰC DH & ĐBBB
DUYÊN HẢI VÀ ĐỒNG BẰNG BẮC BỘ LẦN THỨ XV – NĂM 2023
MÔN: TIẾNG ANH - KHỐI 11
TRƯỜNG THPT CAO BẰNG
Thời gian: 180 phút

L
ĐÁP ÁN ĐỀ THI ĐỀ XUẤT Đáp án gồm: 10 trang

A
CI
PART A. LISTENING: 50 pts

FI
Section 1. 5 x 2.0 = 10 pts
1. B 2. D 3. C 4. B 5. C

OF
Section 2. 5 x 2.0 = 10 pts
1. F 2. T 3. F 4. T 5. F
Section 3. 5 x 2.0 = 10 pts
1. Not fast enough

ƠN
2. C-suite
3. 37%
4. Track key metrics
5. Create a level playing field
NH
Section 4. 10 x 2.0 = 12 pts
1. extraordinary privilege 2. American spirit
3. a presidental candidate 4. wasted out
5. proudest possession 6. struggles and trials
Y

7. reveals 8. health reforms


QU

9. mortgage 10. limitless responsibility


PART B. LEXICO-GRAMMAR: 30 pts
Section 1. 20 x 1.0 = 20 pts

1 C string someone along: để lừa dối ai đó trong một thời gian dài về
M

những gì bạn đang thực sự có ý định làm


2 A perpetual: vĩnh viễn

3 B take on: tuyển dụng ai đó


4 D make a scene: cư xử một cách ồn ào, tức giận ở nơi công cộng
5 A glowing: phát sáng
6 D suited: phù hợp
Y

7 C trudge through something: để làm công việc hoặc một nhiệm vụ


cụ thể chậm và với nỗ lực hoặc khó khăn
DẠ

8 C respectful: tôn trọng


9 A By and large: khi mọi thứ về một tình huống được xem xét cùng
nhau
10 B to do something without fail: không thất bại trong việc làm gì đó
11 B get your own back (on someone): làm điều gì đó khó chịu với ai
đó vì họ đã làm điều gì đó khó chịu với bạn
12 C butt in: để làm gián đoạn một cuộc trò chuyện hoặc thảo luận
hoặc ai đó đang nói chuyện

L
13 A the height of something: thời gian khi một tình huống hoặc sự

A
kiện là mạnh nhất hoặc đầy đủ nhất của hoạt động
14 D outbreak: một thời điểm khi một cái gì đó đột ngột bắt đầu, đặc

CI
biệt là một căn bệnh hoặc một cái gì đó nguy hiểm hoặc khó chịu
15 A suit someone (right) down to the ground: Nếu một cái gì đó phù
hợp với bạn đến tận cùng, thì nó hoàn toàn phù hợp với bạn,

FI
thường là vì nó thuận tiện cho bạn
16 C making a mountain out of a molehill: để làm cho một khó khăn
nhỏ có vẻ như một vấn đề nghiêm trọng

OF
17 B leave someone in the lurch: rời bỏ ai đó vào thời điểm họ cần bạn
ở lại và giúp đỡ
18 B inveigle: để thuyết phục ai đó làm điều gì đó một cách thông
minh và không trung thực, khi họ không muốn làm điều đó

ƠN
19 D pedantic: chú ý quá nhiều đến các quy tắc chính thức hoặc các
chi tiết nhỏ
20 D
NH
Section 2. 10 x 1.0 = 10 pts

1. desensitized để khiến ai đó trải nghiệm điều gì đó, thường là cảm xúc hoặc
nỗi đau, ít mạnh mẽ hơn trước
2. submissive cho phép bản thân bị kiểm soát bởi người hoặc động vật khác
Y

3. newsworthy đủ thú vị để được mô tả trong một báo cáo tin tức


4. armistice một thỏa thuận chính thức giữa hai quốc gia hoặc các nhóm có
QU

chiến tranh để ngừng chiến đấu trong một thời gian cụ thể, đặc
biệt là để nói về hòa bình có thể
5. flora tất cả các thực vật của một địa điểm cụ thể hoặc từ một thời
điểm cụ thể trong lịch sử
6. persuasive khiến bạn muốn làm hoặc tin vào một điều cụ thể
M

7. intricate có rất nhiều bộ phận nhỏ được sắp xếp một cách phức tạp hoặc
tinh tế

8. esoteric rất bất thường và chỉ được hiểu hoặc thích bởi một số ít người,
đặc biệt là những người có kiến thức đặc biệt
9. excavate để loại bỏ lớp đất đang bao phủ các đồ vật rất cũ bị chôn vùi
trong lòng đất để khám phá những điều về quá khứ
10. proficiently theo cách thể hiện kỹ năng và kinh nghiệm
Y
DẠ

PART C. READING: 60 pts


Section 1. 10 x 1.0 = 10 pts
1. C 2. C 3. B 4. B 5. C 6. A 7. A 8. C 9. B 10. D
Section 2. 10 x 1.5 = 15 pts
1. titled
2. halved
3. enthusiatsts

L
4. expertises

A
5. threatens
6. feed

CI
7. pollinate
8. perception
9. diversity

FI
10. unavoidable/inevitable
Section 3. 10 x 1.5 = 15 pts

OF
1. viii 2. iv 3. ix 4. vi 5. x
6. NG 7. No 8. Yes 9. NG 10. Y

ƠN
Section 4. 13 x 1.0 = 13 pts
1. E 2. B 3. G 4. D 5. H
6. mail-order company
7. chain store
NH
8. buying offices
9. celebration
10. big family
11. B
Y

12. C
13. A
QU

Section 5. 7 x 1.0 = 7.0 pts


1. A
2. D
3. H
M

4. B
5. E

6. G
7. C
PART D. WRITING: 60 pts
Y

Section 1. Summarize the following article in not more than 120 words.
(15pts)
DẠ

The mark given to part 1 is based on the following criteria:


1. Write good summary with enough content and clear, logical 5 pts
information.
2. Present the key points and main ideas in the right form of a paragraph. 5 pts
Make sure that no important points have been omitted or distorted.
3. Use your own words or paraphrases with a variety use of synonyms, 5 pts
different sentence structures and word class. You can change the order
of ideas where necessary.

L
Sample text

A
This passage focuses on the history of trades and transaction. The earliest

CI
method of trading is the barter trade, which entails people using goods they own
to exchange for the items they want. This practice is seen as inefficient. A
monetary way of exchange is presented many years later as money is invented.

FI
Money such as beads, shells and hooks are then reworked to gold coins. This
system became more troublesome as time goes on because of thievery and the
weight of coin pouches. The Greek and Roman traders came up with paper

OF
checks, which can be used as cash to deposit the number of gold in them.
Ultimately, present-day governments took on the idea and began the printing of
paper money. Besides this, technology's evolution has presented other means of
exchange such as credit cards and cash cards.

ƠN
Section 2: (15 pts)

1. Completion: 3 pts
2. Content: 4 pts
NH
- Cover the main information in the chart yet not go into too many details.
- Make general remarks and effective comparisons.
3. Organisation: 3 pts
- The ideas are well organized
- The description is sensibly divided into paragraphs
Y

4. Language: 3pts
- Use a wide range of vocabulary and structure
QU

- Good grammar
5. Punctuation and spelling: 2 pt

Sample text
M

The above chart presents the proportion of renewable energy in the total

energy supply in 4 countries from the year 1997 to 2010. Australia and Sweden
both have the lowest proportions of renewable energy in total energy supply out
of the 4 given countries, as they both average between 10 or lower throughout
1997 and 2010. Australia having a slight decrease from 1997 to 2012 while
Y

Sweden has a modest increase. Turkey has a moderate proportion of renewable


energy in total energy supply, starting out with an average below 40 in 1997,
DẠ

peaking at about 40 in 2000 and reaching its lowest in 2010 at about 30. Iceland
has the highest proportion of renewable energy in total energy supply out of the
4 countries presented, having a great increase from about 50 in 1997, rising to
60 in 2000, before hitting a whopping 70 in 2010.
Section 3. (30 points)

1. Content: (10%) - Providing all main ideas and details as required


- Communicating intentions sufficiently and

L
effectively

A
2. Language: (10%) - Demonstration of a variety of vocabulary and
structures appropriate to the level of English

CI
language gifted upper-secondary school students
- Good use and control of grammatical structures
- Good punctuation and no spelling mistakes

FI
- Legible handwriting
3. Organization and - Ideas are well organized and presented with
Presentation: (10%) coherence, cohesion, and clarity

OF
- The essay is well-structured

Sample text

ƠN
Higher education may be the goal that student and parents are currently aiming
for, as evidenced by the annual university entrance exams and the expectations
placed on the results. However, some people argued that higher education is no
longer worth it. There have been examples of successful businessmen and
NH
entreprenuer that forgo higher education such as Bill Gates. Personally, I
bellieve this is not a simple issue that can be viewed from only one perspective.
Firstly, we have to understand about success in life. The only thing that can
provide stable employment and sustain oneself is a job that require a high level
of intellectual capacity, as intellectual work is often rewarded much more than
Y

ordinary labor. However, investing in developing the intellectual capacity


QU

neccessary for high-paying jobs is not a simple task due to the knowledge
applied in higher level and broader job is much more extensive than general
knowledge, and not everyone has the mentality to acquire it. Most importantly,
what demonstate that an applicant can meet the needs of a bussiness the most is
higher education, it also provides the shortest path for learners to outline a clear
M

trajectory for developing their own capabilities, and a university degree serves
as evidence of learning and to some extend, the competance of the degree

holder . Let's pose a simple question: would any hospital hire an unqualified
doctor to treat patients? However, this may not be true when we look at other
fields such as art, which can be pursued through talent than relying on specific
professional like singing or drawing. Even if we consider booming industries
Y

like information technology, which offer high salaries but can be learned
independently without going through traditional schooling or paying any tuition
DẠ

fees. The field of information technology, which is known for its high salaries,
relies heavily on vast open-source resources and a large community that
frequently exchanges knowledge and shares extensive documentation. These are
the kinds of knowledge that even universities may not necessarily be able to
meet.
However, currently the tuition fee for higher education are very expensive , and
not everyone can afford it. Therefore, higher education is not the optimal choice
for everyone considering the financial burden it poses and the limitations it
imposes on access and opportunity.

L
TAPESCRIPT

A
Section 1

CI
Int: Today I’m with the much-loved comedian and writer Jane Clarkson.
Obviously Jane, this year has been quite a turning point for you ...

FI
Jane: Well, I’ll never stop doing comedy, but there were practical reasons for
wanting to take some time off and write a book. I felt my daughter had been
neglected. She was just about to make the tricky transition from primary to

OF
secondary school and I thought she needed her mum around. I seem to have
spent most of her life in a van touring from venue to venue for my comedy act.
And I did enjoy being at home for a bit, although I missed the applause and the
laughter. When I finished writing in the evening, I’d turn the computer off and

ƠN
there’d be nothing, which was hard to get used to.
Int: How was your novel received?
Jane: Well, a lot of male comedians had written books, so there was a bit of a
NH
bandwagon waiting to be jumped on, but with my impeccable timing I jumped
slightly late, when everyone was starting to get heartily sick of comedians’
books. Also there’s a kind of fury coming from some journalists about comics
writing books. They’re absolutely livid, as if they see your book in a bookshop
Y

and they jump up and down, shouting, ‘It’s not fair! Why should she make
money out of writing as well as performing?’
QU

Int: Was it a difficult transition?


Jane: Well, if you think logically, writing is the obvious step. I’ve spent years
trying to make people listen to my anecdotes, so that must count for something!
Also, if you’ve been an observational comedian, which I am, it’s not a great
M

leap to use those skills you’ve developed, like observing odd mannerisms to use
for jokes, and turn them into a book. At least that’s what I felt, but you don’t

become a writer instantly. I’ll have to wait and see whether it was just
beginners’ luck.
Int: I think why people give you a hard time about the novel is surely because
we’re so trapped into thinking Jane Clarkson is a comedian. It’s as though, you
Y

know, you can’t do anything else, which is quite ridiculous because you’ve been
DẠ

writing radio comedy for years.


Jane: Yes, people do become obsessed about what you are. The character I
adopted for my comedy act became rather a burden after a while. When I started
going on stage alone, I was very young and I wasn’t entirely convincing as a
comedian because nervous young women on stage actually frighten audiences.
They’re convinced you’re going to fail and burst into tears, which will be very
embarrassing. So there’s a palpable tension in the room and some audiences
actually boo the female comedians off the stage.

L
Int: How did you deal with that?

A
Jane: Well, I had to counteract that stereotype so I started coming on shouting
and being madder and crosser than any audience could ever be and that defused

CI
the tension. In fact, I rather overdid it and my character got cruder than I ever
really intended. I got so good at it that people got confused between the
everyday Jane and the stage Jane.

FI
Int: What attracted you in the first place to performing, and particularly to
making people laugh?

OF
Jane: Oh, from an early age, I knew I wanted to be an actress. I innocently
thought I was going to be a glamorous film star. The reason I started to do
comedy acts was that in the 1980s a lot of small provincial theatres closed

ƠN
down. In the past, girls would’ve come out of drama school and if they had a
leaning towards comedy they’d join one of these small theatres and play a
variety of comedy roles in all sorts of plays from Shakespeare to contemporary
stuff. All of a sudden, with the demise of these theatres, rooms above pubs
NH
opened up and comedians started telling jokes and developing their acts there. It
was cheap, one performer one microphone, and anyone could do it. In some
ways, it’s a healthier performance art than acting, because with acting you’re at
the mercy of everybody else deciding whether you can work or not. With stand-
up comedy, you might only get paid peanutsbut nobody can stop you from just
Y

driving to a venue, often hundreds of miles in terrible weather, and going on


QU

stage.
Int: But what is it when you’re actually on stage … [fade]
Section 2
A talk from a member of the conservation Society about 'green cleaning' Good
M

morning everyone. It's a pleasure to be here as a representative of the


Conservation Society, to talk to you about "Green Cleaning", in other words

about ways you can help to save the environment at the same time as saving
money.
I'll start with saving money - as we're all interested in that, especially students
who are living on a tight budget. Probably none of you has sat down and
Y

calculated how much you spend on cleaning products each year everything from
DẠ

dishwashing detergent, window cleaners and so on through to shampoos and


conditioners for your hair, and then those disasters products to get stains out of
carpets, or to rescue burnt saucepans. I can see some nods of agreement, even if
you don't spend a lot of time on housework you'd end up spending quite a lot of
money over a period of time, wouldn't you? We can save money on products
and also use products which are cheap, biodegradable and harmless to the
environment - these I will call 'green' products. Unfortunately most cleaning
products on sale commercially are none of these, and many of our waterways
and oceans are polluted with bleach, dioxins, phosphates and artificial

L
colourings and perfumes. Also think how many plastic bottles each household

A
throws away over a year -they'll still be around in land-fill when you are
grandparents! So we often feel there's nothing we can do to make a difference,

CI
but we can. The actual 'recipes' are on handouts you can take at the end of the
talk: The sorts of ingredients I'm referring to are things like bicarbonate of soda,

FI
eucalyptus oil, ammonia, vinegar, lemons, pure soap. Lastly many people find
they're allergic to modem products, so for all you asthma sufferers keep
listening. Nothing in these recipes should cause you any problems, an end to

OF
itching and wheezing!
So let’s start with spills and stains. Soda water is wonderful as an immediate
stain remover: mop up the excess spill don't rub but apply soda water
immediately - it's great for tea coffee wine beer and milk - as is salt or

ƠN
bicarbonate of soda, which will absorb the stain - then vacuum when dry and
shampoo if necessary.
While we are talking about disasters lets quickly look at some others that can be
NH
avoided. Bicarbonate of soda is wonderful for removing smells, especially in
the fridge - an open box in the fridge will eliminate smells for up to three
months. And those terrible burnt saucepans? Either sprinkle with our good
friend bicarb again and leave it to stand, or cover with vinegar and a layer of
cooking salt.
Y

Bring it to the boil and simmer for ten minutes, then wash when cool. Much
QU

cheaper than a new saucepan! Then there are heat rings on wooden furniture.
Simply rub with a mixture of salt and olive oil, or for scratched furniture use
olive oil and vinegar. Now let's look at general cleaning - first the floors. If your
floor covering is made of slate, cork or ceramic tiles or lino it probably only
needs a mop or a scrub with vinegar in a bucket of water. Carpets can be
M

shampooed using a combination of pure soap washing soda, cloudy ammonia


and some boiling water. You put a small amount of this mixture onto the mark

on the carpet, rub with a cloth until it lathers and then wipe off the excess. A
smelly carpet can be deodorized by sprinkling bicarbonate of soda on the
surface, leaving overnight and vacuuming off the next day. Cleaning in the
kitchen, bathroom and toilet is the next section....
Y

Section 3
DẠ

Women are graduating from college at higher rates than ever before. Earning
more than half of bachelor’s degrees last year. Companies are increasingly
focused on the benefits of gender equality in the workplace. However, the
situation for women in the workplace is not changing fast enough.
Women make up 45% at entry level position across the workforce; which
decreases to 32% at management roles. At the top, women only make up
seventeen percent of the Csuite. At the current rate of progress, it would be one
hundred years before we see real equality in the C-suite. Contrary to popular

L
belief, women are leaving organisations at the same rate as men. However,

A
women are less eager to become a top executive citing stress and pressure as the
key deterrents.

CI
Most women see barriers to advancement and feel they have fewer
opportunities to advance due to their gender. While 74% of companies claim

FI
gender equality is a priority for their CEO, less than half of the employees
believe it. And only one-third feel it is a top priority for their direct manager.
More than 90% of women and men believe taking extended family leave will

OF
hurt their career. So while programs widely exist, participation is low.
This inequality even extends to the home. 41 % of working women report doing
more childcare than men, 30% report doing more chores but this does not need

ƠN
to be our reality while these facts are a wake-up call we can affect real change.
Here are five key actions companies can take.
1)Track key metrics to understand the problem: you cannot change why you
cannot see. 2)Demonstrate that gender equality is a top priority: employees must
NH
believe that their leaders want change.
3)Identify and interrupt gender bias : we all have unconscious biases to
overcome.
4)Rethink work: redefine what successful careers look like.
Y

5)Create a level playing field: make sure opportunities and advancement are
equitable. It is up to all of us to take action, to speed the process of gender
QU

equality so everyone benefits, individuals, organisation and society. To learn


more go to the womenintheworkplace.com.
Section 4
Over the past few years as first lady, I have had the extraordinary privilege of
M

traveling all across this country and everywhere I’ve gone and the people I've
met and the stories I’ve heard, I have seen the very best of the American spirit.

See, our life before moving to Washington was, was filled with simple joys.
Saturdays at soccer games, Sundays at grandma’s house, and a date night for
Barack and me was either dinner or movie because as an exhausted mum I
Y

couldn’t stay awake for both. Even back then when Barack was a senator and a
presidential candidate to me he was still the guy who picked me up for our dates
DẠ

in a car that was so wasted out that I could actually see the pavement going by
in a hole in the passenger side door. He was the guy whose proudest possession
was a coffee table he'd found in a dumpster.
Well today, after so many struggles and trials and moments that‘ve tested my
husband in ways I never could have imagined, I have seen first-hand that being
president doesn’t change who you are. No it reveals who you are.

L
When it comes to the health of our families, Barack refused to listen to all those
folks to told him to leave health reforms for another day, another president. He

A
didn’t care whether it was the easy thing to do politically, no that's not how he

CI
was raised. He cared that it was the right thing to do.
When we were first married our combined monthly student loan bill was
actually higher than our mortgage.

FI
Yeah!! We were so young, so in love, and so in debt.

OF
If we wanna give all of our children a foundation for their dreams and
opportunities worthy of their promise. If we wanna give them that sense of
limitless possibility, that belief that here in America there was always
something better out there if you're willing to work for it. Then we must work

ƠN
like never before, and we must once again come together and stand together for
the man we can trust to keep moving this great country forward.
My husband, our president, Barack Obama.
Thank you, God bless you, God bless America.
NH
Y
QU
M

Y
DẠ
KỲ THI HỌC SINH GIỎI CÁC TRƯỜNG THPT CHUYÊN
KHU VỰC DUYÊN HẢI VÀ ĐỒNG BẰNG BẮC BỘ
LẦN THỨ , NĂM 2023

L
ĐỀ THI MÔN: TIẾNG ANH 10
Thời gian: 180 phút (không kể thời gian giao đề)

A
Ngày thi:
ĐỀ XUẤT
(Thí sinh làm bài trực tiếp vào đề)

CI
(Đề thi gồm 12 trang)

FI
Điểm
Giám khảo 1 Giám khảo 2 Số phách
Bằng số Bằng chữ

OF
ƠN
A. LISTENING (50 points):
HƯỚNG DẪN PHẦN THI NGHE HIỂU
• Bài nghe gồm 4 phần; mỗi phần được nghe 2 lần, mỗi lần cách nhau 05 giây; mở đầu và
NH
kết thúc mỗi phần nghe có tín hiệu. Thí sinh có 20 giây để đọc mỗi phần câu hỏi.
• Mở đầu và kết thúc bài nghe có tín hiệu nhạc. Thí sinh có 03 phút để hoàn chỉnh bài trước
tín hiệu nhạc kết thúc bài nghe.
Y

• Mọi hướng dẫn cho thí sinh (bằng tiếng Anh) đã có trong bài nghe
QU

Part 1. You will listen to a short conversation about easy life cleaning services. For
questions 1-5, complete the form below by writing NO MORE THAN ONE WORD OR A
NUMBER for each answer. Write your answers in the corresponding numbered boxes
provided on the answer sheet. (10 points)
M

Easy Life Cleaning services


Basic cleaning package offered

• Cleaning all surfaces


• Cleaning the 1 __________ throughout the apartment
• Cleaning shower, sinks, toilet etc.
Y

Additional services agreed


DẠ

• Every week
− Cleaning the 2 ______________
− Ironing clothes – 3 ______________ only

Page 1 of 15
• Every month
− Cleaning all the 4 _______________ from the inside
Washing down the 5 ______________

L
Your answers:

A
1. 2. 3. 4. 5.

CI
Part 2. You will listen to a recording about going vegan and its effect on the environment. For
questions 6-10, decide whether the following statements are true (T) or false (F). Write your

FI
answers in the corresponding numbered boxes provided on the answer sheet. (10 points)
6. Vegans can consume eggs, milk and honey.
A. True B. False

OF
7. A large-scale study said that eating more meat plays a pivotal role in saving the planet.
A. True B. False
8. Raising livestock for meat results in deforestation.

ƠN
A. True B. False
9. 16% of greenhouse gasses comes from air released from cows’ digestive system.
A. True B. False
10. Beans and chickpeas can be good alternatives to meat in daily diet.
NH
A. True B. False
Your answers:
6. 7. 8. 9. 10.
Y

Part 3. You will hear a radio interview in which two young journalists — called Angus Brown
QU

and Yolanda Zouche — are talking about their work. For questions 11-15, choose the
answers which fit best according to what you hear. Write your answers in the corresponding
numbered boxes provided on the answer sheet. (10 points)
11. In Yolanda’s opinion, what is the most challenging aspect of her job?
M

A. finding suitable images to accompany articles


B. trying to locate interviewees

C. expressing herself coherently within a tight word limit


D. working to demanding time constraints
12. What is it about their work that Angus and Yolanda both enjoy?
A. the variety of the projects they get involved in
Y

B. the challenge of reporting news effectively


DẠ

C. the opportunity to meet interesting people


D. the appeal of searching out information
13. In Angus’s opinion, the advantage of online newspapers over print versions is that they
_______.
Page 2 of 15
A. are able to cover a greater range of topics.
B. can keep up with events as they develop.
C. allow interaction by readers.

L
D. reach a far wider public.
14. What made getting a job in journalism so hard for Angus?

A
A. the extent of competition for posts

CI
B. a low level of encouragement from others
C. a lack of previous professional experience
D. the difficulty of establishing useful contacts

FI
15. Yolanda believes that the essential requirement for a journalist entering the profession is
______.

OF
A. an ability to write persuasively.
B. a clear and logical mind.
C. a resourceful and confident character.
D. a mastery of interviewing techniques.

ƠN
Your answers:
11. 12. 13. 14. 15.
Part 4. You will listen to a recording about co-teaching in Herdorn Middle School. For
NH
questions 16-25, complete the summary by writing NO MORE THAN THREE WORDS in
each gap. Write your answers in the corresponding numbered boxes provided on the answer
sheet. (20 points)
16. Co-teaching is a model of ____________ in which two certified teachers work together in a
Y

classroom which contains students of varying levels of need and ability.


17. There is no _____________ between the roles of the teachers in the classroom.
QU

18. Each team often comprises a content area _____________ and a special education teacher.
19. The first teacher loves talking about individual students through all _____________.
20. She also appreciates _____________ as a good way to help struggling students.
21. The second teacher can get the information or _____________ from another teacher, which
M

is like crosspollinating.
22. Herdorn Middle School receives extra support to meet _____________.

23. Teachers are confident that the _____________ will be further narrowed.
24. The method helps dispel _____________ before students start practicing.
25. By not _____________ students, they can learn together and build on each other’s strengths.
Y

Your answers:
16. 21.
DẠ

17. 22.
18. 23.
19. 24.

Page 3 of 15
20. 25.

B. LEXICO - GRAMMAR (50 points)


Part 1. Choose one of the words marked A, B, C, or D which best completes each of the following

L
sentences. Write your answers in the corresponding numbered boxes. (20 points)
1. This picture book, the few pages _____ are missing, is my favorite.

A
A. for which B. of that C. to which D. of which

CI
2. He tried to run away from the police and was charged with _______ arrest.
A. suffering B. standing C. tolerating D. resisting
3. Your decision will ______ a great strain on our relationship.

FI
A. impose B. propose C. expose D. suppose
4. In _______ to your letter of 30 May, I am writing to accept your offer of £3,575 in compensation.
A. connection B. touch C. answer D. middle

OF
5. Scuba diving? Oh no, count me _______ - I hate being under water!
A. out B. in C. off D. on
6. The study showed that one in twelve women is_______ to develop breast cancer.
A. likely B. alike C. unlike D. like
7. _______ of all modern domestic poultry is the red jungle fowl is widely believed.

ƠN
A. The ancestor B. The ancestor is C. How the ancestor D. That the ancestor
8. Art critics do not all agree on what _______ a painting great.
A. qualities to make B. are the qualities for making
C. qualities make D. do the qualities that make
NH
9. Farmers supply crops with phosphorus in areas ________ have removed it from the soil.
A. because of long years of cultivation B. where long years of cultivation
C. with long years of cultivation D. by long years of cultivation
10. By the time his daughter graduates from the college, Mr. Brown ______.
A. retired B. will have retired C. will be retiring D. has tired
Y

11. Had it not been for the intolerable heat in the hall, they _____ much longer.
A. will stay B. would stay C. would be staying D.would have stayed
QU

12. Having been served dinner, _______.


A. the problem was discussed by the members of the committee.
B. the committee members discussed the problem.
C. it was discussed by the committee members the problem.
D. a discussion of the problem was made by the members of the committee.
M

13. Fast internet service in rural areas provided a new ________ for prosperity.
A. convenience B. opportunity C. possibility D. chance
14. I've been on the ________ all day, and I'm really tired.

A. go B. do C. rope D. take
15. Why don’t we eat on the porch for a ________?
A. similarity B. song C. difference D. change
16. The winning entry will be the first correct answer drawn at ________.
Y

A. hand B. random C. least D. last


17. It is high time for the critics _______ their minds to a new approach.
DẠ

A. to open B. had opened C. opened D. to have opened


18. It’s amazing how Jenny act as though she and Darren_______ serious problems at the moment.
A. hadn’t been having B. hadn’t had C. weren’t having D. aren’t having
19. But the defence argues that Mr. Kwoyelo is a victim, claiming he was abducted as a child while

Page 4 of 15
walking to school and forced to follow ________ for fear of being shot.
A. rules B. guidance C. directions D. orders
20. There was widespread outrage when it was discovered that a known pedophile had been given a job
at the school. “You don’t let the ______ guard the henhouse” said one of the protesters.

L
A. fox B. cat C. duck D. fish
Your answers:

A
1. 5. 9. 13. 17.

CI
2. 6. 10. 14. 18.
3. 7. 11. 15. 19.

FI
4. 8. 12. 16. 20.

OF
Part 2. Complete each sentence with one suitable particle or preposition. Write your answers in the
box provided. (10 points)
1. Could you look ____ these old newspapers for articles about the environment?
2. Can you hold ____ till I’ve finished talking the manager? Then I’ll be right with you.
3. She hasn’t told us the whole story. I’m sure she’s keeping something ____.

ƠN
4. After the first year, I couldn’t cope with university, so I decided to drop ____.
5. Rubber boots are impervious ____ water.
6. It wouldn’t be compatible ____ the public safety to let animals off with a slight punishment.
NH
7. Leisure for study is congenial ____ his tastes.
8. Geology is a science I am not conversant ____.
9. He may be slow at his work, but he is very quick ____ the uptake.
10. The young couple lay ____ money for their old age.
Y

Your answers:
QU

1. 2. 3. 4. 5.
6. 7. 8. 9. 10.

Part 3. Write the correct form of the words given in the brackets. Write your answers in the spaces
provided below. (10 points)
M

1. I was late because I _____________ how much time I will need. ESTIMATE

2. There was ice on the pavement which made it very difficult to walk as it was so SLIP
_____________.
3. Despite the star-studded cast, the film was only _____________ successful. PART
Y

4. How can you _____________ the fact that some people live in mansions while JUST
others live in slums?
DẠ

5. My brother lives in an attractive _____________ part of Paris. RESIDENT


6. The car in front was going very slowly, so John__________________ it. TAKE
7. I _____________ met an old friend last week. EXPECT

Page 5 of 15
8. There’s been yet another _____________ of cholera in Delhi. BREAK
9. The lovers stood, hand in hand, gazing at the _____________ sky. STAR
10. There was a heavy _____________ yesterday afternoon which completely POUR

L
ruined the church Garden Party.

A
Your answers:
1. 2. 3. 4. 5.

CI
6. 7. 8. 9. 10.

C. READING (60 points)

FI
Part 1. For questions 1–10, read the text below and decide which answer (A, B, C or D) best fits each
gap.
FOOTBALL AS AN ART FORM

OF
When filmmakers Douglas Gordon and Phillipe Parreno set out to make an art house movie about the
legendary French footballer Zinedine Zidane, they chose to film just one match between Real Madrid,
the club for which he was playing at the (1) ______, and their great rivals Villareal. But instead of
following the progress of the match, the ninety-minute film would show something that had not been
seen before; the (2) ______ detailed movements of one man during an entire top-level football match.

ƠN
They hoped that the audience would disengage from the match itself, and focus on this portrait of
greatness. Every (3) ______ gesture would be captured and they would see all of the player's grace,
athleticism and competitiveness in great detail.
The (4) ______ film is a fascinating work. Those who are not regular watchers of football will be
astonished at how (5) ______ Zidane becomes actively involved in the game. For much of the ninety
NH
minutes he moves around the field relatively slowly; saying nothing, expressing even less, and only
occasionally (6) ______ into a lethargic jog. And then the ball arrives at his feet, and there is a flurry of
bewildering activity. The cameras (and there are seventeen of them (7) ______ on him) struggle to keep
up. The defenders don't (8) ______ a chance. In a few touches, a couple of checks and feints, Zidane
has (9) ______ them all behind. He crosses from the tightest of (10) ______ and his team-mate is left
with the simplest of headers to score a goal.
Y

1. A. point B. moment C. time D. occasion


2. A. clear B. steady C. precise D. slow
QU

3. A. one B. single C. lone D. sole


4. A. following B. resulting C. concluding D. arising
5. A. partly B. scarcely C. rarely D. hardly
6. A. breaking B. changing C. opening D. starting
7. A. trained B. looking C. pointed D. staring
M

8. A. gain B. hold C. stand D. earn


9. A. missed B. left C. lost D. dropped
10. A. places B. positions C. areas D. angles

Your answers:
1. 2. 3. 4. 5.
6. 7. 8. 9. 10.
Part 2. Read the text and think of the word which best fits each gap. Use only ONE word in
each gap.
Y

Getting ready for Mars


DẠ

The Mars 500 project (1) ______ an experiment that simulated a return mission to Mars. Spending 18
months in a sealed facility in Moscow (2) ______ access to natural light or fresh air, six men were
monitored as they attended (3) ______ their daily duties. A study into (4) ______ each of them coped
with the psychological and physical constraints of the mission has found that there were wide
differences in their wake-sleep patterns. For example, (5) ______ most of the crew began to sleep for

Page 6 of 15
longer periods as the mission progressed and boredom set in, one individual slept progressively less,
resulting (6) ______ him becoming chronically sleep-deprived towards the end of the (7) ______.
Identifying bad sleepers could be important on a real Mars mission, during (8) ______ people are
required to be constantly alert even when days are tediously similar. Researchers warn that for any
astronaut heading to Mars, exciting as the trip might initially seem, (9) ______ could be problems with

L
stress brought on by the monotony of routine. However, they also report that (10) ______ some personal

A
tensions between crew members, there was overall harmony within the group.
Your answers:

CI
1. 2. 3. 4. 5.
6. 7. 8. 9. 10.

FI
Part 3. You are going to read an article about the history of the electric guitar. For
questions 1-10, choose the answer (A, B, C or D) which you think fits best according to the
text.

OF
SUPER HUMANS
Sit down with an anthropologist to talk about the nature of humans, and you are likely to hear
this pearl of wisdom: 'Well, you have to remember that 99 percent of human history was spent
on the open savanna in small bands of hunter-gatherers.' It's a classic scientific cliché, and it's

ƠN
true. Indeed, those millions of ancestral years produced many of our hallmark traits — upright
walking and big brains, for instance. Of course, those useful evolutionary innovations come at a
price: aching backs from our bipedal stance and existential despair from our large, self-
contemplative cerebral cortex.
NH
Compounding the challenges of those trade-offs, the world we have invented is dramatically
different from the one to which our bodies and minds are adapted. Have your dinner delivered to
you instead of chasing it down on foot; log in to Facebook to interact with your nearest and
dearest instead of spending most of the day with them. But this is where the utility of the
Y

anthropologist's cliche for explaining the human condition ends.


QU

The reason for this mismatch between the setting we evolved to live in and the situations we
encounter in our modern era derives from another defining characteristic of our kind, arguably
the most important one: our impulse to push beyond the limitations evolution imposed on us by
developing tools to make us faster, smarter and longer-lived. Science is one such tool — an
M

invention that requires us to break out of our Stone Age seeing-is-believing mindset so that we
can clearly see the next hurdle we have to overcome, be it a pandemic flu or climate change.

You could call it the ultimate expression of humanity's singular drive to aspire to be better than
we are.

To understand how natural selection moulded us into the unique primates we have become, let
Y

us return to the ancestral savanna. There the sun was hotter and nutritious plant foods were
scarcer. In response, our predecessors lost their hair and their molars dwindled as they
DẠ

abandoned a tough vegetarian diet for one focused in part on meat from grassland grazers.
Meanwhile, the selective demands of food scarcities sculpted our distant forebears into having a
body that was extremely thrifty and good at storing calories. Now, having inherited that same
metabolism, we hunt and gather burgers as diabetes becomes a worldwide scourge. Or consider
Page 7 of 15
how our immune systems evolved in a world where one hardly ever encountered someone
carrying a novel pathogen. Today, if you sneeze near someone in an airport, your rhinovirus
could be set free 12 time zones away by the next day.

L
As regards behavior, our abilities abound. We can follow extraordinarily complex scenarios of

A
social interaction and figure out if a social contract has been violated. And we are peerless when
it comes to facial recognition: we even have an area of the cortex in the fusiform gyrus that

CI
specializes in this activity.

The selective advantages of evolving a highly social brain are obvious. It paved the way for us

FI
to finetune our capabilities for reading one another's mental states, to excel at social
manipulation and to deceive and attract mates and supporters. Among Americans, the extent of

OF
social intelligence in youth is a better predictor of adult success in the occupational world than
are academic scores. Indeed, when it comes to social intelligence in primates, humans reign
supreme. The social brain hypothesis of primate evolution is built on the fact that across primate
species the percentage of the brain devoted to the neocortex correlates with the average size of
the social group of that species. This correlation is more dramatic in humans than in any other

ƠN
primate species.

The fact that we have created this world proves a point — namely, that it is in our nature to be
NH
unconstrained by our nature. Science is one of the strangest, newest domains where we
challenge our hominid limits. It also tests our sense of what is the norm, what counts as better
than well and it challenges our sense of who we are. Thanks to science, human life expectancy
keeps extending, our average height increases, our intelligence test scores improve and we
eventually break every world record. But when it comes to humans becoming, on average,
Y

smarter, taller and better at athletics, there is a problem: Who cares about the average? As
individuals, we want to be better than other individuals. Our brain is invidious, comparative and
QU

more interested in contrasts, a state that begins with sensory systems that do not normally tell us
about the quality of a stimulus but instead about the quality relative to the stimuli around it.

1. According to the writer, the anthropological cliché to explain the nature of mankind ______.
M

A. needs some slight modifications


B. requires little analysis
C. should be considered paradoxical

D. is limited in scope
2. Humankind will only be able to use science to progress if ______.
A. ethical considerations are ignored
B. we discard an outdated approach to acquiring knowledge
Y

C. our drive to eliminate barriers continues


DẠ

D. the philosophy we adopt can be widely understood


3. Our ancient ancestors lived in a world where ______.
A. the necessity to hunt for food led to good health
B. it was vitally important to have a balanced diet

Page 8 of 15
C. isolation allowed them to develop immunity from disease
D. their restricted movement protected them from illness
4. The word “peerless” in paragraph 5 is closest in meaning to ______.
A. friendless

L
B. unsurpassed

A
C. uncompetitive
D. flawless

CI
5. Having a highly social brain ______.
A. allows us to create groups with more members
B. prevents us from being misunderstood

FI
C. causes us to be more devious
D. helps us to read other people's minds

OF
6. The way we tend to think ______.
A. forces us to overlook our shortcomings
B. has enhanced our understanding of sense perception
C. distorts our perception of the notion of average
D. makes us less likely to be concerned with absolutes

ƠN
7. The word “invidious” in paragraph 7 mostly means ______.
A. prejudiced
B. preordained
NH
C. unfathomable
D. attitudinal
8. Which of the following square brackets [A], [B], [C], or [D] best indicates where in the
paragraph the sentence "We are no strangers to going out of bounds." can be inserted?
[A] The fact that we have created this world proves a point — namely, that it is in our nature to
Y

be unconstrained by our nature. [B] Science is one of the strangest, newest domains where we
challenge our hominid limits. It also tests our sense of what is the norm, what counts as better
QU

than well and it challenges our sense of who we are. [C] Thanks to science, human life
expectancy keeps extending, our average height increases, our intelligence test scores improve
and we eventually break every world record. [D] But when it comes to humans becoming, on
average smarter, taller and better at athletics, there is a problem: Who cares about the average?
M

As individuals, we want to be better than other individuals. Our brain is invidious, comparative
and more interested in contrasts, a state that begins with sensory systems that do not normally
tell us about the quality of a stimulus but instead about the quality relative to the stimuli around

it.
A. [A] B. [B] C. [C] D. [D]
9. It can be inferred from the passage that
A. there are no limits to human capabilities
Y

B. we will be able to adapt to harsh environments


DẠ

C. humankind's evolutionary path will not be smooth


D. our knowledge of the past is crucial to our future
10. Which of the following is the main idea of the passage?
A. Social intelligence enables Americans to be both academically and professionally successful.

Page 9 of 15
B. Science helps prolong human life and improve human intelligence to break all world records.
C. Our evolutionary limits can be exceeded and that's what sets us apart from other species.
D. A highly evolved social brain paved the way for humans to be able to read and distort others'
thinking.

L
Your answers:

A
1. 2. 3. 4. 5.
6. 7. 8. 9. 10.

CI
Part 4. Read the following passage and do the tasks that follow.
The strange journey that made Great Britain

FI
As evidenced by its geological make-up, the island of Great Britain has a rich and varied pre-
history.
First of all, to clear up a common misconception, Great Britain refers, to the largest island of the

OF
British Isles, (Great in this case meaning Big). It is made up of England, Scotland and Wales,
and is the eighth largest island in the world. There are between around 4,000 and 6,289 other
islands in the group (depending on the size of rock we are talking about and whether you can
count it as an island if it is only one at high tide). Of these, around 200 are inhabited.

ƠN
In order to grasp some of the more difficult ideas behind the prehistory of Great Britain, it helps
to look at the Earth's geological past, starting with its formation 4.5 billion years ago. It is
currently thought that the Earth and the planets were formed from dust left over from when the
Sun was created. What is significant is that the Earth in its infancy was in a largely molten state,
NH
with constant volcanic activity and regular collisions with other bodies (one of which is thought
to have created the Moon).
Although scientific opinion varies on many of these points, the generally accepted sequence of
events is as follows: the temperature began to fall, forming the Earth's crust. At the same time,
Y

water appeared in the atmosphere. The volcanic activity released gases, creating an atmosphere,
and the water condensed, forming the oceans. Molten material solidified, forming continents
QU

which broke apart and re-formed many times over the next couple of billion years. These
continents migrated around the surface of the Earth and it is in the study of this continental drift
that we can trace Great Britain's past.
At three stages in the Earth's geological past, supercontinents were formed, so that there was
only one major land mass, while most of the rest of the Earth was covered with water. These
M

supercontinents have been named Rodinia, Pannotia and Pangaea. The earliest of these, Rodinia,
broke apart 750 million years ago, and 200 million years later, the land masses joined together

again to form the second supercontinent, Pannotia.


Around 500 million years ago, in the period known as the Cambrian, Great Britain was largely
underwater in the southern hemisphere, about the same distance from the South Pole as New
Zealand is today. Scotland was attached to the plate with North America, while England and
Y

Wales were attached to Scandinavia. Around the time of the Silurian period, the two plates
collided, bringing Scotland into contact with England. The collision caused what we call the
DẠ

Caledonian orogeny? - what are now the Scottish Highlands.


In the Devonian period of about 400 million years ago, the continuing rise of the Caledonian
orogeny caused the majority of Great Britain to be above sea level. Africa and South America
then collided with North America and Europe. Around 100 million years later, during what we
Page 10 of 15
call the Carboniferous period, Great Britain was in the equatorial forest belt of Pangaea, the last
supercontinent. It was the vegetation from this forest that created the coal mined in Great Britain
for centuries. Then, around 200 million years ago, Britain migrated to the northern desert belt,
which explains why sandstone is so prevalent in the bedrock of Great Britain.

L
During the Jurassic and Cretaceous periods, between 190 and 65 million years ago, Pangaea

A
broke apart and, due to rifting, Great Britain was flooded by the newly created Atlantic Ocean.
Evidence of this can be found in the chalk under the soil, mainly in southern England. The chalk

CI
is made up of trillions of tiny sea creatures that died and formed a sediment at the bottom of the
sea. After sea levels fell and rose several times, the continents we know today were established
and the island of Great Britain was more or less where it is now.

FI
Except that it was not an island. Throughout all of this movement, it was part of a larger
continent. It was as recently as about 9,000 years ago that it became an island, when the end of

OF
the last ice age made sea levels rise yet again to create the English Channel and cut Great Britain
off from continental Europe. The highlands and lowlands, the chalk cliffs, the layers of
sandstone, the seams of salt, the precious metals, and the fossil fuels; all of these provide clues
to the many, many changes that Great Britain went through in geological history.
Questions 1-5

ƠN
Complete each sentence with the correct ending A-I below.
1. Currently, the island of Great Britain
2. The supercontinent of Rodinia
NH
3. The Scottish Highlands mountain range
4. The coal under the ground in Great Britain
5. Apart from fossil fuels, the land under Great Britain
A. broke apart and later re-formed as Pannotia.
B. caused a rise in sea levels.
Y

C. dates back to a time when it was in an equatorial forest.


D. led to the cooling of the Earth.
QU

E. contains salt, sandstone and chalk.


F. is surrounded by several thousand smaller islands.
G. used to be named Pangaea.
H. was caused by flooding and rifting.
M

I. is the result of two land masses colliding.


Questions 6-10
Complete the timeline below. Choose NO MORE THAN THREE WORDS from the text

for each answer.


Around 4.5 billion years ago: The Earth was formed, remaining in a molten form with a
constantly changing surface due to frequent impacts and 6. ______________________.
Y

The Earth's crust was formed due to the cooling of the planet.
DẠ

Oceans were formed when water from the atmosphere 7. ______________________.

The first continents were formed when molten material solidified.

Page 11 of 15
Around 500 million years ago:
8. ______________________ were united at the time when the Scottish Highlands were formed.

L
Around 300 million years ago:

A
Great Britain was in a(n) 9. ______________________.

CI
Around 200 million years ago:
The presence of sandstone today reveals that Britain was in the northern desert belt.

FI
Around 100 million years ago: The chalk in Britain today comes from a build-up of dead sea
creatures from the time when the area was 10. ______________________ after the Atlantic

OF
Ocean was formed.
Your answers:
1. 2. 3. 4. 5.
6. 7. 8. 9. 10.

ƠN
D. WRITING (50 points)
Part 1. The table below shows information about age, average income per person and
population below poverty line in three states in the USA. (15 points)
Summarise the information by selecting and reporting the main features, and make comparisons
NH
where relevant.
Y
QU
M

……………………………………………………………………………………………………
……………………………………………………………………………………………………
……………………………………………………………………………………………………
Y

……………………………………………………………………………………………………
……………………………………………………………………………………………………
DẠ

……………………………………………………………………………………………………

Page 12 of 15
……………………………………………………………………………………………………
……………………………………………………………………………………………………
……………………………………………………………………………………………………

L
……………………………………………………………………………………………………

A
……………………………………………………………………………………………………

CI
……………………………………………………………………………………………………
……………………………………………………………………………………………………
……………………………………………………………………………………………………

FI
……………………………………………………………………………………………………

OF
……………………………………………………………………………………………………
……………………………………………………………………………………………………
……………………………………………………………………………………………………
……………………………………………………………………………………………………

ƠN
……………………………………………………………………………………………………
……………………………………………………………………………………………………
……………………………………………………………………………………………………
NH
……………………………………………………………………………………………………
……………………………………………………………………………………………………
……………………………………………………………………………………………………
Y

……………………………………………………………………………………………………
……………………………………………………………………………………………………
QU

……………………………………………………………………………………………………
……………………………………………………………………………………………………
……………………………………………………………………………………………………
M

……………………………………………………………………………………………………

Part 2. Write an essay of about 250 words to express your opinion on the following topic. (35 points)

Some people think that the main purpose of school is to turn children into good citizens and
workers, rather than benefit them as individuals.
To what extent do you agree or disagree?
Y
DẠ

……………………………………………………………………………………………………
……………………………………………………………………………………………………
……………………………………………………………………………………………………

Page 13 of 15
……………………………………………………………………………………………………
……………………………………………………………………………………………………
……………………………………………………………………………………………………

L
……………………………………………………………………………………………………

A
……………………………………………………………………………………………………

CI
……………………………………………………………………………………………………
……………………………………………………………………………………………………
……………………………………………………………………………………………………

FI
……………………………………………………………………………………………………

OF
……………………………………………………………………………………………………
……………………………………………………………………………………………………
……………………………………………………………………………………………………
……………………………………………………………………………………………………

ƠN
……………………………………………………………………………………………………
……………………………………………………………………………………………………
……………………………………………………………………………………………………
NH
……………………………………………………………………………………………………
……………………………………………………………………………………………………
……………………………………………………………………………………………………
Y

……………………………………………………………………………………………………
……………………………………………………………………………………………………
QU

……………………………………………………………………………………………………
……………………………………………………………………………………………………
……………………………………………………………………………………………………
M

……………………………………………………………………………………………………
……………………………………………………………………………………………………

……………………………………………………………………………………………………
……………………………………………………………………………………………………
……………………………………………………………………………………………………
Y

……………………………………………………………………………………………………
DẠ

……………………………………………………………………………………………………
……………………………………………………………………………………………………

Page 14 of 15
……………………………………………………………………………………………………
……………………………………………………………………………………………………
……………………………………………………………………………………………………

L
……………………………………………………………………………………………………

A
……………………………………………………………………………………………………

CI
……………………………………………………………………………………………………
……………………………………………………………………………………………………
……………………………………………………………………………………………………

FI
……………………………………………………………………………………………………

OF
……………………………………………………………………………………………………
……………………………………………………………………………………………………
……………………………………………………………………………………………………
……………………………………………………………………………………………………

ƠN
……………………………………………………………………………………………………
……………………………………………………………………………………………………
……………………………………………………………………………………………………
NH
……………………………………………………………………………………………………
-------------- THE END --------------
(Thí sinh không được sử dụng tài liệu. Cán bộ coi thi không giải thích gì thêm)
Y
QU
M

Y
DẠ

Page 15 of 15
KỲ THI HỌC SINH GIỎI CÁC TRƯỜNG THPT CHUYÊN
KHU VỰC DUYÊN HẢI VÀ ĐỒNG BẰNG BẮC BỘ

L
LẦN THỨ , NĂM 2023

IA
HƯỚNG DẪN CHẤM MÔN: TIẾNG ANH 10
Thời gian: 180 phút (không kể thời gian giao đề)
Ngày thi:

IC
(Đáp án gồm 07 trang)

A. LISTENING (50 points):

F
LISTENING

OF
Part 1: 2 points/ correct answer
1. FLOOR(S) 2. FRIDGE 3. SHIRTS 4. WINDOWS 5. BALCONY
Tapescript
JACINTA: Hello, Easy Life Cleaning Services, Jacinta speaking.
CLIENT: Oh hello. I'm looking for a cleaning service for my apartment - do you do
ƠN
domestic cleaning?
JACINTA: Sure.
CLIENT: Well, it’s just a one-bedroom flat. Do you have a basic cleaning package?
JACINTA: Yes. For a one-bedroom flat we’re probably looking at about two hours for a
NH

clean. So we’d do a thorough clean of all surfaces in each room, and


polish them where necessary. Does your apartment have carpets?
CLIENT: No, I don’t have any, but (1) the floor would need cleaning.
JACINTA: Of course - we’d do that in every room. And we’d do a thorough clean of the
kitchen and bathroom.
Y

CLIENT: OK.
JACINTA: Then we have some additional services which you can request if you want -
QU

so for example, we can clean your oven for you every week.
CLIENT: Actually, I hardly ever use that, but (2) can vou do the fridge?
JACINTA: Sure. Would you like that done every week?
CLIENT: Yes, definitely. And would ironing clothes be an additional service you can
do?
M

JACINTA: Yes, of course.


CLIENT: It wouldn’t be much, (3) just my shirts for work that week.

JACINTA: That’s fine. And we could also clean your microwave if you want.
CLIENT: No, I wipe that out pretty regularly so there's no need for that.
JACINTA: We also offer additional services that you might want a bit less often, say
every
month. So for example, if (4) the inside of your windows need cleaning, we
Y

could do that.
DẠ

CLIENT: Yes, that'd be good. I’m on the fifteenth floor, so the outside gets done
regularly by specialists, but the inside does get a bit grubby.
JACINTA: And we could arrange for your curtains to get cleaned if necessary.
CLIENT: No, they’re OK. But (5) would vou be able to do something about the
balcony? It’s quite small and I don't use it much, but it could do with a wash
every month or so.
JACINTA: Yes, we can get the pressure washer onto that.
Part 2: 2 points/ correct answer

L
6. FALSE 7. FALSE 8. TRUE 9. TRUE 10. TRUE
Tapescript

IA
Johnah: Ruby, happy World Vegan Day. I made you this cake.
Ruby: Thanks Johnah. It's definitely vegan. Vegans can be hard to cook for, like

IC
vegetarians. They don't eat meat. (6) But they also rule out other animal products like
eggs, milk, and sometimes even honey. There are different reasons people choose to be
vegan, like animal welfare or as a way to eat healthier. But there's another reason that's been

F
getting a lot of attention recently. (7) Last month, a big study came out looking into the
effect our food has on the environment. And it found that eating less meat is one of the

OF
most important things we can do to help the planet. You see, a huge amount of the
Earth's resources goes into (8) raising livestock for meat. In some places, forests are
cleared to make space for animals and to grow their food. They also use a lot of water,
and these ones in particular create a lot of carbon emissions. (9) Cow burps and farts
account for about 16% of global greenhouse gas emissions. The study says to prevent
ƠN
permanent damage to our planet, the average world citizen needs to eat 75% less beef, 90%
less pork, and 50% fewer eggs. Of course, not everyone's going to go vegan or give up
eating meat completely. (10) But swapping your beef for beans or your chicken for
chickpeas every so often might not be such a bad idea.
NH

Part 3. 2 points/ correct answer

11. D 12. A 13. B 14. A 15. C


Tapescript
Y

Interviewer: Today I’m talking to two young journalists - Angus Brown, a news reporter on
a national daily, and Yolanda Zouche, a features writer on a London evening paper. You’re
QU

both not only successful in your careers, but enthusiastic about them too. What would you
say is the most challenging aspect of your work. Yolanda?
Yolanda: I’m tempted to say nothing really — I like it all, some things more, some less.
Our features are a mix of things that have been thought of and researched and written in
advance, and more urgent, topical pieces with a quick turnaround. I’m sometimes sent out
M

on a story that’s needed for the next day. (11) It’s pretty scary when you know you’ve got
just a few hours and it can involve finding people whose addresses you don’t know.

That’s the same with any story, of course. You’ll have to think up ideas for pictures to
go with it and, write your piece to a fixed word count - but you soon find yourself doing
this automatically.
Interviewer: And what do you both enjoy about your work?
Yolanda: Well, I suppose I’m quite a nosy kind of person, so I love digging out stories
Y

that haven’t been reported — I’ll go from interviewing a singer about a forthcoming tour
DẠ

to investigating some crime, all in one day perhaps. I’d be bored otherwise, I think, and
then it’s all got to be presented in a way that will make people want to read it.
Interviewer: And you, Angus?
Angus: There’s no way I could describe ‘my typical working day’ that’s the sort of question
people often ask. And that’s really why I enjoy it so much I suppose (12) so many
fascinating people to talk to, and so much to learn.
Interviewer: Like several other papers nowadays, Angus, yours has an online version. What
do you think is the greatest significance of the change to digital?

L
Angus: It’s completely altered the way we think about the news. Things move so very

IA
quickly, and we really do need to stay receptive to all the opportunities the medium has
opened up. I think maybe more people are better informed these days. We’ve certainly
become (13) a rolling news operation I can now file a story as soon as it breaks, early

IC
and then update digitally as I find out more. And if I get anything wrong, people are all
too quick to point this out on social media.
Interviewer: Finding a job is not easy for anyone - what was it like for you Angus?

F
Angus: It certainly wasn’t easy. People continually told me that print journalism was dead,
and there was no money in it, (14) but in fact I had to beat 1,800 applicants to get on the

OF
graduate scheme. Before that I’d been on a couple of temporary work experience
placements with a local paper - you can do these to get a taster of the work, but there’s
no salary ... I managed financially because I was doing some part-time teaching at the same
time. You really need some sort of support network of people with influence too luckily the
ƠN
work experience provided that.
Interviewer: Tell us what qualities you think a would-be journalist needs, Yolanda.
Yolanda: There’s no straightforward answer to that - I’ve got an English degree and Angus
is a historian, I believe ... that’s his academic background. There are plenty of good courses
NH

around, and eventually you’ll have to get to grips with some of the technical stuff - like
media law, and so on. (15) But remember no one is ever going to employ a shy retiring
type with no ideas. Being able to write clearly and quickly, and think through a tangle of
information is obviously helpful but these are things you can develop with practice.
Part 4: 1 point/ correct answer
PART 4
Y

16. instructional delivery


QU

17. clear delineation


18. general educator
19. phases
20. flexible grouping
21. lesson plan ideas
M

22. student achievement benchmarks


23. achievement gap

24. misconceptions
25. separating
Tape script
On first glance, this may look like a traditional classroom with one teacher in charge, but on
closer inspection, it appears that there are two teachers leading instructions. Well, that's
Y

because there are. This is Herndon middle school in this class is an example of co teaching.
(16) Co teaching is a model of instructional delivery in which there are two certified
DẠ

teachers in a classroom which contains students of varying levels of need and ability.
The great thing about Lisa and Jennifer is when you walk into their classroom, (17) there's
no clear delineation between this is the general ed teacher in the classroom, this is the
special ed teacher in the classroom. There are two teachers in that classroom and they both
have shared responsibility for every student. We really are making strides at Herndon to
moving away from that mine and their perception of students and who belongs to who. If
they're a Herndon middle school student, they belong to every teacher in this building. And
Lisa and Jennifer exemplify that. (18) Herndon has more than 20 co teaching teams,

L
which usually pair a content area general educator with a special education teacher.

IA
These teams allow teachers to share expertise and actively engage, challenge and motivate
students. Most of my teaching career has been in a traditional model where I was a teacher
for the entire classroom. (19) And what I love about teen teaching is through all phases,

IC
we're talking about individual students and what their needs are. And I love working
with a special education teacher who has the background for (20) ways to help students
that are struggling. So we plan there and then in the classroom with flexible grouping

F
with students that need different things at the same time. We're able to do that together
in the classroom. So I feel much more effective through the entire process. And I'm a first-

OF
year teacher and it's made the transition really easy. And I love how I get the English
expertise from Miss Merrick and then I can kind of bring my special ed learning strategies
expertise to that as well. So it's very helpful because I also co teach with another teacher and
I share a room with another teacher. (21) I also get information from them or lesson plan
ƠN
ideas from them and I can bring them to her and vice versa. So we're kind of like
crosspollinating, which is really great. As part of FCPS’s commitment to closing the
achievement gap, Herndon is one of 30 schools designated as priority schools by the school
system. (22) Herndon middle gets additional support in order to meet student
NH

achievement benchmarks. The staff at Herndon believes that co teaching is an integral part
of student success. (23) I absolutely think this will help close the achievement gap
because we are able to focus more on individual student needs and with more eyes and
more ideas, looking at what students need and then being able to work with them, it helps
tremendously. And I think the students really like it as well. Because they have two different
Y

approaches. They see us cooperating. And I think that lends to a feeling that we're all in this
together. We're all trying to help everyone achieve the most that they can. I definitely think
QU

using co teaching helps close the achievement gap because we have two minds thinking of
lessons, two minds thinking about how we can teach the students in the most effective
manner. (24) It decreases wait time, transition time, and really allows us as teachers to
get to the students and see if there's any misconceptions at all before they start
practicing. And the reason I think that is because when you have two teachers who are
M

working together to truly build and deliver dynamic instruction, that you can meet those
individual needs of every student in the classroom. (25) And I also think that by mixing

students, by not creating 21st century segregation, by here are our achieving kids, here
are non-achieving kids, and separating them, that those students can then learn
together and build on each other's strengths. That's a good.
Y
DẠ
B. LEXICO - GRAMMAR (50 points)
Part 1. Choose the best option A, B, C, or D to complete the following sentences and write your

L
answers in the corresponding numbered boxes. (20 points)
1. C 5. A 9. B 13. B 17. A

A
2. D 6. A 10. B 14. A 18. C
3. A 7. D 11. D 15. B 19. D

CI
4. C 8. C 12. B 16. B 20. A

FI
Part 2. Complete each sentence with one suitable particle or preposition. Write your answers in the
box provided. (10 points)
1. through 2. on 3. back 4. out 5. to

OF
6. with 7. to 8. with 9. on 10. aside
Part 3. Write the correct form of the words given in the brackets. Write your answers in the spaces
provided below. (10 points)
1. underestimated 6. overtook
2. slippery 7. unexpectedly

ƠN
3. partially 8. outbreak
4. justify 9. starry
5. residential 10. downpour
NH
C. READING (60 points)
Part 1. Read the following passage and decide which answer (A, B, C, or D) best fits each
gap. Write your answers in corresponding numbered boxes. (15 points) – (1pt for each correct
answer)
Y

1. C 2. C 3. B 4. C 5. C
6. A 7. A 8. C 9. B 10. D
QU

Part 2. Read the text below and think of the word which best fits each space. Use only ONE
word in each space. Write your answers in the corresponding numbered boxes. (15 points) –
(1.5 pts for each correct answer)
1. was 2. without 3. to 4. how 5. while/whilst/whereas/although/though
6. in 7. mission 8. which 9. there 10. Despite
M

Part 3. Read the passage and choose the best option A, B, C, or D to answer the questions.
Write your answers in the corresponding numbered boxes. (15 points) – (1pt for each correct

answer)
1. D 2. B 3. D 4. B 5. A
6. D 7. A 8. B 9. C 10. C
Part 4. Read the text and do the following tasks. (15 points) – (1.5pts for each correct answer)
Y

1. F 2. A 3. I 4. C 5. E
DẠ

6. volcanic activity 7. condensed 8. Scotland and England


9. equatorial forest (belt) 10. flooded

Page 5 of 7
D. WRITING (50 points)

Part 1: Graph writing (15 points)

L
Contents (1.0 point)

A
- The report MUST cover the following points:
•Introduce the chart (0.2 point) and state the striking features (0.2 point)

CI
•Describe main features with relevant data from the charts and make relevant comparisons (0.6
point)
- The report MUST NOT contain personal opinions. (A penalty of 1 point to 2 points will be

FI
given to personal opinions found in the answer.)
Language use (0.5 points)

OF
The report should:
- demonstrate a wide variety of lexical and grammatical structures,
- have correct use of words (verb tenses, word forms, voice,…); and mechanics (spelling,
punctuations,...).

ƠN
MODEL REPORT
The table provides a comparison of three American states in terms of average income per capita,
age distribution, and the proportion of population living under the poverty line.

Overall, Utal had the youngest population, whereas Florida had the highest percentage of
NH
elderly citizens. Furthermore, even though California turned out to have a higher income per
capita, it registered the highest percentage of population living below the line of poverty.

In terms of age-related statistics, 28% of Utal’s population was under the age of 18, compared
to 17% in California and 16% in Florida. Conversely, Florida had the highest percentage of
Y

citizens above the age of 60, at 24%, while California and Utal had only 13% and 8%
QU

respectively.

With regards to the economic status, the average income per person amounted up to 23 000 and
22 000 in California and Florida respectively, while the average inhabitant in Utal was earning
up to 17 000 per month. However, despite having the highest income per capita, California also
M

registered 16% of population under the poverty line, which was significantly higher than 12% in
Florida and 9% in Utal.

Part 2: Essay writing (35 points)

The mark given to this part is based on the following criteria:


1. Content: (35% of total mark)
Y

a. Providing all main ideas and details as required


b. Communicating intentions sufficiently and effectively
DẠ

2. Organization & Presentation: (30% of total mark)


a. Ideas are well organized and presented with coherence, cohesion, and clarity
b. The composition is well - structured

Page 6 of 7
3. Language: (30% of total mark)
a. Demonstration of a variety of vocabulary and structures appropriate to the level of English language
gifted upper - secondary school students
b. Good use and control of grammatical structures

L
4. Punctuation, and spelling and handwriting (5% of total mark)
a. Good punctuation and no spelling mistakes

A
b. Legible handwriting
MODEL ESSAY

CI
Schools are often considered as places where students learn how to become responsible, productive
members of society, rather than prioritize their individual needs and desires. Despite this, I would argue
that schools should strike a healthy balance between community-oriented and individual-oriented

FI
approaches of teaching children.

There are clear benefits to transforming children into good citizens and productive employees. The

OF
primary advantage has to do with safety. Educating children to understand the need to obey rules and
respect others would make the community safer, as well as fostering a sense of belonging and unity.
Moreover, school teachers ought to equip their students with a wide cluster of skills, such as discipline,
punctuality and co-operation. Having acquired these crucial abilities, school graduates can meet
industry requirements and survive in today’s hyper-competitive job market, thus contributing to the

ƠN
economy.

The role of schools, however, goes beyond this limited view. Instead of turning children into “obedient
slaves” and “robotic workers”, education is crucial for raising well-rounded personalities who pursue
meaningful goals, such as happiness and personal fulfillment. This is because while a law-abiding
NH
citizen stays quiet in case of propaganda, indoctrination, and enslavement, which is currently the case in
Communist North Korea, an active member of society challenges the status quo and reacts as promptly
as possible. Teaching children creativity, critical thinking and logical reasoning skills also helps to
ensure that we have considerate and reliable future generations, who would make new discoveries and
breakthroughs to improve people’s lives.
Y

In conclusion, I believe that schools should aim to provide a balanced curriculum that not only converts
QU

children into law-abiding inhabitants and effective workers but also fosters the development of active
and able members of society.
M

Y
DẠ

Page 7 of 7
KỲ THI HỌC SINH GIỎI CÁC TRƯỜNG THPT CHUYÊN
KHU VỰC DUYÊN HẢI VÀ ĐỒNG BẰNG BẮC BỘ

L
LẦN THỨ XIV, NĂM 2023

A
CI
ĐỀ THI MÔN: TIẾNG ANH 11
ĐỀ ĐỀ NGHỊ Thời gian: 180 phút (Không kể thời gian phát đề)

FI
Ngày thi: /7/2023
(Đề thi gồm 22 trang) (Thí sinh làm bài vào Phiếu trả lời)

OF
A. LISTENING (50 points)
HƯỚNG DẪN PHẦN THI NGHE HIỂU
• Bài nghe gồm 4 phần, mỗi phần được nghe 2 lần, mỗi lần cách nhau 15 giây, mở đầu và kết

ƠN
thúc mỗi phần nghe có tín hiệu.
• Mở đầu và kết thúc bài nghe có tín hiệu nhạc. Thí sinh có 3 phút để hoàn chỉnh bài trước
tín hiệu nhạc kết thúc bài nghe.
NH
• Mọi hướng dẫn cho thí sinh (bằng tiếng Anh) đã có trong bài nghe.

Part 1: You will hear an extract from a lecture about solar eclipses in history. For question
1-5, choose the answer (A, B, C or D) which fits best according to what you hear. (10 pts)
1. The speaker compares a solar eclipse today to a _________.
Y

A. religious experience B. scientific event


QU

C. popular spectacle D. cultural event


2. The speaker says that the dark spot of an eclipse is ___________.
A. simple to predict B. easy to explain
C. randomly occurring D. unable to depict
3. Concerning an eclipse, the ancient Chinese were __________.
M

A. fascinated B. rational C. disturbed D. indifferent


4. For the speaker, the most impressive aspect of an eclipse is the __________.
A. exceptional beauty of the sky B. chance for scientific study

C. effect of the moon on the sun D. effect of the sun on the moon
5. In predicting eclipses, the Babylonians were restricted by the ___________.
A. limited ability to calculate B. inaccurate observations
C. religious attitudes D. cultural beliefs
Y

Your answer:
1 2 3 4 5
DẠ

Part 2: You will listen to TV program about Gordon Ramsay sharing about his job and
decide whether the following sentences are true (T) or false (F). (10 pts)
https://www.youtube.com/watch?v=NMTA6JoU_D8

Page 1 of 22
6. An absolutely stunning dish can be perfected in even 40 or 50 hours.
7. He travelled to France to discover the origin of cookery they do in France.
8. He could communicate fluently in French after one and a half year living in the country.

L
9. His parents bought him his first set of knives before sending him to cooking school.

A
10. When he started out as a chef, his goal was to learn about every ingredient and never feel
frightened or unsure of how to use it.

CI
Your answers
6. 7. 8. 9. 10.

FI
Part 3: You will hear a short conversation between two students. Listen and give short
answers to the questions. Write NO MORE THAN THREE WORDS AND/OR A NUMBER

OF
taken from the recording.
(10 pts) https://ielts-up.com/listening/ielts-listening-sample-5.3.html
11. The total course duration is ________.
12. During the final project students will work in teams of ________.
13. The professor said that the key thing in marketing strategy is to _______.

ƠN
14. The next lecture is in the big classroom on the ________.
15. Students need to take their last week ________.

Your answer:
NH
11 12 13 14 15

Part 4. For questions 16–25, listen to a 5-Minute-Food-Fix Podcast about Tofu puffs and fill
in the missing information. Write NO MORE THAN THREE WORDS taken from the
Y

recording for each answer in the spaces provided. (20 points) (https://player.fm/series/5-
minute-food-fix/tofu-puffs )
QU

The host of the program adores tofu because she thinks tofu is 16 ___________.
One 17__________ of tofu that people can get easily at a grocer is tofu puffs.
Tofu puffs are usually in the 18____________ of the store.
Personally, the host thinks one tofu puff is similar to a 19__________ because it soaks up any
soup.
M

Treat it like a 20__________ ingredient or a pre-cooked soup ingredient.


When it comes to shape, tofu puffs are like 21________.
Be careful when you 22____________. If you overdo it, the tofu puffs will retain all of that.

In that case, you can add some sake or water or stock to 23_______________.
You can add chili, and a couple of handfuls of 24___________.
A dish from tofu puffs and a side of rice can help you get a really 25____________ meal with
very little stress.
Y

Your answer:
DẠ

16 17 18 19 20

Page 2 of 22
21 22 23 24 25

L
B. LEXICO – GRAMMAR (30 points)

A
Part 1. For questions 26-45, choose the best option A, B, C or D to complete the following
sentences and write your answers in the corresponding numbered boxes on the answer sheet.

CI
(20 points)
26. Let’s _______ the place, it looks so gloomy and unpleasant.

FI
A. miss B. abandon C. depart D. disappear
27. The first thing for all of you to remember is that _________ your duties may result in an

OF
instant dismissal.
A. escaping B. neglecting C. resisting D. missing
28. Several soldiers of the squad were taken_______ by the enemy forces.

ƠN
A. capture B. hostage C. kidnap D. torture
29. Patrick is too _______ a gambler to resist placing a bet on the final game.
A. instant B. spontaneous C. compulsive D. continuous
NH
30. Mr. Tanner did his best to fix the faulty oven, but his _______ at repairing electrical
devices wasn’t good enough to succeed.
A. service B. skill C. technique D. craft
31. The inconsiderate driver was _______ for parking his vehicle in the wrong place.
Y

A. inflicted B. condemned C. harassed D. fined


QU

32. Mrs. Hurston was in deep _______ after her husband’s unexpected death.
A. regret B. grief C. lament D. disturbance
33. My older brother is extremely fond of astronomy; he seems to _______ a lot of pleasure
M

from observing the stars.


A. derive B. possess C. seize D. reach

34. I can accept criticism in general, but George really ______ it too far, so I had no
other option but to show my disapproval.
A. carried B. pushed C. put D. made
Y

35. When Mr. Barnaby died, several people ________ their claim to the substantial legacy that
he left.
DẠ

A. placed B. drew C. assumed D. laid


36. Our karate master is as quick as _______.

Page 3 of 22
A. wave B. thunder C. current D. lightning
37. Mr. Smith ate his breakfast in great ________ so as not to miss the bus to Liverpool.

L
A. speed B. pace C. rush D. haste

A
38. The whole situation is getting out of ____________. Let’s do something before it turns into
a bitter row.

CI
A. capacity B. charge C. hand D. discipline
39. I know it works in theory, but try putting it into _______and you’ll find out it’s a

FI
failure.
A. operation B. exercise C. performance D. procedure

OF
40. I wouldn’t have realized the possible risk of setting the factory on fire if Bob hadn’t
_______ my attention to it.
A. sought B. drawn C. showed D. caught

ƠN
41. Eric’s father works as an interpreter. The man is _______ in three languages.
A. fluent B. outspoken C. speechless D. knowledgeable
42. How exactly did you set _______ training the horse to work so well together?
NH
A. up B. to C. about D. out
43. I could tell at a _______ that nothing had changed between Barbara and Edward.
A. glimpse B. blink C. wink D. glance
Y

44. I had no chance to defend myself: the dog ______ for me as soon as I opened the door.
A. went B. ran C. fell D. stood
QU

45. She should have been here but she’s ______ flu.
A. gone through with B. gone down with
C. come in for D. come up against
M

Part 2. Write the correct form of each bracketed word in the numbered space provided in
the column on the right. (10 pts)

46. The number of the nails Harry had bought at the ironmonger’s wasn’t _________ and we
had to borrow a few from our neighbor. (SUFFICE)

47. Our grandfather is so _________ of the modern technologies that he doesn’t even want to
Y

have a TV set in his house. (FEAR)


DẠ

48. At first I was against keeping the news secret, but Frank’s _________ arguments finally
convinced me. (SENSE)

Page 4 of 22
49. Big loans on very ________ conditions are offered by the bank to those who make a large-
scale investment. (FAVOUR)

L
50. It was a _________afternoon for all the representatives, who were busy preparing the
necessary documents. (REST)

A
51. He is the third _________ caught by the guard today intruding into the no-entry territory.

CI
(PASS)
52. Alfred’s irrational reactions may result from the inadequate __________ that he was given

FI
by his foster parents. (BRING)
53. Everything they did was illegal as they hadn’t received any official ______ for excavating

OF
this land. (AUTHOR)
54. There’s no logical accounting for Malcolm’s _________ given that his brothers and sisters
are really generous and considerate. (SELF)

ƠN
55. Because of a sudden fall in temperature the ___________ was so frozen that all the
morning flights had to be cancelled. (RUN)
C. READING (60 points)
NH
Part 1. For questions 56-65, fill each of the following numbered blanks with ONE suitable
word and write your answers in the corresponding numbered boxes on the answer sheet. (15
points)
June 8th is World Oceans Day – an idea instituted by the United Nations to celebrate and
Y

protect our natural heritage. Just as our own central nervous (56)______ controls every part of
QU

our body, so the oceans control every part of our planet. They regulate climate, weather and
ecosystems.
Unfortunately, human activities have led to pollution and the destruction of marine habitats.
Another result of human (57)______ in the natural environment is climate change with its
M

accompanying extreme weather (58)______. A rise in greenhouse gas emissions has led to an

increase in global temperatures and as a consequence sea levels have risen. Plastic pollution is
also a big issue.
In 2016, as (59)______ of World Oceans Day, thousands of people took part in ‘the better bag
Y

challenge’ and promised to use reusable bags instead of plastic ones. 80 per cent of plastic
rubbish (60)______ on land ends up in the sea, usually washed there via (61)______ – where
DẠ

rivers empty into the sea. You may have seen such debris washed up onto your local beach at
low (62)______. Most plastic bags are made of polyethene – a (63)______ compound which is

Page 5 of 22
not biodegradable. Plastic bags break down into tiny visible (64)______, which are ingested by
fish and marine mammals. ‘The better bag challenge’ aims to halt this devastating process and

L
give new (65 )______ to our oceans.

A
CI
Part 2. For questions 66-75, read the passage below and choose the answer A, B, C or D that
fits best according to the text. Write your answers in the corresponding numbered boxes
provided on the answer sheet. (10 points)

FI
In orbit high above Earth, a multi-billion-dollar formation of communications satellites stands
ready to instantly connect pilots, seamen and navigators of all kinds to every available aid

OF
when they find themselves in an emergency. But what if the communication computers on
board these ships and aircraft started acting up or even broke down? The world could go silent,
and leave travellers groping around for directions. Not to worry. There is a backup plan, using

ƠN
technology that was invented in 1835. It's Morse code, the language of dots and dashes that has
survived the assault of higher technology for a century and a half.
Named after its inventor, Samuel F B Morse, the code is a series of combinations of short and
NH
long tones (dots and dashes) representing letters of the alphabet that can be transmitted
manually by a key operator. A telegrapher combines the dots and dashes to form letters and
words. It is a seemingly tedious procedure, but skilled operators can transmit and receive faster
Y

than most secretaries can type. The fastest Morse transmission ever recorded is an amazing 84
QU

words per minute, sent by an operator named T L McElroy in 1951.


Morse telegraphy may seem like a quaint anachronism, with its brass sounder and key
operated by the world's most basic tool, the human finger. However, it is sometimes vital to
M

worldwide communications. When the Mexico City earthquake occurred in 1985 and all the
power went off, calls for help were transmitted in Morse by an amateur radio operator. "We see

the Morse code as a dying art, but we refuse to let it die completely," says Major General Leo
M Childs, the US Army's Chief Signal Officer. "Newer is not always better. Even though it is
old and slow, Morse is still the most reliable in difficult conditions. "
Y

Every merchant vessel bearing the US flag must carry a radio officer who can both transmit
DẠ

and receive Morse code. Under US law, the officer must spend eight hours at sea every day
monitoring the radio for Morse distress signals. Should you ever find yourself adrift at sea in a
lifeboat launch from a sink passenger cruise ship, it will be equipped with a single
Page 6 of 22
communications device: a Morse transmitter that automatically signals a distress call, but is
also equipped with a keyboard in case you happen to know Morse code. Perhaps the best-

L
known bit of Morse code is the call for help - SOS. In the code, these letters form a distinctive

A
pattern (dot dot dot, dash dash dash, dot dot dot) easily recognised in an emergency.

CI
The enduring use of Morse telegraphy is the legacy of a burst of industrialisation in 19th-
Century America, when the railway and telegraph developed side by side. Most of those

FI
railway telegraph lines were used well into the mid-20th century, well after radio, television
and computers became commonplace. Until 1985, the Milwaukee Road had a Morse telegraph

OF
line between Milwaukee and La Crosse that was routinely used to relay orders to train crews.
This Milwaukee operation was shut down quietly in the late Eighties. In many other countries,
however, Morse railway is still used.

ƠN
The military services continue to be the most serious users of Morse telegraphy. While billion-
dollar satellites and sophisticated ground networks are good in theory, such communication
systems can break down on the battlefield. As a matter of prudence, the Army keeps a
NH
functional Morse capability. Morse code signals require much less power to transmit
broadcasting than voice messages. In addition, even an unclear Morse signal can be interpreted,
whereas a distorted voice transmission is virtually useless. The Army annually trains about
Y

2,800 men and women in Morse code for a variety of signal jobs in infantry, artillery,
intelligence and even Special Forces. A Morse transmission will get through when all else fails,
QU

and especially in military conflicts, "he who communicates first, no matter how primitively,
will come out on top," says Major General Childs.
In that case, retired railway telegraph operators will Probably take over the world. Each
M

evening, the amateur radio waves come alive with the Morse transmissions of the retired

operators known among themselves as 'old heads'. "I get on the air and use the code every night
to chat to other old heads," says Craig Becker, the retired Milwaukee telegraph operator who
received the railway's final telegraph message in 1985. "There are a lot of telegraphers around.
Every night you hear them pecking. "Experienced Operators say the Morse code is not so much
Y

a clatter of sounds as a language, because operators do not hear dots and dashes. "You can sit
DẠ

back and hear a conversation," says Becker.

Page 7 of 22
When Morse inaugurated the telegraph service in 1844, he wired from Baltimore to
Washington the now-famed message: "What hath God wrought!" Ever since, the death of

L
Morse code has been regularly predicted. However, although the surface has receded from

A
public view, experts say that they cannot envision an end to its use any time soon. "I can carry

CI
a very small Morse key in my pocket and transmit around the world," says Burke Stinson, a
public relations man for American Telephone & Telegraph Co. "I do not think you will ever

FI
see Morse code die. It is going to be difficult to find another method that is as flexible and
reliable. "

OF
66. The reason the writer mentions the possibility of satellite–based communication systems
failing is to
A. underline the importance of a reliable alternative.

ƠN
B. suggest that satellite communications are unreliable.
C. emphasise the drawbacks of satellite communications.
D. compare Morse to communication satellites.
NH
67. The process of communicating using Morse code
A. involves more than transmitting tones. B. is tedious even for experienced users.
C. is not as laborious as people think. D. is simple enough for anyone who can
Y

type.
68. In the third paragraph, the writer implies that the use of Morse code is
QU

A. old-fashioned, but attracts many enthusiasts.


B. only useful to the armed forces and some hobbyists.
C. being revived by amateur radio operators.
M

D. often the only way to communicate during a crisis.


69. In the fifth paragraph, the writer


A. implies that Morse telegraphy developed faster than the railway.
B. implies that Morse has survived for so long because of the railway.
C. suggests that the railway developed because of Morse telegraphy.
Y

D. explains why Morse telegraphy eventually became redundant.


DẠ

70. The American military


A. needs simple communication methods

Page 8 of 22
B. has recently neglected its Morse capability.
C. allows its forces some Morse equipment.

L
D. offers training for recognising distorted Morse.

A
71. The first message to be transmitted in Morse code

CI
A. symbolised the code’s flexibility. B. was used for commercial purposes.
C. took a long time to reach its destination. D. was sent by Samuel Morse himself.

FI
72. What does the writer mean by saying that Morse code “has survived the assault of higher
technology for a century and a half” in the first paragraph?

OF
A. Even though in the last 150 years newer and more sophisticated ways of doing things have
been invented, Morse is still used.
B. As modern communication systems have progressed rapidly, it is surprising that Morse

ƠN
telegraphy could survive for 150 years.
C. Thanks to the backup plan, Morse telegraphy has survived for 150 years in the face of
technology.
NH
D. Morse code could not have survived for 150 years without the support of modern
technology.
73. What does the word “It” in paragraph 2 refer to?
Y

A. a series of combinations B. sending a message in Morse code


C. representing letters of the alphabet D. the transmission sent by McElroy
QU

74. The phrase “quaint anachronism” in paragraph 3 refers to _______.


A. unimportant thing B. monotonous cycle
C. old-fashioned tool D. irrelevant invention
M

75. Which is the best title for the passage?


A. Morse Code – a Revolutionary Invention


B. Dots and Dashes Still Alive
C. The Comeback of a Forgotten Name
D. The Applications of Morse Telegraphy
Y
DẠ

Part 3. For questions 76-88, read the following passage and do the tasks that follow. (13
points)

Page 9 of 22
Where is Air Travel Going?
New innovations are expanding our concept of flight

L
Between 2006 and 2016, the number of airline passengers worldwide went up by around a

A
billion and a half, to nearly four billion. Given the environmental impact of air travel, it is

CI
imperative that the industry develop aircraft that produce less noise and fewer carbon emissions
than those of the past. Fortunately, we have a number of new technologies and new,

FI
lightweight building materials at our disposal.
Electricity could be key to the future of air travel. Just as electric, driverless cars are set to

OF
revolutionise personal transport, battery-powered automated flying machines could soon
replace traditional taxis. A number of companies worldwide are working on them, convinced
that they will take off both literally and metaphorically. One of the things that make the concept

ƠN
of 'sky taxis' suitable for dense urban environments is the ability to start and end journeys
vertically, with no runway required.
A number of inventors have also shown that it is possible to build flying machines for personal
NH
use on both the roads and in the air, proving that motor pioneer Henry Ford was right when he
predicted the appearance of flying cars back in 1940. The problem is that these vehicles have
very complicated designs, are very expensive and are subject to very strict regulations about
Y

where they can be used; they cannot land just anywhere. They are likely to remain something
of a wealthy person’s toy for some time yet.
QU

However, some passenger planes on scheduled routes could soon be electric, as long as we
find ways of making batteries lighter and more efficient. The most advanced electric aeroplanes
today do not carry passengers and have a maximum range of around 100 km, but at least one
M

company is hoping to introduce a commercial, battery-powered electric aircraft within a


decade. Its plan is for a nine-seat aeroplane with a range of around 500 km.
Since electric aeroplanes need to fly more slowly and at lower altitudes than those with jet
engines, it seems unlikely they will ever be used for long-distance journeys. A mix of fuel and
battery power is certainly possible, however. In fact, this is a very sensible arrangement, as
Y

aeroplanes are required to fly with 45 minutes of reserve power available at all times in case an
DẠ

emergency arises.

Page 10 of 22
In the near future, even the passengers themselves could generate at least some of the power an
aircraft needs to fly. Seats that have the ability to turn body heat into electrical energy are under

L
development. If you feel uncomfortable about the idea of becoming a human battery, the seats

A
might also have plenty of features to help you relax. It has been suggested that aeroplane seats

CI
– which are notoriously lacking in comfort – could one day change shape to match an
individual body. Massage or acupuncture treatments could even be built in. Such luxuries

FI
would be provided to passengers who are willing to pay the top fares, anyway.
Other design innovations seem to be aimed at helping people who find flying unpleasant or

OF
frightening. If you wish you were somewhere else every time you board an aircraft, how would
you like to use a virtual-reality helmet to transport yourself to a different environment? There is
a good chance that ‘sensory headsets’ will be fitted to the headrests of aeroplane seats before

ƠN
long. A more outlandish virtual-reality idea is to make the plane walls seem invisible. By
covering them with special materials that give off light, airlines will be able to show passengers
images of the sky outside the plane. Of course, that is not everyone’s idea of a relaxing
NH
experience – so some people might be very grateful for the sensory headset option!
For those who do enjoy flying, however, a new kind of holiday that puts the plane at the centre
of the experience has already become available – but it does not come cheap. You can fly
Y

around the world in a luxury Boeing 777 aircraft, spending time in ten of the world’s most
exciting cities, for around $160,000. The ‘air cruise’, as the name suggests, is an attempt to
QU

recreate in the air the experience of travelling by sea for pleasure. The aeroplane has lounge
seating, beds, fine dining, a bar and even butlers, and is used exclusively for air cruises lasting
between two weeks and a month.
M

Clearly, standard aircraft are not likely to become so luxurious, but ordinary flyers can at least

expect to get more room for themselves. That is because the shape of commercial aircraft is
expected to change quite fundamentally in the future, as today’s designs are not ideal for fuel
efficiency and smooth movement. Future aircraft will likely be more triangular in shape, with
larger wings that blend into the middle of the plane rather than sticking out from it. These
Y

aeroplanes will be less noisy and they will have more interior space, but there will also be
DẠ

fewer windows. Indeed, if you think about how much cars have changed since their early days,
it is hard to believe that the basic design of aircraft has stayed the same for as long as it has.

Page 11 of 22
Questions 76-88

L
Do the following statements agree with the information given in the Reading Passage. For

A
questions 76-85, choose:

CI
TRUE if the statement agrees with the information
FALSE if the statement contradicts the information

FI
NOT GIVEN if there is no information on this
Write your answers in the corresponding numbered box provided.

OF
76 The popularity of air travel is threatening the environment.
77 Electric power is unlikely to be used in the flight industry.
78 Sky taxis are viewed by some as an opportunity for profit.

ƠN
79 Sky taxis are not a good idea for crowded, big city locations.
80 Henry Ford designed a prototype flying car.
81 It is illegal to land a flying car on a roadway.
NH
82 There are still technical limitations which electric planes must overcome.
83 In the next decade, electric planes are expected to almost double their range.
Questions 84-88
Y

Choose the correct letter A, B, C or D. Write your answers in the corresponding numbered
QU

box provided.
84. What is the writer doing in the fifth paragraph?
A describing why electric planes are not widely used
B pointing out the advantages of battery power
M

C discussing future applications of battery technology


D warning of potential risks associated with electric planes

85. What are we told about innovations in seats in the sixth paragraph?
A They will benefit the airlines more than the passengers.
Y

B They will each make flying a more relaxing experience.


DẠ

C They will do little to ease the discomfort of current seats.


D They are unlikely to be made available to everyone.

Page 12 of 22
86. Future virtual reality innovations could affect anxious passengers
A minimally, as there will be little change.

L
B in several different ways.

A
C by obscuring the flight experience completely.

CI
D because the thrills of flight are emphasised.
87. An air cruise seeks to make

FI
A round-the-world travel attainable for those with little time.
B luxury travel accessible to people from all walks of life.

OF
C the destination more important than the journey.
D air travel emulate a more traditional holiday experience.
88. What is the main point that the writer makes about change in aircraft in the final paragraph?

ƠN
A It is long overdue.
B It will be mostly cosmetic.
C It will be at the expense of comfort.
NH
D It is unlikely to actually take place.

Part 4: In the passage below, seven paragraphs have been removed. Read the passage and
choose from paragraphs A-H the one which fits each gap. There is ONE extra paragraph
Y

which you do not need to use. Write your answers in the corresponding numbered boxes
QU

provided on the answer sheet. (7 points)


Living in a Dream World
Daydreaming can help solve problems, trigger creativity, and inspire great works of art and
M

science. By Josie Glazier.


Most people spend between 30 and 47 per cent of their waking hours spacing out, drifting off,

lost in thought, wool-gathering or building castles in the air. Yale University emeritus
psychology professor Jerome L. Singer defines daydreaming as shifting attention “away from
some primary physical or mental task toward an unfolding sequence of private responses” or,
Y

more simply, “watching your own mental videos.” He also divides daydreaming styles into two
DẠ

main categories: “positive-constructive,” which includes upbeat and imaginative thoughts, and
“dysphoric,” which encompasses visions of failure or punishment.

Page 13 of 22
89.

L
Such humdrum concerns figured prominently in one study that rigorously measured how much

A
time we spend mind wandering in daily life. In a 2009 study, Kane and his colleague Jennifer

CI
McVay asked 72 students to carry Palm Pilots that beeped at random intervals eight times a day
for a week. The subjects then recorded their thoughts at that moment on a questionnaire. The

FI
study found that about 30 per cent of the beeps coincided with thoughts unrelated to the task at
hand and that mind wandering increased with stress, boredom or sleepiness or in chaotic

OF
environments and decreased with enjoyable tasks. That may be because enjoyable activities
tend to grab our attention.
90.

ƠN
We may not even be aware that we are daydreaming. We have all had the experience of
“reading” a book yet absorbing nothing—moving our eyes over the words on a page as our
attention wanders and the text turns into gibberish. “When this happens, people lack what I call
NH
‘meta-awareness,’ consciousness of what is currently going on in their mind,” he says. But
aimless rambling can be productive as they can allow us to stumble on ideas and associations
that we may never find if we intentionally seek them.
Y

91.
QU

So, why should daydreaming aid creativity? It may be in part because when the brain is
floating in unfocused mental space it serves a specific purpose. It allows us to engage in one
task and at the same time trigger reminders of other, concurrent goals so that we do not lose
M

sight of them. There is also the belief that we can boost the creative process by increasing the
amount of daydreaming we do or replaying variants of the millions of events we store in our

brains.
92.
Y

The mind's freedom to wander during a deliberate tuning out could also explain the flash of
insight that may coincide with taking a break from an unsolved problem. A study conducted at
DẠ

the University of Lancaster in England into this possibility found that if we allow our minds to
ramble during a moderately challenging task, we can access ideas that are not easily available

Page 14 of 22
to our conscious minds. Our ability to do so is now known to depend on the normal functioning
of a dedicated daydreaming network deep in our brain.

L
93.

A
CI
It was not until 2007, however, that cognitive psychologist Malia Fox Mason, discovered that
the default network — which lights up when people switch from an attention-demanding

FI
activity to drifting reveries with no specific goals, becomes more active when mind wandering
is more likely. She also discovered that people who daydream more in everyday life show

OF
greater activity in the default network while performing monotonous tasks.
94.
The conclusion reached in this ground-breaking study was that the more complex the mind

ƠN
wandering episode is, the more of the mind it is going to consume. This inevitably leads to the
problem of determining the point at which creative daydreaming crosses the boundary into the
realms of compulsive fantasising. Although there is often a fine dividing line between the two,
NH
one question that can help resolve the dilemma relates to whether the benefits gained from
daydreaming outweigh the cost to the daydreamer’s reputation and performance.
95.
Y

On the other hand, there are psychologists who feel that the boundary is not so easily defined.
QU

They argue that mind wandering is not inherently good or bad as it depends to a great extent on
context. When, for example, daydreaming occurs during an activity that requires little
concentration, it is unlikely to be costly. If, however, it causes someone to suffer severe injury
M

or worse by say, walking into traffic, then the line has been crossed.

A Although these two findings were significant, mind wandering itself was not measured
Y

during the scans. As a result, it could not be determined exactly when the participants in her
DẠ

study were “on task” and when they were daydreaming. In 2009 Smallwood, Schooler and
Kalina Christoff of the University of British Columbia published the first study to directly link

Page 15 of 22
mind wandering with increased activity in the default network. Scans on the participants in
their study revealed activity in the default network was strongest when subjects were unaware

L
they had lost focus.

A
B However, intense focus on our problems may not always lead to immediate solutions. Instead

CI
allowing the mind to float freely can enable us to access unconscious ideas hovering
underneath the surface — a process that can lead to creative insight, according to psychologist

FI
Jonathan W. Schooler of the University of California, Santa Barbara
C Yet to enhance creativity, it is important to pay attention to daydreams. Schooler calls this

OF
“tuning out” or deliberate “off-task thinking.”, terms that refer to the ability of an individual to
have more than just the mind-wandering process. Those who are most creative also need to
have meta-awareness to realise when a creative idea has popped into their mind.

ƠN
D On the other hand, those who ruminate obsessively—rehashing past events, repetitively
analyzing their causes and consequences, or worrying about all the ways things could go wrong
in the future - are well aware that their thoughts are their own, but they have intense difficulty
NH
turning them off. The late Yale psychologist Susan Nolen-Hoeksema does not believe that
rumination is a form of daydreaming, but she has found that in obsessive ruminators, the same
default network as the one that is activated during daydreaming switches on.
Y

E Other scientists distinguish between mundane musings and extravagant fantasies. Michael
Kane, a cognitive psychologist at the University of North Carolina at Greensboro, considers
QU

“mind wandering” to be “any thoughts that are unrelated to one's task at hand.” In his view,
mind wandering is a broad category that may include everything from pondering ingredients
for a dinner recipe to saving the planet from alien invasion. Most of the time when people fall
M

into mind wandering, they are thinking about everyday concerns, such as recent encounters and

items on their to-do list.


F According to Schooler, there are two steps you need to take to make the distinction. First,
notice whether you are deriving any useful insights from your fantasies. Second, it is important
to take stock of the content of your daydreams. To distinguish between beneficial and
Y

pathological imaginings, he adds, “Ask yourself if this is something useful, helpful, valuable,
DẠ

pleasant, or am I just rehashing the same old perseverative thoughts over and over again?” And

Page 16 of 22
if daydreaming feels out of control, then even if it is pleasant it is probably not useful or
valuable.

L
G Artists and scientists are well acquainted with such playful fantasizing. Filmmaker Tim

A
Burton daydreamed his way to Hollywood success, spending his childhood holed up in his

CI
bedroom, creating posters for an imaginary horror film series. Orhan Pamuk, the Turkish
novelist who won the Nobel Prize in Literature in 2006, imagined “another world,” to which he

FI
retreated as a child, Albert Einstein pictured himself running along a light wave—a reverie that
led to his theory of special relativity.

OF
H Like Facebook for the brain, the default network is a bustling web of memories and
streaming movies, starring ourselves. “When we daydream, we're at the center of the universe,”
says neurologist Marcus Raichle of Washington University in St. Louis, who first described the

ƠN
network in 2001. It consists of three main regions that help us imagine ourselves and the
thoughts and feelings of others, draw personal memories from the brain and access episodic
memories.
NH

Part 5: For questions 96-105, you are going to read an article about an art exhibition that
focuses on the subject of whether paintings are authentic or fake. Answer the questions by
Y

choosing from the sections of the article (A - F). The sections may be chosen more than
QU

once. Write your answers in the corresponding numbered boxes provided on the answer
sheet. (15 points)
Napoleon Bonaparte
What characteristics marked out one of history’s greatest military geniuses?
M

A Napoleon Bonaparte stirs the emotions. Some find him heroic and regard his opponents as

reactionary and unimaginative. Others think him mad with ambition and responsible for many
of the sins of his era. Between these extremes there are those who find some aspects of the man
admirable and others regrettable. Of course, Bonaparte was anything but pure, anything but
Y

modest, anything but democratic, and anything but a peacemaker. But in the end, who else that
sat on a throne in Europe could claim to be? Should he be assailed for sins that were so sadly
DẠ

common? What is it about Napoleon Bonaparte that makes him the object of such unique
criticism? Is it because he holds a special place in our imaginations, a place that we hope would

Page 17 of 22
be an example of our better selves? Was his genius, good fortune, and opportunity enough to
condemn him - not so much for what he did, but what he failed to do? In the end is our greatest

L
disappointment in Bonaparte simply that he was merely human?

A
B Napoleon was a military genius in the strategic and tactical handling of armies and although

CI
he provided no large scale reforms of armies (or, indeed, their equipment and techniques), he
excelled at the refinement of an art that already existed. One of the most important factors of

FI
Napoleon's personality and its effect on his abilities as a military commander was his genius to
inspire others. He believed in the maxim that ‘spirit and drive is to the physical as three is to

OF
one’. It was through his system of awards (an appealing to soldiers’ ‘soul in order to electrify
the man’) that Napoleon was so successful in obtaining unquestionable obedience, loyalty and
devotion from his rank and file.

ƠN
C The name Napoleon Bonaparte is surrounded by so vast an array of myth and legend that it is
quite difficult to separate fact from fiction. It is said that Napoleon displayed outstanding
leadership qualities while still at school. But this is undoubtedly the product of the school of
NH
Napoleonic mythology that was systematically promoted for political reasons in 19th century
France, since it hardly squares with the general picture of the taciturn child with an inferiority
complex that has come down to us. Napoleon
Y

was, in truth, a reserved child, resentful of his peers. On the other hand, he excelled at maths –
a qualification that determined his specialisation as an artillery officer. This was a stroke of
QU

luck (one of many that Napoleon benefited from) inasmuch as the artillery was the most
prestigious branch of the army under the old regime. But the biggest stroke of fortune
Napoleon had was to be born when he was – in the age of the French Revolution. The
M

Revolution turned the whole world upside down and presented an ambitious young man

(Napoleon was always ambitious – a consequence of his resentment at his inferior status) with
new and vast opportunities.
D No man previously ever concentrated authority to such a point, nor showed mental abilities
at all comparable to Napoleon’s: an extraordinary power of work; prodigious memory for detail
Y

and fine judgment in their selection; a luminous decision-making capacity and a simple and
DẠ

rapid conception - all placed at the disposal of a sovereign will. And no head of state gave
expression more imperiously than this Corsican to the popular passions of the French of that

Page 18 of 22
day: abhorrence for the emigrant nobility, fear of the ancient régime, dislike of foreigners,
hatred of England, an appetite for conquest evoked by revolutionary propaganda, and the love

L
of glory.

A
E Psychological studies of ‘great men and women’ frequently serve as a fig leaf to disguise the

CI
absence of an understanding of broad socio-historical processes. The study of history is
replaced by trivial personal observations. Instead of science, we have gossip. A careful study of

FI
the character and background of Napoleon Bonaparte can furnish us with many useful insights
into his behaviour. But we are left with a small amount of useful information that can help us to

OF
attain a deeper understanding of Bonaparte. Men and women make their own history, but they
do not make it freely, in the sense that the scope and results of their actions are strictly limited
by the given socio-economic context that is prepared independently of their will.

In which section are the followings mentioned?


ƠN
the idea that an individual’s actions cannot be separated from their historical 96.
Your answers:
NH
context
the view that the writing of history is sometimes agenda-driven 97.
the characteristics that made Napoleon an exceptional leader 98.
Y

the fact that Napoleon was eminently aware of the impact of morale on modern 99.
QU

warfare
the fact that Napoleon is as divisive a character as he is famous 100.
the uncertainty as to whether detractors of Napoleon simply lament the fact he 101.
had the vices and virtues of the common man
M

the view that Napoleon was no great revolutionary of warfare itself 102.

the notion that we are all a product of the time we live in 103.
the fact that Napoleon’s views reflected those of his countrymen 104.
the event that made Napoleon’s ascent possible 105.
Y

D. WRITING (60 points)


DẠ

Part 1. Read the following extract and use your own words to summarize it. Your summary
should be about 140 words. You MUST NOT copy the original. (15 pts)

Page 19 of 22
Part 1: Read the following extract on how camouflaging and mimicry help insects and
use your own words to summarize it. Your summary should be no more than 120 words long.

L
(1.0pt)
Have you ever wondered why soldiers are always clad in green? This is to enable them to

A
camouflage themselves during wartime. Hiding in the jungles, their green attire blend into the

CI
surrounding trees and shrubs, making it difficult for the enemies to spot them.
Long before man make use of camouflaging, insects have already adopted the tactic of disguise
to escape from the clutches of their predators. By having body colors close to those of the rocks

FI
and dried leaves, they catch less attention from the predators and hence escape from being
pursued. However, this kind of disguise works only if the insects remain still in the presence of

OF
their predators.
Butterflies and moths have developed a variety of camouflage strategies since they are quite
defenseless and their predators - birds are abundant in supply. Many moth caterpillars resemble
dead twigs while the young of certain species of butterflies appear like bird droppings. Adult

ƠN
butterflies and moths camouflage themselves too, in attempts to escape from their hunters -
birds who are superior gliders. Possessing wings which resemble dried leaves help certain
butterflies and moths to hide among heaps of dried leaves when predators are around.
NH
Fortunately, not all insects choose the art of disguise to escape from their predators; otherwise,
the world would be so dull and colorless. There are insects which assimilate the bright body
colors of bees and wasps to escape from being pursued by their predators. The concept of
mimicry was derived, owing to the bees and wasps. Long ago, birds have already learnt to
avoid brilliantly colored wasps and bees in fear of their painful stings. Hence, over millions of
Y

years, many harmless insects have assimilated the bees and wasps by imitating their bright
body colors and shapes. In this way, they appear dangerous to their predators and hence ward
QU

them off.
Mimics of the wasps and bees are most commonly found in the gardens. The furry, plump bee-
fly not only appears like the bumble bee in terms of body colors, even its hums sound similar
too. The only difference is that the bee-fly does not have a sting and is hence harmless. The
M

hoverfly is another insect which imitates the body colors of the wasps. Their bodies are striped
yellow and black. The only deviations are that hoverflies do not have stings and they have only

one pair of wings each while wasps have two pairs each. These variations are hardly noticed by
the predators and hence help them to escape.
Part 2: The graph below shows radio and television audiences throughout the day in 1992.
Y

Summarise the information by selecting and reporting the main features, and make
comparisons where relevant.
DẠ

Write at least 150 words

Page 20 of 22
DẠ
Y

M
QU
Y
NH
ƠN
OF
FI
CI
A L

Page 21 of 22
Part 3. Essay writing (30 points)

L
Some people think that children should begin their formal education at a very early age and

A
should spend most of their time on school studies. Others believe that young children should
spend most of their time playing. Compare these two views. Which view do you agree with?

CI
Why?

FI
-------------- HẾT --------------

(Thí sinh không được sử dụng tài liệu. Cán bộ coi thi không giải thích gì thêm)

OF
Họ và tên thí sinh: .............................................................. Số báo danh: ...............................

ƠN
NH
Y
QU
M

Y
DẠ

Page 22 of 22
KỲ THI HỌC SINH GIỎI CÁC TRƯỜNG THPT CHUYÊN
KHU VỰC DUYÊN HẢI VÀ ĐỒNG BẰNG BẮC BỘ
LẦN THỨ XIV , NĂM 2023

L
HƯỚNG DẪN CHẤM MÔN: TIẾNG ANH 11

A
Thời gian: 180 phút (không kể thời gian giao đề)
ĐỀ XUẤT

CI
FI
(Đáp án gồm 10 trang)
A. LISTENING (50 points):
LISTENING

OF
1C 2B 3C 4B 5A
6F 7T 8T 9F 10 T

11. seven weeks 12. five people 13. grab people's 14. ground 15.

ƠN
attention floor assignment

16 versatile 17 common 18 fridge section 19 sponge 20 stir-fry


kind
NH
21 cubes 22 add flavors 23 loosen it up 24 cashew nuts 25
enriched,
delicious,
vegetarian
Y

LISTENING SCRIPT
Part 1: You will hear an extract from a lecture about solar eclipses in history. For question 1-
QU

5, choose the answer (A, B, C or D) which fits best according to what you hear. (10 pts)
Good evening and welcome to this month's Observatory Club lecture. I'm Donald Mackey and
I'm here to talk to you about the solar eclipse in history. A thousand years ago, a total eclipse of
the sun was a terrifying religious experience. But these days, an eclipse is more likely to be
viewed as a tourist attraction than as a scientific or spiritual event. People will literally travel
M

miles to be in the right place at the right time to get the best view of their eclipse. Well, what
exactly causes a solar eclipse? When the world goes dark for a few minutes in the middle of the

day, scientifically speaking, the dark spot itself is easy to explain.


It is the shadow of the moon streaking across a different and, to all intents and purposes, a
seemingly random part of the globe. In the past people often interpreted an eclipse as a danger
signal heralding disaster, and in fact the Chinese were so disturbed by these events that they
Y

included among their gods whose job was to prevent eclipses. But whether or not you are
superstitious, or take a purely scientific view, our earthly eclipses are special in three different
DẠ

ways. Firstly, there can be no doubt that they are very beautiful. It's as if a deep blue curtain has
fallen over the daytime sky, as the sun becomes a black void surrounded by the glow of its outer
atmosphere. But beyond this, total eclipses possess a second more compelling beauty in the eyes
of us scientists. For they offer a unique opportunity for research.

Page 1 of 10
Only during an eclipse can we study the corona and other dim things that are normally lost in
the sun's glare. And thirdly, they are rare. Even though an eclipse of the sun occurs somewhere
on earth, if you sit in your garden and wait, it will take 375 years on average for one to come to
you. If the moon were any larger, eclipses would become a monthly bore. If it was smaller, they

L
simply would not be possible. The ancient Babylonian priests, who spent a fair bit of time

A
staring at the sky, had already noted that there was an 18-year pattern in their recurrence, but
they didn't have the mathematics to predict an eclipse accurately. It was Edmund Halley, the

CI
English astronomer, who knew his maths well enough to predict the return of the comet, which
incidentally bears his name.
Part 2: You will listen to TV program about Gordon Ramsay sharing about his job and decide

FI
whether the following sentences are true (T) or false (F). (10 pts)
https://www.youtube.com/watch?v=NMTA6JoU_D8
GORDON RAMSAY: Cooking is a life skill. For me, it's fundamentally important. It's just as

OF
crucial as keeping face because Latin, history, geography-- no disrespect, but if you're not going
to teach that for the rest of your life, it doesn't come into play. Cooking does, three times a day,
seven days a week,
for the rest of your life to eat. So we don't cook three meals a day, but there's one meal across

ƠN
that day that needs to be absolutely you on a plate, done properly, healthily cooked, and sets you
up, because you are what you eat. So we can never underestimate the importance of food
because that's the fuel, and especially in sport. This is my little nicoise-- a specialized nicoise,
and it's the kind of thing that I pick myself up after cycling 200K. A beautiful piece of tuna.
NH
Brush that first with a little bit of mustard. Stops it from drying out. Now, sesame Just
beautifully roll that down. I always think about foods everywhere I go.
So when I'm training, I'm never very far away from food. I had that extraordinary moment at the
start of Kona two years ago where I was paddling out sort of half past five in the morning
waiting for that amazing cannon to go off. To get rid of the nerves, I just started thinking about
Y

recipes and dishes and how do I get through this 2.8K swim. The garnish-- it's very, very simple.
QU

We got potatoes. Let's start with a little teaspoon of olive oil. We'll get these potatoes nice and
crispy. Cut it in half, and put that side down. So, in my bowl, got some really nice shallot rings.
Olive nicoise, some black olives, some green beans. And then in there, I'm just going to open up
some of these little anchovies. A little touch of vinaigrette. This is classic vinaigrette made with
lots of lemon juice. And then we'll just mix that up nicely with some fresh parsley. Your
M

tomatoes, balsamic vinegar, fresh lemon juice in-- that lifts everything. You work 14, 15 hours a
day to perfect an absolutely stunning dish. It disappears in 2 and 1/2 minutes. And that's why I

said it loses you. You get on that journey and nothing else matters except what you put on the
plate.
Because you start off with these raw ingredients and you go through that journey. 60 minutes
later, you got this bit of magic. Also, it's an incredible passion. It's a huge canvas. Just take your
salad leaves. I want to use the little hearts. Just open them up gently without sort of ripping
Y

them. Get down to the heart, and cut the hearts into fourths. Potatoes-- nice and crispy. And then
DẠ

from there, as they start to cool down, just put a little touch of vinaigrette over there and hit
them with your parsley. Food was my calling, I think, because that was the way I could sort of
disappear. Disappear, travel, learn, and get really excited about something. And my first dream

Page 2 of 10
was to go to France. To understand why were they the sort of foundation of cooking. Why did
they start it? How did they start it? And I disappeared. I became French.
Within 18 months, I was fluent in the language. I was holding my own in a foreign kitchen, and
I was seriously cooking my ass off. Now, for the dressing. Very, very simple. Touch of the

L
vinaigrette, and then some fresh lemon juice. That gives a nice lightness, vibrancy to the dish as

A
well. Just lightly dress them.
My first set of knives was bought by a charity because my parents couldn't afford to send me to

CI
cooking school, and this knife wallet was a "Swimbo." It was with bright, yellow handles. And
trust me, they couldn't cut butter, but they looked the part. My mom had a tear in her eye
because she was upset she couldn't afford it. My first set of whites and my first set of knives. I

FI
still got them. Finally, the tuna.
Get the pan nice and hot. Put a teaspoon of oil in there. Tuna into the pan. 90 seconds each side.

OF
Out. And literally squeeze some fresh lemon juice on there. When you slice it, let the knife do
the work.
Fingers on top and just slice through nice and carefully. On. Three nice slices. Foods your
journey, and there's something quite amazing about the way you cook. Of giving pleasure. My
dream, when I started out as a chef, was to discover every ingredient and never be intimidated

ƠN
and not know what to do with it.
That level of perfection. I think I'm about 97% of the way there. There must be about 3% or 4%
of the ingredients that I still haven't discovered yet, and that's the exciting thing about food. That
NH
is what I call my specialized tuna nicoise.
Part 3: You will hear a short conversation between two students. Listen and give short
answers to the questions. Write NO MORE THAN THREE WORDS AND/OR A NUMBER
taken from the recording.
(10 pts) https://ielts-up.com/listening/ielts-listening-sample-5.3.html
Y

Julie: Hi Kate. How did you like the first lecture?


Kate: Hi Julie. You mean that lecture about brand identity.
QU

Julie: Exactly. When our professor described how brand names are created. As I understood,
he'll teach us practical tips and techniques to create better brands.
And what about course duration?
Kate: I heard it's five weeks. No, five weeks is without the final project. The total duration is
seven weeks.
M

Julie: Oh, I forgot about it. It's the first time I'll be doing a group project. And I'm a bit nervous
to work in a five-people team.

Kate:I think it's going to be okay. He was talking about the right marketing strategy. Did you
hear that?
Julie:Yes, that was the most exciting part of the presentation. The key thing is to grab people's
attention.
Kate: That's true. And how do you like the curriculum?
Y

Julie: I think it's nice. It's great that we'll be learning marketing in detail, as I've always been
DẠ

interested in this subject.


Kate: Me too. And after that, we'll be training to design custom logos. The professor told that
the curriculum has been modified, and we won't learn design this year.
What a pity! The same with product management. But you shouldn't worry, they just combined
these two sections into a larger one called branding. Then it's good. Moreover, they've added
Page 3 of 10
something called e-commerce this year. It's a great alteration and the course is kept up to date.
Online trading is so popular now. And after that we'll learn advertising. Yes, and also analytics.
Really? Won't analytics be replaced with customer attraction?
Julie:Yes, you're right.

L
Kate: I forgot about that.

A
Julie: I think that's all. The professor also teaches a course called Business Strategies. So, we
may take his other course next year. Dear, and where is the next lecture?

CI
Kate: It's in the big classroom on the ground floor.
Julie:Thanks.
Kate: You're welcome. Don't forget to take your last week assignment.

FI
Julie:Okay. Goodbye.
Kate: Okay. Goodbye.
Part 4. For questions 16–25, listen to a 5-Minute-Food-Fix Podcast about Tofu puffs and fill

OF
in the missing information. Write NO MORE THAN THREE WORDS taken from the
recording for each answer in the spaces provided. (20 points) (https://player.fm/series/5-
minute-food-fix/tofu-puffs )
I think about this job sometimes as being about demystifying food.

ƠN
Yeah, so Simon, you've been a food writer, you've edited countless cookbooks, and that's kind of
the job, right?
Very much.
Yeah, like people get scared, people think some ingredients are intimidating, and our job is to be
NH
conduits of information to say, no, you can actually do this.
To peel back the curtain and reveal the wondrousness of that food.
When you just said peel back the curtain, it sounded really gross.
It is.
How awful. Can we just cut that?
Y

Can we start again?


No, that was my favourite bit.
QU

I'm Yumi Stein. And I'm Simon Davis.


And this is your peeling back the curtain five-minute food bit.
Oh, dear God.
So one of the things I wanted to peel back the curtain on was tofu puffs. So tofu in general, and
M

we've talked about this before, there's such a myriad of options and varieties and flavours and
ways to use tofu, but as somebody who really doesn't eat a lot of meat, tofu is something that I

just adore. I find it so versatile and I think there's a lot of mystery around it for people who
aren't familiar with it. I really want to illuminate one kind of tofu that you can buy pretty
commonly. Places like Harris Farm, also Coles and Woolies, I've seen it there, but definitely
Asian Grocers if you're lucky enough to have one in proximity to your home. It's called fried tofu
puffs or tofu puffs and they are usually in the fridge section of the grocer. They're almost ready
Y

to eat, like you could cut open the bag and put one in your mouth and you would not drop dead.
DẠ

That's always good. Which is a bonus for a food staff. But I did want to talk you through how to
get the most out of them. So different sorts of tofu have different properties. The soft silky
custardy one doesn't take on lots of flavor. These ones are spongy. So picture it like a sponge.
It'll soak up any soup that it sits in. It'll be like you could literally squeeze soup out of it. It will

Page 4 of 10
capture a lot of flavors and a lot of wetness and moisture of anything that you cook. And given
that it's already in a cooked state and you don't have to do much to it, treat it like a stir-fry
ingredient or a pre-cooked soup ingredient. That's my advice for tofu puffs. If you've got, say,
some bok choy, one of our favorite ingredients, it's just so cheap, it's so green, it's always fresh

L
and available. So those two things together make something really, really delicious. Let's talk

A
about what you could do with bok choy plus tofu puffs. With the tofu puffs, they come as a cube,
sometimes big, two or three centimeters square, and sometimes smaller, like one centimeter

CI
square. If you want to really maximize how much flavor they'll suck up, then I would cut them in
half. It's a way of handling that you might not be up for. If you just cannot be bothered, then
don't worry. But if you want to think, oh, how can I just shove as much flavor in that, definitely

FI
cutting them in half will do that. And then you want to stir fry the bok choy in a little bit of oil,
say a neutral canola or grapeseed type oil, and then add your flavours, so soy sauce, ginger,

OF
oyster sauce. Don't overdo it though, because they're going to retain all of that. It's not going to
spread. So you might want to even add some sake or water or stock, just kind of loosen it up.
And then toss your fried tofu puffs through, give it a good stir, let them warm up. Corners might
burnish and get a little bit brown and delicious.

ƠN
That sounds good.
I know. And then if you want chili, of course, because chili goes well in everything, and then a
couple of handfuls of cashew nuts. And that's actually kind of like an almost complete meal.
That's a whole dinner, isn't it?
NH
Yeah. I mean, if you wanted a side of rice, by all means. But if you couldn't be bothered, you're
kind of getting a really enriched, delicious, vegetarian meal with very little stress.
That sounds perfect, Stevie.
I know.
If you don't really quite know what a tofu puff is. It sounds like something you could perhaps
Y

rest your feet on while watching TV. It does a bit, doesn't it? It does a bit. I'm going to put up a
photo of what tofu puffs look like in the wild in the shops on our 5-Minute Food Fix Instagram.
QU

Check it out there. And don't forget to tag the shops on our 5-Minute Food Fix Instagram.
Check it out there. And don't forget to tag your friends in your favourite food podcast.
B. LEXICO - GRAMMAR (30 points)
Part 1. Choose the correct answer A, B, C or D to each of the following questions. Write your
M

answers in the corresponding numbered boxes provided.(20 pts)

26. B 27. B 28. D 29. C 30. B 31. D 32. B 33. A 34. A 35. D

36. D 37. D 38. C 39. A 40. B 41. A 42. C 43. D 44. A 45. B

Part 2. Write the correct form of each bracketed word in the numbered space provided in the
Y

column on the right. (10 pts)


DẠ

46. sufficient 47. fearful 48. sensible 49. favourable 50. restless

51.tresspasser 52. upbringing 53.authorization 54. selfishness 55. runways

Page 5 of 10
C. READING (60 points)
Part 1:
56. system 57. Intervention 58. conditions 59. part 60. produced

L
61. estuaries 62. tide 63. chemical 64. particles 65. life

A
Part 2:

CI
66 A
67 C

FI
68 D
69 B

OF
70 C
71 D
72 A

ƠN
73 B
74 C
75 B
NH
PART 3:
76 T
77 F
Y

78 T
QU

79 F
80 F
81 NG
82 T
M

83 F
84 A

85 D
86 B
87 D
Y

88 A
DẠ

PART 4:
89 E
90 B

Page 6 of 10
91 G
92 C
93 H

L
94 A

A
95 F

CI
Part 5.
96 E
97 C

FI
98 D

OF
99 B
100 A
101A
102B

ƠN
103E
104D
105C
NH
D. WRITING (60 points)
Part 1: (15 pts)
Contents (10 points)
- The summary MUST NOT contain personal opinions.
Y

Language use (5 points)


The summary should:
QU

- show attempts to convey the main ideas of the original text by means of paraphrasing
(structural and lexical use),
- demonstrate correct use of grammatical structures, vocabulary, and mechanics (spelling,
punctuations,...),
- maintain coherence, cohesion, and unity throughout (by means of linkers and transitional
M

devices).
Penalties

- A penalty of 1 point to 2 points will be given to personal opinions found in the summary.
- A penalty of 1 point to 2 points will be given to any summary with more than 30% of words
copied from the original.
- A penalty of 1 point will be given to any summary longer than 130 words or shorter than 90
words.
Y

Model answer
DẠ

Camouflaging and mimicry have helped defenseless insects escape from their predators.
Camouflaging requires insects to have body colors close to the surroundings so as to appear less
eye-catching to predators. The moth caterpillars look like dead twigs while certain butterfly
caterpillars resemble bird droppings. Some butterflies and moths have wings that resemble dried

Page 7 of 10
leaves. Mimicry requires harmless insects to adopt the body colors and shapes of the wasps and
bees so as to fool their predators into thinking that they are dangerous. Predators usually avoid
them, thinking they have stings too. The bee-fly and hoverfly assimilate the body colors of the
bumble bee and wasps respectively and the bee-fly even hums like the bumble bee. ( 118 words

L
)

A
Part 2: 15 pts

CI
Contents (10 points)
- The report MUST cover the following points:
•Introduce the chart (2 points) and state the striking features (2 points)

FI
•Describe main features with relevant data from the charts and make relevant comparisons (6
points)
- The report MUST NOT contain personal opinions. (A penalty of 1 point to 2 points will be

OF
given to personal opinions found in the answer.)
Language use (5 points)
The report should:
- demonstrate a wide variety of lexical and grammatical structures,

ƠN
- have correct use of words (verb tenses, word forms, voice,…); and mechanics (spelling,
punctuations,...).
Model answer
The provided line graph delineates the proportion of people watching TV and listening to the
NH
radio who were over 4 years old in a 24 - hour period in the UK from October to December
1992.

From an overall perspective, it is readily apparent that the majority of people listened to the
Y

radio in the morning, while most people watched TV at night. Another striking feature is that
QU

watching TV was the more popular activity compared to listening to the radio for most of the
period in question.

At 6 am, nobody watched TV, yet the figure for this activity started to increase, albeit slightly,
to 5% after two hours only to decline by around 2% at 10 am. The figure then regained its
M

upward streak and surpassed that for the radio at roughly 15% by 2 o’clock, prior to a surge to
the peak at approximately 48% at 8 pm. Just after midnight, however, only 1 out of 10 people

was reported to engage in this pastime. Watching TV began to lose people’s interest as time
went by, yet a marginal rise was recorded before 6 am, culminating the graph at around 4%.
Y

Commencing at around 7% higher, the data of the radio precipitously reached 27% just after 8
am, before witnessing a dramatic drop to 10% nearly 5 hours later. Despite a negligible recovery
DẠ

at 4 pm, the figure went down to the 8% marker, before another minimal rise was registered.
Nevertheless, it fell yet again and after minor fluctuations, ended at the same figure as the other
leisure pursuit.

Page 8 of 10
Part 3: 30 pts
The mark given to part 3 is based on the following criteria:
1. Task achievement (10 points)

L
a. All requirements of the task are sufficiently addressed.

A
b. Ideas are adequately supported and elaborated with relevant and reliable explanations,
examples, evidence, personal experience, etc.

CI
2. Organization (10 points)
a. Ideas are well organized and presented with coherence, cohesion, and unity.
b. The essay is well-structured:

FI
•Introduction is presented with a clear thesis statement introducing the points to be developed.
•Body paragraphs develop the points introduced with unity, coherence, and cohesion. Each body
paragraph must have a topic sentence and supporting details and examples when necessary.

OF
•Conclusion summarizes the main points and offers personal opinions (prediction,
recommendation, consideration,…) on the issue.
3. Language use (5 points)
a. Demonstration of a variety of topic-related vocabulary

ƠN
b. Excellent use and control of grammatical structures
4. Punctuation, spelling, and handwriting (5 points)
a. Correct punctuation and no spelling mistakes
b. Legible handwriting
NH
SUGGESTED ANSWER
Some people think that children should begin their formal education at a very early age and
should spend most of their time on school studies. Others believe that young children should
spend most of their time playing. Compare these two views. Which view do you agree with?
Y

Why?
QU

The question of when children should begin their formal education and how they should spend
their time is a topic of ongoing debate. There are two contrasting views on this matter. Some
people argue that children should start their formal education at a very early age and prioritize
academic studies, while others believe that young children should spend most of their time
playing. Both perspectives have their merits, and the choice between them depends on various
M

factors.

Proponents of early formal education argue that starting at a young age provides children with a
head start in acquiring foundational knowledge and skills. They believe that structured learning
environments can foster cognitive development, enhance early literacy and numeracy skills, and
lay the groundwork for future academic success. Advocates also suggest that early education can
promote discipline, socialization, and a sense of routine, which are valuable for a child's overall
Y

development.
DẠ

On the other hand, supporters of play-based learning emphasize the importance of allowing
children to explore and engage in unstructured activities during their early years. They argue
that play is a natural and essential part of childhood, enabling children to develop creativity,
problem-solving abilities, and social skills. Play-based approaches encourage self-discovery,
imagination, and emotional development, allowing children to learn at their own pace and
Page 9 of 10
develop a love for learning.

In considering these perspectives, it is important to recognize that both formal education and
play have valuable roles in a child's development. A balanced approach that integrates both

L
elements can be beneficial. Young children have a natural curiosity and an innate desire to learn,
and play can serve as a vehicle for acquiring knowledge and skills. Combining play-based

A
activities with intentional learning experiences can provide a well-rounded education that
addresses different aspects of a child's development.

CI
Personally, I lean towards the view that emphasizes the importance of play in early childhood
education. Play is an integral part of a child's development, fostering creativity, imagination, and

FI
social interactions. Through play, children explore and make sense of the world around them,
developing problem-solving abilities and emotional resilience. While formal education has its
benefits, I believe that a play-based approach in the early years allows children to cultivate a

OF
love for learning, which serves as a strong foundation for future academic pursuits.

ƠN
NH
-------------- THE END --------------
Y
QU
M

Y
DẠ

Page 10 of 10
TRƯỜNG THPT CHUYÊN KỲ THI CHỌN HỌC SINH GIỎI CÁC TRƯỜNG THPT CHUYÊN
NGUYỄN TRÃI KHU VỰC DUYÊN HẢI VÀ ĐỒNG BẰNG BẮC BỘ
LẦN THỨ XIV, NĂM 2023
ĐỀ ĐỀ XUẤT

A L
Đề thi gồm 11 trang ĐỀ THI MÔN TIẾNG ANH – LỚP 10
Thời gian: 180 phút (Không kể thời gian giao đề)

CI
SECTION A: LISTENING (50 points)

FI
HƯỚNG DẪN PHẦN THI NGHE HIỂU
• Bài nghe gồm 4 phần; mỗi phần được nghe 2 lần, mỗi lần cách nhau 05 giây; mở đầu và kết

OF
thúc mỗi phần nghe có tín hiệu. Thí sinh có 20 giây để đọc mỗi phần câu hỏi.
• Mở đầu và kết thúc bài nghe có tín hiệu nhạc. Thí sinh có 03 phút để hoàn chỉnh bài trước tín
hiệu nhạc kết thúc bài nghe.
• Mọi hướng dẫn cho thí sinh (bằng tiếng Anh) đã có trong bài nghe

ƠN
Part 1: You will listen to a short conversation about jobs in restaurants. For questions 1-5,
complete the table using NO MORE THAN TWO WORDS AND/ OR A NUMBER for each
answer. (10 points)
NH
Location Job title Responsibilities Pay and conditions
Wivenhoe Street Breakfast supervisor Checking (1) Starting salary 9.50
________ per hour
Y

Making sure Start work at (2)


equipment is clean ________
QU

(3) __________ Junior chef Supporting the senior Annual salary


staff and (4) ______ £23,000
Maintaining stock and No work on every (5)
arranging deliveries _______
M

Part 2: You will listen to a talk about Scandinavia. Listen and decide if the following statements

are true (T) or false (F). (10 points)


6. The presence of state-controlled monopolies on alcohol in both Norway and Denmark
contributes to elevated prices.
7. The Scandinavian nations need substantial taxation to sustain their welfare states.
Y

8. Ikea has been transferred to a parent company in Scandinavia because of lower taxation rates in
Sweden.
DẠ

9. The substantial welfare state of the region is thought to guarantee its financial security, job
stability, and equitable economic distribution.

Page 1
10. Scandinavian citizens receive highly affordable education, access to a free public
transportation system , and a well-operating healthcare service.
Part 3: For questions 11–15, listen to a radio discussion about children who invent imaginary

L
friends and choose the correct answer A, B, C or D which fits best according to what you hear.

A
(10 points)
11. In the incident that Liz describes

CI
A. her daughter asked her to stop the car.
B. she had to interrupt the journey twice.

FI
C. she got angry with her daughter.
D. her daughter wanted to get out of the car.
12. What does the presenter say about the latest research into imaginary friends?

OF
A. It contradicts other research on the subject.
B. It shows that the number of children who have them is increasing.
C. It indicates that negative attitudes towards them are wrong.
D. It focuses on the effect they have on parents.

ƠN
13. How did Liz feel when her daughter had an imaginary friend?
A. always confident that it was only a temporary situation
B. occasionally worried about the friend's importance to her daughter
C. slightly confused as to how she should respond sometimes
NH
D. highly impressed by her daughter's inventiveness
14. Karen says that one reason why children have imaginary friends is that
A. they are having serious problems with their real friends.
B. they can tell imaginary friends what to do.
Y

C. they want something that they cannot be given.


D. they want something that other children haven't got.
QU

15. Karen says that the teenager who had invented a superhero is an example of
A. a very untypical teenager.
B. a problem that imaginary friends can cause.
C. something she had not expected to discover.
M

D. how children change as they get older.


Part 4: For questions 16-25, listen to a recording about the skin care industry in Korea. Write

NO MORE THAN THREE WORDS AND/OR A NUMBER taken from the recording for each
answer. (20 points)
It is anticipated that the skin care industry's revenue will experience growth over the next four
years to more than (16) __________.
Y

Skin care is a big moneymaker for big (17) ___________.


Skin care makes up (18) __________ of the annual revenue of the world beauty industry.
DẠ

According to Larissa Jensen, skin care is actually growing the fastest of all the beauty
categories tracked, including (19) ___________.

Page 2
Wellness is associated with clean eating, the latest (20)___________ and the no-makeup
look.
In South Korea, the latest skin care innovations are (21)___________ and large

L
manufacturing takes place first.

A
Charlotte Cho is a Korean (22) ___________, an entrepreneur, and the author of "The Little
Book of Skin Care”.

CI
She firmly believes Korea has allowed (23)___________ to come to the forefront of a lot of
the skin care trends.

FI
South Korea is both a huge (24) __________ and a large exporter of beauty products.
Many beauty brands from Europe and America are making their products with (25)
___________ chemists and manufacturing plants.

OF
SECTION B: LEXICO-GRAMMAR (40 POINTS)
Part 1: Choose the best answer (A, B, C or D) to each of the following questions and write your
answer (A, B, C or D) in the corresponding numbered boxes. (20 points)

ƠN
26. Major political party is ______ the campaign for tighter gun controls in the wake of last month’s
shooting.
A. procuring B. solidifying C. spearheading D. fulfilling
27. Despite his poor performance in last month’s athletics trials, Jack is still ______ for a medal in
NH
next week’s race.
A. in the running B. by a mile C. on his plate D. on the go
28. In its ______, the city of Bath was a magnet for wealthy and fashionable people from all over
the country.
Y

A. dream B. foothold C. prestige D. heyday


29. Jenny ______ her audition and was immediately offered the part!
QU

A. went up B. scraped through


C. took off D.waltzed through
30. The money laundering scandal lead to the minister’s ______ fall from political power.
A. prodigious B. precipitous C. dismal D. persistent
M

31. The CEO was found guilty of ______; nobody could believe that he had been deceiving the
company for so many months.

A. trafficking B. arson
C. embezzlement D. hooliganism
32. The ______ windows of the house seemed to indicate that it had been derelict for a long time.
A. faced-up B. long-awaited C. semi-detached D. boarded-up
Y

33. The office was closed for a week for refurbishment and now the staff have to deal with the _____
that built up during their absence.
DẠ

A. backstage B. backlash C. backlog D. backdrop


34. Garlic being sold as organic was found to contain pesticide _____ and was removed from the
market.

Page 3
A. elements B. residues C. filaments D. variables
35. Sam has no ______ for the night of the crime and, therefore, he is the main suspect in this case.
A. verdict B. surveillance C. alibi D. witness

L
36. I don’t think it would be wise to try to make Max change his mind about divorcing Barbara.

A
Well, in his place I _____ her at all.
A. would never have married B. needn’t have married

CI
C. would never marry D. must never have married
37. I think Candy _____ the prize if she plays this well during the competition.

FI
A. is in for winning B. is bound to win C. may as well win D. is set for winning
38. Find someone who will let you talk things through, or _____ that, write down your thoughts.
A. except B. failing C. for all of D. given

OF
39. _____, water is required by all living things on Earth and life cannot survive without it.
A. It is known as far B. Known as far as it is
C. As known as far D. As far as is known
40. _____, he tried to fix the machine.

ƠN
A. Not being a computer expert notwithstanding
B. Despite of not being a computer expert
C. Though no computer expert
D. In spite of being no computer expert
NH
41. For the first few months the babies looked so alike I couldn't tell _____.
A. who is whom B. which is which C. which from which D. whom with whom
42. _____, the meeting began.
A. After we have sat down B. All of us having taken the seats
Y

C. Our having seated D. Once we had seated


43. They must have gone away, _____?
QU

A. shouldn’t they B. haven’t they C. mustn’t they D. didn’t they


44. Kate: ‘Did the minister approve the building plans?’
Bill: ‘Not really. He turned them down _____ that the costs were too high.’
A. on the grounds B. provided C. in case D. supposing
M

45. The draw took place yesterday but the competition winners _____.
A. are yet to be announced B. haven’t been yet announced

C. are as yet to have been announced D. haven’t announced yet


Part 2: Give the correct form of each given word to complete the following sentences. Write your
answers in the numbered boxes. (10 points)
46. I am glad that the children have taken advantage of the long walk so ___________. (felicity)
Y

47. The _________ of the technology comes at the end of a long period of innovation. (calcium)
48. Is your name_____________? (hyphen)
DẠ

49. Many people without jobs are living on the___________. (bread)


50. The inspection of a vehicle's steering mechanism and tyres is designed to ensure that it
is ________ in terms of its steering characteristics. (road)

Page 4
51.The majority of infected persons are ____________ and unaware of their condition. (symptom)
52.The two losers got to follow in the footsteps of their fellow ____________ by walking the
plank. (patriotic)

L
53. Casting a vote in a _________ is not exactly the same as advancing an opinion in a survey.

A
(refer)
54. The new system __________ with existing telephone equipment. (face)

CI
55. The principal has taken the students’ petition under _____. (advise)

FI
Part 3. Complete each of the following sentences with suitable preposition(s). Write your answer
in the boxes provided on the answer sheet. (10 points)
56. It was too late to back ___________ the contract.

OF
57. Their warnings about the recent robbery put her ___________ the defense.
58. Alice is going to put ________ the job of a sales assistant in Newcastle.
59. The conflict in the production department boils _______ the employees’ dissatisfaction with
their wages.

ƠN
60. Tiredness can easily creep ________ you while you’re driving.
61. I can only admire his conscientiousness. It’s over and _________ the call of duty.
62. The house was near a park but there was a road in ___________.
63. You’ll have to tell me the truth. Stop trying to pull the wool __________ my eyes.
NH
64. His jokes seemed to go _________ very well with his audience, if their laughter was any
indication.
65. He put it _________ that he was thinking of leaving the company and, as a result, he received
several offers from rival organisations.
Y

SECTION 3: READING (60 points)


QU

Part 1: For questions 66-75, read the following passage and decide which option (A, B, C, or D)
best fits each gap. Write your answers in corresponding numbered boxes on the answer sheet.
(15 points)
NATURE’S CLOCKS
M

Our biological clocks govern almost every aspect of our lives. Our sensitivity to stimuli
(66)__________over the course of the day, and our ability to perform certain functions is subject

to fluctuations. Consequently, there is a (n) (67) ___________ time for tasks such as making
decisions: around the middle of the day. Anything that (68) ___________ physical co-ordination,
on the other hand, is best attempted in the early evening. What is more, there is a dramatic drop in
performance if these activities are (69) __________ out at other times. The risk of accident in a
Y

factory, for example, is 20% higher during the night shift.


Primitive humans lived their lives in tune with daily cycle of light and dark. Today we are
DẠ

(70)__________convinced that we can impose schedules on our lives at (71)___________ Sooner


or later, however, we pay a price for ignoring our natural rhythms. A good example is jet lag,
caused when we confuse our body’s biological clocks by (72) _________several time zones.

Page 5
People suffering from jet lag can take several days to adjust to new time zones, and have a reduced
ability to make decisions, which is a worrying thought, as serious (73)__________ of judgment
can be made. And this may be just the (74) __________ of the iceberg. An increasing number of

L
people suffer from seasonal affective disorder (SAD), a form of depression that can be (75)

A
_________ by living in artificial conditions SAD can be serious, and suffers may even need to
take antidepressant drugs.

CI
66. A. modifies B. ranges C. varies D. wavers
67. A. peak B. summit C. maximum D. optimum

FI
68. A. requests B. demands C. dictates D. stipulates
69. A. made B. done C. carried D. performed
70. A. powerfully B. firmly C. steadily D. highly

OF
71. A. whim B. determination C. will D. desire
72. A. landing B. penetrating C. crossing D. travelling
73. A. errors B. mistakes C. inaccuracies D. fallacies
74. A. peak B. pinnacle C. top D. tip

ƠN
75. A. triggered B. developed C. created D. launched

Part 2: For questions 76-85, fill each of the following numbered blanks with ONE suitable word
and write your answers in the corresponding numbered boxes on the answer sheet. (15 points)
NH
If a picture is (76) ______ a thousand words, a map may equate (77) ______ a million.
Maps, infographics, and photographs blur the (78) ______ between information and art, at once
documenting the state of the world in which information is presented in a (79) _______ dynamic
way. Unlike photographs, however, no map can (80) ______ to be a direct portrait of reality. Every
Y

map, no (81) _______ how simple it appears, is shaped by the viewpoint and the choices of the
person who made it. On (82) ______ of that, every person who reads a map will (83) ______ it
QU

differently than the next. Our interpretations of maps are influenced (84) ______ as much by our
perceptions and personal experiences as by what's actually on the page. For this reason,
understanding subjectivity has become an important part of both geography and cartography. The
long history of cartography has led people to trust maps as a (85) ______ of information about the
M

world.
Part 3: For questions 86-95, read the following passage and circle the best answer to each of

the following questions. Write your answers in corresponding numbered boxes provided on the
answer sheet. (15 points)
Simply ticking the boxes isn’t enough
I have been asked what I think about the idea of ‘Investing in People’. The best answer I can
Y

give is that I think what it tries to achieve - basically making the link between business
improvement and focusing on the needs of the people who work for an organization - is great. My
DẠ

problem is with organizations who subscribe to it as a way to help the ‘get better’, when they don’t
bother to understand where they went wrong in the first place. They need to ask what explicit and
implicit policies and procedures they have in place that prevent their people from being able to do

Page 6
the right thing for the right reasons.
I am sure that there are managers out there who don’t know any better, and assume that to
manage they simply need to put pressure on their people to perform. But people don’t demonstrate

L
high performance because they see the need to do it, and make the choice to do so. They do it

A
because they are connected to the business goals and they see how their contributions can help
achieve them. Such managers may tell themselves they can put a ‘tick’ in the ‘we care about

CI
people’ box. But simply putting ticks in boxes is no good if it doesn’t reflect reality.
I know of a company that was so concerned that its people were doing the ‘right thing’ that

FI
it put in place a series of metrics to measure their effectiveness. So far, so good. But one of the
objectives - making successful sales calls - manifested itself in the metric ‘Number of potential
customers seen in one day’. The sales people obviously focused their efforts on going from one

OF
customer’s office to another, and not on closing deals. Instead of the employees becoming more
effective, they focused on getting the boxes ticked. Good intent; poor thinking.
Another company wanted to improve the speed with which it was able to introduce
new products. Competition was beating it to the market place, and consequently the company was

ƠN
losing market share. Senior management sent out the message to reduce the time spent in getting
products into customers' hands, with the explanation that they couldn't afford delays. This was a
relatively easy task, especially since the time spent testing the products was cut in half to
accomplish the time reduction. The result was new products were introduced in less time than
NH
those of competition – but soon rejected by customers for poor quality. Good intent; reckless
implementation.
A third company I know is trying hard to help employees see that they have some control
over their future. The company instituted a programme with a title like ‘Creating our own future’
Y

or something like that. A good idea; get the people involved in the future of the company. But
instead of the employees becoming motivated to contribute, they saw it as a hollow exercise on
QU

the part of senior management who, in the past, had paid little attention to anything other than
getting the job done so they could report great earnings. Yes, the programme was a big ‘tick the
box’ effort, but that was all it was in the minds of the people that it was designed for.
A final example is of a company that brought in one of their ‘Investing in People’
M

programmes to change the way the company was run. Assessors were running around like crazy,
heling managers examine how they managed. They told managers how they could manage better.

And when the programme was over, the company was able to say that they had done it - it had
invested in its people and life was now good. But the managers simply went back to business as
usual. After all, the assessors were gone, and they had targets to hit.
[95A] All these examples are representative of senior management who see the need to
Y

improve things in their organization, but don't see how to do it. [95B] And when the employees
simply see the programme as a box-ticking exercise, then it's hopeless. If a company is going to
DẠ

go through the effort implied in investing in people, it should make it worthwhile. [95C] Defaulting
on the choice to improve the decision-making process by going through the motions is as lame as
senior management saying their people’s poor performance is not the senior management’s fault.

Page 7
[95D]
86.The writer thinks that putting the concept of “Investing in People” into practice……….
A. frequently results in confusion among the people it is supposed to help

L
B. involves more effort than some organizations are prepared to make

A
C. may create problems where previously there had not been any problems
D. is something that some organizations should not attempt to do

CI
87. The writer’s main point in the second paragraph is that the performance of employees………
A. may be very good even if management is poor

FI
B. cannot be accurately measured by any box-ticking exercise
C. is related to their knowledge of the organization as a whole
D. is not as unpredictable as some managers believe it to be

OF
88. What point does the writer make about the first company he describes?
A. It was not really interested in measuring the effectiveness of employees.
B. The targets that it set for staff were unrealistic.
C. It failed to understand the real needs of its employees.

ƠN
D. The data that it collected did not measure what it was supposed to measure.
89. The word “objectives” in paragraph 3 is closest in meaning to………….
A. purposes B. goals C. motives D. reasons
90. What point does the writer make about the second company he describes?
NH
A. It made what should have been an easy task into a complicated one.
B. It failed to foresee the consequences of an instruction.
C. It misunderstood why a new approach was required.
D. It refused to take into account the views of employees.
Y

91.What does the writer say about the programme introduced by the third company he mentions?
A. Employees did not believe that it had been introduced for their benefit.
QU

B. Employees felt that it was in fact a way of making their jobs even harder.
C.The reason given for introducing it was not the real reason why it was introduced.
D. It was an inappropriate kind of programme for this particular organization.
92. The word "hollow” in paragraph 5 is closest in meaning to……...
M

A. not sincere B. without real value C. empty D. deep


93. The writer says that the programme in his final example………...

A. was too demanding for managers to maintain long-term


B. was treated as a self-contained exercise by managers
C. involved some strange ideas on how managers could improve
D. cause managers to believe that their previous methods had been better
Y

94. The word "it" in the last paragraph refers to………..


A. senior management
DẠ

B. improving things in things in the organiztion


C. organization
D. the need

Page 8
95. Look at the four options [95A], [95B], [95C] and [95D], which indicate where the following
sentence could be added to the passage. Where would the sentence best fit?
For a start, a programme targeted at improving things is only as good as management's

L
ability to motivate their people.

A
A. [95A] B. [95B] C. [95C] D. [95D]

CI
Part 4. For questions 96-105, read the passage and do the following tasks. Write your answers
in the corresponding numbered boxes on the answer sheet. (15 points)

FI
MENDING BROKEN HEARTS
A. Although hearts suffer many maladies-valves leak, membranes become inflamed –
coronary heart disease, which can lead to heart attack and ultimately to heart failure, is the

OF
number one killer of both men and women in the United States, where 500,000 the
annually. Worldwide, it kills 7.2 million people every year. Exacerbated by the Western
lifestyle – motorized transport, abundant meat and cheese, workdays conducted from the
comfort of a well-padded chair – incidence of the disease is soaring.

ƠN
B. To help stem this lethal tide, cardiologists can prescribe such cholesterol-lowering drugs
as statins to help keep arteries clear. They can advise patients to change their habits, or they
can operate to fix a immediate problem. Angioplasty is one procedure, and surgery to
bypass the diseased arteries is another – each year more than 400,000 bypasses are
NH
performed in the U. S. Transplants can replace severely damaged hearts, and artificial ones
can keep people alive while they wait for a donor heart. But in the face of an impending
global epidemic, none of these stopgap measures addresses the essential question: Who
gets heart attacks and why?
Y

C. The human heart beats 100,000 times a day, propelling six quarts (5.6 liters) of blood
through 60,000 miles (96,560.6 kilometers) of vessels – 20 times the distance across the
QU

U. S. from coast to coast. The blood flows briskly, surging out of a ten-ounce (283.5 gram)
heart so forcefully that large arteries, when severed, can send a jet of blood several feet
into the air. Normally the relentless current helps keep blood vessels clean. But where an
artery bends, tiny eddies form, as in a bend in a river. This is where bits of sticky, waxy
M

cholesterol and fat can seep into the artery wall and oxidize, like butter going rancid. Other
matter piles up too. Eventually, the whole mas calcifies into a land of arterial stucco, or

plague.
D. Until recently, cardiologists approached heart diseases as a plumbing problem. Just as
mineral deposits restrict the flow of water through a pipe, an accretion of plague impedes
the flow of blood through an arterial channel. The more crud in the system, the greater
Y

likelihood that a dammed artery will trigger a heart attack. Doctors now dismiss this
“clogged-pipes model” as an idea whose time has passed. It’s just not that simple.
DẠ

E. Most heart attacks are caused by plague embedded within the artery wall that ruptures,
cracking the wall and triggering the formation of a blood clot. The clot flocks the flow of

Page 9
blood to the heart muscle, which can form the lack of oxygen and nutrients. Suddenly, the
pump stops pumping.
F. Contrary to the clogged pipes model, heart attacks generally occur in arteries that have

L
minimal or moderate blockage, and their occurrence depends more on the kind of plague

A
than on the quantity. Scientists have been struggling to figure out what type is most
responsible. Paradoxically, findings suggest that immature, softer plague rich in cholesterol

CI
are more unstable and likely to rupture than the hard, calcified, dense plagues that
extensively narrow the artery channel. But understanding the root cause of the disease will

FI
require much more research. For one thing, human hearts, unlike plumbing fixtures, are
not stamped from a mold. Like the rest of our body parts, they are products of our genes.
G. Don Steffensen was putting duck-hunting decoys out on a small lake one fall afternoon in

OF
South Western Iowa when his heart attack hit. The infarction was massive and unexpected.
It’s likely that Steffensen survived only because a buddy was carrying nitroglycerin tablets
and quickly slipped under his friend’s tongue. Nitroglycerin is used to make dynamite; in
the body, a heavily diluted form releases nitric oxide, which signals the smooth muscle

ƠN
cells in veins and arteries to relax, dilating the vessels.
H. The Steffensen clan is enormous: more than 200 relatives spread over three generations.
many of the youngest are now dispersed from Iowa to New York and beyond. Although
heart trouble is common in the family, it had never struck anyone as unusual. “I attributed
NH
it to diet,” shrugs Tina, a slim 38-year-old and the family’s only vegetarian.
I. It was a reasonable conclusion. The Steffensens were raised on the kind of farm food that
the state is famous for – ham balls, meatloaf, pie, macaroni and cheese – and still popular
even as careers have moved indoors. Driving north through cornfields to meet some of the
Y

family in Buffalo Center, I dined at a restaurant offering deep-fried sandwiches. A single


ham and cheese hoagie – clunked in hot fat and served sizzling – seemed capable of
QU

stopping a heart all on its own.


Questions 96 – 102:
Do the following statements agree with the information given in the passage? WRITE
YES if the statement agrees with the information
M

NO if the statement contradicts the information


NOT GIVEN if there is no information on this

96. Coronary heart disease is the largest culprit behind the deaths in the United States.
97. The Western lifestyle is the largest cause for the increase in the diseases.
98. Measures taken by experts have successfully answered the essential questions about heart
attack.
Y

99. Blood in human body could travel much more distance throughout the body on a daily basis
than that across the U. S. from coast to coast.
DẠ

100. Cholesterol is stored in the arteries to provide energy for various functions of the body.
101. The clogged pipes model is accepted by most doctors and specialists.
102. Scientists have yet to decide the most likely factor that leads to heart attack.

Page 10
Questions 103-105:
Complete each sentence with the correct ending A – G below based on the information in
paragraphs A – E.

L
103. Cardiologists prescribe cholesterol-lowering drugs in order to ___________.

A
104. Artificial hearts are introduced to ______________.
105. Heart diseases in arteries are compared to ___________.

CI
A. reduce the costs of transplantation surgery

FI
B. the blood clots – a kind of arterial stucco, or plague
C. deposits of minerals limiting the flow of water through a pipe
D. smooth the arteries

OF
E. cope with difficult situations
F. braincase the blood from flowing to the heart muscle and interfere with the absorption of oxygen
and nutrients
G. help patients survive until the availability of a donated human heart

SECTION D: WRITING (50 points)


Part 1: (20 points)
ƠN
The chart shows the percentage of male and female teachers in six different types of educational
NH
setting in the UK in 2010.
Summarize the information by selecting and reporting the main features, and make comparisons
where relevant.
Y

Part 2 (30 points): Write an essay to elaborate your opinion on the topic:
What should teenagers do to become responsible digital citizens?
QU

You should use your own ideas, knowledge and experience and support your arguments with
examples and relevant evidence. Write about 250 words.

--------------------------------------- HẾT-------------------------------------------
M

Y
DẠ

Page 11
TRƯỜNG THPT CHUYÊN KỲ THI CHỌN HỌC SINH GIỎI CÁC TRƯỜNG THPT CHUYÊN
NGUYỄN TRÃI KHU VỰC DUYÊN HẢI VÀ ĐỒNG BẰNG BẮC BỘ
LẦN THỨ XIV, NĂM 2023
ĐỀ ĐỀ XUẤT

A L
HƯỚNG DẪN CHẤM: MÔN TIẾNG ANH – LỚP 10

CI
SECTION A: LISTENING (50 points)

FI
Part 1: You will listen to a short conversation about jobs in restaurants. For questions 1-5,
complete the table using NO MORE THAN TWO WORDS AND/ OR A NUMBER for each

OF
answer. (10 points)

1. portions 2. 5.30 am 3. City Road 4. sous chef/ sous-chef 5. Monday

are true (T) or false (F). (10 points)


6. F 7. T 8. F
ƠN
Part 2: You will listen to a talk about Scandinavia. Listen and decide if the following statements

9.T 10. F
NH
Part 3: For questions 11–15, listen to a radio discussion about children who invent imaginary
friends and choose the correct answer A, B, C or D which fits best according to what you hear.
(10 points)
Y

11. B 12.C 13.A 14.C 15.C


QU

Part 4: For questions 16-25, listen to a recording about the skin care industry in Korea. Write
NO MORE THAN THREE WORDS AND/OR A NUMBER taken from the recording for each
answer in the spaces provided. (20 points)
16. 379 billion 17. beauty 18. about 24.9% 19. makeup and 20. fitness fads
M

dollars/ $379 brands fragrance


billion

21. concocted 22. beauty guru 23. indie brands 24. 25. Korean
manufacturing R&D
hub
Y

SECTION B: LEXICO-GRAMMAR (40 POINTS)


DẠ

Part 1: Choose the best answer (A, B, C or D) to each of the following questions and write your
answer (A, B, C or D) in the corresponding numbered boxes. (20 points)

26. C 27. A 28. D 29. D 30. B

Page 1
31. C 32. D 33. C 34. B 35. C
36. B 37. B 38. B 39. D 40. C
41. B 42. B 43. D 44. A 45. C

A L
Part 2: Give the correct form of each given word to complete the following sentences. Write your

CI
answers in the numbered boxes. (10 points)

46. felicitously 47. calcification 48. hyphenated 49. breadline 50. roadworthy

FI
51. 52. compatriots 53. referendum 54. interfaces 55. advisement
asymptomatic

OF
Part 3. Complete each of the following sentences with suitable preposition(s). Write your answer
in the boxes provided on the answer sheet. (10 points)

ƠN
56. out of 57. on 58. in for 59. down to 60. up on

61. above 62. between 63. over 64. down 65. about
NH
SECTION C: READING (60 points)
Part 1: For questions 66-75, read the following passage and decide which option (A, B, C, or D)
best fits each gap. Write your answers in corresponding numbered boxes on the answer sheet.
Y

(15 points)
QU

66. C 67. D 68. B 69. C 70. B


71. C 72. C 73. A 74. D 75. A

Part 2: For questions 76-85, fill each of the following numbered blanks with ONE suitable word
and write your answers in the corresponding numbered boxes on the answer sheet. (15 points)
M

76. worth 77. to 78. line 79. visually 80. claim

81. matter 82. top 83. interprete 84. just 85. source
Part 3: For questions 86-95, read the following passage and circle the best answer to each of
Y

the following questions. Write your answers in corresponding numbered boxes provided on the
DẠ

answer sheet. (15 points)


https://engexam.info/cae-reading-and-use-of-english-practice-tests/cae-reading-and-use-of-
english-practice-test-2/5/

Page 2
86. B 87. C 88. D 89. B 90. B
91. A 92. B 93. B 94. B 95. B

L
Part 4. For questions 96-105, read the passage and do the following tasks. Write your answers

A
in the corresponding numbered boxes on the answer sheet. (15 points)

CI
1.YES 2.NG 3.NO 4.YES 5.NG
6.NG 7.YES 8.D 9. G 10.C

FI
SECTION D: WRITING (50 points)
Part 1: (20 points)

OF
The chart shows the percentage of male and female teachers in six different types of educational
setting in the UK in 2010.
Summarize the information by selecting and reporting the main features, and make comparisons
where relevant.

ƠN
NH
Y
QU

Contents (10 points)


The report MUST cover the following points:
M

- Introduce the bar chart (2 points) and state the striking features (2 points)
+ female teachers dominated in nursery, primary and secondary schools

+ male teachers dominated in private training institutes and universities reversed.


+ equal rates in colleges
- Describe main features with relevant data from the charts and make relevant comparisons (6 points)
Language use (10 points)
Y

The report:
DẠ

- should demonstrate a wide variety of lexical and grammatical structures


- should have correct use of words (verb tenses, word forms, voice...) and mechanics (spelling,
punctuations,)

Page 3
Part 2 (30 points): Write an essay to elaborate your opinion on the topic:
What should teenagers do to become responsible digital citizens?
You should use your own ideas, knowledge and experience and support your arguments with

L
examples and relevant evidence. Write about 250 words.

A
The mark given to part 3 is based on the following criteria:

CI
1. Organization (5 points)
a. Ideas are well organized and presented with coherence, cohesion and unity.

FI
b. The essay is well-structured:
* Introduction is presented with clear thesis statement.
* Body paragraph are written with unity, coherence and cohesion.

OF
Each body paragraph must have a topic sentence and supporting details and examples when
necessary.
* Conclusion summarizes the main points and offers personal opinions (prediction,
recommendation, consideration ...) on the issue.

ƠN
2. Content (15 points)
a. All requirements of the task are sufficiently addressed.
b. Ideas are adequately supported and elaborated with relevant and reliable explanations,
examples, evidence....
NH
3. Language use (10 points)
a. Demonstration of a variety of topic-related vocabulary.
b. Excellent use and control of grammatical structures (verb tenses, word forms, voice...) and
mechanics (spelling, punctuations...)
Y

Listening transcript
QU

Part 1
Source: Part 1, test 2, Cmbridge ielts 18
M

Y
DẠ

Page 4
A L
CI
FI
OF
ƠN
NH
Y

Part 2: https://www.youtube.com/watch?v=-VJX2W3O9Ek&t=174s
QU

Across Scandinavia, the average price of everything from a knitted roll neck to pastries from the
local bakery are some of the highest in the world.
Scandinavia is a region in northern Europe that was historically made up of three kingdoms:
Norway, Sweden, and Denmark. They’re united by their Viking heritage but also their shared
reputation as one of the most expensive regions to live and visit. Whether it’s buying a car or a
M

TV, consumers in Scandinavia pay some of the highest prices in the world. Even buying
groceries is costly with all three countries’ food and drink prices above the European average.

All three countries also have a standard VAT rate of 25%. Amongst all the European nations,
only Hungary has a higher rate. Heading to the bar is also pretty pricey. In Norway, having a
beer or glass of wine will cost 2.5 times the EU average. Both Norway and Sweden have state-
run monopolies on alcohol which helps keep the prices up. This social market model, rather than
Y

a liberal market model like in the U.S., is common across Scandinavia and helps explain why the
cost of living is so high. Scandinavian countries have large welfare states with their social
DẠ

expenditures, as a percentage of GDP, among the highest in the world and this requires high
levels of taxation.

Page 5
Sweden has a top rate of personal income tax of over 60%, while Denmark’s is more than 55%,
both of which are well above the OECD average. In Denmark, if you want to buy a car you have
to pay anywhere between 85% and 150% tax on top of the cost of the vehicle.

L
And the taxes don’t stop there.

A
Let’s say I want to buy a sweater in Denmark.
I pay 300 kronor for it, but how much of that money actually goes to the vendor?

CI
Well, first there’s the 25% VAT - leaving the seller with 240 kronor.
The clothes shop also has to pay a minimum 22% of corporate income tax.

FI
That means that a big portion of the money I paid for my sweater goes to the Danish
government. Not to mention there’s a hefty payroll tax on employees' wages
and the store still has to pay for rent, electricity, and cleaning - all of which are taxed, too.

OF
These taxes mean that for Scandinavian businesses to make a profit,
they need to charge their consumers high prices. For some companies, this has proved too much
of a burden for their business model.
The world’s biggest furniture company IKEA was founded in Sweden but has moved its

ƠN
headquarters to the Netherlands. Through corporate restructuring, the business is now owned by
a non-profit Dutch parent company. In part due to the high taxes in Sweden.
But according to some experts, Scandinavia’s social democrat tendencies have led to a strong
social cohesion and has helped provide political stability. This, in turn, has made their economies
NH
safe havens for outside investors - which is one reason why the Danish, Swedish and Norwegian
currencies called krona are so strong.
This can be tough on tourists with the exchange rate doing them no favors. If the cost of living in
Scandinavia is so expensive, why then are its citizens some of the happiest in the world?
Y

In the past five years, Scandinavian countries have regularly topped the World Happiness
Report. That’s an index that measures overall life satisfaction based on different contributing
QU

factors. Some experts attribute these high satisfaction levels with Scandinavia’s large welfare
state which they say ensures financial security, job security, and economic distribution. In return
for high taxes, citizens get free state education, very cheap child care, a functioning public
transport and a free health service.
M

But a large and expensive social welfare state doesn’t necessarily mean the best.
For example, Norway is the only Scandinavian country that ranks

in the top 10 for adult education levels, amongst OECD countries.


And yet Scandinavians remain happy with the status quo, living within a social corporatist
economy that provides reliable economic welfare.
Having a beer at the end of the day does cost a small fortune, but for people living
Y

and working in Scandinavia, high prices provide a quality of life that’s worth paying for.
Part 3
DẠ

Presenter: Today we're talking about children and their tendency to have imaginary friends. Liz
McManus has a daughter called Caitlin, who's eight now. When she was three, she had an
imaginary friend called Tytner. Liz, tell us about Caitlin and Tytner.

Page 6
Mother: Well, give you an example. One day I was driving Caitlin and Greg, her baby brother,
home when she solemnly informed me that Tytner was hitting the baby. So I said: 'You tell Tytner
that if he does that again, he'll be walking home.' Fifteen seconds later came the inevitable news:

L
'He's just done it again, Mummy.' So I found myself in the embarrassing position of having to pull

A
over, open the back door and say to this imaginary little boy. `Tytner, out, now!' And of course, as
we drove off, Caitlin started crying because her friend was standing on the pavement all alone. I

CI
had to turn back and go through the rigmarole of pulling over and opening the door to pick
him up again.

FI
Presenter: Wow, that's some story! But in fact, Caitlin is no different from many children and her
invented, make-believe friend is far from unusual. As many as 65% of children have had an
imaginary friend at some point in their lives. The latest research suggests that invisible friends,

OF
far from being a cause for concern, should be welcomed by parents because they can help
children to be more creative, confident and articulate, and have more advanced
communication skills. It is thought that these findings will help reverse misconceptions about
children with imaginary friends and that they will come to be seen as having an advantage, rather

ƠN
than a problem that needs to be worried about. Did it worry you, Liz?
Mother: I know it does lots of parents but I never fretted about it, I think I was just amused.
I'd be reading to her and I'd say, 'Is Tytner around?' and she'd say, 'Yes, he's just sitting at the end
of the bed.' He became the centre of her life. She'd have tea parties with him, and he'd go to bed
NH
with her. She was shy and this was her answer. I knew she would grow out of it.
Presenter: Now Liz is one of 15 people taking part in a study of imaginary friends at the Institute
of Education in London, run by Karen Majors, an education psychologist and lecturer at the
institute. Karen, should parents worry about it?
Y

Expert: Well, parents sometimes think, 'Is this healthy and how long should it go on for?' But it
is a normal phenomenon for normal children. And it's very healthy.
QU

Presenter: Why do children invent imaginary friends?


Expert: I think that children create pretend friends for many reasons: as safe, trustworthy best
friends at a time when they are just starting to make real friends; as someone to confide in; and as
someone to play with. Sometimes it is about wish fulfilment; children who cannot have a pet,
M

for example, will invent one. I interviewed one little girl, aged six, who had a pony called Minty
for several years. It went to school with her and the teachers knew all about it. It was a really strong

relationship.
Presenter: Presumably, when they get older, children no longer have these imaginary friends.
Karen?
Expert: Well, my most surprising finding is that children don't always stop having these made-up
Y

playmates when they start school. The imaginary friends often stay with them through their teenage
years, providing comfort and escape - although in secret. One teenager I talked to had invented a
DẠ

superhero to help him through tricky patches. When things hadn't gone well at school, he would
come home and play with the superhero, for whom everything always went well.
Presenter: How should parents treat these invisible people, Karen?

Page 7
Expert: Well, sometimes of course, parents get irritated by them - for instance, if a child insists
on having the playmate at the dinner table with an imaginary setting and glass. Actually I myself
had a friend called Tiger when I was young, who would sit beside me at mealtimes. But I don't

L
think parents should tell children off for this kind of thing, or tell them that their friends are

A
not real. Perhaps the best way is Liz's down-to-earth approach.
Presenter: How did you handle it, Liz?

CI
Mother: Well, I patiently acknowledged Caitlin's playmate but I tried not to get involved. I never
used to have to get out of the friend's way or anything. Other than that one incident in the car,

FI
Caitlin's imaginary friend didn't impinge on my life.
Expert: Yes, I agree that parents should recognize imaginary friends, but they shouldn't try to
overly influence the friendship. Parents who interfere too much risk driving their children's

OF
playmates away. If they try to direct the friends, they could spoil the fantasy altogether.
Presenter: Fascinating subject, thanks for coming in to talk about it, Liz and Karen.

Part 4: https://www.youtube.com/watch?v=rS6Ifm_YecQ&t=274s

ƠN
The global cosmetics industry makes a ton of money.
That includes shampoo, makeup, perfume, cologne, deodorant, soap.
The list goes on.
Analysts expect the industry's revenue to grow within the next four years to more than $379
NH
billion.
But when you break down that number, it's skin care that's driving much of the growth. And it's
not slowing down.
I've used moisturizers upwards in like 250 dollar range.
Y

The most expensive single item would be around 50 dollars.


I try not to spend more than 50 for in essence or moisturizer.
QU

Probably like this really expensive moisturizer that I kind of got conned into buying for about
150 dollars.
Skin care is an increasingly lucrative business for cosmetic companies.
That's why big names like Amazon and the Kardashians are trying to cash
M

in. And there's one country that's been playing an outsized role on skin care. South Korea.
Here's why.

Why is skincare so popular


Skin care is a big moneymaker for big beauty brands.
Over the last five years, skin care has grown so much that it's become the
largest piece of the pie when you break apart the beauty industry by
Y

product category. It makes up about 24.9 percent of a total $52.4 billion in annual revenue.
And the global skin care products market size is expected to reach more
DẠ

than $196 billion by 2024.


But what is making skin care so popular?
And, why are these bottles of face wash and tubs of lotion bringing in so much money?

Page 8
This is Larissa Jensen.
She's a skin care and beauty industry analyst with the NPD Group.
Skin care has been growing very fast.

L
It's actually growing the fastest of all the beauty categories that we track, including makeup and

A
fragrance.
So, it's been the strongest performer for the past couple of years.

CI
What's really driving a lot of that performance is actually wellness and
health, and you know, really natural brands.

FI
That's what's really driving the performance of skincare.
Wellness doesn't mean just being healthy.
In 2019, it means clean eating, the latest fitness fads and the no-makeup look, which actually

OF
does involve some makeup.
At any rate, the wellness trend is driving consumers to take better care of their skin.
This skin-first philosophy did not originate in the U.S.
Instead, it was born on the other side of the world, where skin care is a part of the culture.

ƠN
It's where the latest skin care innovations are concocted and where a large chunk of
manufacturing happens in the first place.
- Hi guys.
It's Charlotte.
NH
This is Charlotte Cho.
She's an entrepreneur, Korean beauty guru and the author of "The Little Book of Skin Care."
She started Soko Glam, an online marketplace full of curated K-beauty products for U.S.
consumers.
Y

I was born and raised in California. I didn't know anything about skin care until I started to live
and work in Seoul, South Korea, and I was completely amazed at the product selection
QU

there was in Korea. And also the real focus on a skin-first philosophy.
So, every woman and man in Korea it seemed really knew how to take care of their skin and at
an early age.
They had categories that just simply did not exist in the U.S., so it was really intriguing to me
M

and I saw results on my own skin. I firmly believe that K-beauty created a skin care wave.
They opened the door to innovations.

They've allowed indie brands to come to the forefront of a lot of the skin care trends.
They've also widened the appetite for new products, new categories.
They've also been a big part of the education around skin care.
Koreas Cosmetics Exports
Y

South Korea's cosmetics exports have exploded in recent years.


In 2014, exports were over 1.7 billion, and then in 2017, they had grown nearly 5 billion.
DẠ

Not only is South Korea a large exporter of beauty products, but is also a huge manufacturing
hub, even for American brands.

Page 9
Local cosmetic production there went from 8.5 billion dollars worth of goods in 2014 and grew
to more than 13 billion in 2017.
I think the impact of K-beauty goes far beyond just brands that are now introduced into this

L
global marketplace.

A
It's actually allowing Korean manufacturing companies to grow rapidly as
well. For example, a lot of European brands and U.S.-based brands are formulating their

CI
products with Korean R&D chemists and manufacturing plants.

FI
OF
ƠN
NH
Y
QU
M

Y
DẠ

Page 10
HỘI CÁC TRƯỜNG CHUYÊN VÙNG ĐỀ THI CHỌN HỌC SINH GIỎI LẦN THỨ
DUYÊN HẢI VÀ ĐỒNG BẰNG BẮC BỘ XVI
TRƯỜNG THPT CHUYÊN NGUYỄN MÔN: TIẾNG ANH - KHỐI 11
TRÃI Thời gian: 180 phút

L
TỈNH HẢI DƯƠNG Đề thi gồm: 18 trang

A
ĐỀ ĐỀ XUẤT

CI
SECTION 1: LISTENING (50 points)
I. You will hear an introduction to a radio phone-in programme about modern lifestyles.

FI
Listen and indicate true (T) or false (F) statements. (10 points)
Statements T F
1. Ron is a record-breaking athlete.

OF
2. Ron thinks an accountant can lead a health and fulfilled life.
3. “Total Living” is believed to be good for athletes.
4. “Total Living” means that we should develop one aspect of our life to the

ƠN
full.
5. According to Ron Clarke, some current health trends are harming us.
II. You will hear a short radio report about how technology is helping archaeologists who
want to learn more about some texts written over 2,000 years ago known as Roman tablets.
NH
Listen and give short answers to the questions. Write NO MORE THAN FIVE WORDS
AND/OR A NUMBER taken from the recording. (10 points)
1. Where did archaeologists discover about 200 tablets?
………………………………………………………………………
Y

2. When did Roman soldiers often use tablets?


QU

………………………………………………………………………
3. In what field is Professor Mike Brady a leading figure?
………………………………………………………………………
4. What were panels on the tablets once filled with?
………………………………………………………………………
M

5. Which type of texts has the new technology already been applied to?
………………………………………………………………………

III. You will hear part of an interview with someone who has been having ballet lessons.
Listen and choose the correct answer. (10 points)
1. What does Rupert say about the fact that he is doing ballet classes?
Y

A. Other people have ridiculed him for it.


B. He expects to be mocked for it.
DẠ

C. It is not as unusual as people might think.


D. People might think it isn’t really true.
2. Rupert says that before he started doing ballet lessons

1
A. he had been doing routine physical fitness training.
B. his knowledge of ballet had been growing.
C. ballet had taken over from football as his greatest interest.
D. he had been considering doing ballroom dancing again.

L
3. Rupert say that when the idea of ballet lessons was suggested to him,

A
A. he thought it was a joke.
B. he was unsure exactly what would be involved.

CI
C. he began to have unrealistic expectations of what he could achieve.
D. he initially lacked the confidence to do it.

FI
4. One of the advantages of ballet that Rupert mentions is that
A. it leads to fewer injuries than other physical activities.
B. it has both physical and mental effects.

OF
C. it is particularly good for certain parts of the body.
D. it is more interesting than other forms of exercise.
5. What does Rupert say about his progress at ballet?

ƠN
A. It has been much more rapid than he had expected.
B. It has made him consider giving up his other training.
C. It has given him greater appreciation of the skills of professionals.
D. It has led him to enroll for certain exams.
NH
IV. Listen to a piece of BBC news about the World War One centenary and fill in the missing
information. Write NO MORE THAN THREE WORDS taken from the recording for each
answer in the spaces provided. (20 points)
People have criticised Germany's government for not doing enough to (1) ___________ the
Y

World War One centenary.


Germany hasn’t spent as much on events as some other (2) ________________ countries.
QU

And the events which have taken place have been seen as (3) ________________by critics.
Traditionally in Germany the First World War is (4)________________the Second World
War. History teaching in German schools tends to focus on (5)______________ of the Nazis rather
than what happened (6) ____________ earlier.
M

And since 1945 there's been a strong (7)_____________ in Germany to anything that might
be seen as (8) _______________ . So many people here are uncomfortable with any anniversary of

a war or a battle.
There's still (9)_________________ among historians about who was responsible for World
War One. But having spent the last 70 years atoning for Nazi guilt, many Germans have
(10)______________ to now take on the blame for the First World War, too. SECTION 2:
Y

LEXICO - GRAMMAR (30 points)


DẠ

I. Choose the word or phrase that best fits the gap in each sentence (20 points)
1. Although the company seems very successful and popular, it has _______ actual money.
Everything is built off loans and debts.

2
A. less or no B. little or no C. many or not D. not any or little
2. There is no point in phoning him. He’s certain _______by now.
A. to leave B. to have left C. left D. having left
3. So little _____ that the neighbors could not settle their differences.

L
A. did they agree B. they agreed C. agreed did they D. they did agree

A
4. Something’s ____________ up, so I’m afraid I won’t be able to make it this afternoon.
A. shown B. pulled C. cropped D. cut

CI
5. Could you lend me some money to _________ me over to the end of the month?
A. hand B. tide C. get D. make

FI
6. Have you considered applying for a job with the ________ service?
A. civil B. civic C. national D. governmental
7. I wonder if you could let me know what the _________ fare to Berlin is?

OF
A. round B. double C. two-way D. return
8. Maria was just walking along the road when someone on a motorbike ________ her handbag.
A. clutched B. grabbed C. clenched D. fumbled

ƠN
9. Fred Astaire is one of my favourite dancers and I love the way he seems to ________ across
the floor.
A. glide B. march C. skid D. dash
10. The new Arts Centre seems to have been ________ construction for quite some time.
NH
A. in B. under C. on D. below
11. The new regulations have _________ up a number of problems for the company.
A. come B. thrown C. got D. moved
12. We’re saving for our holiday, so we’re __________ back on luxuries this month.
Y

A. dropping B. moving C. giving D. cutting


13. Cars have been banned from the city centre, which makes the area much safer for _________.
QU

A. passersby B. onlookers C. pedestrians D. footmen


14. There has been a real boom in _________ electronics over the last three decades.
A. consumer B. customer C. buyer D. client
15. When I realized that I’d left my homework at home, I quickly ________ back to get it.
M

A. crept B. dashed C. crawled D. drifted


16. I know it’s got his name on the cover, but he used a ________.

A. correspondent B. model C. ghostwriter D. fellow


17. The text doesn’t give you the answer explicitly – you have to _________ it from the evidence.
A. convert B. grasp C. reckon D. deduce
18. I couldn’t decide what to write about, when I suddenly _________ upon the idea of doing
Y

something on writer’s block.


DẠ

A. thought B. chanced C. hit D. arrived


19. This fabric has the __________ of silk but it’s very cheap.
A. stroke B substance C. friction D. texture

3
20. You’ll have to __________ your jacket quite hard to get the grass stain off it.
A. polish B. scrub C. pat D. crush
II. Fill each gap with the correct form of the word in brackets. (10 points)
1. She stood there completely _______, so I had no idea at all what she was thinking. (EXPRESS)

L
2. Any actor who becomes known for one role is in danger of becoming _______. (TYPE)

A
3. This school was once _______ as a military hospital during the war. (REQUIRE)
4. So far, the United States has said it cannot agree to _______ all types of antipersonnel mines.

CI
(LAW)
5. In Scotland, there is greater emphasis on _______ by individual schools. (VALUE)

FI
6. The role of clouds is one of the big conundrums of _______. (CLIMATE)
7. Whatever happens, don't let this failure _______ you. (HEART)
8. It is _______ summer, but it's rather autumnal today. (THEORY)

OF
9. She thinks that with a little ________ she can persuade her husband to change his mind.
(FLATTER)
10. The branch of a tree knocked one of the riders _________. (SENSE)

ƠN
SECTION 3: READING (60 points)
I. Fill each gap in the passage below with ONE appropriate word in the space provided. (15
points)
Stuart Hayes had launched himself on a promising career as a swimmer when something
NH
odd happened (1) ______ him at the local pool. Flogging up and down for the umpteenth time, he
suddenly realized how bored he had become with the monotony. Wasn’t there a more interesting
way of (2) ______ sporty, for heaven’s sake? There was and there is: the colour, sweat and sheer
emotion of triathlons. On August 5 and 6, Stuart, 27, will join almost 10,000 athletes in the
Y

London Triathlon, the biggest event of (3) ______ kind in the world.
Triathlons are (4) ______ but boring. Combining swimming, cycling and running in one
QU

physical onslaught, they offer huge variety within a single racing framework. The classic
Olympic distances are a 1,500 m swim, followed by a 40 km bike ride and 10 km run. Hayes, a
world-class triathlete, won last year’s London Triathlon in an impressive 1 hour 47 minutes. “The
worst part is the last 5 km of the run – you’re starting to get really fatigued by (5) ______ ,”he
M

says. So why does he do it? He shrugs. “It’s interesting.”


In Britain, the sport is growing (6) ______ 10 per cent a year. “People are moving away

(7) ______ just running, and are looking for new challenges,” says Nick Rusling, event director
for the London Triathlon. “Triathlons are a great deal more interesting to train for and you can
vary training to fit busy lifestyles, swimming in your lunch break and (8) ______ on. On top of
(9) ______ , people are much more aware of their bodies. They know running is potentially bad
Y

(10) ______ the joints. A triathlon is actually very easy and very smooth, particularly the cycle
DẠ

and swim.”
II. Read the following passage and choose the best answer. (10 points)

4
Over the last century the world has become increasingly smaller. Not geographically, of
course, but in the sense that media, technology and the opening of borders has enabled the
world’s citizens to view, share and gain access to a much wider range of cultures, societies and
world views. In this melting pot that the world has become, today’s child is privy to facets of the

L
human experience that his immediate predecessors had no inkling even existed. It stands to reason

A
that in order to absorb, configure and finally form opinions about this information-laden planet,
children must be supplied with certain tools. Included in this list of ‘tools’ are: education, social

CI
skills, cultural awareness and the acquisition of languages, the most important of these being the
latter. Until recently, a child who had the ability to speak more than one language would have

FI
been considered a very rare entity. This one-language phenomenon could be attributed to a
combination of factors. Firstly, the monolingual environment in which a child was raised played a
strong role, as did the limited, biased education of the past. With regard to immigrants, the sad

OF
fact was that non-native parents tended to withhold the teaching of the mother tongue so that the
child would acquire the ‘more prestigious’ language of the adopted country.
Nowadays, the situation has undergone an almost complete reversal. In the majority of

ƠN
North American and European countries, most children are given the opportunity to learn a
second or even a third language. Children acquire these foreign languages through various and
diverse means. In many countries, learning a foreign language is a compulsory subject in the state
school curriculum. Other children rely on language schools or private tuition to achieve their goal.
NH
In other instances, children are born to bilingual parents, who, if they so desire, may teach the
children two languages.
Bringing up one’s child bilingually is not a decision to be taken lightly. Both parents must
consider long and hard the implications involved in raising a child in a two-language home. This
Y

decision is one of those all-important choices which will affect not only the parents’ lives but also
the life of the child. Raising a child bilingually has a two-fold effect. Firstly, of course, the child
QU

learns the two languages of the parents. Secondly, the parents’ decision will influence factors
which will have a far-reaching effect on the child’s life. Some of these factors include: style and
place of education; diameter of social circle; employment potential and preference; and, most
importantly, the way in which the child views himself and his global environment.
M

One of the more advantageous by-products of being a member of a bilingual family is the
inherent awareness of two different cultures. This bicultural child inherits a wealth of knowledge

brought about by an exposure to: historical backgrounds; traditional songs and folklore; rituals of
marriage; models of social interaction; and therefore, two varying interpretations of the world.
The monolingual child seems to be at a disadvantage in comparison to the bilingual child, who
has a set of languages and an accompanying set of abstract cultural ideas. Practically speaking,
Y

when a child comes from a two-language family, he must be taught both languages in order to
DẠ

communicate with the extended family members. When, for example, the grandparents speak a
language which differs from that of the child’s locale, a monolingual child would be deprived of
the interaction which occurs between grandparents and grandchildren. On the other hand, a

5
bilingual child will not only be able to speak to grandparents, but will also comprehend where
these people have ‘come from’. There will be a shared cultural empathy within the family.
Because all family members can communicate, on both a verbal and cultural level, no one will
feel excluded and the child will develop a sense of rootedness.

L
On a more abstract level, it has been said that a bilingual child thinks differently from a

A
monolingual child. Current research in linguistics indicates that there may be a strong correlation
between bilingualism and cognitive skills. This new research concerns itself with the fact that a

CI
bilingual child has two lexical structures for any given physical or abstract entity. This leads
logically to the assumption that the child also has two associations for many words, as a word can

FI
mean different things in different languages. For example, the word ‘fire’ in many western
hemisphere languages connotes warmth and relaxation. In the Inuit language however, where fire
is a necessity of life, it may connote heat and survival. For the bilingual child, then, vocabulary

OF
items and the abstract idea behind them are both dual in nature and more elastic. Researchers
maintain that this elasticity of ideas may allow the child to think more flexibly and, therefore,
more creatively.

ƠN
1. In the author’s view, the world is becoming a _____.
A. more culturally diverse place.
B. place where only privileged children will prosper.
C. less complex place to live in.
NH
D. much more integrated place.
2. According to the first paragraph, which of the following was true of immigrants?
A. Children were reluctant to use their mother tongue.
B. The mother tongue was considered less important.
Y

C. Parents encouraged children to use their mother tongue.


D. Most parents made it a priority for children to grow up bilingual.
QU

3. The phrase “privy to” in paragraph 1 mostly means _____.


A. acquainted with B. advised of C. apprised of D. in the know about
4. The phrase “attributed to” in paragraph 1 mostly means _____.
A. ascribed to B. associated with C. connected with D. held responsible for
M

5. According to the writer, second or foreign language learning is something _____.


A. people are still apathetic towards.

B. mainly associated with private sector education.


C. that few people take seriously.
D. about which general attitudes have evolved considerably.
6. According to the article, the decision to raise bilingual children is difficult because ______.
Y

A. it may limit the child’s choice of friends.


DẠ

B. though simple for parents, it can impact negatively on children.


C. it may cause children to lose their sense of identity.
D. it needs to be considered from many different angles.

6
7. With regard to the extended family in immigrant situations, the writer feels it is important that
_____.
A. adults try to understand the child’s difficult cultural situation.
B. children are not pressured to speak their parents’ native language.

L
C. adults recognize the child’s need to identify more with local culture.

A
D. children can relate to all aspects of their parents’ native culture.
8. The word “by-products” in paragraph 4 mostly means _____.

CI
A. entailments B. knock-on effects C. side effects D. spin-offs
9. The word “connotes” in paragraph 5 mostly means _____.

FI
A. underpins B. implies C. signifies D. smacks of
10. According to current research, the benefit of learning two languages is that _____.
A. different types of knowledge can be accessed in different languages.

OF
B. bilinguals become more aware the origin of words in languages.
C. it helps to develop different capabilities of the mind.
D. bilinguals develop a greater sense of the value of culture.

ƠN
III. Read the passage and do the tasks that follow. (13 points)
Questions 1 - 6
The reading passage has six paragraphs A-F. Choose the correct heading for each paragraph
from the list below.
NH
List of Headings

i. The productive outcomes that may result from boredom


Y

ii. What teachers can do to prevent boredom


QU

iii. A new explanation and a new cure for boredom


iv. Problems with a scientific approach to boredom
v. A potential danger arising from boredom
vi. Creating a system of classification for feelings of boredom
vii. Age groups most affect by boredom
M

viii. Identifying those most affected by boredom


1. Paragraph A ______
2. Paragraph B ______
Y

3. Paragraph C ______
4. Paragraph D ______
DẠ

5. Paragraph E ______
6. Paragraph F ______

7
Why being bored is stimulating – and useful, too
A We all know how it feels – it’s impossible to keep your mind on anything, time stretches
out, and all the things you could do seem equally unlikely to make you feel better. But
defining boredom so that it can be studied in the lab has proved difficult. For a start, it can

L
include a lot of other mental states, such as frustration, apathy, depression and indifference.

A
There isn’t even agreement over whether boredom is always a low-energy, flat kind of
emotion or whether feeling agitated and restless counts as boredom, too. In his book,

CI
Boredom: A Lively History, Peter Toohey at the University of Calgary, Canada, compares
it to disgust – an emotion that motivates us to stay away from certain situations. ‘If disgust

FI
protects humans from infection, boredom may protect them from “infectious” social
situations,’ he suggests.
B By asking people about their experiences of boredom, Thomas Goetz and his team at the

OF
University of Konstanz in Germany have recently identified five distinct types: indifferent,
calibrating, searching, reactant and apathetic. These can be plotted on two axes – one
running left to right, which measures low positive or negative the feeling is. Intriguingly,

ƠN
Goetz has found that while people experience all kinds of boredom, they tend to specialize
in one. Of the five types, the most damaging is “reactant” boredom with its explosive
combination of high arousal and negative emotion. The most useful is that Goetz calls
“indifferent” boredom: someone isn’t engaged in anything satisfying but still feels relaxed
NH
and calm. However, it remains to be seen whether there are any character traits that predict
the kind of boredom each of us might be prone to.
C Psychologist Sandi Mann at the University of Central Lancashire, UK, goes further. “All
emotions are there for a reason, including boredom,” she says. Mann has found that being
Y

bored makes us more creative. “We’re all afraid of being bored but in actual fact it can lead
to all kinds of amazing things,” she says. In experiments published last year. Mann found
QU

that people who had been made to feel bored by copying numbers out of the phone book
for 15 minutes came up with more creative ideas about how to use a polystyrene cup than a
control group. Mann concluded that a passive, boring activity is best for creativity because
it allows the mind to wander. In fact, she goes so far as to suggest that we should seek out
M

more boredom in our lives.


D Psychologist John Eastwood at York University in Toronto, Canada, isn’t convinced. “If

you are in a state of mind-wandering you are not bored,” he says. “In my view, by
definition boredom is a undesirable sate.” That doesn’t necessarily mean that it isn’t
adaptive, he adds. “Pain is adaptive – if we didn’t have physical pain, bad things would
happen to us. Does that mean that we should actively cause pain? No. But even if boredom
Y

has evolved to help us survive, it can still be toxic if allowed to fester.” For Eastwood, the
DẠ

central feature of boredom is a failure to put our “attention system” into gear. This causes
an inability to focus on anything, which makes time seem to go painfully slowly. What’s
more, your efforts to improve the situation can end up making you feel worse. “People try

8
to connect with the world and if they are not successful there’s that frustration and
irritability,” he says. Perhaps most worryingly, says Eastwood, repeatedly failing to engage
attention can lead to a state where we don’t know what to do any more, and no longer care.
E Eastwood’s team is now trying to explore why the attention system fails. It’s early days but

L
they think that at least some of it comes down to personality. Boredom proneness has been

A
linked with a variety of traits. People who are motivated by pleasure seem to suffer
particularly badly. Other personality traits, such as curiosity, are associated with a high

CI
boredom threshold. More evidence that boredom has detrimental effects comes from
studies of people who are more or less prone to boredom. It seems those who bore easily

FI
face poorer prospects in education, their career and even life in general. But of course,
boredom itself cannot kill – it’s the things we do to deal with it that may put us in danger.
What can we do to alleviate it before it comes to that? Goetz’s group has one suggestion.

OF
Working with teenagers, they found that those who “approach” a boring situation – in other
words, see that it’s boring and get stuck in anyway – report less boredom than those who
try to avoid it by using snacks, TV or social media for distraction.

ƠN
F Psychologist Francoise Wemelsfelder speculates that our over-connected lifestyles might
even be a new source of boredom. “In modern human society there is a lot of
overstimulation but still a lot of problems finding meaning,” she says. So instead of
seeking yet more mental stimulation, perhaps we should leave our phones alone, and use
NH
boredom to motivate us to engage with the world in a more meaningful way.

Questions 7-10
Look at the following people and the list of ideas below. Match each person with the correct idea,
Y

A-E.
7. Peter Toohey
QU

8. Thomas Goetz
9. John Eastwood
10. Francoise Wemelsfelder
List of ideas
M

A. The way we live today may encourage boredom.


B. One sort of boredom is worse than all the others.

C. Levels of boredom may fall in the future.


D. Trying to cope with boredom can increase its negative effects.
E. Boredom may encourage us to avoid an unpleasant experience.
Y

Questions 11-13
DẠ

Complete the summary below. Choose ONE WORD ONLY from the passage for each answer.

9
Responses to boredom
For John Eastwood, the central feature of boredom is that people cannot (11) _________,
due to a failure in what he calls the “attention system”, and as a result they become frustrated and
irritable. His team suggests that those for whom (12) _________ is an important aim in life may

L
have problems in coping with boredom, whereas those who have the characteristic of (13)

A
_________ can generally cope with it.

CI
IV. You are going to read an extract from an article. Seven paragraphs have been removed.
Select from the paragraphs (A-H) the one that fits each gap (1-7). There is one extra paragraph

FI
that you do not need to use. (7 points)
A Race to Remember

OF
Imagine, if you will, driving across sand dunes at a speed of 100 kilometers per hour, with
sand flying up at the windscreen while you try to maintain control of the wheel and your navigator
attempts to maintain their compass steady at 25 degrees north-northwest, so you don't divert from
your course. Now, imagine that scenario taking place over eight hours in a single day and over a

ƠN
period of around 12 days. That is the reality of the world-renowned Dakar Rally.
1
This year the race took place in Saudi Arabia with a total of 310 vehicles, made up of
motorbikes and cars, amongst many other categories, and only 206 of those vehicles crossed the
NH
finish line. One individual that made it through this brutal event was Frederique Le Salles for whom
it was his second rally. “Although my first Dakar Rally was five years ago, I’ve dreamed about
coming back ever since. It took me all that time to raise the funds to enter again, and I’m overjoyed
to finally return.”
Y

2
QU

As these examples show, the Rally truly does get under people’s skin, with the same
competitors returning multiple times. This gives them the benefit of experience, which is an
essential attribute in this competition. Without the modem necessities of GPS, knowing what the
race takes in terms of the trials and tribulations, gives old-timers an important mental advantage
in the race, and many taking part more than once because of this.
M

3
The event originated from the experience of a man called Thierry Sabine who became so

adrift from the course in the 1977 Abidjan-Nice Rally that he realized his detour would make for
a great race in itself. The first Dakar Rally started in Dakar in Africa and finished in Paris,
skirting around the edges of the Sahara Desert, a grueling 10,000 - kilometre trip.
Y

4
It is here that I spoke to competitors about their expectations of and concerns for the race, as
DẠ

well as finding out a bit about their preparation for an unyielding few weeks of fierce motor racing.
The responses I received didn’t vary wildly - participants’ preparation was always meticulous.
5
10
One rally driver, who is on his fifth race, told me that he believes a lot of the rally is down to
luck. “It takes just a split second to make a mistake, and so, it doesn’t matter if you’ve driven
brilliantly for thousands of kilometers, because all you need is one meter to go wrong, and you crash
out. It’s happened to me twice and it’s a completely soul- crushing experience.”

L
6

A
Whether this is truly enough to be ready, only time will tell. While the financial rewards
for winning aren't publicized (although rumor has it that they are relatively low), the accolade of

CI
winning is certainly something money can't buy. However, this doesn't mean that the rally is
unprofitable for all, and there are some enterprising routes to make the rally a little more

FI
lucrative.
7

OF
A. Such is the commitment to compete that one racer, Chris Cork, sold his house in the UK in
order to secure a place on the 2015 Dakar Rally. Unfortunately, he crashed out on the fourth day
and had to undergo a difficult recovery procedure for his injuries. Undeterred, he came back the

ƠN
next year to compete, with financial help from well- wishers who were touched by his story.
B. Although some entrants go it alone, with the bare-minimum requirements needed to
participate, there are many other participants who sign up with companies and receive sponsor
bonuses depending on their performance in each round. While finishing a round may be in the
NH
hands of fate, it is far more beneficial to do it with sponsorship, as many successful rally winners
know.
C. As a motorsport reporter, I’ve been fortunate enough to follow this competition for the last
five years, and it never ceases to amaze me how tough and committed each racer is. Racers are
Y

required to fix their vehicles themselves while on a stage, and there is not even a great
QU

probability that they will be able to finish, with most competitors crashing out or breaking down
on the route.
D. Perhaps the main appeal for these riders is that it offers a sense of adventure that is hard
to parallel. The Rally covers lots of harsh terrain and the course is simply a series of written
directions. In addition to that, if they don’t have any overnight repairs to keep them busy, many
M

drivers find themselves sleeping in tents alongside their vehicles, giving them just enough time to
recuperate and get back on the ride. Indeed, unlike many racing events, there is little luxury to

have, and this has always been the case with the Dakar Rally.
E. There was lot of focus, as with every rally before, on the terrain, and the best ways to cope
with it. 75% of the rally would take place on sand, and a great deal on sand dunes. This means
Y

hill climbs and sheer drops, and little ability to spot landmarks to help you know where you are.
And remember, if you get your directions wrong, it can take you hours to catch up.
DẠ

F. Fortunately, for many people, the Dakar Rally is a labour of love, but budget can make a
significant difference to your opportunities of succeeding. While amateurs may be sleeping aside

11
their vehicles relying on themselves for any repairs, professionals will have a team that can take
care of their every need at the end of each stage.
G. Although for many years Africa was its home, the location of the Rally has not remained
static, it transferred to a different continent altogether, South America, from 2009 to 2019, and

L
more lately to Saudi Arabia, where the vehicles have had to compete with the Middle Eastern

A
deserts, and their scorching days and nearly freezing nights.
H. Others are more concerned with how they’ll keep their momentum up. “This will be our

CI
third race, and we think we’ve finally discovered what we need to really compete with the best.
We’ve come to realise that the race is about resilience and technical ability, and we've been

FI
training with sleep deprivation and motor repair exercises to make sure we’ve got both these
qualities.”

OF
V. You are going to read an article by a psychologist about laughter. For questions 1 - 10,
choose from the sections (A – D). The sections may be chosen more than once. (15 points)
Which section …
1. comments on which person laughs within a verbal exchange?

ƠN
2. uses a comparison with other physical functions to support an idea?
3. gives reasons why understanding laughter supplies very useful insights?
4. refers to someone who understood the self-perpetuating nature of laughter?
5. cites a study that involved watching people without their knowledge?
NH
6. describes laughter having a detrimental effect?
7. criticizes other research for failing to consider a key function of laughter?
8. explains that laughing does not usually take precedence over speaking?
9. describes people observing themselves?
Y

10. encourages checking that a proposition is correct?


QU

Why do people laugh?


Psychologist Robert Provine writes about why and when we laugh.

A. In 1962, what began as an isolated fit of laughter in a group of schoolgirls in Tanzania rapidly
M

rose to epidemic proportions. Contagious laughter spread from one individual to the next and

between communities. Fluctuating in intensity, the laughter epidemic lasted for around two and a
half years and during this time at least 14 schools were closed and about 1,000 people afflicted.
Laughter epidemics, big and small, are universal. Laughter yoga, an innovation of Madan Kataria
of Mumbai, taps into contagious laughter for his Laughter Yoga clubs. Members gather in public
Y

places to engage in laughter exercises to energise the body and improve health. Kataria realised
that only laughter is needed to stimulate laughter – no jokes are necessary. When we hear
DẠ

laughter, we become beasts of the herd, mindlessly laughing in turn, producing a behavioural
chain reaction that sweeps through our group.

12
B. Laughter is a rich source of information about complex social relationships, if you know where
to look. Learning to ‘read’ laughter is particularly valuable because laughter is involuntary and
hard to fake, providing uncensored, honest accounts of what people really think about each other.
It is a decidedly social signal. The social context of laughter was established by 72 student

L
volunteers in my classes, who recorded their own laughter, its time of occurrence and social

A
circumstance in small notebooks (laugh logbooks) during a one-week period. The sociality of
laughter was striking. My logbook keepers laughed about 30 times more when they were around

CI
others than when they were alone – laughter almost disappeared among solitary subjects.

FI
C. Further clues about the social context of laughter came from the surreptitious observation of
1,200 instances of conversational laughter among anonymous people in public places. My
colleagues and I noted the gender of the speaker and audience (listener), whether the speaker or

OF
the audience laughed, and what was said immediately before laughter occurred. Contrary to
expectation, most conversational laughter was not a response to jokes or humorous stories. Fewer
than 20% of pre-laugh comments were remotely jokelike or humorous. Most laughter followed

ƠN
banal remarks such as ‘Are you sure?’ and ‘It was nice meeting you too.’ Mutual playfulness, in-
group feeling and positive emotional tone – not comedy – mark the social settings of most
naturally occurring laughter. Another counterintuitive discovery was that the average speaker
laughs about 46% more often than the audience. This contrasts with the scenario in stand-up
NH
comedy – a type of comedy performance in which a non-laughing speaker presents jokes to a
laughing audience. Comedy performance in general proves an inadequate model for everyday
conversational laughter. Analyses that focus only on audience behaviour (a common approach)
are obviously limited because they neglect the social nature of the laughing relationship.
Y

D. Amazingly, we somehow navigate society, laughing at just the right times, while not
QU

consciously knowing what we are doing. In our sample of 1,200 laughter episodes, the speaker
and the audience seldom interrupted the phrase structure of speech with a ha-ha. Thus, a speaker
may say ‘You are wearing that? Ha-ha,’ but rarely ‘You are wearing … ha-ha… that?’ The
occurrence of laughter during pauses, at the end of phrases, and before and after statements and
M

questions suggests that a neurologically based process governs the placement of laughter. Speech
is dominant over laughter because it has priority access to the single vocalisation channel, and

laughter does not violate the integrity of phrase structure. Laughter in speech is similar to
punctuation in written communication. If punctuation of speech by laughter seems unlikely,
consider that breathing and coughing also punctuate speech. Better yet, why not test my theory of
punctuation by examining the placement of laughter in conversation around you, focusing on the
Y

placement of ha-ha laughs. It's a good thing that these competing actions are neurologically
DẠ

orchestrated. How complicated would our lives be if we had to plan when to breathe, talk and
laugh.

13
SECTION 4: WRITING (60 points)
I. Read the following extract and use your own words to summarize it. Your summary should
be about 140 words long. (15 points)

L
NATIONALISM

A
Until about 1800, people in most of the world were loyal to the places where they and their
families lived. Most did not see themselves as part of a larger state or nation. But the rise of

CI
industry and the need to raise armies prompted rulers to try to inspire a sense of national identity
and common cause. This drive toward lifting the interests of the nation above those of individuals

FI
or groups became known as nationalism. Many historians date modern nationalism from the
French Revolution in the late 1700s. The French monarchy was replaced by a republic, in which
the citizens no longer were expected to see themselves as subjects of the king. Rather, they found

OF
their identity in the abstract concept of France as their mother country. The process was repeated
throughout Europe during the next century.
The philosophical basis of nationalism is that the nation is the most important unit of social

ƠN
and economic life to which all other human activities and desires must yield. Helping to secure
national pride are flags, foods, sports, traditions, histories, folk tales, music, literature, and
culture. There may even be a national religion. Nationalists point to criteria that distinguish
nations from each other, such as a common language, culture, and value. These traits are often
NH
represented by a single ethnic group to which almost all citizens of a nation belong. Many
nations, however, host different ethnic groups side by side, sometimes with violent or politically
disruptive results.
Some ethnic groups refuse to recognize their nation, seeking to secede in order to rule
Y

themselves. Separatist movements in Quebec, Canada, and in the Basque region of Spain have
been active for many years but have not yet succeeded.
QU

Requiring that all speak the same language has been an important means of enforcing
national identity. New nations often attempt to outlaw minority languages. The national language
tends to be the one spoken by the upper classes, resulting in the high-status language replacing the
low-status ones.
M

Y
DẠ

14
II. Writing task 2: (15 points)

A L
CI
FI
OF
ƠN
NH

IV. Writing an essay: (30 points)


Write an essay of about 350 words to express your opinion on the following topic:
Y

“Some people think that you can never become fluent in a language unless you have
QU

spent time living or working in that country. To what extent do you agree?”
M

Y
DẠ

15
HỘI CÁC TRƯỜNG CHUYÊN VÙNG ĐỀ THI CHỌN HỌC SINH GIỎI LẦN THỨ
DUYÊN HẢI VÀ ĐỒNG BẰNG BẮC BỘ XVI
TRƯỜNG THPT CHUYÊN NGUYỄN MÔN: TIẾNG ANH - KHỐI 11
TRÃI Thời gian: 180 phút

L
TỈNH HẢI DƯƠNG Đề thi gồm: 18 trang

A
ĐÁP ÁN

CI
SECTION 1: LISTENING (50 points)
I. You will hear an introduction to a radio phone-in programme about modern lifestyles.

FI
Listen and indicate true (T) or false (F) statements.
1. T 2. F 3. F 4. F 5. T

OF
II. You will hear a short radio report about how technology is helping archaeologists who
want to learn more about some texts written over 2,000 years ago known as Roman tablets.
Listen and give short answers to the questions. Write NO MORE THAN FIVE WORDS

ƠN
AND/OR A NUMBER taken from the recording. (10 points)
1. in one Roman fort
2. writing letters or/and legal documents
3. computer vision
NH
4. wax
5. texts in ink
III. You will hear part of an interview with someone who has been having ballet lessons.
Y

Listen and choose the correct answer. (10 points)


1. B 2. A 3. D 4. B 5. C
QU

IV. Listen to a piece of BBC news about the World War One centenary and fill in the missing
information. Write NO MORE THAN THREE WORDS taken from the recording for each
answer in the spaces provided. (20 points)
1. commemorate 2. European 3. half-hearted 4. overshadowed by
M

5. the crimes 6. a generation 7. aversion 8. glorifying militarism


9. some disagreement 10. little appetite


SECTION 2: LEXICO - GRAMMAR (30 points)
I. Choose the word or phrase that best fits the gap in each sentence (20 points)
1. B 2. B 3. A 4. C 5. B 6. A 7. D 8. B 9. A 10. B
Y

11. B 12. D 13. C 14. A 15. B 16. C 17. D 18. C 19. D 20. B
DẠ

II. Complete the sentences with the correct form of words (10 points).
1. expressionless 1. 2. typecast 3. requisitioned 4. outlaw 5. self-evaluation
6. climatology 7. dishearten 8. theoretically 9. flattery 10. senseless
SECTION 3: READING (60 points)
I. Fill each gap in the passage below with ONE appropriate word in the space provided. (15
points)
1. to 2. being 3. its 4. anything 5. then

L
6. by 7. from 8. so 9. that 10. for

A
II. Read the following passage and choose the best answer. (10 points)

CI
1. A 2. B 3. D 4. A 5. D 6. D 7. D 8. D 9. B 10. C
III. Read the passage and do the tasks that follow. (13 points)

FI
1. iv 2. vi 3. i 4. v 5. viii 6. iii
7. E 8. B 9. D 10. A
11. focus 12. pleasure 13. curiosity

OF
IV. You are going to read an extract from an article. Seven paragraphs have been removed.
Select from the paragraphs (A-H) the one that fits each gap (1-7). There is one extra
paragraph that you do not need to use. (7 points)

ƠN
1. C 2. A 3. D 4. G 5. E 6. H 7. B
V. You are going to read an article by a psychologist about laughter. For questions 1 - 10,
choose from the sections (A – D). The sections may be chosen more than once. (15 points)
1. C 2. D 3. B 4. A 5. C 6. A 7. C 8. D 9. B 10. D
NH
SECTION 4: WRITING (60 points)
I. Students’ answers
The summary must cover the following points:
Y

+ A sense of national pride, or nationalism, arose around the time of the French Revolution.
+ Industrialization and the need to raise armies led to people identifying themselves as being
QU

from a country as opposed to from a tribe or region.


+ Nations are often united by a common language, culture, and value.
+ When a nation includes different ethnic or religious groups, however, political conflict or
violence can result.
M

II. Students’ answers


III. Students’ answers

Audio scripts
Y

I. You will hear part of an interview with someone who has been having ballet lessons.
DẠ

Listen and choose the correct answer. (10 points)


Today I’m talking to opera critic Rupert Christiansen, who is in his forties and has recently
started doing ballet classes. It’s a pretty unusual thing to do, isn’t it?
Well, yes, but one consolation of growing old is that you cease to care what other people
think of your views or activities. So here I am, coming out on the radio as probably the only
balding middle-aged man on the entire planet to take up ballet lessons. Go on, snigger.
I’m not laughing, I’m just interested. Tell me, how did it all start?

L
Well, I have long been an infatuated ballet fan. The way most men think about their

A
football team, I think about the Royal Ballet company – they’re my team and I follow their every

CI
move with nerdy fascination. Recently, I’ve become increasingly frustrated at my lack of
technical knowledge, but the thought of having a bash myself hadn’t crossed my mind since a
schoolboy attempt at ballroom dancing culminated in disaster. Anyway, many years after that

FI
unfortunate event, to stave off bodily decay, I discovered Balance, a wonderful physiotherapy
gym in London, where I began personal training with Hans Ektvedt and his colleagues. Hans

OF
nobly takes me through the necessary boring stuff – weights, pull-ups, crunches – and his
patience and good humour make it endurable and most enjoyable.
So what led you from that to ballet lessons?

ƠN
One day earlier this year, Hans mentioned a newcomer to the Balance team and suggested
I try him for some supplementary lessons. His name is Julien Diaz, and he turns out to be a
remarkable character. A graduate of the Rambert Dance School, he ended up dancing
professionally in Berlin and Amsterdam, before returning to England to set up as a trainer,
NH
specializing in posture and the freeing up of body language. He told me I should start doing ballet
and his disarming manner soon persuaded me to drop my inhibitions. I was under no illusions
about my abilities but I wanted to find out about the art of ballet by trying it myself.
Y

Isn’t it a bit dangerous for someone of your age, if you don’t mind me saying so?
Well, as Julien says, it’s a superb form of exercise. He’d like to see the whole world doing
QU

ballet. It’s only dangerous to muscles if you do it day in day out at the very top level. For almost
anyone else, it’s an injury-preventative activity that tones and elongates every part of your body,
from toes to fingers to head. It’s a training in balance, co-ordination and flexibility, which gets
you to engage your body and brain at the same time. It’s fantastic for getting you to walk with a
M

spring in your step, and it also cleanses the mind – there’s no way you can do a ballet class

without concentrating 100 per cent on what you’re doing.


So what do your lessons consist of?
The sessions last for 90 minutes. We start with what are basically bending and stretching
exercises. Then we move on to the hopping, skipping and turning movements, finishing with
Y

some jumps in both the closed first and open second positions, before cooling down with some
DẠ

excruciating but vital stretches. This is basic stuff, but, believe me, it isn’t easy, and you have to
take it slowly. I’m not unfit, but I can’t do more than a couple of minutes without gasping for rest,
and it’s incredibly difficult to programme it all into one’s muscle memory. You can’t approximate
– you have to get it right, and that involves focusing simultaneously on the correct angle of
turnout, the complementary shaping of the arms, a firm finish to any sequence of movements, and
the fluent placing of the head, neck and shoulders.
Are you going to get really good at this? What kind of progress are you hoping to make?
How much further can I get? Well, I’m not expecting a call from by beloved Royal Ballet.

L
My main aim is to find out something of what it feels like to dance, from the inside – already I

A
find that, as a spectator, my admiration for the professionals has sharpened considerably. Julien

CI
has a potty notion of moulding some of the movements I am painfully acquiring into a dance, and
believes that most people of my age could reach a point at which they could take the lower grades
of Royal Academy of Dance exams. And Hans has noticed that my work with Julien has resulted

FI
in improvements in my regular training. He says that I’m moving better and walking taller, with
firmer core stability and more flexibility in my hips and back. I wonder whether he’s tempted to

OF
have a go himself.
II. You will hear an introduction to a radio phone-in programme about modern lifestyles.
Listen and indicate true (T) or false (F) statements.

ƠN
Good afternoon and welcome to our programme “Modern Lifestyles”. Regular listeners
will remember the Health and Diet programme we broadcast earlier in the year featuring Ron
Clarke, an Australian accountant turned record-breaking athlete.
Ron’s now Managing Director of the five successful Cannons Health Clubs in London
NH
and he’s a firm believer in being positive about life. His philosophy is that, in order to have a
healthy and fulfilled life (which he obviously felt being an accountant, even in Australia, didn’t
offer him), you have to enjoy everything you do. He advocates a healthy diet and exercise as a
Y

means of supporting one’s work, family and social life.


With this in mind he devised the term “ Total Living”. It certainly stood him in good
QU

stead during his successful career as an athlete – and it’s an obvious feature of the health clubs
Ron runs in the city – but he believes it can help everybody. His latest venture’s a book he’s just
written, also entitled “Total Living”, which isn’t just another book of physical exercises, but a
guide to how physical exercise can augment a timetable already filled with a pressurised job and a
M

hectic social life. As the term “Total Living” implies, we should see our lives as a whole, not in

isolated compartments – and this means integrating all the different aspects of our lives. Ron
thinks that too often we don’t build in time for what we need most – in this case, physical
exercise!
You may think that combining work, play, and exercise sounds daunting, but Ron also
Y

argues very much against some current health trends; for example, assuring us that the sun is
DẠ

beneficial for our health and not the danger to our health and longevity which the anti-sun lobby
would have us believe! And then, there’s dieting. How many of you can honestly say you’ve
never considered going on a diet? If you talk to Ron, he will insist that slimming diets should be
avoided at all costs.
Well, we’re fortunate to have Ron back in the studio with us today and he’s going to
answer some of your questions during the next half an hour or so but before …
III. You will hear a short radio report about how technology is helping archaeologists who
want to learn more about some texts written over 2,000 years ago known a Roman tablets.

L
Listen and give short answers to the questions. Write NO MORE THAN FIVE WORDS

A
AND/OR A NUMBER taken from the recording. (10 points)

CI
At the time of the Roman Empire in Europe, around 2,000 years ago, it was common for
information to be written, not on paper, but on things called “tablets”. These were pieces of wood
about the size and thickness of a typical modern envelope.

FI
Hundreds of such tablets have been unearthed from archaeological sites throughout Europe
and the Mediterranean world – nearly 200 were found in one Roman fort alone – and like most of

OF
these discoveries, they have been placed in public collections, mainly in museums in northern
Europe, to be viewed but not, unfortunately, to be read.
This is because, although in some cases traces of writing can still be seen, most are now

ƠN
illegible to the naked eye. But that’s all soon to change because archaeologists hope that with the
help of new technology, their secrets may soon be revealed. Many of the tablets took the form of
legal documents and letters written by Roman soldiers. An example, now at the British Museum,
bears the name of the person who wrote it and the name of the person who received it, plus the
NH
word “transportation”, which you can just make out, but the rest remains a mystery. Now, with
the help of computer techniques, experts hope eventually to be able to read the whole letter.
Professor Mike Brady, a leading figure in what’s known as “computer vision” for many years,
Y

admits that this is the hardest project he’s ever worked on. But the excitement of seeing the latest
ideas in computing applied to such a very ancient problem has the archaeological community
QU

buzzing.
So, in simple terms, why has the writing been preserved and how will it be possible to
“undo” the ageing process? Well, the tablets were made with thin, hollow panels cut across them.
Wax was poured into these and the test was then written into this soft surface using an instrument
M

with a fine metal point. In virtually all cases, the wax has perished and all that can be detected on

the surface of the tablet underneath are scratched. These are too faint to be read, because they are
distorted.
For some time, scientists have attempted to study them with laser photography, but this has
proved fruitless. However, it is now hoped that by enhancing images of the tablets on computer,
Y

their original messages will become legible again. If this is the case, a whole new source of
DẠ

historical information will be opened up, and this promises advances and new knowledge for
many decades to come. The new technology has already been used on texts in ink as well, and in
the future, it will be applied to damaged surfaces of many kinds.
IV. Listen to a piece of BBC news about the World War One centenary and fill in the missing
information. Write NO MORE THAN THREE WORDS taken from the recording for each
answer in the spaces provided. (20 points)

L
People have criticised Germany's government for not doing enough to commemorate the

A
World War One centenary.

CI
Germany hasn’t spent as much on events as some other European countries. And the events
which have taken place have been seen as half-hearted by critics.
Traditionally in Germany the First World War is overshadowed by the Second World War.

FI
History teaching in German schools tends to focus on the crimes of the Nazis rather than what
happened a generation earlier.

OF
And since 1945 there's been a strong aversion in Germany to anything that might be seen as
glorifying militarism. So many people here are uncomfortable with any anniversary of a war or a
battle.

ƠN
There's still some disagreement among historians about who was responsible for World War One.
But having spent the last 70 years atoning for Nazi guilt, many Germans have little appetite to now
take on the blame for the First World War, too.
NH
Y
QU
M

Y
DẠ
KỲ THI CHỌN HỌC SINH GIỎI CÁC TRƯỜNG THPT CHUYÊN
KHU VỰC DUYÊN HẢI VÀ ĐỒNG BẰNG BẮC BỘ

L
LẦN THỨ XIV, NĂM 2023

A
ĐỀ THI MÔN: TIẾNG ANH - LỚP 10

CI
ĐỀ ĐỀ XUẤT Thời gian: 180 phút (Không kể thời gian giao đề)
Đề thi gồm 11 trang Ngày thi: 16/7/2023

FI
SECTION A: LISTENING (50 points)
HƯỚNG DẪN PHẦN THI NGHE HIỂU
• Bài nghe gồm 4 phần; mỗi phần được nghe 2 lần, mỗi lần cách nhau 05 giây; mở đầu và kết

OF
thúc mỗi phần nghe có tín hiệu. Thí sinh có 20 giây để đọc mỗi phần câu hỏi.
• Mở đầu và kết thúc bài nghe có tín hiệu nhạc. Thí sinh có 03 phút để hoàn chỉnh bài trước tín
hiệu nhạc kết thúc bài nghe.
• Mọi hướng dẫn cho thí sinh (bằng tiếng Anh) đã có trong bài nghe

ƠN
Part 1. For questions 1-5, listen to a student asking for information about a healthy eating
society and supply the blanks with the missing information. Write NO MORE THAN TWO
WORDS OR NUMBERS taken from the recording for each answer in the space provided.
Healthy Eating Society Weekly Plan
NH
Day Activity Time Location of meal

Restaurant: 1.______________
Wednesday 7.30 High Street
food

Thursday Prepare a meal together 8.00 2.______________


Y

4.______________ in
QU

Friday Restaurant: European food 3.______________


town

Saturday 5.______________ 12.00 canteen

Part 2. You will hear a conversation between a boy, Carl, and a girl, Susanna, about a school
M

concert. For questions 6-10, decide whether the following statements are true (T) or false (F).
Write your answers in the corresponding numbered boxes provided on the answer sheet. (10
points)

No. Statements True False


6. Susanna feels shy about playing her violin in public.
7. Carl and Susanna share the same opinion about practising their
instruments regularly.
Y

8. Susanna's parents refuse to allow her to give up violin lessons.


DẠ

9. Carl's aim is to have a career in music.


10. Susanna thinks she would enjoy working in another country.

Page 1 of 11
Part 3. For questions 11-15, listen to a radio interview in which a psychologist, Colin Fraser,
talks about cultural identity and choose the correct answer A, B, C, or D which fits best
according to what you hear. Write your answers in the corresponding numbered boxes provided.

L
11. When discussing his own cultural identity, Colin reveals______
A. his resilience to changing cultures.

A
B. his unorthodox family background.
C. his ability to adapt.

CI
D. his feeling of alienation.
12. What does Colin regard as the defining aspect of a person’s cultural identity?
A. the sense of birth right
B. the emotion it generates

FI
C. the physical proximity to heritage
D. the symbols of tradition
13. What is the influence of a culture attributed to?

OF
A. the dissemination of wisdom
B. connection between societies
C. knowledge of one’s background
D. the practice of archaic rituals
14. According to Colin, what makes a culture successful on the global scene?

ƠN
A. its capacity for tolerance
B. its isolation from the mainstream
C. its aptitude for resolving conflicts
D. its ability to be self-effacing
15. During the conversation, Colin is_______
NH
A. distinguishing between birthplace and residence.
B. advocating the celebration of heritage.
C. highlighting the differences in societies.
D. addressing the issues raised by conflicting cultures.

Part 4. For questions 16-25, listen to a report on G7 Summit held in France and supply the
Y

blanks with the missing information. Write NO MORE THAN FOUR WORDS taken from the
recording for each answer in the space provided.
QU

- The leaders of the major industrialized democracies have gathered in France for the G7 summit in
an attempt to 16.____________________ amid sharp differences over a clutch of global issues that
risk further dividing a group of countries already struggling to 17.____________________.
- Speaking before the presumably 18.____________________ summit, the European Council
President acknowledged it would be hard to find common ground at a time when
M

19.____________________ has never been more important


- A grim array of disputes and problems await the leaders, with a trade war between China and the

United States deteriorating, European governments struggling to uphold the collapsing


20.___________________, and global condemnation growing over illegal fires which are
21.___________________ the Amazon.
- Trudeau sat down with British Prime Minister for discussions that focused on post-Brexit
22.___________________. Trudeau also met with Japanese Prime Minister, where he highlighted
Y

strong ties forged with G7 allies as a 23.___________________.


DẠ

- Donald Trump threatened to introduce high tariffs on French 24.___________________ in


retaliation for French president Emmanuel Macron’s tax on global technology companies.
Emmanuel Macron has said he hopes to convince world leaders to pull back from trade war and
25.___________________ at the G7 summit, despite signs that will be a daunting task

Page 2 of 11
SECTION B: LEXICO- GRAMMAR (40 points)
Part 1. For questions 26-45, choose the best option A, B, C, or D to complete the following
sentences and write your answers in the corresponding numbered boxes provided on the answer

L
sheet. (20 points)

A
26. Unfortunately, Jamie’s plans to tour around Australia didn’t _______ due to a lack of finances.
A. pan out B. pull off C. knuckle down D. waltz through

CI
27. If you are given a ________ anaesthetic during an operation, you are still aware of what is
going on around you.
A. local B. tropical C. varied D. released
28. Insurance companies had to ________ £10 million in storm damage claims.

FI
A. dip in B. rip off C. bail out D. cough up
29. The well-known ________ clash between the President and the rebel leader is not making
things easier.

OF
A. character B. mood C. enemy D. personality
30. Their ________ religious beliefs were of great comfort in troubled times.
A. age-long B. long-gone C. strongly-held D. time-honored
31. They are a real ________ organization; they are only interested in making a profit.
A. devil-may-care B. fly-by-night C. open-handed D. down-to-earth

ƠN
32. Although he came to work the day before his retirement, everyone knew he was just ________.
A. going with the flow B. going through the motions
C. going against the grain D. going along with them
33. She was ________ away from the company with promises of better terms and conditions from a
rival firm.
NH
A. enhanced B. enticed C. manipulated D. raided
34. Lawrence suggested ________ ourselves at the beginning of the long project in order to avoid
exhaustion.
A. pacing B. rushing C. plodding D. racing
35. All three siblings agreed to ________ their resources in order to start their new business
venture.
Y

A. fuse B. blend C. mingle D. pool


36. Although citizen-centered schemes involve residents in debates, full political ________ is kept
QU

by local councilors and MPs.


A. autonomy B. autarchy C. autocracy D. authority
37. Our guests are from a _______ of society and so should reflect most point of views.
A. cross-section B. cross-fire C. cross-purpose D. cross-reference
38. Mr Jones is _______, unfortunately, so you’ll have to call again tomorrow.
A. indifferent B. inimitable C. indisposed D. incongruous
M

39. The only room available was, to say the least, _______. There was no carpet, no curtains, and
the only furniture was a bed and a small beside table.

A. snug B. dreary C. stark D. cushy


40. My daughter has a very sunny ________, unlike my son who is often moody and
uncommunicative.
A. behavior B. condition C. disposition D. nurture
41. I’m in a real ________ and I just don’t know what to do.
A. dilemma B. paradox C. query D. hunch
Y

42. The Minister was accused of ________ the truth.


A. distorting B. revising C. reforming D. shifting
DẠ

43. Lionel Messi was ________ over his team’s victory against Brazil.
A. delighted B. ecstatic C. jubilant D. blissful
44. Something’s ________ up, so I’m afraid I won’t be able to make it this afternoon.
A. shown B. pulled C. cropped D. cut

Page 3 of 11
45. We don’t want him to suspect we’re giving him a surprise party. Make sure you don’t
____________.
A. break the ice B. kick the bucket C. spill the beans D. sweep the board

L
Part 2. For questions 46-55, fill each gap with the correct form of the words in brackets. Write
your answer in the boxes provided on the answer sheet. (10 points)

A
46. Like his famous _______, young Washington had a brave, adventurous spirit. (NAME)
47. She has a softly ___________voice that would melt anyone listening to her singing. (FLUENT)

CI
48. The history of the territory is ___________ illustrated in the book. (PICTURE)
49. Don’t you think she is ______? She always looks good in her photographs. (PHOTO)
50. Hardly a day goes by without Tim being __________ of eating sweets. (DESIRE)

FI
51. She resembled an army commander whom nothing could put the __________ on.
(FRIGHTEN)
52. In an overscheduled world, children and caregivers are encouraged to take part in open-ended
plays, a ____________ experience that creates intergenerational conversations. (FREE)

OF
53. Halloween has been grossly _________ over the last years. (COMMODITY)
54. Until now, there is hardly any measure to lessen the _________ regulation and taxation on
small businesses. (ONUS)
55. About $200 million in taxes weren’t paid because of ____________ income. (REPORT)

ƠN
Part 3. For questions 56-65, complete each of the following sentences with suitable
preposition(s). Write your answer in the boxes provided on the answer sheet. (10 points)
56. The station is _____________ walking distance of the hotel.
57. I’m so tired after work that I often drop _____________ in front of the TV.
58. The decision was deferred _____________ a later meeting.
NH
59. That awful new office block is a real blot _____________ the landscape.
60. I don’t know how Nicole survives, living all away _____________ the sticks.
61. Carl and I just spent the whole day lazing _____________ listening to music.
62. Unfortunately, most of the photos Terry took were ______________ focus.
63. Why don’t we meet _____________ here again on the 12th, when I get back from Germany?
64. The prime minister has come _____________ fire during this election campaign for being slow
Y

to respond to events.
65. Keep plodding _____________ and you’ll finish your novel eventually.
QU

SECTION C: READING (60 points)


Part 1. For questions 66-75, read the following passage and decide which option (A, B, C, or D)
best fits each gap. Write your answers in corresponding numbered boxes on the answer sheet. (15
points)
M

Despite the continued 66.________ of those early town perks, it wasn't until the Depression that
modern Hershey started to take shape. Perhaps the only town in the country actually to
67_________ during the 1930s, it thrived because Hershey vowed his Utopia would never see a

breadline. lnstead he 68_________ a massive building boom that gave rise to the most visited
buildings in today's Hershey and delivered wages to more than 600 workers. He admitted that his
69____________ were partly selfish: "lf I don't provide work for them, I'll have to feed them. And
since building materials are now at their lowest cost levels, I'm going to build and give them jobs."
Y

He seems to have 70__________ no expense; most of the new buildings were strikingly
471________. The first to be finished was the three-million-dollar limestone Community Center,
DẠ

home to the 1,904-seat Venetian-style Hershey Community Theater, which has played 72______
since 1933 to touring Broadway shows and to music, dance, and opera performances. lt offers just
as much to look at when the lights are on and the curtains closed. The floors in the 73_________
named Grand Lobby are polished ltalian lava rock, surrounded by marble walls and capped with a

Page 4 of 11
bas-relief ceiling showing sheaves of wheat, beehives, swans, and scenes from Roman mythology.
With the 74________ inner foyer, Hershey thumbed his nose even harder at the ravages of the
Depression: The arched ceiling is tiled in gold, the fire curtain bears a painting of Venice, and the

L
ceiling is 75_______with 88 tiny lightbulbs to re-create a star-lit night.
66. A. flexibility B. rigidity C. elasticity D. resilience

A
67. A. prosper B. decline C. get on D. flower
68. A. trusted B. funded C. accounted D. stocked

CI
69. A. pretensions B. objections C. preoccupation D. intentions
70. A. spared B. spent C. allowed D. justified
71. A. impoverished B. unattractive C. poor D. opulent

FI
72. A. hosting B. housing C. host D. homogeneously
73. A. aptly B. inappropriately C. seemingly D. frightfully
74. A. dizzying B. gaudy C. dazzling D. bland
75. A. holed B. studded C. supported

OF
D. magnified

Part 2. For questions 76-85, fill each of the following numbered blanks with ONE suitable word
and write your answers in the corresponding numbered boxes on the answer sheet. (15 points)
THE CHANGING FACE OF WORKING LIFE

ƠN
The accepted concept of a career 76._______ followed a similar pattern for decades. After
completing their education, people would enter the adult world of work, 77.___________ down on
to a job which they would likely remain from that point 78.____________. Not only would this
occupation provide their income for their entire working life, it would also allow them a healthy
pension when they retired and moved into 79.___________ age. Over the past twenty years,
NH
80.________, the relationship between a wage earner and their chosen profession has changed
enormously. Today, the idea of a ‘job-for-life’ has all 81._________ disappeared, to be replaced by
an unforgiving world of unstable employment. Some observers even argue that current society to pit
old 82._________ young in a constant battle to find work of some description, all against a
83.__________ of increasing debt and economic difficulties.
Y

At the same time, the government regularly releases figures that suggest the economy is prospering,
evidencing this claim with the fact that the unemployment rate continues to fall annually. There are
QU

indeed more jobs available. However, a huge number of these are casual, temporary or short-term
positions, all of 84.__________ are low-paid and create little in the way of tax income for the
government. This has a number of debilitating long-term effects, not 85.___________ because this
assurance of a growing economy is based more in myth than fact.
M

Part 3. For questions 86-95, read the following passage and circle the best answer to each of the
following questions. Write your answers in corresponding numbered boxes provided on the
answer sheet. (15 points)

The Human Immune System


The human immune system is composed of both an innate and an adaptive immune system.
First, humans have an innate immune system that is intrinsic in all organisms, and it functions
particularly through establishing biological barriers and creating biochemical reactions that
immediately respond with a maximal effort in order to destroy infectious microbes. [A] Second,
Y

humans have an adaptive immune system, which can only be found in vertebrates with jaws. [B] The
adaptive immune system gains an immunological memory from previously encountered germs, so it is
DẠ

able to prevent these specific microbes from causing further infection. [C] With these dual capacities
of fighting infection and acquiring resistance to germs, humans can maximize their immunity. [D]

Page 5 of 11
A person’s innate immune system has many complex barriers and biochemical reactions
designed to ward off infections. The most visible one is the skin, which keeps most bacteria, fungi, and
viruses from ever entering the body, but humans also have mucus, which traps germs that reside in the

L
body’s tissue. In addition to such biological material, there are other internal barriers like gastric acids,
tears, saliva, urine, and various chemicals that either destroy or flush out germs. Even involuntary

A
functions like sneezing and coughing are barriers that serve to expel germs. Beyond these, there are
biochemical reactions that come from leukocytes, which are found in the blood. Leukocytes are white

CI
blood cells that effectively clear out cellular debris, create inflammation near an infection, summon
immune cells to the inflammation, activate several other chemical reactions, and even destroy tumors.
However, perhaps the most important action these cells perform is activating a human’s adaptive
immune system, which is essential in not only curing current diseases but also preventing future

FI
infections.
With an adaptive immune system, cells learn how to best combat pathogens and develop a
higher resistance to them. Like the innate immune system, this involves chemical reactions and cellular

OF
cooperation. Unlike the innate immune system, this system doesn’t respond very quickly or with its
full strength all at once. Instead, it uses its time and energy to provide cells with an immunological
memory to the pathogens they encounter, making them more resistant to recurring infections (similarly
to how a vaccination works). Certain white blood cells called T-cells are the principal actors in this
system; these identify “self” cells with the same DNA and distinguish them from any foreign cells with

ƠN
different DNA. After this, they seek and destroy these foreign cells, whether they are invading
microbes or infected host cells. T-cells also mediate the responses from the innate and adaptive
immune systems so that the body can effectively exterminate the infection.
After destroying infectious cells, the body uses B-cells to develop antibodies, or specialized
NH
proteins that prevent future infections. A B-cell is designed to connect with an individual type of
antigen created by an infectious cell. The B-cell uses this antigen to produce antibodies that seek out
and neutralize infectious bacteria, fungi, and viruses. However, the most important process comes after
the infection disappears: these B-cells will duplicate, and their progeny will manufacture the same
antibodies. Thus, the body will constantly produce antibodies that successfully fight off a specific
infection, and the body can successfully fight off any subsequent infections from this pathogen. In
Y

addition to this, B-cells also mark antigens for leukocytes to attack, thus making them and microbes
easier targets for the biochemical reaction.
QU

An interesting feature of the human immune system is how it affects infants both before and
after birth. When babies are first born, they do not have very many previously formed antibodies, so
they have a greater risk of infection than adults do. However, they ward off many infections by
temporarily obtaining the mother’s antibodies from breast milk and nutrients passed through the
placenta. Also interesting is the very inception of the fetus among such an aggressive immune system:
somehow, the fetus, which doesn’t have its mother’s exact DNA, is ignored by the mother’s T-cells
M

and B-cells. Scientists currently have a few theories about this phenomenon. For instance, the uterus
may not be monitored by white blood cells, or it may produce special proteins that suppress any local

immune responses. Nonetheless, the fact that the immune system restrains its programming for
reproductive development continues to puzzle many scientists.
86. The word intrinsic in the passage is closest in meaning to ________.
A. fundamental B. auxiliary C. detrimental D. extraordinary
Y

87. Which of the following square brackets [A], [B], [C], or [D] best indicates where in the
paragraph the sentence ‘However, this particular action never changes to counter specific threats
DẠ

of infection’ can be inserted?


A. [A] B. [B] C. [C] D. [D]

Page 6 of 11
88. According to passage 2, which bodily fluid initiates biochemical reactions in a human’s innate
immune system?
A. urine B. blood C. saliva D. tears

L
89. The word mediate in the passage is closest in meaning to _________.

A
A. interrupt B. magnify C. contemplate D. coordinate
90. Based on the information in paragraph 3, what can be inferred about the adaptive immune

CI
system?
A. Because it takes so long to act, it is less effective in purging infectious cells than the innate
immune system.

FI
B. Even though it takes longer to act, it is more effective in long term immunity than the innate
immune system.

OF
C. Because humans already have an innate immune system, this system is unnecessary and only
used as a substitute.
D. It works differently from the innate immune system, so the two are completely independent
of one another.

ƠN
91. The word progeny in the passage is closest in meaning to _________.
A. willingness B. mechanism C. offspring D. mutation
92. According to paragraph 4, what do B-cells produce?
A. antigens B. antibodies C. leukocytes D. pathogens
NH
93. The word inception in the passage is closest in meaning to __________.
A. conception B. invulnerability C. contamination D. consumption
94. According to paragraph 5, where do newborn babies get most of their antibodies?
A. from their own white blood cells B. from immune cells in the uterus
Y

C. from mucus and other barriers D. from breast milk and the placenta
QU

95. Based on the information in paragraph 5, what can be inferred about how the mother’s immune
system should scientifically be affecting the fetus?
A. It should protect the fetus from infection.
B. It should help develop cells in the fetus.
C. It should be attacking foreign fetal cells.
M

D. It should be exposing the fetus to microbes.


Part 4. For questions 96-105, read the passage and do the following tasks. Write your answers in
the corresponding numbered boxes on the answer sheet. (15 points)
THE SWIFFER
For a fascinating tale about creativity, look at a cleaning product called the Swiffer and how
Y

it came about, urges writer Jonah Lehrer. In the story of the Swiffer, he argues, we have the key
elements in producing breakthrough ideas: frustration, moments of insight and sheer hard work. The
DẠ

story starts with a multinational company which had invented products for keeping homes spotless,
and couldn't come up with better ways to clean floors, so it hired designers to watch how people
cleaned. Frustrated after hundreds of hours of observation, they one day noticed a woman do with a
paper towel what people do all the time: wipe something up and throw it away. An idea popped into

Page 7 of 11
lead designer Harry West's head: the solution to their problem was a floor mop with a disposable
cleaning surface. Mountains of prototypes and years of teamwork later, they unveiled the Swiffer,
which quickly became a commercial success.

L
Lehrer, the author of Imagine, a new book that seeks to explain how creativity works, says
this study of the imagination started from a desire to understand what happens in the brain at the

A
moment of sudden insight. 'But the book definitely spiralled out of control,' Lehrer says. 'When you
talk to creative people, they'll tell you about the 'eureka' moment, but when you press them they

CI
also talk about the hard work that comes afterwards, so I realised I needed to write about that, too.
And then I realised I couldn't just look at creativity from the perspective of the brain, because it's
also about the culture and context, about the group and the team and the way we collaborate.'

FI
When it comes to the mysterious process by which inspiration comes into your head as if
from nowhere, Lehrer says modern neuroscience has produced a 'first draft' explanation of what is
happening in the brain. He writes of how burnt-out American singer Bob Dylan decided to walk

OF
away from his musical career in 1965 and escape to a cabin in the woods, only to be overcome by a
desire to write. Apparently 'Like a Rolling Stone' suddenly flowed from his pen. 'It's like a ghost is
writing a song,' Dylan has reportedly said. 'It gives you the song and it goes away.' But it's no ghost,
according to Lehrer.
Instead, the right hemisphere of the brain is assembling connections between past influences and

ƠN
making something entirely new. Neuroscientists have roughly charted this process by mapping the
brains of people doing word puzzles solved by making sense of remotely connecting information. For
instance, subjects are given three words - such as 'age', 'mile' and 'sand' - and asked to come up with a
single word that can precede or follow each of them to form a compound word. (It happens to be
'stone'.) Using brain-imaging equipment, researchers discovered that when people get the answer in an
NH
apparent flash of insight, a small fold of tissue called the anterior superior temporal gyrus suddenly
lights up just beforehand. This stays silent when the word puzzle is solved through careful analysis.
Lehrer says that this area of the brain lights up only after we've hit the wall on a problem. Then the brain
starts hunting through the 'filing cabinets of the right hemisphere' to make the connections that produce
the right answer.
Y

Studies have demonstrated it's possible to predict a moment of insight up to eight seconds before it
arrives. The predictive signal is a steady rhythm of alpha waves emanating from the brain's right
QU

hemisphere, which are closely associated with relaxing activities. 'When our minds are at ease-when those
alpha waves are rippling through the brain - we're more likely to direct the spotlight of attention towards
that stream of remote associations emanating from the right hemisphere,' Lehrer writes. 'In contrast, when
we are diligently focused, our attention tends to be towards the details of the problems we are trying to
solve.' In other words, then we are less likely to make those vital associations. So, heading out for a walk
or lying down are important phases of the creative process, and smart companies know this. Some now
M

have a policy of encouraging staff to take time out during the day and spend time on things that at first
glance are unproductive (like playing a PC game), but day-dreaming has been shown to be positively
correlated with problem-solving. However, to be more imaginative, says Lehrer, it's also crucial to

collaborate with people from a wide range of backgrounds because if colleagues are too socially intimate,
creativity is stifled.
Creativity, it seems, thrives on serendipity. American entrepreneur Steve Jobs believed so. Lehrer
describes how at Pixar Animation, Jobs designed the entire workplace to maximise the chance of strangers
bumping into each other, striking up conversations and learning from one another. He also points to a
Y

study of 766 business graduates who had gone on to own their own companies. Those with the greatest
DẠ

diversity of acquaintances enjoyed far more success. Lehrer says he has taken all this on board, and despite
his inherent shyness, when he's sitting next to strangers on a plane or at a conference, forces himself to
initiate conversations. As for predictions that the rise of the Internet would make the need for shared
working space obsolete, Lehrer says research shows the opposite has occurred; when people meet face-to-

Page 8 of 11
face, the level of creativity increases. This is why the kind of place we live in is so important to innovation.
According to theoretical physicist Geoffrey West, when corporate institutions get bigger, they often
become less receptive to change. Cities, however, allow our ingenuity to grow by pulling huge numbers of

L
different people together, who then exchange ideas. Working from the comfort of our homes may be
convenient, therefore, but it seems we need the company of others to achieve our finest 'eureka' moments.

A
Questions 96-100

CI
Do the following statements agree with the information given in Reading Passage? For questions 96-100,
choose:
TRUE if the statement agrees with the information

FI
FALSE if the statement contradicts the information
NOT GIVEN if there is no information on this

OF
Write your answers in the corresponding numbered box provided.
96. It did not take long for the Swiffer to develop once the idea for it was conceived.
97. Lehrer was driven by his own experience of the ‘eureka’ moment.
98. Lehrer refers to the singer Bob Dylan in order to propose particular approaches to regaining

ƠN
lost creativity.
99. Neuroscientists discovered from the word puzzle experiment that one part of the brain only
becomes active when a connection is made suddenly.
100.Scientists know a moment of insight is coming because there is greater activity in the right side
NH
of the brain.
Your answers
96. ______ 97. ______ 98. ______ 99. ______ 100. ______
For questions 101-105, complete the notes below. Choose ONE WORD ONLY from the passage for
each answer. Write your answers in the corresponding numbered box provided.
Y

HOW OTHER PEOPLE INFLUENCE OUR CREATIVITY


QU

• Steve Jobs: made changes to the (101)__________ to encourage interaction at Pixar.


• Lehrer: company owners must have a wide range of (102)__________ to do well.
it’s important to start (103)__________ with new people
the (104)__________ has not replaced the need for physical contact.
• Geoffrey West: living in (105)__________ encourages creativity.
M

Your answers
101. 102. 103. 104. 105.

______________ ______________ ______________ ______________ ______________

D. WRITING (50 points)


Part 1: The charts below show reasons for travel and the main issues for the travelling public in
the US in 2019. (20 points)
Y

Summarise the information by selecting and reporting the main features and make comparisons
where relevant. You should write about 150 words.
DẠ

Page 9 of 11
A L
CI
FI
OF
ƠN
NH
________________________________________________________________________________

________________________________________________________________________________

________________________________________________________________________________
Y

________________________________________________________________________________
QU

________________________________________________________________________________

________________________________________________________________________________

________________________________________________________________________________
M

________________________________________________________________________________

________________________________________________________________________________

________________________________________________________________________________

________________________________________________________________________________
Y

________________________________________________________________________________
DẠ

________________________________________________________________________________

Page 10 of 11
Part 2. With the advent of Chat GPT, people think that students would enormously benefit from
this A.I chatbot for their studies. However, many others believe that without teachers, students
will do more harm than good when they use this type of chatbot. (30 points)

L
Discuss both views and give your opinion. You should write an essay of about 250 words.

A
________________________________________________________________________________

CI
________________________________________________________________________________

________________________________________________________________________________

FI
________________________________________________________________________________

________________________________________________________________________________

OF
________________________________________________________________________________

________________________________________________________________________________

ƠN
________________________________________________________________________________

________________________________________________________________________________

________________________________________________________________________________
NH
________________________________________________________________________________

________________________________________________________________________________

________________________________________________________________________________
Y

________________________________________________________________________________
QU

________________________________________________________________________________

________________________________________________________________________________

________________________________________________________________________________
M

________________________________________________________________________________

________________________________________________________________________________

________________________________________________________________________________

________________________________________________________________________________
Y

________________________________________________________________________________
DẠ

-------------- HẾT --------------


(Thí sinh không được sử dụng tài liệu. Cán bộ coi thi không giải thích gì thêm.

Page 11 of 11
KỲ THI HỌC SINH GIỎI CÁC TRƯỜNG THPT CHUYÊN
KHU VỰC DUYÊN HẢI VÀ ĐỒNG BẰNG BẮC BỘ

L
LẦN THỨ XIV, NĂM 2023

A
CI
HƯỚNG DẪN CHẤM: MÔN TIẾNG ANH – LỚP 10

FI
A. LISTENING (50 points)
Part 1. For questions 1-5, listen to a student asking for information about a healthy eating society
and supply the blanks with the missing information. Write NO MORE THAN TWO WORDS OR

OF
NUMBERS taken from the recording for each answer in the space provided.
(10 points – 2 points/a correct answer)
1. vegetarian 2. Wednesford 3. 7.00 4. covered market 5. Coffee Club

ƠN
Part 2. You will hear a conversation between a boy, Carl, and a girl, Susanna, about a school
concert. For questions 6-10, decide whether the following statements are True or False by putting
a tick (√) in the corresponding column. (10 points)
NH
6. F 7. F 8. T 9. F 10. T

Part 3. For questions 11-15, listen to a radio interview in which a psychologist, Colin Fraser, talks
about cultural identity and choose the correct answer A, B, C, or D which fits best according to what
Y

you hear. Write your answers in the corresponding numbered boxes provided.
QU

11. C 12. B 13. C 14. A 15. B

Part 4. For questions 16-25, listen to a report on G7 Summit held in France and supply the blanks
with the missing information. Write NO MORE THAN FOUR WORDS taken from the recording for
each answer in the space provided.
M

16. show a united front 21. ravaging


17. speak with one voice 22. trade relations/ties


18. confrontational 23. key priority
19. cooperation 24. wine (imports)
Y

20. Iran nuclear deal 25. heal growing divisions


DẠ
B. LEXICO – GRAMMAR (40 points)
Part 1. For questions 26-45, choose the best option A, B, C, or D to complete the following sentences

L
and write your answers in the corresponding numbered boxes provided on the answer sheet. (20
points – 1 point/a correct answer)

A
26.A 27.A 28.D 29.D 30.C 31.B 32.B

CI
33.B 34.A 35.D 36.A 37.A 38.C 39.C
40.C 41.A 42.A 43.C 44.C 45.C

FI
Part 2. For questions 46-55, fill each gap with the correct form of the words in brackets. Write your
answer in the boxes provided on the answer sheet.
(10 points – 1 point/a correct answer)

OF
46. Namesake 51. Frighteners
47. Mellifluous 52. Freewheeling
48. Pictorially 53. Commodified/commoditized
49. Photogenic 54. Onerous

ƠN
50. Desirous 55. Underreported

Part 3. For questions 56-65, complete each of the following sentences with suitable preposition(s).
NH
Write your answer in the boxes provided on the answer sheet.
(10 points – 1 point/a correct answer)

56. within 57. off 58. to 59. on 60. in


Y

61. around 62. out of 63. up 64. under 65. away


QU

C. READING (60 points)


Part 1. For questions 66-75, read the following passage and decide which option (A, B, C, or D) best
fits each gap. Write your answers in corresponding numbered boxes on the answer sheet. (15 points
M

– 1.5 points/a correct answer)


66. D 67. A 68. B 69. D 70. A

71. D 72. C 73. A 74. C 75. B

Part 2. For questions 76-85, fill each of the following numbered blanks with ONE suitable word and
write your answers in the corresponding numbered boxes on the answer sheet.
Y

(15 points – 1.5 points/a correct answer)


DẠ

76. path 77. settling 78. onward(s) 79. old 80. however
81. but 82. against 83. backdrop 84. which 85. least
Part 3. For questions 86-95, read the following passage and circle the best answer to each of the
following questions. Write your answers in corresponding numbered boxes provided on the answer

L
sheet. (15 points – 1.5 points/a correct answer)

A
86. A 87. A 88. B 89. D 90. B
91. C 92. B 93.A 94. D 95. C

CI
Part 4. For questions 96-105, read the passage and do the following tasks. Write your answers in the
corresponding numbered boxes on the answer sheet.
(15 points – 1.5 points/a correct answer)

FI
96. FALSE 97. NOT GIVEN 98. FALSE 99. TRUE 100. TRUE
101. workplace 102. acquaintances 103. conversations 104. Internet 105 cities

OF
D. WRITING (50 points)
Part 1: 20 points

ƠN
Contents (10 points)
The report MUST cover the following points:
- Introduce the charts (2 points) and state the overall trends and striking features (2 points)
- Describe main features with relevant data from the charts and make relevant comparisons (6 points)
NH
Language use (10 points)
The report:
- should demonstrate a wide variety of lexical and grammatical structures
- should have correct use of words (verb tenses, word forms, voice...) and mechanics (spelling,
Y

punctuations,)
QU

Part 2: (30 points)


The mark given to part 3 is based on the following criteria:
1. Organization (5 points)
a. Ideas are well organized and presented with coherence, cohesion and unity.
M

b. The essay is well-structured:


* Introduction is presented with clear thesis statement.

* Body paragraph are written with unity, coherence and cohesion.


Each body paragraph must have a topic sentence and supporting details and examples when
necessary.
Y

* Conclusion summarizes the main points and offers personal opinions (prediction,
recommendation, consideration ...) on the issue.
DẠ

2. Content (15 points)


a. All requirements of the task are sufficiently addressed.
b. Ideas are adequately supported and elaborated with relevant and reliable explanations,
examples, evidence....

L
3. Language use (10 points)
a. Demonstration of a variety of topic-related vocabulary.

A
b. Excellent use and control of grammatical structures (verb tenses, word forms, voice...) and

CI
mechanics (spelling, punctuations...)
THE END

FI
OF
ƠN
NH
Y
QU
M

Y
DẠ
TRƯỜNG THPT CHUYÊN KỲ THI HỌC SINH GIỎI CÁC TRƯỜNG THPT CHUYÊN
TỈNH TUYÊN QUANG KHU VỰC DUYÊN HẢI VÀ ĐỒNG BẰNG BẮC BỘ
LẦN THỨ XIV, NĂM 2023

L
ĐỀ THI ĐỀ XUẤT ĐỀ THI MÔN: TIẾNG ANH 11

A
Thời gian: 180 phút (Không kể thời gian phát đề)
(Đề thi có 20 trang)
Ngày thi: 15/7/2023

CI
A. LISTENING

FI
Part 1. For questions 1-5, you will hear a talk about epidemiology. Listen and decide whether
the following sentences are true (T) or false (F). Write your answers in the corresponding

OF
numbered boxes on the answer sheet. (10 points)
1. Epidemiology involves a interdisciplinary approach to the study of human health and disease.
2. Epidemiology is commonly identified with both infectious and non-infectious diseases.
3. The epidemiologic triangle is made up of three components: an external agent, a host

ƠN
and an environment.
4. Large-scale social distancing recommended by a team of scientists from Imperial
College London aimed to break the link between the external agent and the host.
5. The team’s recommendations became public policy in many countries.
NH
Your answer

1. 2. 3. 4. 5.

Part 2. For question 6-10, listen to a talk about the future of fashion and answer the
Y

questions. WRITE NO MORE THAN FOUR WORDS taken from the recording for each
answer in the recording numbered boxes provided.
QU

6. What is capable of forcasting fashion trends with high precision?


7. As evidenced by H& M’s unsold clothes, what could have untold repercussions?
8. Besides superfluous production, what aspect of fashion could be diminished thanks to the
adoption of machine learning?
M

9. To create new clothes, what will Amazon’s AI designer be doing aside from duplicating well-
known styles?

10. What is ZOZO?

Your answer

6.______________________
Y

7.______________________
DẠ

8.______________________
9.______________________
10.______________________

1
Part 3. You will hear an interview with Maria Stefanovich, co-founder of a creativity group
which organises workshop executives. For questions 11-15, choose the answer (A, B, C or D)

L
which fits best according to what you hear.

A
11. Corporations appreciate mask-making workhops because______
A. no one wants negative faces at the office.

CI
B. unhappy employees won’t come to work.
C. they realise how their employees see them.

FI
D. their employees change their approach.
12. Companies are turning to creative workshops because they have acknowledged that ______
A. unproductive employees are a financial burden.

OF
B. the traditional work environment has its limitations.
C. there is an increase in absenteeism.
D. employees are working too hard without enjoying it.
13. The employees at the firm “Play” ______

ƠN
A. change positions frequently to lessen boredom.
B. have business cards indicating their jobs.
C. do not have stereotyped ideas about their jobs.
D. dress up like comic book characters.
NH
14. The companies that show most interest in creative workshops are surprising because ______
A. they usually have creative employees to begin with.
B. their employees are the one who have to present regularly.
C. there are many other exciting workshops they would prefer.
Y

D. their employees should be used to being funny.


15. Maria mentions the traditional companies that have held workshops in order to ______.
QU

A. boast about the clients her company has helped.


B. show that they have a narrow list of clients.
C. downplay the serious reputations of the films.
D. point out the diversity of those trying different approaches.
M

Your answer

11. 112. 13. 14. 15.


Part 4. For questions 16-25, you will listen to a presenter talking abour a phenomenon in the
nature. Complete the summary by writing NO MORE THAN THREE WORDS in each gap.
Write your answers in the corresponding numbered boxes on the answer sheet. (20 points – 2
points/a correct answer)
Y

(16)_________ are the building blocks of plant life. Plants naturally absorb nutrients from
DẠ

the soil through their root system. However, farmers turn to (17)_________ when facing poor soil
or massive erosion or leaching. Farmers tend to overuse fertilizer as a precautionary measure
because it is difficult to (18)_________ the amount of fertilizer needed. Excess fertilizer can

2
runoff into bodies of water causing (19)_________, characterized by the rapid accumulation in the
population of algae in marine water systems. Dense layers of algae form an (20)_________ on the
surface of the water, blocking other plants in the water from getting the sunlight they need to

L
survive. Because water cannot support aquatic life, plants die off and sink to the bottom of the

A
water body, where (21)_________ feast ont the dead bodies. This decomposition process
consumes plenty of (22)_________ and animals that rely on oxygen to breathe can, in fact,

CI
suffocate. A vicious cycle of degradation in aquatic life can be observed and a (23)_________ is
created. If this process occurs in lakes, native species can be (24)_________ and give way to

FI
invasives. If this happens in the ocean, the incidence of coral bleaching is inevitable.
(25)_________ and farming activities can lead to nutrient-rich runoff, also known as clear-
cutting.

OF
Your answer

16.______________________
17.______________________
18.______________________

ƠN
19.______________________
20.______________________
21.______________________
NH
22.______________________
23.______________________
24.______________________
25.______________________
Y

B. LEXICO – GRAMMAR (30 points)


QU

Part 1. For questions 26-45, choose the best option A, B, C or D to complete the following
sentences and write your answers in the corresponding numbered boxes provided. (20 points)
26. Google is the first major company to ______ refuse China’s demands for control.
A. blissfully B.sorely C. steadfastly D. woefully
27. Although it takes quite a bit of time at the begginning, once you have acquired the basic
M

knowledge, a quick learner like you will surely ______.


A. forge ahead B. plunge ahead C. plough ahead D. press ahead

28. Do you think it’s better to ______ your anger than to supress it?
A. assert B. repress C. arouse D. vent
29. Breaking his leg dealt a ______ to his chances of becoming a professional footballer.
A. thump B. strike C. blow D. hit
Y

30. I don’t know ______ the new manager.


DẠ

A. what to get B. what to make of C. how to get of D. how to make of


31. Jenny decided to draw a line ______ her recent breakup and move on.
A. at B. under C. over D. with

3
32. It’s hard to believe that so many completely untrue stories are spread on social media. We are
living in an era where ______ information is more credible than fact.
A. full- scale B. low-key C. post-true D. short-lived

L
33. I only asked to move her car but she made such a(n) ______ about it.

A
A.song and dance B. short and sweet C.cut and dried D. open and shut
34. 25 people have replied to the invitation, but I’ve ______ and put out 30 chairs.

CI
A. teetered on the brink B. erred on the side of caution
C. been on a zaror’s edge D. been in the teeth

FI
35. She’s a bit down in the ________ at the moment – her husband has just lost his job.
A. world B. bottom C. heart D. dumps
36. The company cannot accept_________for injuries resulting from improper use of rental

OF
equipment
A. validity B. liability C. compensation D. privilege
37. You’ll just have to _________ yourself to the fact that you can’t always have what you want.
A. acknowledge B. concede C. allow D. reconcile

ƠN
38. A whole _______ of measures was tried in an attempt to get them to give up cigarettes.
A. battery B. wood C. generation D. stream
39. Only the most basic and essential facts are required, stop adding more information, you are
making _______ of the presentation, Emily!
NH
A. a song B. a dish C. a meal D. a scale
40. _______ tests assume that individuals have instrinc talents and limitations as well as a natural
predisposition toward success or failure in various areas based on their innate qualities.
A. Achievement B. Perception C. Acumen D. Aptitude
Y

41. If you dare to go against everyone’s expectations, you will be _______.


A. off for it B. for one C. for it D. off and on
QU

42. Jason has _______ a fantastic job with one of our top newpapers.
A. earned B. launched C. won D. landed
43. Despite dismal failures in the past, James still _______ his ambitions of playing professional
soccer.
M

A. nurses B. cradles C. breeds D. rears


44. As a man with an eye for neatness around the place, Tom always ensured that things were

_______.
A. hale and hearty B. spick and span C. bright and breezy D. short and sweet
45. The books are a good cover to cover read for the recommended age group, with enough detail
to ________ the imagination of inquiring young minds.
Y

A. incense B. foment C. kindle D. pique


Your answer
DẠ

26. 27. 28. 29. 30. 31. 32. 33. 34. 35.
36. 37. 38. 39. 40. 41. 42. 43. 44. 45.

4
Part 2. For questions 46-55, give the correct form of each given word to complete the following
sentences and write your answers in the corresponding numbered provided. (10 points)
46. The loveliest scenes, he found, were comprised of the simplest, most natural _______________

L
of native plants. (POSITION)

A
47. Living in cities and towns, kids these days are much more
than we ever were at their age. (STREET)

CI
48. As in all the chapters, David maintain the excitement visually with pictures and page designs
that make such a book a _______________. (PAGE)

FI
49. There are islands in Antartica and parts of northern Canada that are uninhabitable due to the
________________ of the weather. (CLEMENT)
50. A(n) ________________ political outlook can be misguided or even dangerous, because it

OF
doesn’t take the lessons of the past into account. (HISTORY)
51. Mind-alerting drugs weren’t necessary, since this was an evening of pure ______________
spontaneous and harmless fun. (ADULT)
52. The ________________ headed by Mr.Smith would be looking to develop hotel facilities

ƠN
adjoining the stadium. (CONSORT)
53. People living in very hot climates need to ensure that they get enough fluids into their bodies
to prevent ________________. (HYDRATE)
54. Once a fine-looking manor, and easily the largest and grandest building for miles around, the
NH
Riddle House was now damp, ______________, and unoccupied. (RELIC)
55. I believe he has committed a(n) _____________ sin, so his punishment is well-deserved.
(PARDON)
Your answer
Y

46. 51.
47. 52.
QU

48. 53.
49. 54.
50. 55.
M

C. READING (60 points)


Part 1. For questions 56-65, fill each of the following numbered blanks with ONE suitable word

and write your answers in the corresponding numbered provided. (15 points)
Concentration is good in exams, bad in orange juice. Concentration (56) ______ when you
manage to focus on one thing to the exclusion of all others, and concentrating on that one thing
allows you to stop worrying about a lot of other things. Sometimes your mind concentrates when
Y

you don’t want it to. Maybe you can’t get something (57) ______ of your head, such as a problem
you have to face up to, or an embarrassing situation you’ve been in. That’s why collecting things
DẠ

as a hobby is popular; it (58) ______ your mind off other things. Indeed, some people seem to
prefer looking after and cataloguing their collections to actually (59) ______ anything with them,
because this is when the absorbing, single- minded concentration happens.
5
The natural (60) ______ for concentration is 45 minutes. That’s why half an hour for a television
programme seems too short (61) ______ an hour seems too long. But many people's lives are
devoid of concentration. Modern culture is served up in small, easily digestible chunks (62)

L
______ require only a short attention span although young people can concentrate on computer

A
games for days at a (63) ______ .
Sticking out the tongue can aid concentration. This is because you can’t (64) ______ yourself

CI
with talking at the same time and other people won’t dare to (65) ______ your thoughts, because
you look like an idiot! It would probably be better to concentrate your own mind before having it

FI
concentrated for you, but only people with a will of iron choose that route.
Edited from https://www.theguardian.com/lifeandstyle/2006/sep/23/weekend.guybrowning
Your answer

OF
56. 58. 60. 62. 64.
57. 59. 61. 63. 65.

Part 2. For questions 66-75, read the passage below and choose the answer A, B, C or D that

ƠN
fits best according to the text. Write your answers in the corrresponding numbered boxes
provided. (10 points)
LEARNING TO RUN
An article published recently in the prestigious scientific journal Nature is shedding new
NH
light on an important, but hitherto little appreciated, aspect of human evolution. In this article,
Professors Dennis Bramble and Daniel Lieberman suggest that the ability to run was a crucial
factor in the development of our species. According to the two scientists, humans possess a number
of anatomical features that make them surprisingly good runners. ‘We are very confident that
Y

strong selection for running (A) ____was instrumental in the origin of the modern human body
form,’ says Bramble, a biology professor at the University of Utah. Traditional thinking up to now
QU

has been that the distinctive, upright body form of modern humans has come about as a result of
the ability to walk, and that running is simply a by-product of walking.
Furthermore, humans have usually been regarded as poor runners compared to such
animals as dogs, horses or antelopes. However, this is only true if we consider fast running, or
M

sprinting, over short distances. Even an Olympic athlete can hardly run as fast as a horse can
gallop, and can only keep up a top speed for fifteen seconds or so. Horses, antelopes and

greyhounds, on the other hand, can run at top speed for several minutes, clearly outperforming us
in this respect. But when it comes to long-distance running, humans do astonishingly well
(B)_____ They can maintain a steady pace for miles, and their overall speed compares favourably
with that of horses or dogs.
Y

Bramble and Lieberman examined twenty-six anatomical features found in humans. One
of the most interesting of these is the nuchal ligament, a band of tissue that extends from a ridge
DẠ

on the base of the skull to the spine. When we run, it is this ligament that prevents our head from
pitching back and forth or from side to side. Therefore, we are able to run with steady heads, held
high. The nuchal ligament (C)_____is not found in any other surviving primates, although the
6
fossil record shows that Homo erectus, an early human species that walked upright, much as we
do, also had one. Then there are our Achilles tendons at the backs of our legs, which connect our
calf muscles to our heel bones - and which have nothing to do with walking. When we run, these

L
behave like springs, helping to propel us forward. Furthermore, we have low, wide shoulders,

A
virtually disconnected from our skulls, an anatomical adaptation which allows us to run more
efficiently. Add to this our light forearms, which swing out of phase with the movement of our legs

CI
to assist balance, and one begins to appreciate the point that Bramble and Lieberman are trying to
make.

FI
But what evolutionary advantage is gained from being good long-distance runners? One
hypothesis is that this ability may have permitted early humans to obtain food more effectively.
‘What these features and fossil facts appear to be telling us is that running evolved in order for our

OF
direct ancestors to compete with other carnivores for access to the protein needed to grow the big
brains that we enjoy today,’ says Lieberman. Some scientists speculate that early humans may have
pursued animals for miles in order to exhaust them before killing them. Running would also have
conferred an advantage before weapons were invented: early humans might have been scavengers,

ƠN
eating the meat and marrow left over from a kill by lions or other large predators. They may have
been alerted to the existence of a freshly-killed carcass by vultures (D)____, and the faster they
got to the scene of the kill, the better.
‘Research on the history of human locomotion has traditionally been contentious,’ says
NH
Lieberman. ’At the very least, I hope this theory will make many people have second thoughts
about how humans learned to run and walk and why we are built the way we are.’
66. According to the text, the human ability to run ________
A. was only recently described in a scientific journal.
Y

B. is now regarded as more important than the ability to climb trees.


C. played an important part in human evolution.
QU

D. is surprising when we consider evolutionary trends.


67. According to the text, scientists used to believe ________
A. that the human body owes its form to the ability to walk.
B. the human ability to walk adversely affected the ability to run.
M

C. that only modern humans could walk upright.


D. that humans can run because they stand upright.

68. According to the text, humans________


A. are better runners than most other animals. B. are not good at running short distances.
C. cannot run at top speed for long distances. D. compare unfavourably with horses and
dogs.
Y

69. It appears that the nuchal ligament________


A. is found only in modern primates. B. is associated with the ability to run.
DẠ

C. prevents the head from moving D. is a unique anatomical feature.


70. The text implies that________
A. we do not need calf muscles in order to walk.
7
B. without shoulders we could not run very fast.
C. the movement of our forearms is out of phase.
D. our Achilles tendons are an adaptation for running.

L
71. The pronoun “these” in the third paragraph refers to________

A
A. legs B. tendons C. muscles D. bones
72. According to the text, early humans________

CI
A. killed animals by exhausting them. B. may have evolved big brains for running.
C. competed with other animals for food. D. could probably run before they could

FI
walk.
73. Professor Lieberman hopes to________
A. dispel any remaining doubts about the nature of the human body.

OF
B. prove conclusively that humans did not always walk in an upright position.
C. make people reconsider previously-held ideas about human anatomy.
D. inform people of the real reason why humans are able to run and walk.
74. Which of the following spaces can the relative clause “- which came at the expense of the

ƠN
historical ability to live in trees -” fit?
A. (A) B. (B) C. (C) D. (D)
75. The word “conferred” in the fourth paragraph can be best replaced by________.
A. give out B. bring about C. refer to D. make out
NH
Your answer
66. 67. 68. 69. 70. 71. 72. 73. 74. 75.
Y

Part 3. For questions 76-88, read the following passage and do the tasks that follow. (13 points)
A
QU

Since 1901, the Nobel Prize has been honoring men and women from all corners of the globe for
outstanding achievements in physics, chemistry, medicine, literature, and for work in peace. The
foundations for the prize were laid in 1895 when Alfred Nobel wrote his last will, leaving much
of his wealth to the establishment of the Nobel Prize.
M

B
Alfred Nobel was born in Stockholm on October 21, 1833. His father Immanuel Nobel was an

engineer and inventor who built bridges and buildings in Stockholm. In connection with his
construction work, Immanuel Nobel also experimented with different techniques for blasting
rocks. Successful in his industrial and business ventures, Immanuel Nobel was able, in 1842, to
bring his family to St. Petersburg. There, his sons were given a first-class education by private
Y

teachers. The training included natural sciences, languages and literature. By the age of 17,
Alfred Nobel was fluent in Swedish, Russian, French, English and German. His primary interests
DẠ

were in English literature and poetry as well as in chemistry and physics. Alfred’s father, who
wanted his sons to join his enterprise as engineers, disliked Alfred’s interest in poetry and found
his son rather introverted.
8
C
In order to widen Alfred’s horizons, his father sent him abroad for further training in chemical
engineering. During a two year period, Alfred Nobel visited Sweden, Germany, France and the

L
United States. In Paris, the city he came to like best, he worked in the private laboratory of

A
Professor T. J. Pclouze, a famous chemist. There he met the young Italian chemist Ascanio
Sobrero who, three years earlier, had invented nitroglycerine, a highly explosive liquid. But it

CI
was considered too dangerous to be of any practical use. Although its explosive power greatly
exceeded that of gunpowder, the liquid would explode in a very unpredictable manner if

FI
subjected to heat and pressure. Alfred Nobel became very interested in nitroglycerine and how it
could be put to practical use in construction work. He also realized that the safety problems had
to be solved and a method had to be developed for the controlled detonation of nitroglycerine.

OF
D
After his return to Sweden in 1863, Alfred Nobel concentrated on developing nitroglycerine as
an explosive. Several explosions, including one (1864) in which his brother Emil and several
other persons were killed, convinced the authorities that nitroglycerine production was

ƠN
exceedingly dangerous. They forbade further experimentation with nitroglycerine within the
Stockholm city limits and Alfred Nobel had to move his experimentation to a barge anchored on
Lake Malaren. Alfred was not discouraged and in 1864 he was able to start mass production of
nitroglycerine. To make the handling of nitroglycerine safer Alfred Nobel experimented with
NH
different additives. He soon found that mixing nitroglycerine with kieselguhr would turn the
liquid into a paste which could be shaped into rods of a size and form suitable for insertion into
drilling holes. In 1867 he patented this material under the name of dynamite. To be able to
detonate the dynamite rods he also invented a detonator (blasting cap) which could be ignited by
Y

lighting a fuse. These inventions were made at the same time as the pneumatic drill came into
general use. Together these inventions drastically reduced the cost of blasting rock, drilling
QU

tunnels, building canals and many other forms of construction work.


E
The market for dynamite and detonating caps grew very rapidly and Alfred Nobel also proved
himself to be a very skillful entrepreneur and businessman. Over the years he founded factories
M

and laboratories in some 90 different places in more than 20 countries. Although he lived in Paris
much of his life he was constantly traveling. When he was not traveling or engaging in business

activities Nobel himself worked intensively in his various laboratories, first in Stockholm and
later in other places. He focused on the development of explosives technology as well as other
chemical inventions including such materials as synthetic rubber and leather, artificial silk, etc.
By the time of his death in 18%, he had 355 patents.
Y

F
Intensive work and travel did not leave much time for private life. At the age of 43, he was
DẠ

feeling like an old man. At this time he advertised in a newspaper “Wealthy, highly-educated
elderly gentleman seeks the lady of mature age, versed in languages, as secretary and supervisor
of household.” The most qualified applicant turned out to be an Austrian woman, Countess
9
Bertha Kinsky. After working a very short time for Nobel she decided to return to Austria to
marry Count Arthur von Suttner. In spite of this Alfred Nobel and Bertha von Suttner remained
friends and kept writing letters to each other for decades. Over the years Bertha von Suttner

L
became increasingly critical of the arms race. She wrote a famous book, Lay Down Your Arms

A
and became a prominent figure in the peace movement. No doubt this influenced Alfred Nobel
when he wrote his final will which was to include a Prize for persons or organizations who

CI
promoted peace. Several years after the death of Alfred Nobel, the Norwegian
Storting (Parliament) decided to award the 1905 Nobel Peace Prize to Bertha von Suttner.

FI
G
Alfred Nobel died in San Remo, Italy, on December 10, 1896. When his will was opened it came
as a surprise that his fortune was to be used for Prizes in Physics, Chemistry, Physiology or

OF
Medicine, Literature and Peace. The executors of his will were two young engineers, Ragnar
Sohlman and Rudolf Lilljequist. They set about forming the Nobel Foundation as an
organization to take care of the financial assets left by Nobel for this purpose and to coordinate
the work of the Prize-Awarding Institutions. This was not without its difficulties since the will

ƠN
was contested by relatives and questioned by authorities in various countries.
H
Alfred Nobel’s greatness lay in his ability to combine the penetrating mind of the scientist and
inventor with the forward-looking dynamism of the industrialist. Nobel was very interested in
NH
social and peace-related issues and held what were considered radical views in his era. He had a
great interest in literature and wrote his own poetry and dramatic works. The Nobel Prizes
became an extension and a fulfillment of his lifetime interests.
For questions 76-82, decide whether the following statements are True (T), False (F) or Not
Y

Given (NG). Write your answers in the corresponding numbered boxes provided
Do the following statements agree with the information given in Reading Passage?
QU

In boxes 76-81 on your answer sheet, write


TRUE if the statement is true
FALSE if the statement is false
NOT GIVEN if the information is not given in the passage
M

76. The first Nobel Prize was awarded in 1895.


77. Nobel’s father wanted his son to have a better education than what he had had.

78. Nobel was an unsuccessful businessman.


79. Bertha von Suttner was selected by Nobel himself for the first peace prize.
80. The Nobel Foundation was established after the death of Nobel
81. Nobel’s social involvement was uncommon in the 1800s.
Y

For questions 82-88, complete the notes below using NO MORE THAN TWO WORDS from
the passage. Write your answers in the corresponding numbered boxes provided
DẠ

Education:
Having accumulated a great fortune in his business, Nobel’s father determined to give his son the
best education and sent him abroad to be trained in 82______ during Nobel’s study in Paris, he
10
worked in a private laboratory, where he came in contact with a young engineer 83______ and
his invention nitroglycerine, a more powerful explosiven than 84______
Benefits in construction works:

L
Nobel became really interested in this new explosive and experimented on it. But nitroglycerine

A
was too dangerous and was banned for experiments within the city of 85______. So Nobel had to
move his experiments to a lake. To make nitroglycerine easily usable, Nobel invented dynamite

CI
along with 86______ while in the meantime 87______ became popular, all of which
dramatically lowered the 88______ of construction works.

FI
Your answer:
76. 77. 78. 79. 80. 81.

OF
82. 83. 84. 85. 86. 87. 88.

Part 4: In the passage below, seven paragraphs have been removed. Read the passage and
choose from paragraphs A-H the one which fits each gap. There is ONE extra pragraph which

ƠN
you do not need to use. Write your answers in the corresponding numbered boxes provided on
the answer sheet. (7 points)
“What do you want to be when you grow up?” a class of fresh-faced 12-year-olds were asked
upon commencing secondary school. Their new English teacher- Mrs Marcus- asked this question
NH
every year and it seemed to fire the imagination of every child. Usually there was a smattering of
professions, vocations and trades, along with some interesting surprises. This lot did not disappoint
89.
It turns out they were a highly varied lot: doctor, nurse, lawyer, judge, electrician, archeologist,
Y

businesswoman, vet, police officer, hairdresser, actor, shop assistant. There was trouble containing
their enthusiasm, with some throwing out more than one idea. A few had non-specific ambitions,
QU

‘I don’t know. I want to travel,’ and ‘I just want to go to university.’ All of them had opinions,
some stronger than others, but opinions nonetheless.
90.
I’m particularly interested in the differences between that generation and the current one.
M

‘Hopes and dreams,’ she replies immediately. ‘Whether your classmates achieved those things or
not is irrelevant. The important thing is you had ideas about your future; you had aspirations. When

I have asked that question in recent years, instead if setting their sights on becoming a scientist, a
lawyer or an artist, the best some children could think of was going on the dole, being famous, or
being the boss of a gang,’ she says.
91.
Y

It is a vicious circle that becomes increasingly difficult to break. It was crucial for my peers
and I that we knew people who worked and we could make decisions about our ambitions based
DẠ

on some knowledge. We had the benefit of seeing our parents, relatives and neighbours going to
work, returning from work, talking about their jobs, or their time at university. These experiences

11
informed our ideas, ambitions and, let’s face it, our expectations, too. We wouldn’t have dreamt of
the alternative. After all, work and study were our means to get ahead and make our way in life.
92.

L
This lies at the very core of a gang’s appeal. The aimlessness of some youths’ experience is

A
replaced by the rigid system of rules, rituals, and codes of behavior that members follow, and
which gives them a purpose and adds much-needed structure to their lives. In many cases, the gang

CI
becomes a surrogate family, providing security, camaraderie and a sense of belonging. These
powerful inducements exert a strange power over vulnerable teenagers.

FI
93.
I ask Margaret what it is that can drive such a change. ‘I’ve known many young boys who have
turned over a new leaf,’ she says. ‘The key is intervention at the grassroots level. Community

OF
programmes that keep kids off the streets and involve them in pro-social activities are great
deterrents. Strong after-school programmes that meet children’s needs for supervision are also
successful in reducing attractionto gang-related activities. These cost money, though, and
authorities are often not willing to spend,’ she explains, ‘and sadly, some kids fall through the

ƠN
cracks.’
94.
What chance for rehabilitation do they have, I wonder, when they cling to their gangs even in
these circumstances? Admittedly, the need for survival plays a role since those in prison rely on
NH
their fellow gang members for protection. After all, prison is no picnic and is possibly more
dangerous an environment than the outside world. But even in here, there is hope.
95.
‘You can’t make anyone succeed, but you can help them to see that success in life is possible
Y

outside of the narrow confines of the gang,’ says Margaret. ‘If we give young people opportunities
QU

to bring about a change in their circumstances, they can build a happy future.’ Let’s hope that the
next time Margaret asks ‘the question,’ there will be some scientists, entrepreneurs and plumbers
in the group.

Paragraphs
M

A. Success, however, can be measured in a myriad of ways, and for those without traditional role
models, gang culture becomes increasingly alluring as a way to make something of themselves.

‘They’re not inherently bad kids,’ says Marcus, ‘they just have no direction and no one to look up
to. Were they to attach value to work and education, their whole outlook on life would change and
they wouldn’t need what gang membership provides.’
B. Though there is no conclusive evidence, many critics of popular media believe exposure to
Y

violent films and song lyrics, particularly rap music, has a negative influence; glamorising gang
life and encouraging at-risk youths to join gangs or to participate in gang-related crime as a means
DẠ

of gaining a sense of belonging and empowerment.


C. Those who do join in a gang inevitably end up in a downward spiral, losing any moral
foundation they had and hurtling headlong into a life of violence and crime. And yet, even when
12
they are placed in juveline detention centres, or worse- in adult prisons- some maintain their
allegiance to the gang and look upon their membership as a badge of honour; a mark of success,
not failure.

L
D. Thick and fast came the replies. ‘Teacher,’ sai a bespectacled girl in the front row. Mrs Marcus

A
smiled to herself. The prospective teachers always sat as close as possible to the board, eager and
serious. ‘Football player!’ shouted a tall lad from the back, raising his arms in victory as though

CI
he’d just scored a winning goal against Argentina at the World Cup. Mrs Marcus knew he’d be a
live wire in class. ‘Prime minister,’ said another, garnering a round of applause as well as ridiculous

FI
from his classmates.
E. How has it come to this? A recent report has found that children in some areas of the country
have so little contact with working people that the concept of employment is almost foreign. They

OF
live in the so-called ‘welfare ghettos’ where more than half of the working -age population depends
on out-of-work benefits. In many families, unemployment is intergenerational with grandparents
and parents living on the dole.
F. Thankfully, in many cases the lure if temporary. It becomes nothing more than a phase that plays

ƠN
to their fantasies or rebellion and desire for high drama, in part fueled by pop culture through music
and films that glamorise thug life. In time, these wannabe gangsters find other interests and reject
the values of the gang.
G. Fast forward twenty years and Mrs Margaret Marcus is now teaching at an inner-city school in
NH
a large metropolis. Hoping to get some insight from this forty-year veteran of the education system,
I’m interviewing her about the challenges faced by young people today. ‘So you became a
journalist instead of a teacher’, she says with a twinkle in the eye. Yes, I was that child in the specs
long ago. Before getting down to business, we reminisce for a few moments about my classmates.
Y

H. There are many organisations that are working positively with young people in gangs, both
inside and outside of detention, and helping them through some very difficult times in their lives.
QU

With this help they can stop their slide into crime and violence, and make the tough transition of
evolving into productive, responsible and law abiding of society.

Your answer
M

89. 90. 91. 92. 93. 94. 95.


Part 5: For questions 96-105, you are going to read an extract about food ans cooking. Answer
the questions by choosing from the sections of the extract (A - D). The sections may be chosen
more than once. Write your answers in the corresponding numbered boxes provided on the
answer sheet. (15 points)
Y

A. Monosodium Gluctamate
Good food is one of the life’s pleasures and even 1,200 years ago, oriental cooks knew that
DẠ

food tasted better when prepared with a soup stock made from a type of seaweed. But it was only
in 1908 that Japanese scientists identified the ingredient responsible for enhancing flavour.

13
That ingredient is known today by its scientific name, monosodium glutamate. It is often
reffered to as MSG and it is an amino acid found in virtually all foods. The bound form is linked
to other amino acids in proteins and is manufactured in the human body. The free form of glutamate

L
in foods enhances food flavours. Tomaoes, cheese and mushrooms are just some free glutamate

A
rich foods. Free glutamate content increases during ripening, bringing out a fuller taste in many
foods and is made as a flavour enhancer by a fermentation process similar to that used for making

CI
soy and vinegar.
People have long known about the four basic tastes-sweet, sour, salty and bitter. But now

FI
a fifth basic taste called umani has been recognised. This is imparted to foods by glutamate and is
responsible for the savoury taste of many foods, such as tomatoes and cheese.
B. Organic Food & Bunisess!

OF
Organic farmers pride themselves on fostering sustainable agriculture, but it remains to be
seen if the industry’s rapid growth is in fact sustainable.
One challenge facing the industry is to bring the price of organic products more in line with
those of conventional products. The price of organic ingredients is improving but demand still

ƠN
outpaces suppply. However, supply issues are overshadowed by the fact that the organic foods
sector continues to grow faster than the food industry as a whole, fundamentally due to the natural
alliance between organic foods and processed foods. Firstly, organic foods earmarked for
processing do not have to be as comestically perfect as their fresh counterparts. In addition,
NH
freezing or tinning reduces many of the shelf-life problems associated with fresh produce. It was
only a question of time before mainstream food companies woke up to these synergies.
The pioneers of the organic food industry view the growing presence of major food
companies in their markets as a mixed blessing. Many smaller companies fear that the ideals of
Y

organic agrilculture will be compromised. Other think major food companies will help persuade
consumers to buy organic products through the power of their branding.
QU

C. Chilli
Capsicums, commonly known as chillis, come in all dimensions and colours from the tiny,
pointed, extremely hot, bird’s eye chilli, to the large, mild, fleshy peppers like the Anaheim.
Indigenous to Central and South America and the West Indies, they were cultivated long before
M

the Spanish conquest, which was the eventual cause of their introduction to Europe. Chillis along
with tomatoes, avocados, vanilla and chocolate changed the flavours of the known world. Today,

there are around 400 different varieties of chillis grown. They are easy to cultivate and are one of
the world’s most widely distributed crops, available for sale at most food outlets.
In 1902, a method was developed for measuring the strength of a given variety of capsicum,
ranking it on predetermined scale. This originally meant tasting the peppers, but nowadays it can
Y

be done more accurately with the help of computers to rate the peppers in units to indicate parts
per million of capsaicin. This potent chemical not only causes the fiery sensation, but also triggers
DẠ

the brain to produce endorphins, natural painkillers that promote a sense of well-being.
D. Writing about cooking

14
Two cookery writers are often credited with the present revival of interest in food and
cooking. Elizabeth David discovered her taste for good food when she lived with a French family
for two years. After returning to England she learnt to cook so that she could reproduce some of

L
the food that she had come to appreciate in France. Her first book appeared when rationing was

A
still in force after the war and most of the ingredients she had so lovingly described were not
available. At the time her books was read rather than used, and it created a yearning for good

CI
ingredients and for a way of life that saw more in food than mere sustenance. Her late books
confirmed her position as the most inspirational and influential cookery writer in the English

FI
language. She shared with Jane Grigson an absorbing interest in the literature of cookery.
Jane Grigson was brought up in the north-east of England, where there is a strong tradition
of good eating, but it was not until she began to spend time in France that she became really

OF
interested in food. She was renowned for her fine writing on food and cookery, often catching the
imagination with a deftly chosen fragment of history or poetry, but never failing to explain the
“why” as well as the “how” of cookery.
In which section are the following mentioned?

ƠN
96. a group of foods that changed the way an area of the world cooked
97. a period of time access to food was restricted
98. a comparison of the process of producing a substance with that used for some other foods,
too
NH
99. the global popularity of a particular food
100. an interest in discovering more about unfamiliar types of food
101. the discrepancy between the amount of a type of food produced and the demand for it
102. a substance that reinforces the savoury aspect of food
Y

103. a way of determining the strength of a foodstuff


104. using literay forms to talk about food dishes
QU

105. worries about the ethical future of a food industry


(CPE Practice tests)
Your answer
96. 97. 98. 99. 100. 101. 102. 103. 104. 105.
M

D. WRITING (60 points)


Y

Part 1. Read the following extract and use your own words to summarize it. Your summary
should be about 140 words. You MUST NOT copy the original. (15 pts)
DẠ

Achieving Peak Performance


There are seven steps to achieve peak performance. The first step is to lead at well-rounded life.
High achievers, according to experts, are obsessed people who take work home and then labor
15
over it until bedtime. Furthermore, research has also shown that such people tend to peak early
and then go into a decline or level off. They then become addicted to work itself, with less concern
for results.

L
High performers, in contrast, are willing to work hard but within strict limits. For them, work is

A
not everything and they know how to relax. They are able to leave work at the office. They value
close friendships and family life, and spend a healthy amount of time with their families.

CI
The second step is to select a carcer you care about. Studies show that high performers choose
work they truly prefer, and spend over two-thirds of their working hours doing it and only one-

FI
third on disliked tasks. They want internal satisfaction and not just external results such as pay
rises and promotions. In the end, of course, they often have both. Since they enjoy what they do,
they produce better work and the rewards are higher.

OF
Rehearsing each challenge or task mentally is the third step to achieving peak performance. Before
any difficult or important situation - a public presentation, a board meeting, a key tennis match,
for example - most peak performers run their desired actions through in their minds over and over
again. Nearly all of us day-dream. about important coming events, but idle day-dreaming is not

ƠN
the same as a deliberate mental workout that sharpens the skills to be used in the activity.
In order to achieve peak performance, you also have to seek results, not perfection. Many
ambitious and hardworking people are so obsessed with perfection that they produce very little
work. It has been found that those with perfectionist tendencies earned considerably less a year
NH
than those who did not have such tendencies. In contrast, high performers are almost always free
of the compulsion to be perfect. They do not think of their mistakes as failures, but they learn from
mistakes so that they can do better the next time
The next step is to be willing to take risks. Most people are willing to settle for jobs which they
Y

think are secure, even if that also means mediocrity and boredom, rather than take changes. High
performers, on the other hand, are able to take risks because they would carefully consider how
QU

they would adjust and how they would salvage the situation if, in reality they did fail. Constructing
a 'worst-case' scenario allows them to make a rational choice.
The penultimate step to achieving peak performance is not to underestimate your own, potential.
Most of us think we know our own limits, but much of what we "know' is not knowledge at all. It
M

could be a belief which is erroneous and self-limiting. These types of beliefs are the biggest barriers
to achieving high-level performance. Too many of us set our individual limits far below what we

can actually achieve. High performers, on the contrary, are able to ignore artificial barriers. They
concentrate instead on their own feelings, on their functioning, on the momentum of their effort
and are therefore free to achieve peak levels.
Finally, compete with yourself, not with others. High performers focus more on improving on their
Y

own previous efforts than on competing with others.


Such are the skills of high performers. If you want to make the most of your talents and to live up
DẠ

to your fullest potential, learn to use these skills.


__________________________________________________________________
__________________________________________________________________
16
__________________________________________________________________
__________________________________________________________________

L
__________________________________________________________________
__________________________________________________________________

A
__________________________________________________________________

CI
__________________________________________________________________
__________________________________________________________________

FI
__________________________________________________________________
__________________________________________________________________

OF
__________________________________________________________________
__________________________________________________________________
__________________________________________________________________
__________________________________________________________________

ƠN
__________________________________________________________________
__________________________________________________________________
__________________________________________________________________
NH
__________________________________________________________________
__________________________________________________________________
__________________________________________________________________
__________________________________________________________________
Y

__________________________________________________________________
QU

__________________________________________________________________

Part 2. The bar chart below illustrates five different industries’ percentage share of Brazil’s
economy in 2009 and 2019 with a forecast for 2029.
M

Summarise the information by selecting and reporting the main features, and make
comparisons where relevant. (15 points)

Y
DẠ

17
Industries as a percentage of Brazil's economy
45%

L
40%

A
35%
30%

CI
25%
20%
15%

FI
10%
5%

OF
0%
Finance Food Manufacturing Tourism Oil, coal and
processing gas
2009 2019 2029

__________________________________________________________________

ƠN
__________________________________________________________________
__________________________________________________________________
__________________________________________________________________
NH
__________________________________________________________________
__________________________________________________________________
__________________________________________________________________
__________________________________________________________________
Y

__________________________________________________________________
QU

__________________________________________________________________
__________________________________________________________________
__________________________________________________________________
__________________________________________________________________
M

__________________________________________________________________
__________________________________________________________________

__________________________________________________________________
__________________________________________________________________
__________________________________________________________________
Y

__________________________________________________________________
DẠ

__________________________________________________________________
__________________________________________________________________
__________________________________________________________________
18
Part 3. Essay writing (30 points)
Some people enjoy change and look forward to new experiences. Others like their lives to remain
the same and do not like any changes. Discuss both views and give your opinion.

L
Give reasons and relevant examples to support your answer. You should write at least 350 words.

A
__________________________________________________________________
__________________________________________________________________

CI
__________________________________________________________________
__________________________________________________________________

FI
__________________________________________________________________
__________________________________________________________________

OF
__________________________________________________________________
__________________________________________________________________
__________________________________________________________________

ƠN
__________________________________________________________________
__________________________________________________________________
__________________________________________________________________
__________________________________________________________________
NH
__________________________________________________________________
__________________________________________________________________
__________________________________________________________________
__________________________________________________________________
Y

__________________________________________________________________
QU

__________________________________________________________________
__________________________________________________________________
__________________________________________________________________
__________________________________________________________________
M

__________________________________________________________________

__________________________________________________________________
__________________________________________________________________
__________________________________________________________________
__________________________________________________________________
Y

__________________________________________________________________
DẠ

__________________________________________________________________
__________________________________________________________________
__________________________________________________________________
19
__________________________________________________________________
__________________________________________________________________

L
__________________________________________________________________
__________________________________________________________________

A
__________________________________________________________________

CI
__________________________________________________________________
__________________________________________________________________

FI
__________________________________________________________________
__________________________________________________________________

OF
__________________________________________________________________
__________________________________________________________________
__________________________________________________________________
__________________________________________________________________

ƠN
__________________________________________________________________
__________________________________________________________________
__________________________________________________________________
NH
__________________________________________________________________
__________________________________________________________________
__________________________________________________________________
__________________________________________________________________
Y

__________________________________________________________________
QU

__________________________________________________________________
__________________________________________________________________
__________________________________________________________________
__________________________________________________________________
M

__________________________________________________________________
__________________________________________________________________

__________________________________________________________________
__________________________________________________________________
__________________________________________________________________
Y

__________________________________________________________________
- THE END -
DẠ

20
TRƯỜNG THPT CHUYÊN KỲ THI HỌC SINH GIỎI CÁC TRƯỜNG THPT CHUYÊN
TỈNH TUYÊN QUANG KHU VỰC DUYÊN HẢI VÀ ĐỒNG BẰNG BẮC BỘ
LẦN THỨ XIV, NĂM 2023

L
MÔN: TIẾNG ANH – LỚP 11

A
HƯỚNG DẪN CHẤM

CI
(Hướng dẫn chấm gồm có 4 trang)

A. LISTENING (50 points)

FI
Part 1. For questions 1-5, you will hear a talk about epidemiology. Listen and decide whether
the following sentences are true (T) or false (F). Write your answers in the corresponding

OF
numbered boxes on the answer sheet. (10 points– 2 points/a correct answer)
1. T 2. F 3. T 4. F 5. T
Part 2. For question 6-10, listen to a talk about the future of fashion and answer the questions.
WRITE NO MORE THAN FOUR WORDS taken from the recording for each answer in the

ƠN
recording numbered boxes provided. (10 points– 2 points/a correct answer)
6. cognitive computing
7. trendspotting mistakes
8. environmental footprint
NH
9. analyzing images
10. (a) body measurement suit
Part 3. You will hear an interview with Maria Stefanovich, co-founder of a creativity group
which organises workshop executives. For questions 11-15, choose the answer (A, B, C or D)
Y

which fits best according to what you hear. (10 points– 2 points/a correct answer)
QU

11. D 12. B 13. C 14. A 15. D


Part 4. For questions 16-25, you will listen to a presenter talking abour a phenomenon in the
nature. Complete the summary by writing NO MORE THAN THREE WORDS in each gap.
Write your answers in the corresponding numbered boxes on the answer sheet. (20 points – 2
M

points/a correct answer)


16. Nutrients

17. ferlilizer
18. gauge
19. algal bloom
20. impenetrable roof
Y

21. decomposers
22. dissolved oxygen
DẠ

23. positive feedback loop


24. suppressed
25. eutrophication
1
B. LEXICO – GRAMMAR (30 points)
Part 1. For questions 26-45, choose the best option A, B, C or D to complete the following
sentences and write your answers in the corresponding numbered boxes provided. (20 points –

L
1 point/a correct answer)

A
26. C 27. A 28. D 29. C 30. B 31. B 32. C 33. A 34. B 35. D
36. B 37. D 38. A 39. C 40. D 41. C 42. D 43. A 44. B 45. C

CI
Part 2. For questions 46-55, give the correct form of each given word to complete the following

FI
sentences and write your answers in the corresponding numbered provided. (10 points- 1 point/a
correct answer)
46. juxtaposition 51. unadulterated

OF
47. streetwise 52. consortium
48. page-turner 53. dehydration
49. inclemency 54. derelict
50. ahistorical 55. unpardonable

C. READING (60 points)


ƠN
Part 1. For questions 56-65, fill each of the following numbered blanks with ONE suitable word
NH
and write your answers in the corresponding numbered provided. (15 points– 1.5 points/a
correct answer)
56. 58. takes 60. span 62. that 64. distract
happens/occurs
57. out 59. doing 61. whilst/ 63. time 65. interrupt
Y

whereas/ albeit/
QU

while

Part 2. For questions 66-75, read the passage below and choose the answer A, B, C or D that
fits best according to the text. Write your answers in the corrresponding numbered boxes
provided. (10 points – 1 point/a correct answer)
M

66. C 67. A 68. C 69. B 70. D 71. B 72. C 73. C 74. A 75. B

Part 3. For questions 76-88, read the following passage and do the tasks that follow. (13 points–
1 point/a correct answer)
Y

76. F 77. NG 78. F 79. F 80. T 81. T


DẠ

82. 83. 84. 85. 86. 87. 88. cost


chemical Ascanio gunpower Stockholm detonator pnematic
engineering Sobrero drill

2
Part 4: In the passage below, seven paragraphs have been removed. Read the passage and
choose from paragraphs A-H the one which fits each gap. There is ONE extra pragraph which

L
you do not need to use. Write your answers in the corresponding numbered boxes provided on

A
the answer sheet. (7 points – 1 point/a correct answer)

CI
89. D 90. G 91. E 92. A 93. F 94. C 95. H

FI
Part 5: For questions 96-105, you are going to read an extract about food ans cooking. Answer
the questions by choosing from the sections of the extract (A - D). The sections may be chosen
more than once. Write your answers in the corresponding numbered boxes provided on the

OF
answer sheet. (15 points – 1 point/a correct answer)

96. C 97. A 98. D 99. A 100. D 101. D 102. C 103. B 104. C 105. B

ƠN
D. WRITING (60 points)
Part 1. Read the following extract and use your own words to summarize it. Your summary
should be about 140 words. You MUST NOT copy the original. (15 pts)
a) Contents (10 points)
NH
The summary MUST cover the following points:
* Main topic: (2 points)
* Supporting ideas: (8 points)
b) Language use (5 points)
Y

The summary:
QU

- should show attempts to convey the main ideas of the original text by means of paraphrasing
(structural and lexical use)
- should demonstrate correct use of grammatical structures, vocabulary, and mechanics (spelling,
punctuations, ...)
- should maintain coherence, cohesion, and unity throughout (by means of linkers and
M

transitional devices)

Suggested Answer:
There are seven steps which can lead to peak performance. Firstly, have a well-balanced life work
hard but know when and how to relax and enjoy your life. The second step is to choose a career
you love since internal satisfaction gives better results and rewards. The next step is to rehearse a
Y

task mentally before actually doing it. Another step is not to be a perfectionist but to be a risk-
DẠ

taker, and to pursue results and learn from mistakes. High performers never underestimate
themselves but concentrate on their capabilities. Lastly, compete only with yourself. not with
others, (98 words)

3
Part 2: (15 points)
Contents (10 points)
The report MUST cover the following points:

L
- Introduce the charts (2 points) and state the overall trends and striking features (2 points)

A
- Describe main features with relevant data from the charts and make relevant comparisons (6
points)

CI
Language use (5 points)
The report:

FI
- should demonstrate a wide variety of lexical and grammatical structures,
- should have correct use of words (verb tenses, word forms, voice...) and mechanics (spelling,

OF
Part 3: (30 points)
The mark given to part 3 is based on the following criteria:
1. Organization (5 points)
a. Ideas are well organized and presented with coherence, cohesion and unity.

ƠN
b. The essay is well-structured:
* Introduction is presented with clear thesis statement.
* Body paragraph are written with unity, coherence and cohesion.
Each body paragraph must have a topic sentence and supporting details and examples when
NH
necessary.
* Conclusion summarizes the main points and offers personal opinions (prediction,
recommendation, consideration ...) on the issue.
2. Content (15 points)
Y

a. All requirements of the task are sufficiently addressed.


b. Ideas are adequately supported and elaborated with relevant and reliable explanations,
QU

examples, evidence....
3. Language use (10 points)
a. Demonstration of a variety of topic-related vocabulary.
b. Excellent use and control of grammatical structures (verb tenses, word forms, voice...) and
M

mechanics (spelling, punctuations...)


- THE END -

Y
DẠ

You might also like